Duane E. Haines
Neuroanatomy
An Atlas of
Structures, Sections,
and Systems.
SIXTH EDITION
Contents
Preface to Sixth Edition
v
Preface to the First Edition
vii
Acknowledgments
ix
Chapter 1
Introduction and Reader’s Guide
1
Including Rationale for Labels and Abbreviations
8
Chapter 2
External Morphology of the Central Nervous System
9
The Spinal Cord: Gross Views and Vasculature
10
The Brain: Lobes, Principle Brodmann Areas, Sensory-Motor Somatotopy
13
The Brain: Gross Views, Vasculature, and MRI
16
The Cranial Nerves in MRI
38
The Insula: Gross View and MRI
45
The Meninges, Cisterns, and Meningeal and Cisternal Hemorrhages
46
The Ventricles and Ventricular Hemorrhages
52
Chapter 3
Dissections of the Central Nervous System
55
Lateral, Medial, and Ventral Aspects
56
Overall Views
59
Chapter 4
Internal Morphology of the Brain in Slices and MRI
63
Brain Slices in the Coronal Plane Correlated with MRI
63
Brain Slices in the Axial Plane Correlated with MRI
73
Chapter 5
Internal Morphology of the Spinal Cord and Brain in Stained Sections
83
The Spinal Cord with CT and MRI
84
Arterial Patterns Within the Spinal Cord With Vascular Syndromes
94
The Degenerated Corticospinal Tract
96
The Medulla Oblongata with MRI and CT
98
Arterial Patterns Within the Medulla Oblongata With Vascular Syndromes
110
The Cerebellar Nuclei
112
The Pons with MRI and CT
116
Arterial Patterns Within the Pons With Vascular Syndromes
124
The Midbrain with MRI and CT
126
Arterial Patterns Within the Midbrain With Vascular Syndromes
136
The Diencephalon and Basal Nuclei with MRI
138
Arterial Patterns Within the Forebrain With Vascular Syndromes
158
Chapter 6
Internal Morphology of the Brain in Stained Sections:
Axial-Sagittal Correlations with MRI
161
Axial-Sagittal Correlations
162
Chapter 7
Synopsis of Functional Components, Tracts, Pathways, and Systems
173
Components of Cranial and Spinal Nerves
174
Orientation
176
Sensory Pathways
178
Motor Pathways
190
Cerebellum and Basal Nuclei
204
Optic, Auditory, and Vestibular Systems
220
Limbic System
232
xi
xii Contents
Chapter 8
Anatomical-Clinical Correlations: Cerebral Angiogram, MRA, and MRV
239
Cerebral Angiogram, MRA, and MRV
240
Blood Supply to the Choroid Plexi
251
Overview of Vertebral and Carotid Arteries
252
Chapter 9
Q&A’s: A Sampling of Study and Review Questions, Many in the USMLE Style,
All With Explained Answers
253
Sources and Suggested Readings
297
Index
301
Preface to the Sixth Edition
revious editions of Neuroanatomy have endeavored 1) to provide a
drawing and its companion stained section in the anatomical orienta-
P
structural basis for understanding the function of the central ner-
tion. At the same time, it introduces, on the same set of pages, the im-
vous system; 2) to emphasize points of clinical relevance through use
portant concept that CNS anatomy, both external and internal, is ori-
of appropriate terminology and examples; and 3) to integrate neuro-
ented differently in MRI or CT. It is the clinical orientation issue that
anatomical and clinical information in a format that will meet the edu-
will confront the student/clinician in the clinical setting. It is certainly
cational needs of the user. The goal of the sixth edition is to continue
appropriate to introduce, and even stress, this view of the brain and
this philosophy and to present structural information and concepts in
spinal cord in the basic science years.
an even more clinically useful and relevant format. Information learned
Third, new images have been included in chapter 8. These include,
in the basic science setting should flow as seamlessly as possible into the
but are not limited to, new examples of general vessel arrangement in
clinical setting.
MRA, examples of specific vessels in MRI, and some additional exam-
I have received many constructive suggestions and comments from
ples of hemorrhage.
my colleagues and students. This is especially the case for the modifi-
Fourth, additional examples of cranial nerves traversing the sub-
cations made in Chapters 2, 5, 7, 8, and 9 in this new edition. The
arachnoid space are included. In fact, the number of MRI showing cra-
names of the individuals who have provided suggestions or comments
nial nerves has been doubled. In addition, each new plate starts with a
are given in the Acknowledgments. This thoughtful and helpful input
gross anatomical view of the nerve (or nerves) shown in the succeed-
is greatly appreciated and has influenced the preparation of this new
ing MRI in that figure.
edition.
Fifth, additional clinical information and correlations have been in-
The major changes made in the sixth edition of Neuroanatomy are as
cluded. These are in the form of new images, new and/or modified fig-
follows:
ure descriptions, and changes in other portions of the textual elements.
First, recognizing that brain anatomy is seen in clear and elegant de-
Sixth, in some instances, existing figures have been relocated to im-
tail in MRI and CT, and that this is the primary way the brain is viewed
prove their correlation with other images. In other instances, existing
in the health care setting, additional new images have been incorporated
figures have been repeated and correlated with newly added MRI or
into this new edition. Every effort has been made to correlate the MRI
CT so as to more clearly illustrate an anatomical-clinical correlation.
or CT with brain or spinal cord anatomy by relating these images on the
Seventh, a new chapter (chapter 9), consisting of approximately 240
same page or on facing pages. New MRI or CT have been introduced
study and review questions and answers in the USMLE style, has been
into chapter 2 (spinal cord, meningeal hemorrhages correlated with the
added. All of these questions have explained answers keyed to specific
meninges, cisterns, hemorrhage into the brain, hemorrhage into the
pages in the Atlas. Although not designed to be an exhaustive set, this
ventricles correlated with the structure of the ventricles), chapter 5
new chapter should give the user of this atlas a unique opportunity for
(spinal cord and brainstem), and chapter 8 (vascular).
self-assessment.
Second, the structure of the central nervous system should be avail-
Two further issues figured prominently in the development of this
able to the student (or the medical professional for that matter) in a for-
new edition. First, the question of whether to use eponyms in their
mat that makes this information immediately accessible, and applica-
possessive form. To paraphrase one of my clinical colleagues “Parkin-
ble, to the requirements of the clinical experience. It is commonplace
son did not die of his disease (Parkinson disease), he died of a stroke;
to present brain structure in an anatomical orientation (e.g., the colli-
it was never his own personal disease.” There are rare exceptions, such
culi are “up” in the image and the interpeduncular fossa is “down”).
as Lou Gehrig’s disease, but the point is well taken. McKusick
However, when the midbrain is viewed in an axial MRI or CT, the re-
(1998a,b) has also made compelling arguments in support of using the
verse is true: the colliculi are “down” in the image and the interpedun-
non-possessive form of eponyms. It is, however, acknowledged that
cular fossa is “up”. There are many good reasons for making brainstem
views differ on this question—much like debating how many angels
images available in an anatomical orientation and for teaching this view
can dance on the head of a pin. Consultation with my neurology and
in the academic setting. These reasons are recognized in this book. On
neurosurgery colleagues, a review of some of the more comprehensive
the other hand, the extensive use of MRI or CT in all areas of medi-
neurology texts (e.g., Rowland, 2000; Victor and Ropper, 2001), and
cine, not just the clinical neurosciences, requires that students be
the standards established in The Council of Biology Editors Manual for
clearly aware of how brain and spinal cord structure is viewed, and used, in
Authors, Editors, and Publishers (1994) and the American Medical As-
the clinical environment. To address this important question, a series of
sociation’s Manual of Style (1998) clearly indicate an overwhelming
illustrations, including MRI or CT, are introduced in the spinal cord
preference for the non possessive form. Recognizing that many users
and brainstem sections of chapter 5. These images are arranged to show
of this book will enter clinical training, it was deemed appropriate to
1) the small colorized version of the spinal cord or brainstem in an
encourage a contemporary approach. Consequently, the non posses-
anatomical orientation; 2) the same image flipped bottom-to-top into
sive form of the eponym is used.
a clinical orientation; and 3) the clinical orientation of the colorized
The second issue concerns use of the most up-to-date anatomical
line drawing followed by T1 and T2 MRI and/or CT at levels compa-
terminology. With the publication of Terminologia Anatomica (Thieme,
rable to the line drawing and corresponding stained section. This ap-
New York, 1998), a new official international list of anatomical terms
proach retains the inherent strengths of the full-page, colorized line
for neuroanatomy is available. This new publication, having been
v
vi Preface to the Sixth Edition
adopted by the International Federation of Associations of Anatomists,
References:
supersedes all previous terminology lists. Every effort has been made
Council of Biology Editions Style Manual Committee. Scientific Style and
to incorporate any applicable new or modified terms into this book.
Format—The CBE Manual for Authors, Editors, and Publishers. 6th Ed.
The number of changes is modest and related primarily to directional
Cambridge: Cambridge University Press, 1994.
terms: posterior for dorsal, anterior for ventral, etc. In most cases, the
Federative Committee on Anatomical Terminology. Terminologia Ana-
previous term appears in parentheses following the official term, i.e.,
tomica. Thieme, Stuttgart and New York, 1998.
posterior (dorsal) cochlear nucleus. It is almost certain that some changes
Iverson, MA et al. American Medical Association Manual of Style—A Guide
have eluded detection; these will be caught in subsequent printings.
for Authors and Editors. 9th Ed. Baltimore: Williams & Wilkins, 1998.
Last, but certainly not least, the sixth edition is a few pages
McKusick, VA. On the naming of clinical disorders, with particular ref-
longer than was the fifth edition. This results exclusively from the
erence to eponyms. Medicine 1998;77: 1-2.
inclusion of more MRI and CT, a better integration of anatomical-
McKusick, VA. Mendelian Inheritance in Man, A Catalog of Human Genes
clinical information, including more clinical examples (text and il-
and Genetic Disorders. 12th Ed. Baltimore: The Johns Hopkins Uni-
lustrations), and the inclusion of Study/Review and USMLE style
versity Press, 1998.
questions with explained answers.
Rowland, LP. Merritt’s Neurology. 10th Ed. Baltimore: Lippincott Williams
& Wilkins, 2000.
Duane E. Haines
Victor, M and Ropper, AH. Adams and Victor’s Principles of Neurology. 7th
Jackson, Mississippi
Ed. New York: McGraw-Hill, Medical Publishing Division, 2001.
Preface to the First Edition
his atlas is a reflection of, and a response to, suggestions from pro-
dents in the laboratory and greatly enhances their ability to grasp and
T
fessional and graduate students over the years I have taught human
retain information on CNS connections. While this atlas does not at-
neurobiology. Admittedly, some personal philosophy, as regards
tempt to teach clinical concepts, a chapter correlating selected views
teaching, has crept into all parts of the work.
of angiograms and CT scans with morphological relationships of cere-
The goal of this atlas is to provide a maximal amount of useful in-
bral arteries and internal brain structures is included. These examples
formation, in the form of photographs and drawings, so that the initial
illustrate that a clear understanding of normal morphological relation-
learning experience will be pleasant, logical, and fruitful, and the re-
ships, as seen in the laboratory, can be directly transposed to clinical
view process effective and beneficial to longterm professional goals. To
situations.
this end several guiding principles have been followed. First, the entire
This atlas was not conceived with a particular audience in mind. It
anatomy of the central nervous system (CNS), external and internal,
was designed to impart a clear and comprehensive understanding of
has been covered in appropriate detail. Second, a conscientious effort
CNS morphology to its readers, whoever they may be. It is most obvi-
has been made to generate photographs and drawings of the highest
ously appropriate for human neurobiology courses as taught to med-
quality: illustrations that clearly relay information to the reader. Third,
ical, dental, and graduate students. In addition, students in nursing,
complementary information always appears on facing page. This may
physical therapy, and other allied health curricula, and psychology as
take the form of two views of related structures such as brainstem or
well, may also find its contents helpful and applicable to their needs.
successive brain slices or a list of abbreviations and description for a
Inclusion and integration of blood vessel patterns, both external and in-
full-page figure. Fourth, illustrations of blood supply have been in-
ternal, and the summary pathway drawings may be useful to the indi-
cluded and integrated into their appropriate chapters. When gross
vidual requiring a succinct, yet comprehensive review before taking
anatomy of the brain is shown, the patterns of blood vessels and rela-
board exams in the neurological, neurosurgical, and psychiatric spe-
tionships of sinuses appear on facing pages. The distribution pattern of
cialties.
blood vessels to internal CNS structures is correlated with internal
The details in some portions of this atlas may exceed that found in
morphology as seen in stained sections. Including information on ex-
comparable parts of other atlases. If one is to err, it seems more judi-
ternal vascular patterns represents a distinct departure from what is
cious to err on the side of greater detail than on the side of inadequate
available in most atlases, and illustrations of internal vessel distribution
detail. If the student is confronted with more information on a partic-
are unique to this atlas.
ular point than is needed during the initial learning process, he or she
There are other features which, although not unique in themselves,
can simply bypass the extra information. However, once the initial
do not usually appear in atlas format. In the chapter containing cross-
learning is completed, the additional information will be there to en-
sections, special effort has been made to provide figures that are accu-
hance the review process. If students have inadequate information in
rate, clear, and allow considerable flexibility in how they can be used
front of them it may be difficult, or even impossible, to fill in missing
for both teaching and learning. The use of illustrations that are one-half
points that may not be part of their repertoire of knowledge. In addi-
photograph and one-half drawing is not entirely novel. In this atlas,
tion, information may be inserted out of context, and, thereby, hinder
however, the sections are large, clearly labeled, and the drawing side
the learning experience.
is a mirror-image of the photograph side. One section of the atlas is de-
A work such as this is bound to be subject to oversights, and for such
voted to summaries of a variety of major pathways. Including this ma-
foibles, I am solely responsible. I welcome comments, suggestions, and
terial in a laboratory atlas represents a distinct departure from the stan-
corrections from my colleagues and from students.
dard approach. However, feedback over the years strongly indicates
that this type of information in atlas format is extremely helpful to stu-
Duane E. Haines
vii
Acknowledgments
s was the case in previous editions of this book, my colleagues and
A. Rosenquist, M. Schwartz, J. Scott, V. Seybold, D. Smith, S. Sten-
A
students in both medical and graduate programs have been most
saas, D. Tolbert, F. Walberg, S. Walkley, M. Woodruff, M. Wyss,
gracious in offering their suggestions and comments. I greatly appreci-
and B. Yezierski. The stained sections used in this atlas are from the
ate their time and interest in the continuing usefulness of this book.
teaching collection in the Department of Anatomy at West Virginia
As changes were being contemplated for this new edition, input on
University School of Medicine.
potential modifications was solicited from faculty as well as students in
Dr. R. Brent Harrison (former Chairman of Radiology, UMMC),
an effort to ascertain how these changes might impact on the usefulness
Dr. Robert D. Halpert (current Chairman of Radiology, UMMC) and
of this Atlas. These individuals went out of their way to review the doc-
Dr. Gurmett Dhilon (Neuroradiology) generously continue to give me
uments that were provided and to give insightful, and sometimes
full access to all their facilities. I would like to express a special thanks
lengthy, comments on the pros and cons of the ideas being considered.
to Mr. W. (Eddie) Herrington (Chief CT/MRI Technologist) and Mr.
This input was taken into consideration as the initial plans were modified
Joe Barnes (Senior MRI Technologist) for their outstanding efforts to
and finalized by the author and then incorporated into this new edition.
supply new images and their special efforts to generate images at spe-
The faculty who gave generously of their time and energy were Drs. A.
cific planes for this new edition. In the same vein, Drs. G. Dhilon and
Agmon, C. Anderson, R. Baisden, S. Baldwin, J. L. Culberson, B. Hal-
S. Crawford also made special attempts to get specific MRI at special
las, J. B. Hutchins, T. Imig, G. R. Leichnetz, E. Levine, R. C. S. Lin, J.
planes. I am also deeply appreciative to several technologists and nurses
C. Lynch, T. McGraw-Ferguson, G. F. Martin, G. A. Mihailoff, R. L.
in the CT/MRI suite, and particularly to Master Johnathan Barnes, for
Norman, R. E. Papka, H. J. Ralston, J. Rho, L. T. Robertson, J. D.
being such cooperative “patients” as we worked to generate scans that
Schlag, K. L. Simpson, and C. Stefan. The students who offered helpful
matched stained sections in the Atlas as closely as possible.
and insightful comments were A. Alqueza (medical student, University
Modifications, both great and small, to the artwork and labeling
of Florida at Gainesville), A. S. Bristol (graduate student, University of
scheme, as well as some new renderings, were the work of Mr.
California at Irvine), L. Simmons (medical student, Vanderbilt Univer-
Michael Schenk (Director of Biomedical Illustration Services). Mr. Bill
sity), J. A. Tucker (medical student, The University of Mississippi Med-
Armstrong (Director of Biomedical Photography) produced outstand-
ical Center), S. Thomas (graduate student, University of Maryland at
ing photographs of gross specimens and slices, CTs, MRIs, and MRAs.
College Park), and M. Tomblyn (medical student, Rush Medical Col-
I am very appreciative of the time, effort, and dedication of these indi-
lege). I greatly appreciate their comments and suggestions.
viduals to create the very best artwork and photographs possible for
I would also like to thank my colleagues in the Department of
this new edition. Ms. Katherine Squires did all the typing for the sixth
Anatomy at The University of Mississippi Medical Center (UMMC) for
edition. Her excellent cooperation, patience, and good-natured repar-
their many helpful suggestions and comments. My colleagues in the
tee with the author were key elements in completing the final draft in
Department of Neurosurgery at UMMC (Drs. A. Parent [Chairman],
a timely manner.
L. Harkey, J. Lancon, J. Ross, D. Esposito, and G. Mandybur) and in
This sixth edition would not have been possible without the inter-
the Department of Neurology at UMMC (especially Drs. J. Corbett
est and support of the publisher, Lippincott Williams & Wilkins. I want
[Chairman], S. Subramony, H. Uschmann, and M. Santiago) have of-
to express thanks to my editor, Ms. Betty Sun (Acquisitions Editor), to
fered valuable input on a range of clinical issues. I am especially in-
Mr. Dan Pepper (Associate Managing Editor), to Ms. Erica Lukenich
debted to Dr. J. A. Lancon (Neurosurgery) for his significant contri-
(Editorial Assistant), Ms. Jennifer Weir (Associate Production Man-
butions to this new edition. These include his willingness to participate
ager), and to Mr. Joe Scott (Marketing Manager) for their encourage-
as co-author of Chapter 9 and his careful review of all new clinical in-
ment, continuing interest, and confidence in this project. Their coop-
formation added to the book. I would also like to thank Ms. Amanda
eration has given me the opportunity to make the improvements seen
Ellis, B.S.N., for keeping my friend John on track.
herein.
I am indebted to the following individuals for their careful review
Last, but certainly not least, I would like to express a special thanks
of previous editions of the book: Drs. B. Anderson, R. Borke, Patricia
to my wife, Gretchen. She put up with me while these revisions were
Brown, Paul Brown, T. Castro, B. Chronister, A. Craig, E. Dietrichs,
in progress, carefully reviewed all changes in the text and all ques-
J. Evans, B. Falls, C. Forehand, R. Frederickson, E. Garcis-Rill, G.
tions/answers, and was a tangible factor in getting everything done. I
Grunwald, J. King, A. Lamperti, K. Peusner, C. Phelps, D. Rosene,
dedicate this edition to Gretchen.
ix
CHAPTER
1
Introduction
and
Reader’s Guide
2
Introduction and Reader’s Guide
t a time when increasing numbers of atlases and text-
The present atlas addresses these points. The goal is not
A
books are becoming available to students and instruc-
only to show external and internal structure per se but also
tors, it is appropriate to briefly outline the approach used in
to demonstrate that the relationship between brain anatomy
this volume. Most books are the result of 1) the philosophic
and MRI/CT, the blood supply to specific areas of the CNS
approach of the author/instructor to the subject matter and
and the arrangement of pathways located therein, the neu-
2) students’ needs as expressed through their suggestions
roactive substances associated with pathways, and examples
and opinions. The present atlas is no exception, and as a re-
of clinical deficits are inseparable components of the learn-
sult, several factors have guided its further development.
ing experience. An effort has been made to provide a for-
These include an appreciation of what enhances learning in
mat that is dynamic and flexible—one that makes the learn-
the laboratory and classroom, the inherent value of corre-
ing experience an interesting and rewarding exercise.
lating structure with function, the clinical value of under-
standing the blood supply to the central nervous system
The relationship between blood vessels and specific brain
(CNS), and the essential importance of integrating anatomy
regions (external and/or internal) is extremely important
with clinical information and examples. The goal is to make
considering that approximately 50% of what goes wrong in-
it obvious to the user that structure and function in the CNS
side the skull, producing neurological deficits, is vascular-
are integrated elements and not separate entities.
related. To emphasize the value of this information, the dis-
Most neuroanatomic atlases approach the study of the
tribution pattern of blood vessels is correlated with external
CNS from fundamentally similar viewpoints. These atlases
spinal cord and brain anatomy (Chapter 2) and with inter-
present brain anatomy followed by illustrations of stained
nal structures such as tracts and nuclei (Chapter 5), re-
sections, in one or more planes. Although variations on this
viewed in each pathway drawing (Chapter 7), and shown in
theme exist, the basic approach is similar. In addition, most
angiograms, MRAs, and MRVs (Chapter 8). This approach
atlases do not make a concerted effort to correlate vascular
has several advantages: 1) the vascular pattern is immediately
patterns with external or internal brain structures. Also,
related to the structures just learned, 2) vascular patterns
most atlases include little or no information on neurotrans-
are shown in the sections of the atlas in which they belong,
mitters and do not integrate clinical examples and informa-
3) the reader cannot proceed from one part of the atlas to
tion with the study of functional systems.
the next without being reminded of blood supply, and 4) the
Understanding CNS structure is the basis for learning path-
conceptual importance of the distribution pattern of blood
ways, neural function, and for developing the skill to diagnose
vessels in the CNS is repeatedly reinforced.
the neurologically impaired patient. Following a brief period
The ability to diagnose a neurologically compromised pa-
devoted to the study of CNS morphology, a significant por-
tient is specifically related to a thorough understanding of
tion of many courses is spent learning functional systems. This
pathway structure, function, blood supply, and the rela-
learning experience may take place in the laboratory because
tionships of this pathway to adjacent structures. To this end
it is here that the student deals with images of representative
Chapter 7 provides a series of semidiagrammatic illustrations
levels of the entire neuraxis. However, few attempts have
of various clinically relevant pathways. Each figure shows 1)
been made to provide the student with a comprehensive and in-
the trajectory of fibers that comprise the entire pathway; 2)
tegrated guide—one that correlates, 1) external brain anatomy
the laterality of fibers comprising the pathway, this being an
with MRI and blood supply; 2) meninges and ventricles with
extremely important concept in diagnosis; 3) the positions
examples of meningeal, ventricular, and brain hemorrhage;
and somatotopy of fibers comprising each pathway at repre-
3) internal brain anatomy with MRI, blood supply, the orga-
sentative levels; 4) a review of the blood supply to the en-
nization of tracts and nuclei and selected clinical examples; 4)
tire pathway; 5) important neurotransmitters associated
summaries of clinically relevant pathways with neurotrans-
with fibers of the pathway; and 6) examples of deficits seen
mitters, numerous clinical correlations, and the essential con-
following lesions of the pathway at various levels through-
cept of laterality; and 5) includes a large variety of images such
out the neuraxis. This chapter is designed to be used by itself
as angiogram, computed tomography (CT), magnetic reso-
or integrated with other sections of the atlas; it is designed to
nance imaging
(MRI), magnetic resonance angiography
provide the reader with the structural and clinical essentials
(MRA), and magnetic resonance venography (MRV).
of a given pathway in a single illustration.
Introduction and Reader’s Guide
3
The advent and common use of imaging methods (MRI,
The Brain and Related Structures in CT
MRA, and MRV) mandates that such images become an inte-
STRUCTURE/FLUID/SPACE
GREY SCALE
gral part of the educational process when teaching and/or
Bone, acute blood
Very white
learning clinically applicable neuroscience. To this end, this
Enhanced tumor
Very white
book contains about 175 MRI and CT images and 12 MRA and
Subacute blood
Light grey
MRV. All of these images are directly correlated with external
Muscle
Light grey
brain anatomy such as gyri and sulci, internal structures in-
Grey matter
Light grey
cluding pathways and nuclei, cranial nerves and adjacent struc-
White matter
Medium grey
tures, or they demonstrate examples of hemorrhages related
Cerebrospinal fluid
Medium grey to black
to the meninges and ventricles or the parenchyma of the brain.
Air, Fat
Very black
Imaging the Brain (CT and MRI): Imaging the brain in
vivo is now commonplace for the patient with neurological
The advantages of CT are 1) it is rapidly done, which is
deficits that may indicate a compromise of the central nervous
especially important in trauma; 2) it clearly shows acute and
system. Even most rural hospitals have, or have easy access to,
subacute hemorrhages into the meningeal spaces and brain;
CT or MRI. With these facts in mind, it is appropriate to make
3) it shows bone (and skull fractures) to advantage; and 4)
a few general comments on these imaging techniques and what
it is less expensive than MRI. The disadvantages of CT are
is routinely seen, or best seen, in each. For details of the meth-
1) it does not clearly show acute or subacute infarcts or is-
ods and techniques of CT and MRI consult sources such as
chemia, or brain edema; 2) it does not clearly differentiate
Grossman (1996), Lee et al. (1999), or Buxton (2002).
white from grey matter within the brain nearly as well as
Computed Tomography (CT): In CT, the patient is
MRI; and 3) it exposes the patient to ionizing radiation.
passed between a source of x-rays and a series of detectors.
Magnetic Resonance Imaging (MRI): The tissues
Tissue density is measured by the effects of x-rays on atoms
of the body contain proportionately large amounts of pro-
within the tissue as these x-rays pass through the tissue.
tons (hydrogen). Protons have a positive nucleus, a shell of
Atoms of higher number have a greater ability to attenuate
negative electrons, and a north and south pole; they func-
(stop) x-rays while those with lower numbers are less able to
tion like tiny spinning bar magnets. Normally, these atoms
attenuate x-rays. The various attenuation intensities are
are arranged randomly in relation to each other due to the
computerized into numbers (Hounsfield units or CT num-
constantly changing magnetic field produced by the elec-
bers). Bone is given the value of +1,000 and is white, while
trons. MRI uses this characteristic of protons to generate
air is given a value of
1,000 and is black. Extravascular
images of the brain and body.
blood, an enhanced tumor, fat, the brain (grey and white
When radio waves are sent in short bursts into the mag-
matter), and cerebrospinal fluid form an intervening contin-
net containing the patient, they are called a radiofrequency
uum from white to black. A CT image of a patient with sub-
pulse (RP). This pulse may vary in strength. When the fre-
arachnoid hemorrhage illustrates the various shades seen in a
quency of the RP matches the frequency of the spinning pro-
CT (Fig. 1-1). In general, the following table summarizes the
ton, the proton will absorb energy from the radio wave (res-
white to black intensities seen for selected tissues in CT.
onance). The effect is two-fold. First, the magnetic effects
of some protons are cancelled out and second, the magnetic
effects and energy levels in others are increased. When the
RP is turned off, the relaxed protons release energy (an
“echo”) that is received by a coil and computed into an im-
age of that part of the body.
The two major types of MRI images (MRI/T1 and
MRI/T2) are related to the effect of RP on protons and the
reactions of these protons (relaxation) when the RP is
turned off. In general, those cancelled out protons return
slowly to their original magnetic strength. The image con-
structed from this time constant is called T1 (Fig. 1-2). On
the other hand, those protons that achieved a higher energy
level
(were not cancelled-out) lose their energy more
rapidly as they return to their original state; the image con-
structed from this time constant is T2 (Fig. 1-3). The cre-
1-1
Computed Tomography (CT) in the axial plane of a patient
ation of a T1-weighted image versus a T2-weighted image is
with subarachnoid hemorrhage. Bone is white, acute blood (white)
based on a variation in the times used to receive the “echo”
outlines the subarachnoid space, brain is grey, and cerebrospinal fluid
in third and lateral ventricles is black.
from the relaxed protons.
4
Introduction and Reader’s Guide
The Brain and Related Structures in MRI
NORMAL
T1
T2
Bone
Very black
Very black
Air
Very black
Very black
Muscle
Dark grey
Dark grey
White matter
Light grey
Dark grey
Grey matter
Dark grey
Light grey
Fat
White
Grey
CSF
Very black
Very white
ABNORMAL
T1
T2
1-2
A sagittal T1 weighted Magnetic Resonance Image (MRI).
Brain is grey and cerebrospinal fluid is black.
Edema
Dark grey
Light grey to white
Tumor
Variable
Variable
Enhanced tumor
White
(Rarely done)
Acute infarct
Dark grey
Light grey to white
The following table summarizes the white to black inten-
Subacute infarct
Dark grey
Light grey to white
sities seen in MRI images that are T1-weighted versus T2-
Acute ischemia
Dark grey
Light grey to white
weighted. It should be emphasized that a number of varia-
Subacute ischemia
Dark grey
Light grey to white
tions on these two general MRI themes are routinely seen in
the clinical environment.
The advantages of MRI are 1) it can be manipulated to vi-
sualize a wide variety of abnormalities or abnormal states
Chapter 2
within the brain; and 2) it can show great detail of the brain
This chapter presents 1) the gross anatomy of the spinal cord
in normal and abnormal states. The disadvantages of MRI
and its principal arteries; 2) the external morphology of the
are 1) it does not show acute or subacute subarachnoid hem-
brain, accompanied by MRIs and drawings of the vascula-
orrhage or hemorrhage into the substance of the brain in any
ture patterns from the same perspective; 3) cranial nerves
detail; 2) it takes a much longer time to do and, therefore,
as seen in specimens and in MRI; and 4) the meninges and
is not useful in acute situations or in some types of trauma;
ventricular spaces. Emphasis is placed on correlating exter-
3) it is, comparatively, much more expensive than CT, and
nal brain and spinal cord anatomy with the respective vas-
4) the scan is extremely loud and may require sedation in
cular patterns and on correlating external brain structures
children.
and cranial nerves as seen in specimens with how the same
The ensuing discussion briefly outlines the salient features
structures appear in MRI. Information concerning the orga-
of individual chapters. In some sections, considerable flexi-
nization of the meninges includes clinical correlations, ex-
bility has been designed into the format; at these points,
amples of extradural, so-called “subdural”, and subarach-
some suggestions are made as to how the atlas can be used.
noid hemorrhages in CT and examples of cisterns in MRI.
In addition, new clinical correlations and examples have
The section showing the structure and relations of the ven-
been included and a new chapter of USMLE-style review
tricular system now includes samples of hemorrhage into
questions has been added.
lateral, third, and fourth ventricles.
Chapter 3
The dissections in Chapter 3 offer views of some of those
brain structures introduced in Chapter 2. Certain structures
and/or structural relationships—for example, the orienta-
tion of the larger association bundles—are particularly
suited to such a presentation. This chapter uses a represen-
tative series of dissected views to provide a broader basis for
learning human neuroanatomy. Because it is not feasible to
illustrate every anatomic feature, the views and structures
selected are those that are usually emphasized in medical
neurobiology courses. These views provide basic informa-
1-3
A sagittal T2 weighted Magnetic Resonance Image (MRI).
Brain is grey, blood vessels frequently appear black, and cerebrospinal
tion necessary to make more detailed dissections, if appro-
fluid is white.
priate, in a particular learning situation.
Introduction and Reader’s Guide
5
Chapter 4
nuclei of cranial nerves. This scheme continues rostrally
into the caudal nuclei of the dorsal thalamus and the poste-
The study of general morphology of the hemisphere and
rior limb of the internal capsule. In addition to the coloring
brainstem is continued in the two sections of Chapter 4. The
of the artwork, each page has a key that specifies the struc-
first section contains a representative series of unstained
ture and function of each colored structure. This approach
coronal slices of brain, each of which is accompanied, on the
emphasizes anatomical-clinical integration.
same page, by MRIs. The brain slice is labeled (by complete
Semidiagrammatic representations of the internal blood
names), and the MRIs are labeled with a corresponding ab-
supply to the spinal cord, medulla, pons, midbrain, and fore-
breviation. The second section contains a series of unstained
brain follow each set of line drawings and stained sections.
brain slices cut in the axial plane, each of which is accompa-
This allows the immediate, and convenient, correlation of
nied, again on the same page, by MRIs. Labeling of the axial
structure with its blood supply as one is studying the inter-
slices is as done for the coronal slices.
nal anatomy of the neuraxis. In addition, tables that summarize
The similarities between the brain slices and the MRIs are
the vascular syndromes of the spinal cord, medulla, pons, midbrain,
remarkable, and this style of presentation closely integrates
and forebrain are located on the pages facing each of these vas-
anatomy in the slice with that as seen in the corresponding
cular drawings. While learning or reviewing the internal
MRI. Because the brain, as sectioned at autopsy or in clini-
blood supply to these parts of the neuraxis, one can also cor-
cal pathologic conferences, is viewed as an unstained speci-
relate the deficits seen when the same vessels are occluded.
men, the preference here is to present the material in a for-
It is essential to successful diagnosis to develop a good un-
mat that will most closely parallel what is seen in these
derstanding of what structure is served by what vessel.
clinical situations.
The diencephalon and basal nuclei section of this chapter
uses ten cross-sections to illustrate internal anatomy. It
should be emphasized that 8 of these 10 sections (those parallel to
Chapter 5
each other) are all from the same brain.
This chapter has been revised with special emphasis on in-
The internal anatomy of the brainstem is commonly
creasing the correlation between anatomical and clinical in-
taught in an anatomical orientation. That is, posterior struc-
formation. This new edition retains the quality and inherent
tures, such as the vestibular nuclei and colliculi, are “up” in
strengths of the line drawings and the stained sections being
the image, while anterior structures, such as the pyramid
located on facing pages in this chapter. However, an innov-
and crus cerebri, are “down” in the image. However, when
ative approach (described below) is introduced that allows
the brainstem is viewed in the clinical setting, as in CT or
the use of these images in their classic Anatomical Orienta-
MRI, this orientation is reversed. In the clinical orientation,
tion and, at the same time, their conversion to the Clinical
posterior structures (4th ventricle, colliculi) are “down” in
Orientation so universally recognized and used in clinical
the image while anterior structures (pyramid, basilar pons,
imaging techniques.
crus cerebri) are “up” in the image.
Chapter 5 consists of six sections covering, in sequence,
Recognizing that many users of this book are pursuing a
the spinal cord, medulla oblongata, cerebellar nuclei, pons,
health care career (as a practitioner or teacher of future clin-
midbrain, and diencephalon and basal nuclei, all with MRI.
icians), it is essential to introduce MRI and CT of the brain-
In this format, the right-hand page contains a complete im-
stem into chapter 5. This accomplishes two important points.
age of the stained section. The left-hand page contains a la-
First, it allows correlation of the size, shape, and configura-
beled line drawing of the stained section, accompanied by a
tion of brainstem sections (line drawings and stained slices)
figure description, and a small orientation drawing. The sec-
with MRI and CT at comparable levels. Second, it offers the
tion part of the line drawing is printed in a 60% screen of
user the opportunity to visualize how nuclei, tracts (and their
black, and the leader lines and labels are printed at 100%
somatotopy) and vascular territories are represented in MRI
black. This gives the illustration a sense of depth and tex-
and CT. Understanding the brain in the Clinical Orientation
ture, reduces competition between lines, and makes the il-
(as seen in MRI or CT) is extremely important in diagnosis.
lustration easy to read at a glance.
To successfully introduce MRI and CT in the brainstem por-
Beginning with the first spinal cord level (coccygeal, Fig-
tion of chapter 5, a continuum from Anatomical Orientation
ure 5-1), the long tracts that are most essential to under-
to Clinical Orientation to MRI needs to be clearly illustrated.
standing how to diagnose the neurologically impaired
This is achieved by 1) placing a small version of the colorized
patient are colored. These tracts are the posterior column-
line drawing on the facing page (page with the stained section)
medial lemniscus system, the lateral corticospinal tract, and
in Anatomical Orientation; 2) showing how this image is
the anterolateral system. In the brainstem, these tracts are
flipped top to bottom into a Clinical Orientation; and 3) fol-
joined by the colorized spinal trigeminal tract, the ventral
lowing this flipped image with (usually) T1 and T2 MRls at
trigeminothalamic tract, and all of the motor and sensory
levels comparable to the accompanying line drawing and
6
Introduction and Reader’s Guide
stained section (Fig. 1-4). This approach retains the anatom-
ical strengths of the spinal cord and brainstem sections of
chapter 5 but allows the introduction of important concepts
regarding how anatomical information is arranged in images
utilized in the clinical environment.
Every effort has been made to use MRI and CT that match,
as closely as possible, the line drawings and stained sections in
the spinal cord and brainstem portions of chapter 5. Recog-
nizing that this match is subject to the vicissitudes of angle and
individual variation, special sets of images were used in chap-
ter 5. The first set consisted of T1- and T2-weighted MRI
1-5
Computed Tomography (CT) of a patient following injection
generated from the same individual; these are identified, re-
of a radiopaque contrast media into the lumbar cistern. In this exam-
spectively, as “MRI, T1-weighted” and “MRI, T2-weighted”
ple, at the medullary level (a cisternogram), neural structures appear
in chapter 5. The second set consisted of CT images from a
grey and the subarachnoid space appears light.
patient who had an injection of the radiopaque contrast me-
dia Isovue-MR 200 (iopamidol injection 41 %) into the lum-
The juxtaposition of MRI to stained section extends into
bar cistern. This contrast media diffused throughout the spinal
the forebrain portion of chapter 5. Many anatomic features
and cranial subarachnoid spaces, outlining the spinal cord and
seen in the forebrain stained sections are easily identified in
brainstem (Fig. 1-5). Images at spinal levels show neural
the adjacent MRI. These particular MRI are not labeled so
structures as grey surrounded by a light subarachnoid space;
as to allow the user to develop and practice his/her inter-
this is a “CT myelogram”. A comparable image at brainstem
pretive skills. The various subsections of chapter 5 can be
levels (grey brain, light CSF) is a “CT cisternogram”. These
used in a variety of ways and will accommodate a wide range
designations are used in chapter 5. While all matches are not
of student and/or instructor preferences.
perfect, not all things in life or medicine are, the vast major-
ity of matches between MRI, CT, and drawings/sections are
Chapter 6
excellent and clearly demonstrate the intended points.
The three-dimensional anatomy of internal structures in the
CNS can also be studied in stained sections that correlate sim-
ilar structures in different planes. The photographs of stained
axial and sagittal sections and of MRIs in Chapter 6 are orga-
nized to provide four important levels of information. First,
Anatomical orientation
Clinical orientation
the general internal anatomy of brain structures can be easily
identified in each photograph. Second, axial photographs are
on left-hand pages and arranged from dorsal to ventral (Fig-
ures 6-1 to 6-9), whereas sagittal photographs are on right-
hand pages and arranged from medial to lateral (Figures 6-2
to 6-10). This setup, in essence, provides complete repre-
sentation of the brain in both planes for use as independent
MRI, T1 weighted image
study sets (axial only, sagittal only) or as integrated/corre-
lated sets (compare facing pages). Third, because axial and
sagittal sections are on facing pages and the plane of section of
each is indicated on its companion by a heavy line, the reader
can easily visualize the positions of internal structures in more
MRI, T2 weighted image
than one plane and develop a clear concept of three-dimen-
sional topography. In other words, one can identify structures
dorsal or ventral to the axial plane by comparing them with
the sagittal, and structures medial or lateral to the sagittal
plane by comparing them with the axial. Such comparisons fa-
cilitate a more full understanding of three-dimensional rela-
CT cisternogram
tionships in the brain. Fourth, the inclusion of MRIs with rep-
resentative axial and sagittal stained sections provides
1-4
An example showing anatomical and clinical orientations of a
excellent examples of the fact that structures seen in the
brainstem level and the corresponding T1 MRI, T2 MRI, and CT cister-
teaching laboratory are easy to recognize in clinical images.
nogram. For additional examples and details see chapter 5, pages 84-133.
Introduction and Reader’s Guide
7
These MRIs are also not labeled so as to allow the user to de-
general format as the preceding figures. Photocopies of
velop his/her interpretive skills.
these blank master drawings can be used by the student for
learning and/or review of any pathway and by the instruc-
tor to teach additional pathways not included in the atlas or
Chapter 7
as a substrate for examination questions. The flexibility of
This chapter provides summaries of a variety of clinically
information as presented in Chapter 7 extends equally to
relevant CNS tracts and/or pathways and has four features
student and instructor.
that enhance student understanding. First, the inclusion of
pathway information in atlas format broadens the basis one
Chapter 8
can use to teach functional neurobiology. This is especially
the case when pathways are presented in a style that en-
This chapter contains a series of angiograms (arterial and
hances the development of diagnostic skills. Second, each
venous phases), magnetic resonance angiography (MRA)
drawing illustrates, in line color, a given pathway com-
images, and magnetic resonance venography (MRV) im-
pletely, showing its 1) origins, longitudinal extent, course
ages. The angiograms are shown in lateral and anterior-
throughout the neuraxis and termination; 2) laterality—an
posterior projections—some as standard views with corre-
all-important issue in diagnosis; 3) point of decussation, if
sponding digital subtraction images. MRA and MRV tech-
applicable; 4) position in representative cross sections of the
nology are noninvasive methods that allow for the visualiza-
brainstem and spinal cord; and 5) the somatotopic organi-
tion of arteries (MRA) and veins and venous sinuses (MRV).
zation of fibers within the pathway, if applicable. The blood
There are, however, many situations when both arteries and
supply to each pathway is reviewed on the facing page.
veins are seen with either method. Use of MRA and MRV
Third, a brief summary mentions the main neuroactive sub-
is commonplace, and this technology is an important diag-
stances associated with cells and fibers composing particular
nostic tool. A number of new vascular images have been in-
segments of the pathway under consideration. The action of
cluded in this revised version of Chapter 8.
the substance, if widely agreed on, is indicated as excitatory
(
) or inhibitory (
). This allows the reader to closely correlate
Chapter 9
a particular neurotransmitter with a specific population of projec-
tion neurons and their terminals. The limitations of this ap-
A primary goal in the study of functional human neurobi-
proach, within the confines of an atlas, are self-evident. The
ology is to become a competent health care professional.
transmitters associated with some pathways are not well
Another, and equally significant, goal is to pass examina-
known; consequently, such information is not provided for
tions. These may be course examinations, the National
some connections. Also, no attempt is made to identify sub-
Board Subject Exam (some courses require these), or stan-
stances that may be colocalized, to discuss their synthesis or
dardized tests, such as the USMLE Step 1 and Step 2, given
degradation, or to mention all neurotransmitters associated
at key intervals and taken by all students.
with a particular cell group. The goal here is to introduce
The questions comprising chapter 9 were generated in
the reader to selected neurotransmitters and to integrate and
the recognition that examinations are an essential part of the
correlate this information with a particular pathway, circuit,
educational process. Whenever possible, and practical,
or connection. Fourth, the clinical correlations that accompany
these questions are in the USMLE Step 1 style (single best
each pathway drawing provide examples of deficits resulting from
answer). These questions emphasize 1) anatomical and clin-
lesions, at various levels in the neuraxis, of the fibers composing that
ical concepts and correlations; 2) the application of basic hu-
specific pathway. Also, examples are given of syndromes or
man neurobiology to medical practice; and 3) how neuro-
diseases in which these deficits are seen. The ways in which
logical deficits and diseases relate to damage in specific parts
these clinical correlations can be used to enrich the learning
of the nervous system. In general, the questions are grouped
process are described in Figure 7-3 on page 176.
by chapter. However, in some instances, questions draw on
The drawings in this section were designed to provide the
information provided in more than one chapter. This is
maximum amount of information, to keep the extraneous
sometimes essential in an effort to make appropriate
points to a minimum, and to do it all in a single, easy-to-fol-
structural/functional/clinical correlations. At the end of
low illustration. A complete range of relevant information
each group of questions the correct answers are provided
is contained in each drawing and in its description as ex-
and explained. Included with the explanation is a reference
plained in the second point above.
to the page (or pages) containing the answer, be that answer
Because it is not possible to anticipate all pathways that
in the text or in a figure. Although not exhaustive, this list
may be taught in a wide range of neurobiology courses, flex-
of questions should provide the user of this atlas with an ex-
ibility has been designed into Chapter 7. The last figure in
cellent opportunity for self-assessment covering a broad
each section is a blank master drawing that follows the same
range of clinically relevant topics.
8
Introduction and Reader’s Guide
Rationale for Labels and Abbreviations
o universally accepted way to identify specific features
Chapters 2 and 4). This uses the complete word(s) on the
N
or structures in drawings or photographs exists. The
larger image of a brain structure while using the shorter ab-
variety of methods seen in currently available atlases reflects
breviation on the smaller image of the MRI.
the personal preferences of the authors. Such is the case in
The abbreviations used in this atlas do not clutter the il-
the present endeavor. The goal of this atlas is to present ba-
lustration; they permit labeling of all relevant structures and
sic functional and clinical neuroanatomy in an understand-
are adequately informative while stimulating the think-
able and useful format.
ing-learning process. The abbreviations are, in a very real
Among currently available atlases, most figures are la-
sense, mnemonics. When learning gyri and sulci of the oc-
beled with either the complete names of structures or with
cipital lobe, for example, one realizes that the abbreviation
numbers or letters that are keyed to a list of the complete
“LinGy” in the atlas could only mean “lingual gyrus.” It could
names. The first method immediately imparts the greatest
not be confused with other structures in other parts of the
amount of information; the second method is the most suc-
nervous system. Regarding the pathways, “RuSp” could
cinct. When using the complete names of structures, one
mean only “rubrospinal tract” and “LenFas,” the “lenticular
must exercise care to not compromise the quality or size of
fasciculus.” As the reader learns more and more terminol-
the illustration, the number of structures labeled, or the size
ogy from lectures and readings, he or she will be able to use
of labels used. Although the use of single letters or numbers
these abbreviations with minimal reference to the accompa-
results in minimal clutter on the figure, a major drawback is
nying list. In addition, a subtle advantage of this method of
the fact that the same number or letter may appear on sev-
labeling is that, as the reader looks at the abbreviation and
eral different figures and designate different structures in all
momentarily pauses to ponder its meaning, he or she may
cases. Consequently, no consistency occurs between num-
form a mental image of the structure and the complete
bers and letters and their corresponding meanings as the
word. Because neuroanatomy requires one to conceptualize
reader examines different figures. This atlas uses a combi-
and form mental images to more clearly understand CNS
nation of complete words and abbreviations that are clearly
relationships, this method seems especially useful.
recognized versions of the complete word.
References:
In response to suggestions made by those using this book
Bruxton, RB. Introduction to Functional Magnetic Resonance
over the years, the number of abbreviations in the sixth edi-
Imaging, Principles and Techniques. Cambridge: Cambridge
tion has been reduced, and the number of labels using the
University Press, 2002.
complete name has been increased. Simultaneously, com-
Grossman, CB. Magnetic Resonance Imaging and Computed To-
plete names and abbreviations have been used together in
mography of the Head and Spine.
2nd Ed. Baltimore:
some chapters to the full advantage of each method. For ex-
Williams & Wilkins, 1996.
ample, structures are labeled on a brain slice by the com-
Lee, SH, Roa, KCVG, and Zimmerman, RA. Cranial MRI
plete name, but the same structure in the accompanying
and CT. 4th Ed. New York: McGraw-Hill Health Profes-
MRI is labeled with a corresponding abbreviation (see
sions Division, 1999.
CHAPTER
2
External Morphology
of the
Central Nervous System
10
External Morphology of the Central Nervous System
Posterior View
C2 Posterior root (PR)
Dura
Posterior spinal
artery
Arachnoid
C3 PR
Denticulate
ligament
C4 PR
Posterior spinal
medullary artery
C5 PR
Anterior View
C2 Anterior root (AR)
Dura
Denticulate
ligament
C3 AR
Arachnoid
Anterior spinal
medullary artery
C4 AR
Anterior spinal
artery
C5 AR
2-1
Posterior (upper) and anterior (lower) views showing the gen-
Figure 2-3 on facing page) follow their respective roots. The posterior
eral features of the spinal cord as seen at levels C2-C5. The dura and
spinal artery is found medial to the entering posterior rootlets (and the
arachnoid are reflected, and the pia is intimately adherent to the spinal
dorsolateral sulcus), while the anterior spinal artery is in the anterior
cord and rootlets. Posterior and anterior spinal medullary arteries (see
median fissure (see also Figure 2-2, facing page).
The Spinal Cord
11
Posterior View
Sulci:
Posterior median
Posterior intermediate
Posterolateral
C7 Posterior root
Spinal (posterior root) ganglion
Fasciculus gracilis
Fasciculus cuneatus
Anterior View
Anterior spinal artery
C7 Anterior root
Anterior radicular
artery
Anterior funiculus
Anterior median fissure
2-2
Posterior (upper) and anterior (lower) views showing details
of the spinal cord as seen in the C7 segment. The posterior (dorsal) root
ganglion is partially covered by dura and connective tissue.
Posterior spinal arteries
Arterial
vasocorona
Basilar artery
Posterior inferior
cerebellar arteries
Vertebral arteries
Anterior spinal artery
Posterior spinal
medullary artery
Posterior radicular
artery (on dorsal root)
Sulcal arteries
Anterior spinal
medullary artery
Anterior radicular artery
(on ventral root)
Segmental artery
2-3
Semidiagrammatic representation showing the origin and gen-
medullary arteries) arise at intermittent levels and serve to augment
eral location of principal arteries supplying the spinal cord. The ante-
the blood supply to the spinal cord. The artery of Adamkiewicz is an
rior and posterior radicular arteries arise at every spinal level and serve
unusually large spinal medullary artery arising usually on the left in low
their respective roots and ganglion. The anterior and posterior spinal
thoracic or upper lumbar levels (T9-L1). The arterial vasocorona is a
medullary arteries (also called medullary feeder arteries or segmental
diffuse anastomotic plexus covering the cord surface.
12
External Morphology of the Central Nervous System
A
B
C
Thoracic
cord
T9
Dura and
arachnoid
LuSaCd
Lumbar and
sacral cord
L1
(LuSaCd)
SaCoCd
Sacral and
coccygeal
Lumbar
cord
cistern
(SaCoCd)
FTInt
Conus
medullaris
CaEq
Filum
terminale
internum
(FTInt)
L5
Cauda
equina
(CaEq)
S1
Posterior
root
ganglion
Dura and
arachnoid
2-4
Overall posterior (A,B) and sagittal MRI (C, T2-weighted)
end of the spinal cord. This space contains the anterior and posterior
views of the lower thoracic, lumbar, sacral, and coccygeal spinal cord
roots from the lower part of the spinal cord that collectively form the
segments and the cauda equina. The dura and arachnoid are retracted
cauda equina. The filum terminale internum also descends from the
in A and B. The cauda equina is shown in situ in A, and in B the nerve
conus medullaris through the lumbar cistern to attach to the inner sur-
roots of the cauda equina have been spread laterally to expose the conus
face of the dural sac. The dural sac ends at about the level of the S2 ver-
medullaris and filum terminale internum. This latter structure is also
tebra and is attached to the coccyx by the filum terminale externum
called the pial part of the filum terminale. See Figures 5-1 and 5-2 on
(also see Fig. 2-47 on page 47). A lumbar puncture is made by insert-
pages 84-87 for cross-sectional views of the cauda equina.
ing a large gauge needle (18-22 gauge) between the L3 and L4 verte-
In the sagittal MRI (C), the lower portions of the cord, the filum
bra or L4 and L5 vertebra and retrieving a sample of cerebrospinal fluid
terminale internum, and cauda equina are clearly seen. In addition, the
from the lumbar cistern. This sample may be used for a number of di-
intervertebral discs and the bodies of the vertebrae are clear. The lum-
agnostic procedures.
bar cistern is an enlarged part of the subarachnoid space caudal to the
The Brain: Lobes
13
Central sulcus
Precentral sulcus
Postcentral sulcus
Lobes
Parietooccipital
sulcus
Frontal
A
Parietal
Temporal
Occipital
Preoccipital
Lateral sulcus
notch
Limbic
Insular
Central sulcus
Paracentral sulcus
Marginal sulcus (marginal
ramus of the cingulate
Cingulate sulcus
sulcus)
Corpus callosum
Parietooccipital
sulcus
Fornix
B
Diencephalon
Calcarine
sulcus
Preoccipital notch
Collateral sulcus
2-5
Lateral (A) and medial (B) views of the cerebral hemisphere
of the cortex is made up of long and short gyri that are separated from
showing the landmarks used to divide the cortex into its main lobes.
each other by the central sulcus of the insula. The insula, as a whole, is
On the lateral aspect, the central sulcus (of Rolando) separates
separated from the adjacent portions of the frontal, parietal, and tem-
frontal and parietal lobes. The lateral sulcus (of Sylvius) forms the bor-
poral opercula by the circular sulcus.
der between frontal and temporal lobes. The occipital lobe is located
On the medial aspect, the cingulate sulcus separates medial portions
caudal to an arbitrary line drawn between the terminus of the parieto-
of frontal and parietal lobes from the limbic lobe. An imaginary con-
occipital sulcus and the preoccipital notch. A horizontal line drawn
tinuation of the central sulcus intersects with the cingulate sulcus and
from approximately the upper two-thirds of the lateral fissure to the
forms the border between frontal and parietal lobes. The parieto-
rostral edge of the occipital lobe represents the border between pari-
occipital sulcus and an arbitrary continuation of this line to the preoc-
etal and temporal lobes. The insular cortex (see also Figs. 2-46 on page
cipital notch separate the parietal, limbic, and temporal lobes from the
45 and 3-1 on page 56) is located internal to the lateral sulcus. This part
occipital lobe.
14
External Morphology of the Central Nervous System
Postcentral gyrus
3,1,2
Precentral gyrus
5
Pars opercularis
Surpamarginal gyrus
7
6
4
8
40
Angular gyrus
39
19
Pars triangularis
22
18
44
45
17
41
A
42
Pars orbitalis
47
3,1,2
Anterior paracentral gyrus
5
Posterior paracentral gyrus
8
6
4
7
Cuneus
19
18
Calcarine sulcus
17
B
18
19
Lingual gyrus
2-6
Lateral (A) and medial (B) views of the cerebral hemisphere
triangularis, and a pars orbitalis. A lesion that is located primarily in ar-
showing the more commonly described Brodmann areas. In general,
eas 44 and 45 (shaded) will give rise to what is called a Broca aphasia,
area 4 comprises the primary somatomotor cortex, areas 3,1, and 2 the
also called expressive or nonfluent aphasia.
primary somatosensory cortex, and area 17 the primary visual cortex.
The inferior parietal lobule consists of supramarginal (area 40) and
Area 41 is the primary auditory cortex, and the portion of area 6 in the
angular (area 39) gyri. Lesions in this general area of the cortex
caudal part of the middle frontal gyrus is generally recognized as the
(shaded), and sometimes extending into area 22, will give rise to what
frontal eye field.
is known as Wernicke aphasia, also sometimes called receptive or flu-
The inferior frontal gyrus has three portions: a pars opercularis, pars
ent aphasia.
The Brain: Lobes
15
Precentral gyrus (primary somatomotor cortex)
Posrcentral gyrus (primary somatosens ry cortex)
A
Anterior paracentral gyrus (somatomotor)
Posterior paracentral gyrus (somatosensory)
B
Left
inferior
visual
quadrant
2-7
Lateral (A) and medial (B) views of the cerebral hemisphere
ing the hand and upper extremity areas, and the medial third repre-
showing the somatotopic organization of the primary somatomotor
senting the trunk and the hip. Lesions of the somatomotor cortex re-
and somatosensory cortices. The lower extremity and foot areas are lo-
sult in motor deficits on the contralateral side of the body while lesions
cated on medial aspects of the hemisphere in the anterior paracentral
in the somatosensory cortex result in a loss of sensory perception from
(motor) and the posterior paracentral (sensory) gyri. The remaining
the contralateral side of the body.
portions of the body extend from the margin of the hemisphere over
The medial surface of the right hemisphere (B) illustrates the posi-
the convexity to the lateral sulcus in the precentral and postcentral
tion of the left portions of the visual field. The inferior visual quadrant
gyri.
is located in the primary visual cortex above the calcarine sulcus while
In general, the precentral gyrus can be divided into three regions:
the superior visual quadrant is found in the cortex below the calcarine
the lateral third representing the face area, the middle third represent- sulcus.
16
External Morphology of the Central Nervous System
Longitudinal fissure
Superior frontal
gyrus (SFGy)
Middle frontal
gyrus (MFGy)
Superior frontal
sulcus (SFSul)
Precentral
sulcus (PrCSul)
Precentral
gyrus (PrCGy)
Precentral
Central
gyrus (PrCGy)
sulcus (CSul)
Central
Postcentral
sulcus (CSul)
gyrus (PoCGy)
Supramarginal
Postcentral sulcus
gyrus
Superior parietal lobule
Occipital gyri
Anterior
cerebral
arteries
MFGy
SFGy
ACA
territory
SFSul
PrCSul
PrCGy
CSul
Falx
PoCGy
cerebri
2-8
Dorsal view of the cerebral hemispheres showing the main gyri
Note the area of infarction representing the territory of the anterior
and sulci and an MRI (inverted inversion recovery—lower left) and a
cerebral artery (ACA).
CT (lower right) identifying structures from the same perspective.
The Brain: Gross Views, Vasculature, and MRI
17
Frontopolar
branches of ACA
Branches of MCA (M4)
Orbitofrontal
Prerolandic
Rolandic
Callosomarginal branches
Parietal and
(from ACA)
temporal
Paracentral branches
(from ACA)
Internal parietal branches
(from ACA)
Branches of PCA
Parieto-occipital sulcus
Temporal (P3)
Parieto-occipital (P4)
Calcarine (P4)
2-9
Dorsal view of the cerebral hemispheres showing the location and posterior (PCA) cerebral arteries. Gyri and sulci can be identified
and general branching patterns of the anterior (ACA), middle (MCA), by a comparison with Figure 2-8 (facing page).
Superior cerebral veins
Superior
sagittal sinus
Rolandic vein
To superficial middle
cerebral vein and
inferior anastomotic
vein
Greater anastomotic
vein (Trolard)
Superior cerebral veins
To sinus confluens
2-10
Dorsal view of the cerebral hemispheres showing the location
Figure 2-8 (facing page). See Figures 8-4 and 8-5 (pp. 243-244) for
of the superior sagittal sinus and the locations and general branching
comparable angiograms (venous phase) of the superior sagittal sinus.
patterns of veins. Gyri and sulci can be identified by a comparison with
18
External Morphology of the Central Nervous System
Precentral gyrus (PrCGy)
Central sulcus (CSul)
Precentral sulcus (PrCSul)
Postcentral gyrus (PoCGy)
Superior frontal gyrus
Postcentral sulcus (PoCSul)
Superior parietal lobule
Superior frontal sulcus
Supramarginal gyrus
Middle frontal gyrus (MFGy)
Interparietal sulcus
Inferior frontal sulcus (IFSul)
Angular gyrus
Inferior frontal gyrus:
Pars opercularis (PoP)
Pars triangularis (PTr)
Pars orbitalis (POrb)
Occipital
gyri (OGy)
Lateral sulcus (LatSul)
Superior temporal gyrus (STGy)
Superior temporal sulcus (STSul)
Middle temporal gyrus (MTGy)
Preoccipital notch
PrCGy
PrCSul
CSul
PoCSul
MFGy
PoCGy
LatSul
IFSul
OGy
PoP
MTGy
PTr
POrb
STGy
STSul
2-11
Lateral view of the left cerebral hemisphere showing the
principal gyri and sulci and an MRI (inversion recovery) identifying
many of these structures from the same perspective.
The Brain: Gross Views, Vasculature, and MRI
19
Central sulcus
Anterior and posterior
parietal branches of MCA
Rolandic branches
of MCA
Angular branches
Prerolandic branches
of MCA
of MCA
Posterior temporal
branches of MCA
Orbitofrontal branches
of MCA
Orbital branches
of anterior cerebral artery
Middle temporal
branches of MCA
Middle cerebral artery (MCA)
in lateral sulcus
Anterior temporal
branches of MCA
2-12
Lateral view of the right cerebral hemisphere showing the
sphere represent the M4 segment. Terminal branches of the posterior
branching pattern of the middle cerebral artery. Gyri and sulci can be
and anterior cerebral arteries course over the edges of the temporal and
identified by comparison with Figure 2-11 (facing page). The middle
occipital lobes, and parietal and frontal lobes, respectively (see Figure
cerebral artery initially branches in the depths of the lateral sulcus (as
2-9 on page 17). See Figure 8-1 (p. 240) for a comparable angiogram
M2 and M3 segments); these branches seen on the surface of the hemi-
of the middle and anterior cerebral arteries.
Rolandic vein
Greater anastomotic vein (Trolard)
Superior sagittal sinus
Superior cerebral veins
Superior cerebral
veins
Inferior anastomotic
vein (Labbé)
Straight sinus
Superficial middle
Sinus confluens
cerebral vein
Transverse sinus (TS)
To sphenoparietal sinus
To cavernous sinus
Inferior cerebral veins
To sphenoparietal sinus
Inferior cerebral veins
Occipital sinus
to sphenoparietal sinus
Temporal cerebral veins
TS
To sigmoid sinus
2-13
Lateral view of the right cerebral hemisphere showing the lo-
sinuses are also indicated. See Figures 8-2 (p. 241) and 8-11 (p. 250)
cations of sinuses and the locations and general branching patterns of
for comparable angiogram and MRV of the sinuses and superficial
veins. Gyri and sulci can be identified by comparison with Figure 2-11
veins.
(facing page). Communications between veins and sinuses or between
20
External Morphology of the Central Nervous System
Frontal pole
Olfactory bulb
Olfactory sulcus (OlfSul)
Gyrus rectus (GyRec)
Orbital gyri (OrbGy)
Olfactory tract
Temporal pole
Optic nerve
Infundibulum
Optic chiasm
Uncus (Un)
Optic tract (OpTr)
Mammillary
Interpeduncular
body (MB)
fossa (IPF)
Parahippocampal
Inferior temporal
gyrus
gyrus
Collateral sulcus
Crus cerebri (CC)
Substantia nigra
Occipitotemporal
Cerebral
gyri
aqueduct (CA)
Colliculi (Col)
Lingual gyrus
Occipital gyri
Occipital pole
GyRec
Anterior
OrbGy
cerebral
artery
OlfSul
Middle
OpTr
cerebral
OpTr
artery
Hypothalamus
MB
Un
Un
IPF
CC
IPF
Col
Temporal lobe
CA
Col
Cerebellum
2-14
Ventral view of the cerebral hemispheres and diencephalon sion recovery—lower left; T2-weighted—lower right) showing many
with the brainstem caudal to midbrain removed and two MRIs (inver- structures from the same perspective.
The Brain: Gross Views, Vasculature, and MRI
21
Orbitofrontal branches
of MCA
Orbital branches of ACA
Middle cerebral
artery (MCA)
Anterior cerebral
MCA in lateral sulcus
artery (ACA)
Lenticulostriate
branches of MCA
Anterior temporal
Posterior cerebral
branch of PCA
artery (PCA)
(P3 segment)
Posterior temporal
branch of PCA
(P3 segment)
Parieto-occipital
branch of PCA
(P4 segment)
Calcarine branch of PCA
(P4 segment)
2-15
Ventral view of the cerebral hemisphere with the brainstem
on page 25. Shown here are P3 (origin of temporal arteries) and P4 (ori-
removed, which shows the branching pattern of the posterior cerebral
gin of calcarine and parietooccipital arteries) segments. Gyri and sulci
artery (PCA) and some branches of the anterior and middle cerebral
can be identified by comparison with Figure 2-14 (facing page).
arteries. The P1 and P2 segments of the PCA are shown on Figure 2-21
Sphenoparietal sinus
Intercavernous sinuses
Anterior
Cavernous sinus
Posterior
Superior petrosal sinus
Pineal
Inferior petrosal sinus
Great cerebral vein
Sigmoid sinus
Straight sinus
Internal jugular vein
—inferior sagittal sinus
—superior cerebellar veins
TS
Transverse sinus (TS)
Sinus confluens
2-16
Ventral view of the cerebral hemisphere, with brainstem re-
are the main tributaries of that sinus. See Figures 8-5 (p. 245), 8-9
moved, showing the locations and relationships of the main sinuses.
(p. 248), and 8-11 (p. 250) for comparable MRV of the transverse
Gyri and sulci can be identified by comparison with Figure 2-14 (facing
sinus.
page). The listings preceded by an en-dash (-) under principal sinuses
22
External Morphology of the Central Nervous System
Frontal pole
Olfactory bulb
Longitundinal fissure
Olfactory sulcus (OlfSul)
Orbital sulci
Orbital gyri (OrbGy)
Gyrus rectus (GyRec)
Olfactory tract
Temporal pole (TPole)
Basilar pons (BP)
Uncus
Occipitotemporal
sulcus
Parahippocampal
gyrus
Occipitotemporal
gyri
Collateral
sulcus
Glossopharyngeal
Middle cerebellar
nerve
peduncle (MCP)
Flocculus
Facial nerve
Vestibulocochlear
Vagus nerve
nerve
Abducens nerve
Medulla
Olive (inferior);
Decussation
olivary eminence
of pyramids
Cerebellum (Cbl)
OlfSul
GyRec
OrbGy
Tpole
Trigeminal
nerve
BP
BP
MCP
Fourth
ventricle
Cbl
2-17
Ventral view of the cerebral hemispheres, diencephalon,
brainstem, and cerebellum and two MRIs (both T1-weighted images)
that shows structures from the same perspective. A detailed view of the
ventral aspect of the brainstem is seen in Figure 2-20 on page 24.
The Brain: Gross Views, Vasculature, and MRI
23
Anterior cerebral artery
Internal carotid artery
Middle cerebral artery (MCA)
Optic nerve,
chiasm, and tract
Posterior communicating artery
Lenticulostriate
Oculomotor nerve
branches of MCA
Superior cerebellar artery
Posterior cerebral artery
Trochlear nerve
Basilar artery
Trigeminal nerve
Abducens nerve
Facial and
AICA
vestibulocochlear nerves
Anterior inferior
Branches of AICA
cerebellar artery (AICA)
Posterior inferior
PICA
cerebellar artery (PICA)
PSA
Vertebral artery
Branches of PICA
Posterior spinal artery (PSA)
Anterior spinal artery
2-18
Ventral view of the cerebral hemispheres, diencephalon,
son with Figure 2-17 (facing page). Details of the cerebral arterial cir-
brainstem, and cerebellum, which shows the arterial patterns created
cle and the vertebrobasilar arterial pattern are shown in Figure 2-21 on
by the internal carotid and vertebrobasilar systems. Note the cerebral
page 25. See Figure 8-9 and 8-10 (pp. 248-249) for comparable MRA
arterial circle (of Willis). Gyri and sulci can be identified by compari-
of the cerebral arterial circle and its major branches.
Superior ophthalmic vein
-from area of ophthalmic artery
Sphenoparietal sinus
-middle cerebral vein
Cavernous sinus
-cerebral vein
Superior petrosal sinus
Intercaverous sinuses
-cerebellar veins
-inferior cerebral veins
-tympanic veins
Inferior petrosal sinus
-veins of pons and medulla
Basilar plexus
-auditory veins
Sigmoid sinus
Internal jugular vein
Transverse sinus
Anterior vertebral
-emissary veins
venous plexus
-inferior cerebral veins
-inferior cerebellar veins
Occipital sinus
Sinus confluens
-posterior internal vertebral
-straight sinus
venous plexus
-superior sagittal sinus
2-19
Ventral view of the cerebral hemispheres, diencephalon, principal sinuses and veins. The listings preceded by a dash (-) under
brainstem, and cerebellum showing the locations and relationships of principal sinuses are the main tributaries of that sinus.
24
External Morphology of the Central Nervous System
Gyrus rectus
Olfactory tract
Optic nerve
Infundibulum
(cranial nerve II)
Optic chiasm
Mammillary body
Optic tract
Interpeduncular
fossa
Oculomotor nerve
(cranial nerve III)
Basilar pons
Crus cerebri
Trochlear nerve
(cranial nerve IV)
Trigeminal nerve
Parahippocampal
(cranial nerve V)
gyrus
Abducens nerve
(cranial nerve VI)
Middle cerebellar peduncle
(brachium pontis)
Facial nerve
(cranial nerve VII)
Intermediate nerve
Vestibulocochlear nerve
Flocculus
(cranial nerve VIII)
Olive (inferior);
olivary eminence
Glossopharyngeal nerve
(cranial nerve IX)
Retroolivary sulcus
Vagus nerve
(postolivary sulcus)
(cranial nerve X)
Choroid plexus
Hypoglossal nerve
(cranial nerve XII)
Preolivary sulcus
(exit of XIIth Nr.)
Accessory nerve
Pyramid
(cranial nerve XI)
Anterior median fissure
Brs of posterior inferior
cerebellar artery
Decussation
of pyramids
2-20
Detailed ventral view of the diencephalon and brainstem
the hypoglossal nerve on that side; the general position of the (spinal)
with particular emphasis on cranial nerves and related structures. The
accessory nerve is shown on the right by the dark line.
dots on the left side represent the approximate position of the roots of
The Brain: Gross Views, Vasculature, and MRI
25
Vessels
Structures
Medial striate artery
Anterior communicating artery
A2
Olfactory tract
Anterior cerebral artery
A1
Optic chiasm
Optic nerve
Posterior communicating artery
Anterior perforated substance
Ophthalamic artery
Anterior and polar
Internal carotid artery
temporal arteries
Uncal artery
Middle cerebral
artery
M1
M2
Optic tract
Mammillary body
Infundibulum
Lenticulostriate arteries
Crus cerebri
Anterior choroidal artery
P1
Posterior cerebral artery
Oculomotor nerve (III)
P2
Posterior choroidal arteries
Trochlear nerve (IV)
Quadrigeminal artery
Basilar pons
Superior cerebellar artery
Trigeminal nerve (V)
Abducens nerve (VI)
Pontine arteries
Facial nerve (VII)
Middle cerebellar
peduncle
Basilar artery
Vestibulocochlear
nerve (VIII)
Choroid plexus
Anterior inferior
Glossopharyngeal nerve (IX)
cerebellar artery
Vagus nerve (X)
Labyrinthine artery
Accessory nerve (XI)
Posterior inferior
cerebellar artery
Hypoglossal nerve (XII)
Olive (inferior);
Posterior spinal artery
olivary eminence
Vertebral artery
Cerebellum
Pyramid
Anterior spinal artery
2-21
Ventral view of the brainstem showing the relationship of
p. 242 for details). Lateral to the internal carotid bifurcation is the
brain structures and cranial nerves to the arteries forming the verte-
M1 segment of the middle cerebral artery (MCA), which divides and
brobasilar system and the cerebral arterial circle (of Willis). The pos-
continues as the M2 segments (branches) on the insular cortex. The
terior spinal artery usually originates from the posterior inferior
M3 branches of the MCA are those located on the inner surface of the
cerebellar artery (left), but it may arise from the vertebral (right).
opercula, and the M4 branches are located on the lateral aspect of the
Although the labyrinthine artery may occasionally branch from the
hemisphere. Between the basilar bifurcation and the posterior com-
basilar (right), it most frequently originates from the anterior infe-
municating artery is the P1 segment of the posterior cerebral artery;
rior cerebellar artery (left). Many vessels that arise ventrally course
P2 is between the posterior communicator and the first temporal
around the brainstem to serve dorsal structures. The anterior cere-
branches. See Figure 8-9, 8-10, and 8-12 (pp. 248, 249, 251) for
bral artery consists of A1 (between the internal carotid bifurcation
comparable MRA of the cerebral arterial circle and vertebrobasilar
and the anterior communicating artery) and segments A2-A5 which
system. See Figure 8-12 on p. 251 for blood supply of the choroid
are distal to the anterior communicating artery (see Figure 8-3 on
plexus.
26
External Morphology of the Central Nervous System
Lateral geniculate
body
Crus cerebri
Trochlear nerve
Optic tract
Optic chiasm
Middle cerebellar
Optic nerve
peduncle
Infundibulum
Trigeminal nerve
Vestibulocochlear
motor root
nerve
Trigeminal nerve
Facial nerve
sensory root
Olive (inferior),
Basilar pons
olivary eminence
Abducens nerve
Retroolivary sulcus
(postolivary sulcus)
Pyramid
Preolivary sulcus
2-22
Lateral view of the left side of the brainstem emphasizing
brainstem. Compare with Figure 2-24 on the facing page. The cere-
structures and cranial nerves on the ventral aspect of the thalamus and bellum and portions of the temporal lobe have been removed.
Anterior cerebral
artery
Olfactory tract
Medial olfactory stria
Lateral olfactory stria
Optic nerve
Optic chiasm
Anterior perforated
substance
Infundibulum
Optic tract
Mammillary body
Posterior perforated
substance
Crus cerebri
Trochlear nerve
Basilar pons
Lateral geniculate body
Trigeminal nerve
Abducens nerve
Medial geniculate body
Facial nerve
Middle cerebellar peduncle
Vestibulocochlear nerve
Pyramid
2-23
View of the ventral aspect of the diencephalon and part of the Note structures of the hypothalamus, cranial nerves, and optic struc-
brainstem with the medial portions of the temporal lobe removed. tures, including the lateral geniculate body.
The Brain: Gross Views, Vasculature, and MRI
27
Fornix
Choroid plexus, third ventricle
Optic tract
Posterior choroidal arteries
Thalamogeniculate artery
Lateral geniculate body
Dorsal thalamus
Posterior cerebral artery
Mammillary body
Medial geniculate body
Quadrigeminal artery
Superior colliculus
Posterior communicating
Crus cerebri
artery
Internal carotid artery
Brachium of inferior colliculus
Inferior colliculus
Oculomotor nerve
Superior cerebellar artery
Trochlear nerve
Trigeminal nerve
Motor root
Sensory root
Superior cerebellar peduncle
Anterior medullary velum
Basilar artery
Middle cerebellar peduncle
Anterior inferior
cerebellar artery
Vestibulocochlear nerve
Facial nerve
Labyrinthine artery
Posterior inferior
Abducens nerve
cerebellar artery
Glossopharyngeal nerve
Choroid plexus,
Vagus nerve
fourth ventricle
Hypoglossal nerve
Restiform body
Accessory nerve
Cuneate tubercle
Gracile tubercle
Posterior inferior cerebellar artery
Posterior spinal artery
Anterior spinal artery
Vertebral artery
2-24
Lateral view of the brainstem and thalamus showing the rela-
tively, are shown as dashed lines. Compare with Figure 2-22 on the fac-
tionship of structures and cranial nerves to arteries. Arteries that serve
ing page. See Figure 8-7 (p. 246) for comparable angiogram of the ver-
dorsal structures originate from ventrally located parent vessels. The
tebrobasilar system. See Figure 8-12 on p. 251 for blood supply of the
approximate positions of the posterior spinal and labyrinthine arteries,
choroid plexus.
when they originate from the vertebral and basilar arteries, respec-
Anterior cerebral artery
Anterior
A2
communicating
A1
artery
Middle cerebral artery (M1)
Hypothalamus
Posterior communicating
Crus cerebri
artery
Red nucleus
Posterior cerebral artery
P2
P1
Cerebral aqueduct
Cortical branches of
posterior cerebral artery
2-25
A proton density MRI through basal regions of the hemi-
2-21 on page 25. See Figure 8-9 and 8-10 (pp. 248-249) for compa-
sphere and through the midbrain showing several major vessels that
rable MRA of the cerebral arterial circle.
form part of the cerebral arterial circle (of Willis). Compare to Figure
28
External Morphology of the Central Nervous System
Anterior paracentral gyrus (APGy)
Central sulcus (CSul)
Paracentral sulcus (ParCSul)
Posterior paracentral gyrus (PPGy)
Precentral sulcus (PrCSul)
Marginal sulcus (MarSul)
Precuneus (PrCun)
Cingulate gyrus (CinGy)
Superior frontal
gyrus (SFGy)
Parieto-occipital
sulcus (POSul)
Cingulate sulcus
Cuneus (Cun)
(CinSul)
Calcarine sulcus
(CalSul)
Lingual gyrus
(LinGy)
Sulcus of corpus
callosum (SulCC)
Isthmus of cingulate gyrus
Paraterminal gyri
Occipitotemporal gyri
Parolfactory gyri (ParolfGy)
Parahippocampal gyrus
Uncus
Temporal pole
Rhinal sulcus
APGy
PrCSul
CSul
PPGy
ParCSul
MarSul
SulCC
CinGy
PrCun
CinSul
POSul
ParolfGy
Cun
CalSul
LinGy
MarSul
SFGy
Corpus callosum
POSul
CalSul
Colloid cyst
Internal cerebral
vein
2-26
Midsagittal view of the right cerebral hemisphere and dien-
A colloid cyst (colloid tumor) is a congenital growth usually dis-
cephalon, with brainstem removed, showing the main gyri and sulci
covered in adult life once the flow of CSF through the interventricular
and two MRI (both T1-weighted images) showing these structures
foramina is compromised (obstructive hydrocephalus). The patient
from the same perspective. The lower MRI is from a patient with a
may have headache, unsteady gait, weakness of the lower extremities,
small colloid cyst in the interventricular foramen. When compared to
visual or somatosensory disorders, and/or personality changes or con-
the upper MRI, note the enlarged lateral ventricle with resultant thin-
fusion. Treatment is usually by surgical removal.
ning of the corpus callosum.
The Brain: Gross Views, Vasculature, and MRI
29
Internal frontal branches
Paracentral branches
Callosomarginal branch
Internal parietal branches
of ACA
Parietooccipital
Pericallosal branch
branches of PCA
of ACA
Frontopolar branches
of ACA
Orbital branches of ACA
Anterior cerebral artery (ACA)
Calcarine branch of PCA
Posterior temporal branches of PCA
Posterior cerebral artery (PCA)
Anterior temporal branches of PCA
2-27
Midsagittal view of the cerebral hemisphere and dien-
to serve medial regions of the frontal and parietal lobes, and the same
cephalon showing the locations and branching patterns of anterior and
relationship is maintained for the occipital and temporal lobes by
posterior cerebral arteries. The positions of gyri and sulci can be ex-
branches of the posterior cerebral artery. See Figures 8-1 (p. 240) and
trapolated from Figure 2-26 (facing page). Terminal branches of the
8-7 (p. 246) for comparable angiogram of anterior and posterior cere-
anterior cerebral artery arch laterally over the edge of the hemisphere
bral arteries.
Inferior sagittal sinus
Posterior vein of corpus callosum
Superior sagittal sinus
Internal occipital veins
TV
Veins of the
caudate nucleus
Straight sinus
Septal veins
Sinus
confluens
Transverse
sinus
Superior
Anterior cerebral vein
cerebellar vein
Occipital
Basal vein
sinus
Great
Internal cerebral vein
cerebral vein
2-28
Midsagittal view of the cerebral hemisphere and dien-
(facing page). TV = Terminal vein (superior thalamostriate vein). See
cephalon that shows the locations and relationships of sinuses
Figures 8-2 (p. 241) and 8-11 (p. 250) for comparable angiogram (ve-
and the locations and general branching patterns of veins. The
nous phase) and MRV showing veins and sinuses.
position of gyri and sulci can be extrapolated from Figure
2-26
30
External Morphology of the Central Nervous System
Anterior paracentral gyrus (APGy)
Central sulcus (CSul)
Paracentral sulcus (PCSul)
Posterior paracentral gyrus (PPGy)
Superior frontal gyrus (SFGy)
Marginal sulcus (MarSul)
Body of corpus callosum (BCorC)
Precuneus (PCun)
Sulcus of the
Splenium of corpus
corpus callosum (SulCorC)
callosum (SplCorC)
Cingulate gyrus (CinGy)
Parieto-occipital
sulcus (POSul)
Cingulate sulcus
(CinSul)
Cuneus (Cun)
Lingual gyrus
Genu of corpus
(LinGy)
callosum (GCorC)
Calcarine sulcus
(CalSul)
Septum
Rostrum of corpus
callosum (RCorC)
Fornix (For)
Midbrain tegmentum (MidTeg)
Cerebellum (Cbl)
Basilar pons (BP)
Tonsil of cerebellum (Ton)
Pontine tegmentum (PonTeg)
Medulla (Med)
SFGy
PCSul
APGy
CSul
PPGy
MarSul
BCorC
PCun
SulCorC
SplCorC
POSul
CinGy
Cun
CinSul
GCorC
CalSul
LinGy
RCorC
For
Cbl
MidTeg
BP
PonTeg
Med
Ton
2-29
A midsagittal view of the right cerebral hemisphere and di-
encephalon with the brainstem and cerebellum in situ. The MRI (T1-
weighted image) shows many brain structures from the same perspec-
tive.
The Brain: Gross Views, Vasculature, and MRI
31
Dorsal thalamus (DorTh)
Body of fornix (For)
Septum pellucidum (Sep)
Choroid plexus of third ventricle
Massa intermedia
Interventricular foramen
Stria medullaris thalami
Column of fornix
Habenula
Anterior
commissure (AC)
Suprapineal
recess
Lamina terminalis
Posterior
commissure
Pineal (P)
Supraoptic recess
Superior
colliculus (SC)
Optic chiasm
(OpCh)
Quadrigeminal
Hyth
cistern (QCis)
Inferior
colliculus (IC)
Optic nerve
Cerebral
aqueduct (CA)
Anterior medullary
velum (AMV)
Fourth ventricle
(ForVen)
Infundibulum (In)
Infundibular recess
Mammillary body (MB)
Hypothalamic sulcus
Posterior inferior
Oculomotor nerve
cerebellar artery
Interpeduncular fossa (IpedFos)
Medulla
Basilar pons (BP)
For
DorTh
Sep
Internal cerebral vein
P
AC
Tentorium cerebelli
Hypothalamus
QCis
OpCh
SC
In
IC
Pituitary gland
AMV
MB
ForVen
IpedFos BP
CA
2-30
A midsagittal view of the right cerebral hemisphere and di-
image) shows these brain structures from the same perspective. Hyth
encephalon with the brainstem in situ focusing on the details primarily
hypothalamus.
related to the diencephalon and third ventricle. The MRI (T1-weighted
32
External Morphology of the Central Nervous System
A
D
Midbrain
Anterior
Anterior
quadrangular
lobe (AntLb)
lobule
Posterior
quadrangular
lobule
Posterior
Primary
superior
fissure
fissure
E
Superior semilunar
Hemisphere
lobule
Bpon
Vermis (Ver)
AntLb
SCP
B
Fourth ventricle
Basilar pons (Bpon)
Medulla (Med)
Flocculus (Fl)
Tonsil (Ton)
F
Biventer
lobule
Med
Gracile
lobule
Ton
Inferior
semilunar
PostLb
lobule
Hemisphere
Vermis (Ver)
Ver
C
Colliculi:
Anterior
Superior
Cerebellar peduncles:
lobe (AntLb)
Inferior
Superior (SCP)
G
Middle (MCP)
Inferior
Primary fissure
AntLb
Horizontal
MCP
fissure
Fl
Flocculus (Fl)
Posterior
Tonsil (Ton)
lobe (PostLb)
Nodulus
Med
PostLb
2-31
Rostral (A, superior surface), caudal (B, inferior surface),
with cerebellar structures seen in axial MRIs at comparable levels (D,
and an inferior view (C, inferior aspect) of the cerebellum. The view
E). Structures seen on the inferior surface of the cerebellum, such as
in C shows the aspect of the cerebellum that is continuous into the
the tonsil (F), correlate closely with an axial MRI at a comparable level.
brainstem via cerebellar peduncles. The view in C correlates with su-
In G, note the appearance of the margin of the cerebellum, the general
perior surface of the brainstem (and middle superior cerebellar pe-
appearance and position of the lobes, and the obvious nature of the
duncles) as shown in Figure 2-34 on page 34.
middle cerebellar peduncle. All MRI images are T1-weighted.
Note that the superior view of the cerebellum (A) correlates closely
The Brain: Gross Views, Vasculature, and MRI
33
A
B
II,III
V
II,III
IV
I
V
Midbrain (Mid)
Primary
fissure (PriFis)
PriFis
Basilar
pons (Bpon)
Mid
VI
VII
Fourth
Bpon
VII
ventricle
(ForVen)
ForVen
Medulla
Med
VIII
(Med)
VIII
X
X
IX
IX
Posterolateral
fissure (PostLatFis)
II,III
IV
V
C
PriFis
Mid
VI
Bpon
VII
ForVen
Med
X IX
VIII
2-32
A median sagittal view of the cerebellum (A) showing its re-
Lobules I-V are the vermis parts of the anterior lobe; lobules VI-IX
lationships to the midbrain, pons, and medulla. This view of the cere-
are the vermis parts of the posterior lobe; and lobule X (the nodulus)
bellum also illustrates the two main fissures and the vermis portions of
is the vermis part of the flocculonodular lobe. Note the striking simi-
lobules I-X. Designation of these lobules follows the method devel-
larities between the gross specimen (A) and a median sagittal view of
oped by Larsell.
the cerebellum in a T1-weighted MRI (B) and a T2-weighted MRI (C).
Peduncles
Middle cerebellar
Superior cerebellar
Inferior colliculus
Trochlear nerve
Flocculus
Crus cerebri
Trigeminal nerve:
Sensory root
Motor root
Basilar pons
2-33
Lateral and slightly rostral view of the cerebellum and brain-
relative positions of, and distinction between, motor and sensory roots
stem with the middle and superior cerebellar peduncles exposed. Note
of the trigeminal nerve. See page 40, Figure 2-41D for an MRI show-
the relationship of the trochlear nerve to the inferior colliculus and the
ing the trochlear nerve.
34
External Morphology of the Central Nervous System
Internal cerebral vein
Superior colliculus (SC)
Inferior colliculus (IC)
Pineal
Frenulum
Pulvinar nuclear
complex (PuNu)
Medial
geniculate
body (MGB)
Brachium of
PulNu
superior
colliculus
SC
MGB
LGB
Brachium of
Lateral
inferior
geniculate
colliculus
body (LGB)
IC
Crus cerebri
Crus cerebri
Trochlear nerve
Trochlear nerve
(cranial nerve IV)
(cranial nerve IV)
Superior cerebellar
Anterior medullary velum
peduncle
Facial colliculus
Middle cerebellar
Sulcus limitans
peduncle
Superior fovea
Inferior cerebellar peduncle
Striae medullares of
(juxtarestiform body and
fourth ventricle
restiform body)
Lateral recess of
fourth ventricle
Vestibular area
Restiform body
Tela choroidea (cut edge)
Inferior fovea
Vagal trigone
Level of obex
Hypoglossal trigone
Tuberculum cuneatum (cuneate tubercle)
Tuberculum gracile
(gracile tubercle)
Posterolateral sulcus
Trigeminal tubercle (tuberculum cinerum)
Posterior intermediate sulcus
Cuneate fasciculus
Gracile fasciculus
Posterior median sulcus
2-34
Detailed dorsal view of the brainstem, with cerebellum re-
tuberculum cinereum is also called the trigeminal tubercle (tubercu-
moved, providing a clear view of the rhomboid fossa (and floor of the
lum trigeminale) because it is the surface representation of the spinal
fourth ventricle) and contiguous parts of the caudal diencephalon. The
trigeminal tract and its underlying nucleus. Figure 3-10 on page 61 also
dashed line on the left represents the position of the sulcus limitans and
shows a comparable view of the brainstem and the posterior portions
the area of the inferior cerebellar peduncle is shown on the right. The
of the diencephalon.
The Brain: Gross Views, Vasculature, and MRI
35
Vessels
Structures
Choroid plexus, third ventricle
Pineal
Habenula
Medial thalamus
Brachium of superior colliculus
Thalamogeniculate arteries
Superior
Lateral thalamus
colliculus
Pulvinar nucleus
Internal capsule
Choroid plexus,
lateral ventricle
Medial
Lateral geniculate body
and lateral
posterior choroidal arteries
Medial geniculate body
Quadrigeminal artery
Brachium of inferior colliculus
Superior cerebellar artery:
Crus cerebri
Medial branch
Trochlear nerve (IV)
Lateral branch
Inferior colliculus
Superior cerebellar peduncle
Anterior medullary velum
Facial colliculus
Vestibular area
Inferior cerebellar peduncle
Middle cerebellar peduncle
Choroid plexus, fourth ventricle
Hypoglossal trigone
Anterior inferior
Glossopharyngeal nerve (IX)
cerebellar artery
Vagal nerve (X)
Accessory nerve (XI)
Posterior inferior
cerebellar artery
Restiform body
Vagal trigone
Trigeminal tubercle (tuberculum cinereum)
Cuneate tubercle
Posterior spinal artery
Gracile tubercle
Gracile fasciculus
Cuneate fasciculus
2-35
Dorsal view of the brainstem and caudal diencephalon show-
tion to serving the medulla, branches of the posterior inferior cerebel-
ing the relationship of structures and some of the cranial nerves to ar-
lar artery also supply the choroid plexus of the fourth ventricle. The
teries. The vessels shown in this view have originated ventrally and
tuberculum cinereum is also called the trigeminal tubercle. See Figure
wrapped around the brainstem to gain their dorsal positions. In addi-
8-12 on p. 251 for blood supply of the choroid plexus.
36
External Morphology of the Central Nervous System
Medial
geniculate body
Lateral geniculate
Brachium of
body
inferior colliculus
Superior colliculus
Crus cerebri
Inferior colliculus
Trochlear nerve
Superior cerebellar
Motor root of
peduncle
trigeminal nerve
Middle cerebellar
Sensory root of
peduncle
trigeminal nerve
Vestibulocochlear
Lateral recess of
nerve
fourth ventricle
Restiform body
Basilar pons
Posterior inferior
cerebellar artery
Tuberculum
cinereum
(trigeminal tubercle)
2-36
Lateral view of the left side of the brainstem emphasizing
temporal lobe have been removed. Compare with Figure 2-38 on the
structures that are located dorsally. The cerebellum and portions of the facing page.
Medial eminence
of fourth ventricle
Superior cerebellar peduncle
Facial colliculus
Middle
cerebellar peduncle
Superior fovea
Vestibular area
Striae medullares
Lateral recess
Foramen of Luschka
Hypoglossal trigone
Sulcus limitans
Vagal trigone
Restiform body
Cuneate tubercle
Inferior fovea
Gracile tubercle
Tela choroidea
(cut edge)
2-37
The floor of the fourth ventricle (rhomboid fossa) and imme-
diately adjacent structures. Also compare with Figure 2-34 on page 34.
The Brain: Gross Views, Vasculature, and MRI
37
Fornix
Choroid plexus, third ventricle
Optic tract
Posterior choroidal arteries
Thalamogeniculate artery
Lateral geniculate body
Dorsal thalamus
Posterior cerebral artery
Mammillary body
Medial geniculate body
Quadrigeminal artery
Superior colliculus
Posterior communicating
Crus cerebri
artery
Internal carotid artery
Brachium of inferior colliculus
Inferior colliculus
Oculomotor nerve
Superior cerebellar artery
Trochlear nerve
Trigeminal nerve
Motor root
Sensory root
Superior cerebellar peduncle
Anterior medullary velum
Basilar artery
Middle cerebellar peduncle
Anterior inferior
cerebellar artery
Vestibulocochlear nerve
Facial nerve
Labyrinthine artery
Posterior inferior
Abducens nerve
cerebellar artery
Glossopharyngeal nerve
Choroid plexus,
fourth ventricle
Vagus nerve
Hypoglossal nerve
Restiform body
Accessory nerve
Cuneate tubercle
Gracile tubercle
Posterior inferior cerebellar artery
Posterior spinal artery
Anterior spinal artery
Vertebral artery
2-38
Lateral view of the brainstem and thalamus, which shows the
shown as dashed lines. Arteries that distribute to dorsal structures orig-
relationship of structures and cranial nerves to arteries. The approxi-
inate from large ventral vessels. Compare with Figure 2-36 on the fac-
mate positions of the labyrinthine and posterior spinal arteries, when
ing page.
they originate from the basilar and vertebral arteries, respectively, are
38
External Morphology of the Central Nervous System
A
Optic nerve
Optic chiasm
Infundibulum
Optic tract
Crus cerebri
Mammillary
body
Interpeduncular
fossa
B
Optic nerve
Bulb of eye
Optic chiasm
Temporal lobe
Mammillary
body
Optic tract
Interpeduncular
fossa
Uncus
Midbrain
Crus cerebri
tegmentum
C
Dorsal thalamus
Frontal lobe
Interpeduncular
fossa
Optic nerve
Basilar pons
Bulb of eye
D
Anterior communicating
artery
Optic nerve
Optic chiasm
Anterior cerebral
Optic tract
artery, A1 segment
Interpeduncular
fossa
Infundibulum
Midbrain
2-39
Inferior view of the hemisphere showing the optic nerve (II),
cerebral artery (D) is the most common site of supratentorial (carotid
chiasm, tract, and related structures (A). The MRIs of cranial nerve II
system) aneurysms. Rupture of aneurysms at this location is one of the
are shown in axial (B, T1-weighted; D, T2-weighted) and in oblique
more common causes of spontaneous subarachnoid hemorrhage. The
sagittal (C, T1-weighted) planes. Note the similarity between the ax-
proximity of these vessels to optic structures and the hypothalamus (D)
ial planes, especially (B), and the gross anatomical specimen. In addi-
explain the variety of visual and hypothalamic disorders experienced by
tion, note the relationship between the anterior cerebellar artery, an-
these patients. A lesion of the optic nerve results in blindness in that
terior communicating artery, and the structures around the optic
eye and loss of the afferent limb of the pupillary light reflex. Lesions
chiasm (D).
in, or caudal to, the optic chiasm result in deficits in the visual fields of
The anterior communicating artery or its junction with the anterior both eyes.
The Cranial Nerves
39
artery
Optic chiasm
Middle cerebral
artery
Posterior communicating
Posterior cerebral
artery
artery
Oculomotor nerve
Basilar artery
Superior cerebellar
artery
Basilar pons
B
Optic tract
Posterior cerebral
Bulb of the eye
artery
Superior cerebellar
artery
Oculomotor nerve
C
Internal carotid
Oculomotor
artery
nerve
Oculomotor
Temporal lobe
nerve
Basilar pons
Uncus
(rostral portion)
Fourth ventricle
(rostral portion)
D
Corpus callosum
Dorsal thalamus
Frontal lobe
Superior colliculus
Interpeduncular
fossa
Inferior colliculus
Cerebellum
Optic chiasm
Basilar pons
Oculomotor nerve
2-40
Inferior view of the hemisphere showing the exiting fibers of
position of the oculomotor nerve in the interpeduncular fossa rostral
the oculomotor nerve (III), and their relationship to the posterior cere-
to the basilar pons and caudal to optic structures.
bral and superior cerebellar arteries (A). The MRIs of cranial nerve III
That portion of the posterior cerebral artery located between the
are shown in sagittal (B, T2-weighted; D, T1-weighted) and in axial
basilar artery and the posterior communicating artery (A) is the P1 seg-
(C, T1-weighted) planes. Note the relationship of the exiting fibers of
ment. The most common site of aneurysms in the infratentorial area
the oculomotor nerve to the posterior cerebral and superior cerebel-
(vertebrobasilar system) is at the bifurcation of the basilar artery, also
lar arteries (A, B) and the characteristic appearance of the III nerve as
called the basilar tip. Patients with aneurysms at this location may pre-
it passes through the subarachnoid space toward the superior orbital fis-
sent with eye movement disorders and pupillary dilation due to dam-
sure (C). The sagittal section (D) is just off the midline and shows the
age to the root of the third nerve (A,B).
40
External Morphology of the Central Nervous System
A
Mammillary body
Interpeduncular
Lamina terminalis
fossa
Supraoptic recess
Cerebral aqueduct
Optic chiasm
Oculomotor nerve
Basilar pons
Infundibular recess
B
Posterior cerebral
Optic tract
artery
Superior cerebellar
artery
Optic nerve
Oculomotor nerve
Basilar pons
C
Posterior cerebral
Oculomotor nerve
artery
Posterior cerebral
Superior cerebellar
artery
artery
Interpeduncular
Crus cerebri
fossa
Midbrain
Anterior lobe
of cerebellum
D
Anterior cerebral artery
Middle cerebral artery
Hypothalamus
Optic tract
Interpeduncular fossa
Mammillary body
Posterior cerebral artery
Crus cerebri
Trochlear nerve
Midbrain tegmentum
(in ambient cistern)
Fourth ventricle
(rostral portion)
2-41
A median sagittal view of the brainstem and diencephalon (A)
nerves of the midbrain. The third nerve exits via the interpeduncular
reveals the position of the oculomotor nerve (III) in relation to adjacent
fossa to innervate four major extraocular muscles (see Fig. 7-15 on
structures. The MRI in B and C show the position of the oculomotor
page 201) and, through the ciliary ganglion, the sphincter pupillae
nerve in sagittal (B, T1-weighted) and in axial (C, T2-weighted) planes.
muscles. Damage to the oculomotor nerve may result in paralysis of
Note the relationship of the oculomotor nerve to the adjacent posterior
most eye movement, a dilated pupil, and loss of the efferent limb of
cerebral and superior cerebellar arteries (B, C). Also compare these im-
the pupillary light reflex, all in the ipsilateral eye. The fourth nerve is
ages with that of figure 2-40B on page 39. In D (T2-weighted), the
unique in that it is the only cranial nerve to exit the posterior (dorsal)
trochlear nerve is seen passing through the ambient cistern around the
aspect of the brainstem and is the only cranial nerve motor nucleus to
lateral aspect of the midbrain (compare with Fig. 2-32 on page 32).
innervate, exclusively, a muscle on the contralateral side of the mid-
The oculomotor (III) and trochlear (IV) nerves are the cranial
brain.
The Cranial Nerves
41
A
Basilar pons
Trigeminal nerve
Abducens nerve
Facial nerve
Flocculus
Vestibulocochlear
Pyramid
nerve
B
Internal carotid artery
Temporal lobe
Trigeminal nerve
Basilar artery
Middle cerebellar
Trigeminal nerve
peduncle
Basilar pons
Fourth ventricle
Pontine tegmentum
Cerebellum
C
Temporal lobe
Trigeminal
Trigeminal
ganglion
ganglion
Basilar artery
Trigeminal nerve
Superior cerebellar
artery
Tegmentum
of pons
Basilar pons
Fourth ventricle
Anterior lobe
of cerebellum
Third ventricle
D
E
Midbrain
Crus cerebri
tegmentum
Interpeduncular
Root of trigeminal
fossa
nerve
Sensory root of the
trigeminal nerve
Basilar pons
Basilar pons
Pyramid
2-42
The trigeminal nerve (V) is the largest of the cranial nerve
in the coronal plane (D, E). In addition, the MRI in C clearly illustrates
roots exiting the brainstem (A). It exits at an intermediate position on
the position of the trigeminal ganglion in the middle cranial fossa.
the lateral aspect of the pons roughly in line with cranial nerves VII, IX,
Trigeminal neuralgia (tic douloureux) is a lancinating paroxysmal
and X. The fifth nerve, and these latter three, are mixed nerves in that
pain within the V2-V3 territories frequently triggered by stimuli
they have motor and sensory components. The trigeminal nerve is
around the corner of the mouth. The causes are probably multiple and
shown in axial MRI (B, T1-weighted; C, T2-weighted) and in coronal
may include neurovascular compression by the superior cerebellar
planes (D, E, both T1-weighted images). Note the characteristic ap-
artery (see the apposition of this vessel to the nerve root in C), multi-
pearance of the root of the trigeminal nerve as it traverses the sub-
ple sclerosis, tumors, and ephaptic transmission within the nerve or
arachnoid space (B and C), the origin of the trigeminal nerve, and the
ganglion.
position of the sensory root of the nerve at the lateral aspect of the pons
42
External Morphology of the Central Nervous System
A
Abducens nerve
Facial nerve
Vestibulocochlear
nerve
Vestibulocochlear
nerve
Facial nerve
Glossopharyngeal
nerve
Pyramid
Vagus nerve
Olivary eminence
Hypoglossal nerve
B
Basilar artery
Cochlea
Abducens nerve
Semicircular canals
Pons-medulla junction
Vestibulocochlear
nerve
Lateral recess of
fourth ventricle
Facial nerve
Fourth ventricle
Tonsil of cerebellum
Abducens nerve
C
Cochlea
Cochlea
Cochlear portion of
VIIIth nerve (CPVIII)
Semicircular canals
VPVIII
Semicircular canals
CPVIII
Vestibular portion of
VIIIth nerve (VPVIII)
Fourth ventricle
Basilar pons
Pontine tegmentum
Cerebellum
D
Cochlear portion of
VIIIth nerve
Anterior inferior
cerebellar artery
Cochlea
Cochlea
Semicircular canals
Semicircular canals
Vestibular portion of
Cerebellar vermis
VIIIth nerve
Cerebellar
Cerebellar
tonsil
hemisphere
2-43
The cranial nerves at the pons medulla junction are the ab-
anterior
inferior cerebellar artery also enters the internal acoustic
ducens (VI), the facial (VII), and the vestibulocochlear (VIII) (A). The
meatus.
facial and vestibulocochlear nerves both enter the internal acoustic
The so-called acoustic neuroma, a tumor associated with the eighth
meatus, the facial nerve distributing eventually to the face through the
nerve, is actually a vestibular schwannoma since it arises from the
stylomastoid foramen, and the vestibulocochlear nerve to structures
neurilemma sheath of the vestibular root. Most patients with this tu-
of the inner ear. MRI in the axial plane, B, C, D, (all T2-weighted im-
mor have hearing loss, tinnitus and equilibrium problems, or vertigo.
ages) show the relationships of the vestibulocochlear root and the fa-
As the tumor enlarges (to more than about 2 cm) it may cause facial
cial nerve to the internal acoustic meatus. Also notice the character-
weakness (seventh root), numbness (fifth root), or abnormal corneal
istic appearance of the cochlea (B, C) and the semicircular canals (C).
reflex (fifth or seventh). Treatment is usually by surgery, radiation
In addition to these two cranial nerves, the labyrinthine branch of the
therapy, or a combination thereof.
The Cranial Nerves
43
A
Facial nerve
Abducens nerve
Vestibulocochlear
nerve
Glossopharyngeal
Olivary eminence
nerve
Vagus nerve
Postolivary sulcus
Preolivary sulcus
Hypoglossal nerve
B
Preolivary sulcus
Pyramid
Retroolivary sulcus
(postolivary sulcus)
Olive (inferior)
Glossopharyngeal
Glossopharyngeal
nerve
nerve
Flocculus
Restiform body
Fourth ventricle
Tonsil of cerebellum
Cerebellum
C
Pyramid
Olive (inferior)
Vagus nerve
Retroolivary sulcus
Fourth ventricle
Tonsil of cerebellum
Cerebellum
D
Pyramid
Olivary eminence
Vagus nerve
Postolivary sulcus
Restiform body
Vagus nerve
Fourth ventricle
Tonsil of cerebellum
2-44
The glossopharyngeal (IX) and vagus (X) nerves (A) exit the
cle is smaller. The ninth and tenth cranial nerves and the spinal portion
lateral aspect of the medulla via the postolivary sulcus; the ninth nerve
of the accessory nerve (XI) exit the skull via the jugular foramen.
exits rostral to the row of rootlets comprising the tenth nerve (A).
Glossopharyngeal neuralgia is a lancinating pain originating from
These nerves are generally in line with the exits of the facial and
the territories served by the ninth and tenth nerves at the base of the
trigeminal nerves; all of these are mixed nerves. The exit of the glos-
tongue and throat. Trigger events may include chewing and swallow-
sopharyngeal nerve (A, B) is close to the pons-medulla junction and
ing. Lesions of nerves passing through the jugular foramen (IX, X, XI)
correlates with the corresponding shape (more rectangular) of the
may result in loss of the gag reflex (motor limb via ninth nerve), and
medulla. The vagus nerve exits at a slightly more caudal position (A,
drooping of the ipsilateral shoulder accompanied by an inability to turn
C, D); the shape of the medulla is more square and the fourth ventri-
the head to the opposite side against resistance (eleventh nerve).
44
External Morphology of the Central Nervous System
A
Facial nerve
Abducens nerve
Vestibulocochlear
nerve
Glossopharyngeal
Olivary eminence
nerve
Vagus nerve
Postolivary sulcus
Preolivary sulcus
Hypoglossal nerve
B
Pyramid
Preolivary sulcus
Hypoglossal nerve
Olivary eminence
Postolivary sulcus
Restiform body
Vagus nerve
Tonsil of cerebellum
C
Hypoglossal nerve
Medulla
Tonsil of cerebellum
Cerebellum
2-45
The hypoglossal nerve (XII) (A) exits the inferolateral aspect
teristic position of the hypoglossal nerve in the subarachnoid space and
of the medulla via the preolivary sulcus. This cranial nerve exits in line
its relation to the overall shape of the medulla. This shape is indicative
with the abducens nerve found at the pons-medulla junction and in line
of a cranial nerve exiting at more mid-to-caudal medullary levels. In B,
with the exits of the third and fourth nerves of the midbrain. The
note its relationship to the preolivary sulcus and olivary eminence. The
twelfth nerve exit is characteristically located laterally adjacent to the
hypoglossal exits the base of the skull by traversing the hypoglossal
pyramid, which contains corticospinal fibers.
canal. A lesion of the hypoglossal nerve results in a deviation of the
In axial MRI (B, T2-weighted; C, T1-weighted), note the charac- tongue to the ipsilateral side on attempted protrusion.
The Insula
45
Precentral gyrus (PrCGy)
Superior frontal
Central sulcus (CSul)
gyrus
Postcentral gyrus
(PoCGy)
Middle frontal
gyrus (MFGy)
Gyri longi (GyLon-long
gyri of the insula)
Gyri breves
(GyBr-short gyri
of the insula)
Transverse temporal
Central sulcus
gyrus (TrTemGy)
of the insula (CSulIn)
Limen insulae (LimIn)
Temporal lobe (TLob)
PrCGy
PoCGy
CSul
MFGy
TrTemGy
GyBr
GyLon
CSulIn
LimIn
TLob
CSul
PrCGy
PoCGy
MFGy
CSulIn
GyBr
GyLon
TLob
2-46
Lateral view of the left cerebral hemisphere with the cortex
overlying the insula removed. Structures characteristic of the insular
cortex, and immediately adjacent areas, are clearly seen in the two
MRIs in the sagittal plane through lateral portions of the hemisphere
(inversion recovery—upper; T1-weighted image—lower).
46
External Morphology of the Central Nervous System
Comparison of Cerebral versus Spinal Meninges
CEREBRAL
SPINAL
Dura
Dura
• adherent to inner table of skull (no epidural space)
• separated from vertebrae by epidural space
• composed of two fused layers (periosteal and meningeal),
• composed of one layer (spinal dura only; vertebrae
which split to form sinuses
have their own periosteum)
Arachnoid (outer part of leptomeninges)
Arachnoid (outer part of leptomeninges)
• attached to dura in living condition (no subdural space)
• attached to dura in living condition (no subdural
space)
• arachnoid villi (in superior sagittal sinus)
• no arachnoid villi
• arachnoid trabeculae
• few or no arachnoid trabeculae but larger arachnoid septae
• subarachnoid space with many cisterns
• subarachnoid space with one cistern
Pia (inner part of leptomeninges)
Pia (inner part of leptomeninges)
• intimately adherent to surface of brain
• intimately adherent to surface of cord
• no pial specializations
• specializations in the form of denticulate ligaments,
filum terminale, and linea splendens
• follows vessels as they pierce the cerebral cortex
• follows vessels as they pierce the cord
Meningitis, Meningeal, Hemorrhages, Meningioma
A wide variety of disease processes and lesions may involve the
commonly seen in younger patients, are usually detected immediately
meninges; only a few examples are mentioned here.
or within a few hours after the precipitating incident. Chronic subdural
Bacterial infections of the meninges (bacterial meningitis) are
hematomas, usually seen in the elderly, are frequently of unknown ori-
commonly called leptomeningitis because the causative organisms are
gin; may take days or weeks to become symptomatic; and cause a pro-
usually found in the subarachnoid space and involve the pia and arach-
gressive change in mental status of the patient. This lesion appears
noid. The organism seen in about one-half of adult cases is Streptococcus
“long and thin,” compared to an epidural hematoma, follows the sur-
pneumoniae, while in neonates and children up to about 1 year it is Es-
face of the brain, and may extend for considerable distances (see Fig.
cherichia coli. The patient becomes acutely ill (i.e., confusion, fever, stiff
2-48 on p. 48 and Fig. 2-51 on p. 51). Treatment is surgical evacua-
neck, stupor), may have generalized or focal signs/symptoms, and, if
tion (for larger or acute lesions) or close monitoring for small, asymp-
not treated rapidly, will likely die. Treatment is with appropriate an-
tomatic, or chronic lesions.
tibiotics. Patients with viral meningitis may become ill over a period
The most common cause of subarachnoid hemorrhage is trauma.
of several days, experience headache, confusion, and fever, but, with
In approximately 80% of patients with spontaneous (nontrau-
supportive care, will recover after an acute phase of approximately 1-2
matic) subarachnoid hemorrhage, the precipitating event is rupture
weeks. These patients usually recover with no permanent deficits.
of an intracranial aneurysm. Symptomatic bleeding from an arteriove-
The most common cause of an epidural (extradural) hematoma
nous malformation occurs in approximately 5% of cases. Blood collects
is a skull fracture that results in a laceration of a major dural vessel, such
in, and percolates through, the subarachnoid space and cisterns (see
as the middle meningeal artery. In approximately 15% of cases, bleed-
Fig. 2-51 on page 51). Sometimes, the deficits seen (assuming the pa-
ing may come from a venous sinus. The extravasated blood dissects the
tient is not in coma) may be a clue as to location, especially if cranial
dura mater off the inner table of the skull; there is no preexisting (ex-
nerves are nearby. Onset is sudden; the patient complains of an excru-
tradural) space for the blood to enter. These lesions are frequently
ciating headache and may remain conscious, become lethargic and dis-
large, lens (lenticular) shaped, may appear loculated, and are “short
oriented, or may be comatose. Treatment of an aneurysm is to surgi-
and thick” compared to subdural hematomas (see Fig. 2-48 on page
cally occlude it (by clip or coil), if possible, and to protect against the
48). The patient may lapse into a coma and, if the lesion is left un-
development of vasospasm. During surgery, some blood in the sub-
treated, death may result. In some cases, the patient may initially be
arachnoid space and cisterns may be removed.
unconscious followed by a lucid interval (the patient is wide awake),
Tumors of the meninges (meningiomas) are classified in different
then subsequently deteriorate rapidly and die; this is called “talk and
ways but they usually arise from arachnoid cap/stem cells (a small
die.” Treatment of choice for large lesions is surgical removal of the
number are dural in origin) around the villi or at places where vessels
clot and coagulation of the damaged vessel.
or cranial nerves penetrate the dura-arachnoid. These tumors grow
Tearing of bridging veins (veins passing from the brain outward
slowly (symptoms may develop almost imperceptibly over years), are
through the arachnoid and dura), usually the result of trauma, is a com-
histologically benign, may result in hyperostosis of the overlying skull,
mon cause of subdural hematoma. This designation is somewhat a
and frequently contain calcifications. In decreasing order, menin-
misnomer because the extravasated blood actually dissects through a
giomas are found in the following locations: parasagittal area
falx
specialized, yet structurally weak, cell layer at the dura-arachnoid in-
(together 29%), convexity 15%, sella 13%, sphenoid ridge 12%, and
terface; this is the dural border cell layer. There is no preexisting “sub-
olfactory groove 10%. Treatment is primarily by surgical removal, al-
dural space” in the normal brain. Acute subdural hematomas, more
though some meningiomas are treated by radiotherapy.
The Meninges, Cisterns, and Meningeal and Cisternal Hemorrhages
47
Superior sagittal sinus
Arachnoid villus
Skull
Lateral lacunae
Cerebrum
Dura mater
Arachnoid mater
Arachnoid trabeculae
Pia mater
Transverse
sinus
Falx cerebri
Tentorium
cerebelli
Cerebellum
Cistern
Skull
Dura mater
Subarachnoid space
Arachnoid mater
Cerebral vessel
and branch
Pia mater
Arachnoid trabeculae
Vertebrae
Spinal nerves
Spinal vessel
Dura mater
Dura mater
Intervertebral
ligament
Epidural space
Conus medullaris
Vertebra
Cauda equina
Lumbar cistern
Arachnoid mater
Filum terminale (interum)
Denticulate ligament
Pia mater
Coccygeal ligament (filum terminale externum)
Coccyx
2-47
Semidiagrammatic representation of the central nervous sys-
fourth ventricles. It circulates through the ventricular system (small ar-
tem and its associated meninges. The details show the relationships of
rows) and enters the subarachnoid space via the medial foramen of Ma-
the meninges in the area of the superior sagittal sinus, on the lateral as-
gendie and the two lateral foramen of Luschka. In the living situation
pect of the cerebral hemisphere, and around the spinal cord. Cere-
the arachnoid is attached to the inner surface of the dura. There is no
brospinal fluid is produced by the choroid plexi of lateral, third, and
actual or potential subdural space.
48
External Morphology of the Central Nervous System
A
B
C
D
Hemorrhage in brain
2-48
Examples of epidural (extradural) hemorrhages (A, B) and of
where the blood is replaced by fluid, and the subacute phase by the
acute (C) and subacute (D) subdural hematoma. Note the lenticular
middle arrow where fresher blood has entered the lesion. Note the ex-
shape of the epidural lesions (A, B), their loculated appearance, and
tent of this lesion on the surface of the cortex and its narrowness com-
their location external to the substance of the brain. In contrast, the
pared to epidural lesions. The patient in D also has small hemorrhages
acute subdural lesion (C) is quite thin and extends over a longer dis-
into the substance of the brain, the larger of these in the region of the
tance on the cortex.
genu of the internal capsule. Images A-D are CT. For additional com-
In D, the subdural hematoma has both chronic and subacute phases.
ments on epidural and subdural hemorrhages see page 46.
The chronic phase is indicated by the upper two and lower two arrows
The Meninges, Cisterns, and Meningeal and Cisternal Hemorrhages
49
A
B
C
Blood in frontal lobe
Blood in third
Temporal horn
ventricle
Blood in cerebral
aqueduct
2-49
Examples of hemorrhages into the substance of the brain that,
Blood in the substance of the brain and in the ventricular system may
in some cases, have also resulted in blood in the ventricular system.
also result from trauma (C). In this example (C), blood is seen in the
The large hemorrhages into the hemisphere (A, B) have resulted in en-
frontal lobe and in the third ventricle and cerebral aqueduct. The en-
largement of the ventricles, a midline shift, and, in the case of A, a
larged temporal horns (C) of the lateral ventricles are consistent with
small amount of blood in the posterior horn of the lateral ventricle. In
the interruption of CSF flow through the cerebral aqueduct (noncom-
these examples, the lesion is most likely a result of hemorrhage from
municating hydrocephalus). Images A-C are CT.
lenticulostriate branches of the M1 segment.
50
External Morphology of the Central Nervous System
A
Paracallosal cistern
Lamina terminalis
Quadrigeminal cistern
cistern
B
Chiasmatic cistern
Fourth ventricle
Interpeduncular cistern
C
Prepontine cistern
Premedullary cistern
D
Cisterna magna
B
Lamina terminalis cistern
Sylvian cistern
Optic tract
Crural cistern
Midbrain
Interpeduncular cistern
Ambient cistern
Quadrigeminal cistern
Inferior colliculus
C
Prepontine cistern
Trigeminal nerve
Basilar artery
Superior
Basilar pons
cerebellopontine
cistern
Fourth ventricle
D
Premedullary cistern
Medulla
Inferior
cerebellopontine
Cisterna magna
cistern
2-50
A median sagittal MRI (A, T2-weighted) of the brain show-
contain arteries and veins, roots of cranial nerves, and, of course, cere-
ing the positions of the major cisterns associated with midline struc-
brospinal fluid. Consequently, the subarachnoid space and cisterns are
tures. Axial views of the midbrain
(B, T1-weighted), pons
(C,
continuous one with the other. In addition, the subarachnoid space
T2-weighted), and medulla (D, T2-weighted) represent the corre-
around the brain is continuous with that around the spinal cord. Com-
sponding planes indicated in the sagittal view (A).
pare these cisterns with blood-filled parts of the subarachnoid space and
Cisterns are the enlarged portions of the subarachnoid space that cisterns in Figure 2-51 on the facing page.
The Meninges, Cisterns, and Meningeal and Cisternal Hemorrhages
51
A
B
Subdural
hemorrhage
Lamina terminalis
cistern
Supraoptic recess
Sylvian
Interpeduncular
cistern
cistern
Crural
cistern
Temporal
horn
Blood on
insular cortex
Midbrain
Ambient
Quadrigeminal
cistern
cistern
C
D
Lamina terminalis
Third
cistern
ventricle
Sylvian
Blood on
cistern
insula
Interpeduncular
cistern
Crural
cistern
Ambient
Cerebellum
cistern
Blood on tentorium
Rostral part
cerebelli
of fourth
ventricle
2-51
Blood in the subarachnoid space and cisterns. In these CT ex-
blood on the cortex of the insula. In C, the blood is located around the
amples, blood occupies the subarachnoid space and cisterns, outlining
midbrain (crural and ambient cisterns), extends into the Sylvian cis-
these areas in white. Consequently, the shape of the cisterns is indi-
tern, and into the cistern of the lamina terminalis. The sharp interface
cated by the configuration of the white area, the white area represent-
between the lamina terminalis cistern (containing blood) and the third
ing blood.
ventricle (devoid of blood) represents the position of the lamina ter-
Around the base of the brain (A), it is easy to identify the cisterns
minalis. In D, blood is located in cisterns around the pons but avoids
related to the midbrain, the supraoptic recess which is devoid of blood,
the rostral part of the fourth ventricle. Compare these images with the
and blood extending laterally into the Sylvian cistern. In some cases
locations of some of the comparable cisterns as seen in Figure 2-50 on
(B), subdural hemorrhage may penetrate the arachnoid membrane and
the facing page. Images A-D are CT.
result in blood infiltrating between gyri, such as this example with
52
External Morphology of the Central Nervous System
Massa intermedia
Body of lateral
Pineal recess
ventricle
Third ventricle
Suprapineal recess
Anterior horn of
Posterior commissure
lateral ventricle
Pineal
Interventricular
Atrium of lateral ventricle
foramen
(and glomus choroideum)
Posterior horn of
Anterior commissure
lateral ventricle
Tectum
Lamina terminalis
Cerebral aqueduct
Infundibular recess
Supraoptic recess
Fourth ventricle
Optic chiasm
Infundibulum
Lateral recess of fourth ventricle
Mammillary body
Foramen of
Amygdaloid nuclear complex
Luschka
Inferior horn of lateral ventricle
Dorsal cerebellomedullary
cistern (cisterna magna)
Bordering Structures
Ventricular Space
Genu of corpus callosum
Anterior horn of lateral ventricle
Head of caudate nucleus
Septum pellucidum
Body of lateral ventricle
(ventral to body of corpus callosum)
Body of caudate nucleus
Third ventricle
Fornix
Suprapineal recess
Amygdaloid nuclear complex
Inferior horn of
lateral ventricle
Tail of caudate nucleus
Hippocampal formation
Cerebral aqueduct
Splenium of
corpus callosum
Atrium of lateral ventricle
(contains glomus choroideum)
Optic radiations
Tapetum
Lateral recess of
fourth ventricle
Fourth ventricle
Posterior horn of lateral ventricle
2-52
Lateral (above) and dorsal (below) views of the ventricles and
colors are continued in Figure 2-53 on the facing page. Note the rela-
the choroid plexus. The dashed lines show the approximate positions
tionships between the choroid plexus and various parts of the ventric-
of some of the important structures that border on the ventricular
ular system. The large expanded portion of the choroid plexus found
space. The choroid plexus is shown in red and structures bordering on
in the area of the atrium is the glomus (glomus choroideum). See Fig-
the various portions of the ventricular spaces are color-coded; these
ure 8-12 on p. 251 for details of blood supply to the choroid plexus.
The Ventricles and Ventricular Hemorrhage
53
Corpus callosum
A
Caudate nucleus
(body)
Corpus callosum
B
(body)
Caudate nucleus (body)
Anterior horn of
lateral ventricle
Stria terminalis
Septum
pellucidum
Corpus callosum (body)
Septum
C
pellucidum
Fornix (F)
Body of lateral
Caudate nucleus
ventricle
Body of lateral
(head)
ventricle
Choroid plexus (CP)
Interventricular
Corpus callosum
foramen
(rostrum)
Fornix
F
Anterior
commissure
CP
Third ventricle
Third ventricle
Gyrus rectus
Dorsal thalamus
Hypothalamus
Massa intermedia
Optic chiasm
Mammillary body
B
C
D
Hypothalamus
E
A
F
Caudate nucleus
D
G
Fornix
Dorsal thalamus
Third ventricle
Amygdaloid nuclear
complex
Optic tract
Inferior horn of
lateral ventricle
Body of lateral
ventricle
E
Corpus callosum
Pulvinar
Optic radiations
Optic radiations
Tapetum
Tapetum
F
Pineal
G
Cerebral aqueduct
Inferior horn of
lateral ventricle
Hippocampal formation
Corpus callosum
(splenium)
Caudate nucleus (tail)
Atrium of lateral
Calcarine
ventricle
sulcus
Hippocampal formation
Calcar avis
Posterior horn of
lateral ventricle
2-53
Lateral view of the ventricular system and corresponding
and the majority of structures labeled have some direct relevance to the
semidiagrammatic cross-sectional representations from rostral (A) to
ventricular space at that particular level. The color-coding corresponds
caudal (G) identifying specific structures that border on the ventricu-
to that shown in Figure 2-52 on the facing page.
lar space. In the cross-sections, the ventricle is outlined by a heavy line,
54
External Morphology of the Central Nervous System
A
B
Anterior horn of
lateral ventricle
Anterior horn
Atrium of
Third
lateral ventricle
ventricle
Posterior horn of
lateral ventricle
C
Temporal horn of
lateral ventricle
Basilar pons
Tegmentum of pons
Fourth ventricle
Cerebellum
D
Pons-medulla junction
Fourth ventricle
Lateral recess of
fourth ventricle
Cerebellum
2-54
Examples of hemorrhage occupying portions of the ventric-
third ventricle (B). Blood also clearly outlines central portions of the
ular system (ventricular hemorrhage). In these CT images, blood ap-
fourth ventricle (C) and caudal portions of the fourth ventricle (D), in-
pears white within the ventricles. Consequently, the shape of the ven-
cluding an extension of blood into the left lateral recess of the fourth
tricular system is outlined by the white area, and the specific portion
ventricle. In addition to these images, Figure 2-49 on page 49 shows
of the ventricular system is correspondingly labeled.
blood in the cerebral aqueduct and in the most inferior portions of the
Note blood in the anterior horn, atrium, and posterior horn of the
third ventricle. Images A-D are CT.
lateral ventricles (A, B), and blood clearly outlining the shape of the
CHAPTER
3
Dissections of the
Central Nervous System
56
Dissections of the Central Nervous System
Central sulcus
Gyri:
Superior parietal lobule
Precentral
Postcentral
Supramarginal gyrus
Long insular gyri
Short insular gyri
Central sulcus of insula
Gyri:
Transverse temporal
Superior temporal
Superior temporal sulcus
3-1
Lateral view of the right cerebral hemisphere with the inferior
removed to show the insular cortex, transverse temporal gyri, and re-
and parts of the middle frontal gyri and precentral and postcentral gyri
lated structures.
Superior longitudinal
fasciculus
Uncinate fasciculus
External capsule
3-2
Dissection of the lateral aspect of the right cerebral hemisphere subcortical white matter. This dissection is deep to that shown in Fig-
showing the locations and relationships of some of the main bundles of ure 3-1 (above) and superficial to that shown in Figure 3-3 on page 57.
Lateral, Medial, and Ventral Aspects
57
Superior
longitudinal
fasciculus
Corona radiata
Lenticular
nucleus
Uncinate fasciculus
Occiptofrontal
fasciculus
3-3
Dissection of the lateral aspect of the right cerebral hemisphere
lus. The lenticular nucleus is shown in situ, lateral to the internal cap-
showing the relationship between fibers radiating from the internal
sule. This is a deeper dissection of the specimen shown in Figure 3-2
capsule (corona radiata) and those of the superior longitudinal fascicu-
on page 56.
Internal capsule (IC):
Posterior limb
Genu
Anterior limb
Optic radiations
Retrolenticular limb of IC
3-4
Dissection of the lateral aspect of the right cerebral hemisphere
This is a deeper dissection of the specimen shown in Figure 3-3
showing the internal capsule and the concavity left by removal of the
(above).
lenticular nucleus. Note the other bundles of subcortical white matter.
58
Dissections of the Central Nervous System
Cingulum
Calcarine sulcus
Corpus callosum
Spiral fibers of hippocampus
3-5
Dissection of the medial aspect of the left cerebral hemisphere
showing the cingulum and spiral fibers of the hippocampus.
Optic:
Infundibulum
Nerve
Chiasm
Amygdaloid
Tract
complex
Inferior horn of
Crus cerebri
lateral ventricle
Hippocampus
Lateral geniculate
body
Calcar avis
Medial geniculate
body
Posterior horn of
lateral ventricle
3-6
Overview of a dissection showing the ventral aspect of the
cerebral hemispheres. Note the structures related to ventricular spaces
and the structures located at the mesencephalon-diencephalon inter-
face. A number of structures in addition to those labeled can be iden-
tified.
Overall Views
59
Optic chiasm
Infundibulum
Optic nerve
Olfactory tract
Anterior perforated
substance
Tuber cinereum
Crus cerebri
Amygdaloid complex
Mammillary body
Temporal horn
Optic tract
Hippocampus
Posterior perforated
substance
Substantia nigra
Medial geniculate body
Red nucleus
Lateral geniculate body
Periaqueductal gray
Brachium of
superior colliculus
Pulvinar
Choroid plexus
Splenium of
Superior colliculus
corpus callosum
Great cerebral vein
3-7
Detailed view of a dissection showing the ventral aspects of the
crus cerebri; and the relationship of hypothalamic structures on the
cerebral hemispheres; this is of the same specimen shown in Figure
ventral aspect of the brain. In addition to those labeled, other struc-
3-6 on page 58. Note the continuum of optic nerve, chiasm, and tract
tures can be identified.
to the lateral geniculate body; the relationship of the optic tract to the
60
Dissections of the Central Nervous System
Corpus callosum
Head of caudate
Septum pellucidum
Transverse
Dorsal thalamus
temporal gyrus
Fornix
Choroid plexus
3-8
Dissected view of the brain from the dorsal aspect showing
of insular and transverse temporal gyri, the fornix, and other structures
structures associated with the lateral ventricles. Note the appearance
in addition to those labeled.
Column of fornix
Anterior horn of
lateral ventricle
Head of caudate
Interventricular
foramen
Anterior nucleus
of thalamus
Massa
intermedia
Third ventricle
Pineal
Habenula
Colliculi
3-9
Dissected view of the brain from the dorsal aspect showing lat-
tufts of choroid plexus identify the locations of the interventricular
eral and third ventricles, the dorsal surface of the diencephalon, the in-
foramina. Note the massa intermedia traversing the third ventricle and
sula and transverse temporal gyri, and the colliculi. The majority of the
other structures in addition to those labeled.
fornix and the roof of the third ventricle have been removed. The small
Overall Views
61
Transverse
Fornix
cerebral fissure
Suprapineal recess
Brachium of superior colliculus
Pineal
Caudate nucleus
Brachium of inferior colliculus
Optic radiations
Choroid
plexus
Tapetum
Pulvinar
Glomus
SC
Brachium
of SC
Temporal
inferior horn
Lateral
IC
geniculate
CC
CC
body
Hippocampus
*
*
Medial
geniculate
Frenulum
body
Anterior medullary
velum
Cerebellar peduncles:
Middle
Flocculus
Superior
Inferior
Lateral recess,
fourth ventricle
Posterior column:
Tubercles
Fasciculi
3-10
A dissection showing caudal diencephalic structures, several
culi (SC), the inferior colliculi (IC), and the crus cerebri (CC), as seen
telencephalic structures, and the interface of the mesencephalon with
from the dorsal aspect, are identified. The asterisks represent the exit
caudal parts of the thalamus. On the right side, note the continuation
points of the trochlear nerves. For further details of the dorsal brain-
between the fornix and hippocampus; on the left, these structures have
stem, see Figure 2-34 on page 34. Note structures in addition to those
been removed to expose the underlying pulvinar. The superior colli-
labeled.
CHAPTER
4
Internal Morphology
of the Brain in
Slices and MRI
Brain Slices in the Coronal Plane with MRI
Orientation to Coronal MRIs: When looking at a coronal
To reinforce this concept, the rostral surface of each coronal
MRI image, you are viewing the image as if you are looking at
brain slice was photographed. So, when looking at the slice, the
the face of the patient. Consequently, the observer’s right is the
observer’s right field of view is the left side of the brain slice. This
left side of the brain in the MRI and the left side of the patient’s
view of the slice correlates exactly with the orientation of the
brain. Obviously, the concept of what is the left side versus what
brain as seen in the accompanying coronal MRIs.
is the right side of the patient’s brain is enormously important
when using MRI (or CT) to diagnose a neurologically impaired
individual.
64
Internal Morphology of the Brain in Slices and MRI
Cingulate gyrus (CinGy)
Cingulum (Cin)
Body of corpus
callosum (BCorCl)
Anterior horn of lateral
ventricle (AH of Lven)
Head of caudate nucleus
(HCaNu)
Septum pellucidum
Anterior limb of
internal capsule
(ALIntCap)
Rostrum of corpus
callosum (RCorCl)
Putamen (Put)
External capsule
(ExtCap)
Subcallosal gyrus
Insula (In)
Claustrum (Cl)
Extreme capsule
Middle cerebral
(ExtrmCap)
artery
Temporal lobe
(TemLb)
Nucleus accumbens
(NuAcc)
Optic nerve (OpNr)
CinGy
BCorCl
Cin
AH of
HCaNu
LVen
BCorCl
ALIntCap
AH of
In
LVen
Put
ExtrmCap
ExtCap
RCorCl
Cl
NuAcc
ExtrmCap
TemLb
OpNr
Olfactory tract
4-1
The rostral surface of a coronal section of brain through the an-
MRI images (both are inversion recovery) are at the same plane and
terior limb of the internal capsule and the head of the caudate nucleus. The two
show many of the structures identified in the brain slice.
Coronal Brain Slice—MRI Correlation
65
Head of caudate nucleus
(HCaNu)
Body of corpus
callosum (BCorCl)
Septum (Sep)
Anterior limb of
internal capsule (ALintCap)
Corona radiata (CorRad)
Column of fornix
(ColFor)
Putamen (Put)
External Capsule
(ExtCap)
Insula (In)
Anterior
Claustrum (Cl)
commissure (AC)
Extreme capsule (ExtCap)
Claustrum
Globus pallidus (GP)
Ventral striatum
Ventral pallidum
Middle cerebral
artery
Supraoptic recess
Optic chiasm (OpCh)
Infundibulum (Inf)
Uncus
BCorCl
Sep
HCaNu
CorRad
Sep
ALIntCap
ColFor
CorRad
In
Put
GP
GP
Cl
AC
AC
AC
Optic
Third
tract
ventricle
OpCh
Inf
ExtrmCap
4-2
The rostral surface of a coronal section of brain through the level
ages (both are inversion recovery) are at the same plane and show many
of the anterior commissure and the column of the fornix. The two MRI im-
of the structures identified in the brain slice.
66
Internal Morphology of the Brain in Slices and MRI
Body of corpus callosum (BCorCl)
Anterior tubercle of
thalamus (AntTub)
Head of caudate nucleus
(HCaNu)
Body of lateral
ventricle (BLatVen)
Internal capsule (IntCap)
Corona radiata (CorRad)
Stria terminalis and
terminal vein
Putamen (Put)
Insula (In)
External capsule
Globus pallidus (GP)
(ExtCap)
Insula (In)
Claustrum (Cl)
Extreme capsule
Column of fornix
(ExtrmCap)
(ColFor)
Third ventricle (ThrVen)
Column of fornix
(ColFor)
Amygdaloid nuclear
complex (AmyNu)
Amygdaloid nuclear
complex (AmyNu)
Hypothalamus
Optic Tract (OpTr)
BCorCl
BLatVen
Body of
HCaNu
fornix
CorRad
IntCap
AntTub
Anterior
ExtCap
nucleus
Put
In
Ventral
ThrVen
anterior
GP
nucleus
OpTr
OpTr
Hyth
AmyNu
Hippo-
ThrVen
campus
4-3
The rostral surface of a coronal section of brain through the level
quately expose the anterior tubercles of the thalamus. The terminal vein
of the anterior tubercle of the thalamus and the column of the fornix just cau-
is also called the superior thalamostriate vein. The two MRI images
dal to the anterior commissure. Portions of the columns of the fornix
(both are inversion recovery) are at the same plane and show many of
and the septum (drawn in as black lines) were removed to more ade-
the structures identified in the brain slice.
Coronal Brain Slice—MRI Correlation
67
Body of corpus callosum (BCorCl)
Body of lateral ventricle (BLatVen)
Septum
Body of fornix (BFor)
Body of caudate nucleus
(BCaNu)
Anterior nucleus
Ventral anterior nucleus
of thalamus (AntNu)
of thalamus (VA)
Stria terminalis and
terminal vein
Posterior limb of
internal capsule (PLIntCap)
Corona radiata (CorRad)
Ventral anterior nucleus
of thalamus (VA)
Putamen (Put)
Mammillothalamic
tract
Insula (In)
External capsule
(ExtCap)
Globus pallidus (GP)
Claustrum
Extreme capsule
Third ventricle (ThrVen)
(ExtrmCap)
Dorsomedial nucleus
Amygdaloid nuclear
of thalamus
complex (AmyNu)
Mammillary body (MB)
Optic Tract
Hippocampal formation
Inferior horn of lateral
ventricle (IHLatVen)
Oculomotor nerve
Hippocampal formation (Hip)
Interpeduncular fossa
Posterior cerebral artery
Basilar pons (BP)
BLatVen
BCaNu
BCorCl
BCorCl
AntNu
BFor
BLatVen
AntNu
VA
BFor
VA
PLintCap
PLIntCap
Thalamus
Put
Put
In
IHLatVen
Hip
GP
Hip
MB
IHLatVen
BP
4-4
The rostral surface of a section of brain through the anterior nu-
pallidus is clearly divided into its lateral and medial segments in the brain
cleus of the thalamus, mammillothalamic tract, and mammillary bodies. The
slice. Additionally, the terminal vein is also called the superior thala-
two MRI images (both are inversion recovery) are at the same plane and
mostriate vein.
show many of the structures identified in the brain slice.The globus
68
Internal Morphology of the Brain in Slices and MRI
Body of corpus callosum (BCorCl)
Body of fornix (BFor)
Body of lateral ventricle (BLatVen)
Stria terminalis and
terminal vein
Body of caudate nucleus
(BCaNu)
Corona radiata (CorRad)
Dorsomedial nucleus
of thalamus (DMNu)
Third ventricle and
Ventral lateral nucleus
massa intermedia
of thalamus (VL)
Posterior limb of internal
capsule (PLIntCap)
External capsule
Putamen (Put)
Claustrum
Insula (In)
Internal medullary
Extreme capsule
lamina (IML)
Globus pallidus (GP)
Subthalamic nucleus
Third ventricle (ThrVen)
Red nucleus
Tail of caudate nucleus
Substantia nigra (SN)
Inferior horn of
lateral ventricle
Hippocampal formation (Hip)
Interpeduncular fossa (IPF)
Optic tract
Basilar pons (BP)
Crus cerebri (CC)
Corticospinal fibers
BFor
BCorCl
BFor
BCaNu
DMNu
ThrVen
IML
VL
ThrVen
PLIntCap
In
Put
Put
SN
GP
CC
Hip
IPF
SN
IPF
BP
4-5
The rostral surface of a coronal section of brain through caudal
are at the same plane and show many of the structures identified in the
parts of the ventral lateral nucleus, the massa intermedia, the subthalamic nu-
brain slice. The terminal vein is also called the superior thalamostriate
cleus, and basilar pons. The two MRI images (both are inversion recovery)
vein.
Coronal Brain Slice—MRI Correlation
69
Body of corpus callosum (BCorCl)
Body of fornix (BFor)
Body of lateral ventricle (BLatVen)
Body of caudate nucleus
(BCaNu)
Stria terminalis and
terminal vein
Corona radiata
Lateral dorsal nucleus
Dorsomedial nucleus
of thalamus
of thalamus (DMNu)
Ventral posterolateral
Internal medullary
nucleus of thalamus (VPL)
lamina (IML)
External capsule
Centromedian nucleus
of thalamus
Claustrum
Insula
Putamen (Put)
Ventral posteromedial
nucleus of thalamus
Posterior limb
of internal
Tail caudate nucleus
capsule (PLIntCap)
Inferior horn of lateral
Optic tract
ventricle (IHLatVen)
Hippocampal formation
Third ventricle
(Hip)
Interpeduncular fossa (IPF)
Red nucleus (RNu)
Crus Cerebri (CC)
Trigeminal nerve (TriNr)
Substantia nigra
Basilar pons
BFor
BCorCl
BLatVen
IML
BCaNu
DMNu
PLIntCap
VPL
RNu
Put
Hip
CC
TriNr
TriNr
IPF
BP
4-6
The rostral surface of a coronal section of brain through the lat-
is at the same plane and shows many of the structures identified in the
eral dorsal and centromedian nuclei, rostral midbrain (red nucleus), and cor-
brain slice. The terminal vein is also called the superior thalamostriate
ticospinal fibers in the basilar pons. The MRI image (inversion recovery)
vein.
70
Internal Morphology of the Brain in Slices and MRI
Body of corpus callosum (BCorCl)
Body of fornix (BFor)
Body of lateral ventricle (BLatVen)
Fimbria of fornix (FFor)
Body of caudate nucleus
Stria terminalis and
(BCaNu)
terminal vein
Pulvinar (Pul)
Pulvinar (Pul)
Retrolenticular limb of
internal capsule
Medial geniculate
nucleus (MGNu)
Posterior commissure
Lateral geniculate
nucleus (LGNu)
Tail of caudate
nucleus
Inferior horn of lateral
ventricle (HLatVen)
Lateral geniculate
Hippocampal
nucleus (LGNu)
formation (Hip)
Cerebral aqueduct (CA)
Pretectal area (PrTecAr)
Periaqueductal gray
Decussation of superior
cerebellar peduncle
Middle cerebellar peduncle (MCP)
Pyramid
BCorCl
BLatVen
BCaNu
FFor
BFor
Pul
Pul
Pul
MGNu
PrTecAr
LGNu
LGNu
IHLatVen
Hip
MGNu
Basilar
Basilar
pons
pons
CA
Trigeminal nerve
4-7
The rostral surface of a coronal section of brain through the pul-
same plane and show many of the structures in the brain slices. The
vinar, medial, and lateral geniculate nuclei, the basilar pons, and middle cere-
terminal vein is also called the superior thalamostriate vein. For details
bellar peduncle. The two MRI images (both inversion recovery) are at the
of the cerebellum see Figures 2-31 to 2-33 on pp. 32 and 33.
Coronal Brain Slice—MRI Correlation
71
Body of corpus callosum (BCorCl)
Crus of fornix (CrF)
Fimbria of fornix (FFor)
Body of lateral ventricle (BLatVen)
Pulvinar (Pul)
Body of caudate
nucleus (BCaNu)
Retrolenticular limb
of internal capsule
Fimbria of hippocampus
Stria terminalis and
terminal vein
Pineal
Hippocampal
Tail caudate nucleus
formation (Hip)
Inferior horn of lateral
ventricle (IHLatVen)
Superior colliculus (SC)
Cerebral aqueduct (CA)
Periaqueductal gray (Pag)
Flocculus
Medulla (Med)
Middle cerebellar peduncle (MCP)
FFor
Splenium
BCorCl
of corpus
callosum
Pul
BLatVen
BLatVen
SC
IHLatVen
SC
Hip
Hip
CA
Pag
Pag
MCP
Med
MCP
Pyramid
4-8
The rostral surface of a coronal section of brain through the pul-
tures identified in the brain slice. The terminal vein is also called the
vinar nucleus, the superior colliculus, the middle cerebellar peduncle, and the
superior thalamostriate vein. For details of the cerebellum see Figures
rostral portion of the medulla oblongata. The two MRI images (both are
2-31 to 2-33 on pp. 32 and 33.
inversion recovery) are at the same plane and show many of the struc-
72
Internal Morphology of the Brain in Slices and MRI
Splenium of corpus
callosum (SpCorCl)
Towards posterior horn of
lateral ventricle (PHLatVen)
Hippocampal
commissure
(HipCom)
Crus of fornix (CrFor)
Hippocampal
formation
Optic Radiations
(OpRad)
Towards inferior horn
of lateral ventricle
(IHLatVen)
Tapetum (Tap)
Inferior colliculus (IC)
Trochlear nerve
Superior cerebellar
peduncle (SCP)
Middle cerebellar peduncle (MCP)
Fourth ventricle (ForVen)
Restiform body
Medulla (Med)
Pyramid (Py)
CrFor
PHLat
SpCorCl
Ven
HipCom
Tap
Hip
IHLatVen
IC
OpRad
SCP
IC
MCP
Med
ForVen
Py
4-9
The rostral surface of a coronal section of brain through the sple-
of the lateral ventricles. The two MRI images (both are inversion re-
nium of corpus callosum, the inferior colliculus, the middle cerebellar pe-
covery) are at the same plane and show many of the structures identi-
duncle in the base of the cerebellum, and the rostral portion of the
fied in the brain slice. For details of the cerebellum see Figures 2-31 to
medulla oblongata. The plane of the section is also through the atrium
2-33 on pp. 32 and 33.
Internal Morphology
of the Brain in
Slices and MRI
Brain Slices in the Axial Plane with MRI
Orientation to Axial MRIs: When looking at an axial MRI
using MRI (or CT) in the diagnosis of the neurologically impaired
image, you are viewing the image as if standing at the patient’s
patient.
feet and looking toward his or her head while the patient is lying
To reinforce this concept, the ventral surface of each axial slice
on his or her back. Consequently, and as is the case in coronal im-
was photographed. So, when looking at the slice, the observer’s
ages, the observer’s right is the left side of the brain in the MRI
right is the left side of the brain slice. This view of the slice cor-
and the left side of the patient’s brain. It is absolutely essential to
relates exactly with the orientation of the brain as seen in the ac-
have a clear understanding of this right-versus-left concept when
companying axial MRIs.
74
Internal Morphology of the Brain in Slices and MRI
Body of corpus callosum
(toward the genu)
Corona radiata (CorRad)
Caudate nucleus
(CaNu)
Stria terminalis and
terminal vein
Corpus callsoum (CorCl)
Body of lateral
ventricle (BLatVen)
Body of corpus callosum
(toward the splenium)
Anterior
forceps
CorCl
CaNu
CorCl
CorRad
BLatVen
Septum
pellucidum
Posterior
forceps
4-10
Ventral surface of an axial section of brain through dorsal por-
sion recovery) are at a similar plane and show some of the structures
tions of corpus callosum. The plane of the section just touches the upper
identified in the brain slice. The terminal vein is also called the superior
portion of the body of caudate nucleus. The two MRI images (both inver-
thalamostriate vein.
Axial Brain Slice—MRI Correlation
75
Cingulate gyrus
Anterior cerebral arteries
Genu of corpus
calllosum
Head of caudate
nucleus (HCaNu)
Anterior horn of lateral
ventricle (AHLatVen)
Stria terminalis and
terminal vein
Body of fornix
Anterior nucleus of
Anterior tubercle
thalamus
Corona radiata (CorRad)
Ventral anterior
nucleus of thalamus
Lateral thalamic nuclei
Tail of caudate
nucleus
Dorsomedial nucleus
of thalamus
Lateral ventricle (LatVen)
Tail of caudate nucleus
Crus of fornix
Splenium of corpus callosum
AHLatVen
Caudate
nucleus
LatVen
Putamen
HCaNu
CorRad
Internal
capsule
Septum
pellucidum
Dorsal
thalamus
Atrium of
lateral
ventricle
4-11
Ventral surface of an axial section of brain through the splenium
sion recovery—left; T2-weighted—right) are at a comparable plane
of corpus callosum and the head of the caudate nucleus. This plane includes
and show some of the structures identified in the brain slice. The ter-
only a small portion of the dorsal thalamus. The two MRI images (inver-
minal vein is also called the superior thalamostriate vein.
76
Internal Morphology of the Brain in Slices and MRI
Anterior horn of lateral
Genu of corpus callosum
ventricle (AHLatVen)
Septum pellucidum
Stria terminalis and
terminal vein
Head of caudate nucleus (HCaNu)
Genu of internal
Column of fornix (ColFor)
capsule (GIntCap)
Anterior limb of
Anterior nucleus of thalamus
internal capsule (ALIntCap)
Ventral anterior nucleus
Putamen (Put)
of thalamus
Choroid plexus in third ventricle
Globus pallidus (GP)
Insula (In)
Claustrum
Third ventricle (ThrVen)
Posterior limb of internal
capsule (PLIntCap)
Internal medullary lamina
Dorsomedial nucleus
Ventral lateral nucleus
of thalamus (DMNu)
of thalamus
Habenular nucleus
Centromedian nucleus
(Hab)
of thalamus
Ventral posterolateral
nucleus of thalamus
Retrolenticular limb of
internal capsule (RLIntCap)
Tail of caudate nucleus
Fimbria of hippocampus
Pulvinar (Pul)
Hippocampal formation (Hip)
Atrium of lateral
ventricle (ALatVen)
Crus of fornix
Optic radiations
Tapetum
Splenium of corpus callosum (SpCorCl)
Posterior horn of lateral ventricle (PHLatVen)
ALIntCap
AHLatVen
HCaNu
ALIntCap
GIntCap
Put
GIntCap
PLIntCapr
ColFor
PLIntCap
ThrVen
Lateral
GP
thalamic
Lateral
nuclei
thalamic
nuclei
DMNu
RLIntCap
ALatVen
RLIntCap
Pul
Hab
PHLatVen
Hip
ThrVen
SpCorCl
4-12
Ventral surface of an axial section of brain through the genu of
structures identified in the brain slice. The arrowheads in the brain slice
the corpus callosum, head of caudate nucleus, centromedian nucleus, and dor-
and in the MRIs are pointing to the mammillothalamic tract. The ter-
sal portions of the pulvinar. The two MRI images (inversion recovery—
minal vein is also called the superior thalamostriate vein.
left; T2-weighted—right) are at the same plane and show many of the
Axial Brain Slice—MRI Correlation
77
Genu of corpus callosum
Anterior commissure (AC)
Subcallosal gyri
Head of caudate nucleus (HCaNu)
Anterior limb of internal
capsule (ALIntCap)
Column of fornix (ColFor)
Third ventricle
Putamen (Put)
Mammillothalamic tract (MtTr)
Globus pallidus (GP)
External capsule
Insula (In)
Claustrum
Posterior limb of internal
Extreme capsule
capsule (PLIntCap)
Ventral posterior
thalamic nuclei
Subthalamic nucleus
Brachium of superior
colliculus
Red nucleus (RNu)
Pulvinar
Lateral geniculate
nucleus (LGNu)
Tail of caudate nucleus
Retrolenticular limb of
internal capsule
Tapetum (Tap)
Fimbria of hippocampus
Optic radiation (OpRad)
Hippocampal formation (Hip)
Atrium of lateral ventricle
(ALatVen)
Medial geniculate
nucleus (MGNu)
Posterior horn of lateral ventricle
(PHLatVen)
Pineal
Superior colliculus (SC)
Splenium of corpus callosum (SpCorCl)
AC
HCaNu
ColFor
ALIntCap
In
ColFor
Put
AC
GP
MtTr
MtTr
PLIntCap
RNu
LGNu
Massa
Lateral
intermed.
MGNu
thalamic
nuclei
Pul
Tap
Hip
Dorsomedial
nucleus
SC
ALatVen
Crus of
fornix
PHLatVen
OpRad
SpCorCl
4-13
Ventral surface of an axial section of brain through the anterior
can be discerned on the right side of the brain. The MRI images (both
commissure, column of fornix, medial and lateral geniculate nuclei, and supe-
T2-weighted) are at approximately the same plane and show many of
rior colliculus. The medial and lateral segments of the globus pallidus are vis-
the structures identified in the brain slice.
ible on the slice. The lateral and medial segments of the globus pallidus
78
Internal Morphology of the Brain in Slices and MRI
Hypothalamus (HyTh)
Anterior cerebral arteries (ACA)
Lamina terminalis
Head of
caudate nucleus
Third ventricle (ThrVen)
Nucleus accumbens
Optic tract (OpTr)
Anterior perforated
substance
Uncus
Crus cerebri (CC)
Amygdaloid
nuclear complex
Inferior horn of
lateral ventricle
(IHLatVen)
Mammillary body (MB)
Interpeduncular
Hippocampal
fossa (IPF)
formation
Lateral geniculate
nucleus
Substantia
nigra (SN)
Tail of caudate nucleus
Hippocampal formation (Hip)
Decussation of superior
cerebellar peduncle
Choroid plexus in
inferior horn
Inferior colliculus (IC)
Periaqueductal gray
Cerebellum (Cbl)
Cerebral Aqueduct (CA)
ACA
OpTr
ThrVen
HyTh
ThrVen
Un
MB
CC
SN
IPF
IHLatVen
Hip
CA
IC
Posterior
cerebral
artery
Posterior horn
lateral ventricle
Cbl
4-14
Ventral surface of an axial section of brain through the hypo-
planes and show many of the structures identified in the brain slice. For
thalamus, mammillary body, crus cerebri, and inferior colliculus. The two MRI
details of the cerebellum see Figures 2-31 to 2-33 on pp. 32 and 33.
images (inversion recovery—left; T2-weighted—right) are at similar
Axial Brain Slice—MRI Correlation
79
Basilar artery (BA)
Corticospinal fibers
Medial lemniscus (ML)
Basilar pons (BP)
Tegmentum of pons (TegP)
Fourth ventricle (ForVen)
Nucleus coeruleus
Superior cerebellar
peduncle (SCP)
Vermis of anterior lobe
Hemisphere of anterior lobe
of cerebellum (VCbl)
of cerebellum (HCbl)
Hemisphere of posterior lobe
of cerebellum
BA
Temporal
lobe
BP
ML
HCbl
TegP
HCbl
VCbl
SCP
VCbl
ForVen
Occipital
Occipital
lobe
lobe
4-15
Ventral surface of an axial section of brain through rostral
plane and show many of the structures identified in the brain slice. For
parts of the basilar pons and the anterior lobe of the cerebellum. The two MRI
details of the cerebellum see Figures 2-31 to 2-33 on pp. 32 and 33.
images (T2-weighted—left; inversion recovery—right) are at the same
80
Internal Morphology of the Brain in Slices and MRI
Basilar artery (BA)
Basilar pons (BP)
Trigeminal nerve (TriNr)
Medial lemniscus
Tegmentum of pons (TegP)
Middle cerebellar
peduncle (MCP)
Fourth ventricle (ForVen)
Dentate nucleus (DNu)
Vermis of posterior lobe
Hemisphere of posterior lobe
of cerebellum (VCbl)
of cerebellum (HCbl)
Temporal
lobe
BP
BA
Trigeminal
TriNr
ganglion
Temporal lobe
TriNr
MCP
ForVen
Lesion
BP
in BP
TegP
HCbl
TegP
ForVen
BA
BP
TegP
MCP
ForVen
DNu
VCbl
4-16
Ventral surface of an axial section of brain through the middle
lower) are at the same planes and show many of the structures identi-
regions of the basilar pons, the exit of the trigeminal nerve, the fourth ventri-
fied in the brain slice. Note the lesion in the basilar pons (upper right).
cle, and the cerebellar nuclei. The three MRI images (inverted inversion
For details of the cerebellum see Figures 2-31 to 2-33 on pp. 32
recovery—upper left; T2-weighted—upper right; T1-weighted—
and 33.
Axial Brain Slice—MRI Correlation
81
Basilar artery
Basilar pons
Anterior median fissure (AMF)
Pyramid (Py)
Preolivary sulcus (PreOIS)
Olivary eminence (OlEm)
Vestibulocochlear nerve
Vagus and glossopharyngeal
nerves
Retroolivary sulcus
(Postolivary sulcus) (PoOIS)
Restiform body (RB)
Medial lemniscus
Tonsil of cerebellum (TCbl)
Hemisphere of posterior lobe
Fourth ventricle (ForVen)
of cerebellum (HCbl)
Vermis of posterior lobe
of cerebellum (VCbl)
AMF
Py
PreOlS
OlEm
PoOlS
RB
TCbl
TCbl
ForVen
HCbl
VCbl
OlEm
Lesion-Lateral
medullary syndrome
RB
ForVen
4-17
Ventral surface of an axial section of brain through portions of
show many of the structures identified in the brain slice. Note the lateral
the medulla oblongata, just caudal to the pons-medulla junction and the
medullary lesion (lower), also known as the posterior inferior artery
posterior lobe of the cerebellum. The three MRI images (T1-weighted—
syndrome or the lateral medullary syndrome (of Wallenberg). For de-
upper left and right; T2-weighted—lower) are at the same plane and
tails of the cerebellum see Figures 2-31 to 2-33 on pp. 32 and 33.
CHAPTER
5
Internal Morphology of the Spinal Cord
and Brain in Stained Sections
Basic concepts that are essential when one is initially learning how
and the lateral corticospinal tract (grey). In the brainstem, these
to diagnose the neurologically impaired patient include 1) an un-
spinal tracts are joined by the spinal trigeminal tract and ventral
derstanding of cranial nerve nuclei and 2) how these structures re-
trigeminothalamic fibers (both are light green). The long tracts are
late to long tracts. The importance of these relationships is clearly
color-coded on one side only, to emphasize 1) laterality of function
seen in the combinations of deficits that generally characterize le-
and dysfunction, 2) points at which fibers in these tracts may de-
sions at different levels of the neuraxis. First, deficits of only the
cussate, and 3) the relationship of these tracts to cranial nerves.
body that may present as motor or sensory losses (long tracts) on
A color key appears on each page. This key identifies the var-
the same, or opposite, sides are indicative of spinal cord lesions
ious tracts and nuclei by their color and specifies the function of
(e.g., Brown-Sequard syndrome). Spinal cord injuries character-
each structure on each page. This approach not only emphasizes
istically have motor and sensory levels; these are the lowest functional
anatomical and clinical concepts, but also lends itself to a variety
levels remaining in the compromised patient. Second, cranial
of instructional settings.
nerve deficits (on one side of the head) in combination with long
Correlation of MRI and CT with Spinal Cord and
tract signs (on the opposite side of the body) characterize lesions
Brainstem: As one is learning basic anatomical concepts it is
in the brainstem (e.g., lateral medullary or Weber syndromes).
essential to consider how this information may be used in the
These patterns of loss are frequently called alternating or crossed
clinical environment. To this end, MRI (T1- and T2-weighted)
deficits. In these examples cranial nerve signs are better localizing
and CT (myelogram/cisternogram) images are introduced into
signs than are long tract signs. A localizing sign can be defined as an
the spinal cord and brainstem sections of this chapter (see also
objective neurologic abnormality that correlates with a lesion (or
Chapter 1). To show the relationship between basic anatomy and
lesions) at a specific neuroanatomical location (or locations).
how MRI and CT are viewed, a series of self-explanatory illus-
Third, motor and sensory deficits on the same side of the head and
trations are provided on each set of facing pages in these sections.
body are usually indicative of a lesion in the forebrain.
This continuum of visual information consists of (1) a small ver-
Color Coded Cranial Nerve Nuclei and Long Tracts:
sion of the colorized line drawing in an Anatomical Orientation,
Cranial nerve nuclei are coded by their function: pink, sensory;
(2) a top-to-bottom flip of this illustration that brings it into a
red, motor. These structures are colored bilaterally to make it easy
Clinical Orientation, and (3) a CT (spinal cord) or MRI and CT
to correlate cranial nerve and long tract function on both sides of
(brainstem) that follows this clinically oriented image. Every ef-
the midline. For example, one can easily correlate damage to the
fort is made to identify and use MRI and CT that correlate, as
hypoglossal nerve root and the adjacent corticospinal fibers on one
closely as possible, with their corresponding line drawing and
side while comparing this pattern with the clinical picture of a lat-
stained section. This approach recognizes and retains the
eral medullary syndrome on the other side.
strength of the anatomical approach, introduces essential clinical
Long tracts are color-coded beginning at the most caudal spinal
concepts while at the same time allowing the user to customize
cord levels (e.g., see Figures 5-1 and 5-2), with these colors ex-
the material to suit a range of educational applications.
tending into the dorsal thalamus (see Figure 5-30) and the posterior
limb of the internal capsule (see Figures 5-31 and 5-32). The col-
orized spinal tracts are the fasciculus gracilis (dark blue), the fasci-
*The dark and light blue colors represent information originating from lower
culus cuneatus (light blue)*, the anterolateral system (dark green),
and upper portions of the body, respectively.
84
Internal Morphology of the Spinal Cord and Brain in Stained Sections
The Spinal Cord With CT
85
86
Internal Morphology of the Spinal Cord and Brain in Stained Sections
The Spinal Cord With CT
87
88
Internal Morphology of the Spinal Cord and Brain in Stained Sections
The Spinal Cord With CT
89
90
Internal Morphology of the Spinal Cord and Brain in Stained Sections
The Spinal Cord With CT
91
92
Internal Morphology of the Spinal Cord and Brain in Stained Sections
The Spinal Cord With CT
93
94
Internal Morphology of the Spinal Cord and Brain in Stained Sections
Arterial Patterns Within The Spinal Cord With Vascular Syndromes
95
96
Internal Morphology of the Spinal Cord and Brain in Stained Sections
Degenerated Corticospinal Tract
97
98
Internal Morphology of the Spinal Cord and Brain in Stained Sections
The Medulla Oblongata With MRI and CT
99
100
Internal Morphology of the Spinal Cord and Brain in Stained Sections
The Medulla Oblongata With MRI and CT
101
102
Internal Morphology of the Spinal Cord and Brain in Stained Sections
The Medulla Oblongata With MRI and CT
103
104
Internal Morphology of the Spinal Cord and Brain in Stained Sections
The Medulla Oblongata With MRI and CT
105
106
Internal Morphology of the Spinal Cord and Brain in Stained Sections
The Medulla Oblongata With MRI and CT
107
108
Internal Morphology of the Spinal Cord and Brain in Stained Sections
The Medulla Oblongata With MRI and CT
109
110
Internal Morphology of the Spinal Cord and Brain in Stained Sections
Arterial Patterns Within The Medulla Oblongata With Vascular Syndromes
111
112
Internal Morphology of the Spinal Cord and Brain in Stained Sections
The Cerebellar Nuclei
113
114
Internal Morphology of the Spinal Cord and Brain in Stained Sections
The Cerebellar Nuclei
115
116
Internal Morphology of the Spinal Cord and Brain in Stained Sections
The Pons With MRI and CT
117
118
Internal Morphology of the Spinal Cord and Brain in Stained Sections
The Pons With MRI and CT
119
120
Internal Morphology of the Spinal Cord and Brain in Stained Sections
The Pons With MRI and CT
121
122
Internal Morphology of the Spinal Cord and Brain in Stained Sections
The Pons With MRI and CT
123
124
Internal Morphology of the Spinal Cord and Brain in Stained Sections
Arterial Patterns Within The Pons With Vascular Syndrome
125
126
Internal Morphology of the Spinal Cord and Brain in Stained Sections
The Midbrain With MRI and CT
127
128
Internal Morphology of the Spinal Cord and Brain in Stained Sections
The Midbrain With MRI and CT
129
130
Internal Morphology of the Spinal Cord and Brain in Stained Sections
The Midbrain With MRI and CT
131
132
Internal Morphology of the Spinal Cord and Brain in Stained Sections
The Midbrain With MRI and CT
133
134
Internal Morphology of the Spinal Cord and Brain in Stained Sections
The Midbrain With MRI and CT
135
136
Internal Morphology of the Spinal Cord and Brain in Stained Sections
Arterial Patterns Within The Midbrain With Vascular Syndromes
137
138
Internal Morphology of the Spinal Cord and Brain in Stained Sections
The Diencephalon and Basal Nuclei With MRI
139
140
Internal Morphology of the Spinal Cord and Brain in Stained Sections
The Diencephalon and Basal Nuclei With MRI
141
142
Internal Morphology of the Spinal Cord and Brain in Stained Sections
The Diencephalon and Basal Nuclei With MRI
143
144
Internal Morphology of the Spinal Cord and Brain in Stained Sections
The Diencephalon and Basal Nuclei With MRI
145
146
Internal Morphology of the Spinal Cord and Brain in Stained Sections
The Diencephalon and Basal Nuclei With MRI
147
148
Internal Morphology of the Spinal Cord and Brain in Stained Sections
The Diencephalon and Basal Nuclei With MRI
149
150
Internal Morphology of the Spinal Cord and Brain in Stained Sections
The Diencephalon and Basal Nuclei With MRI
151
152
Internal Morphology of the Spinal Cord and Brain in Stained Sections
The Diencephalon and Basal Nuclei With MRI
153
154
Internal Morphology of the Spinal Cord and Brain in Stained Sections
The Diencephalon and Basal Nuclei With MRI
155
156
Internal Morphology of the Spinal Cord and Brain in Stained Sections
The Diencephalon and Basal Nuclei With MRI
157
158
Internal Morphology of the Spinal Cord and Brain in Stained Sections
Arterial Patterns Within the Forebrain With Vascular Syndromes
159
CHAPTER
6
Internal Morphology of the Brain in Stained
Sections: Axial-Sagittal Correlations with MRI
Although the general organization of Chapter 6 has been de-
in this chapter gives the reader an opportunity to compare in-
scribed in Chapter 1 (the reader may wish to refer back to this
ternal brain anatomy, as seen in stained sections, with those
section), it is appropriate to reiterate its unique features at this
structures as visualized in clinical images generated in the same
point. Each set of facing pages has photographs of an axial stained
plane. Even a general comparison reveals that many features, as
section (left-hand page) and a sagittal stained section (right-hand
seen in the stained section, can be readily identified in the adja-
page). In addition to individually labeled structures, a heavy line
cent MRI.
appears on each photograph. This prominent line on the axial
This chapter is also organized so that one can view structures in
section represents the approximate plane of the sagittal section
either the axial or the sagittal plane only. Axial photographs appear
located on the facing page. On the sagittal section this line signi-
on left-hand pages and are sequenced from dorsal to ventral (odd-
fies the approximate plane of the corresponding axial section.
numbered Figures 6-1 through 6-9), while sagittal photographs are
The reader can identify features in each photograph and then, us-
on the right-hand pages and progress from medial to lateral (even-
ing this line as a reference point, visualize structures that are lo-
numbered Figures 6-2 through 6-10). Consequently, the user can
cated either above or below that plane (axial to sagittal compar-
identify and follow structures through an axial series by simply flip-
ison) or medial or lateral to that plane
(sagittal to axial
ping through the left-hand pages or through a sagittal series by flip-
comparison). This method of presentation provides a format for
ping through the right-hand pages. The inherent flexibility in this
reconstructing and understanding three-dimensional relation-
chapter should prove useful in a wide variety of instructional/
ships within the central nervous system.
learning situations. The drawings shown in the following illustrate
The magnetic resonance image (MRI) placed on every page the axial and sagittal planes of the photographs in this chapter.
Fig. 6-6
Fig. 6-4
Fig. 6-8
Fig. 6-2
Fig. 6-10
Axial Planes
Fig. 6-1
Fig. 6-3
Fig. 6-5
Sagittal
Fig. 6-7
Planes
Fig. 6-9
162
Internal Morphology of the Brain in Stained Sections
CorCl
Anterior horn of
lateral ventricle
Sep
CaNu,H
Put
For
VA
AntNu
IntCap:
AL
VL
G
DMNu
Pl
CM
Cl
Hab
VPL
PulNu
StTer
CaNu,T
Hip
CP
Com
Hip
Hip,F
OpRad
Atrium of
lateral ventricle
6-1
Axial section through the head of the caudate nucleus and several
shown in Figure 6-2 (facing page). Many of the structures labeled in
key thalamic nuclei
(anterior, centromedian, pulvinar, habenular). The this photograph can be clearly identified in the adjacent T1-weighted
heavy line represents the approximate plane of the sagittal section MRI.
Abbreviations
AntNu Anterior nucleus of thalamus
HipCom Hippocampal commissure
CaNu,H Caudate nucleus, head
IntCap,AL Internal capsule, anterior limb
CaNu,T Caudate nucleus, tail
IntCap,G Internal capsule, genu
CI Claustrum
IntCap,PL Internal capsule, posterior limb
CM Centromedial nucleus of thalamus
OpRad Optic radiations
CorCI Corpus callosum
PulNu Pulvinar nuclear complex
CP Choroid plexus
Put Putamen
DMNu Dorsomedial nucleus of thalamus
Sep Septum pellucidum
For Fornix, column
StTer Stria terminalis
Hab Habenular nucleus
VA Ventral anterior nucleus of thalamus
Hip Hippocampal formation
VL Ventral lateral nucleus of thalamus
Hip,F Hippocampus, fimbria
VPL Ventral posterolateral nucleus
Axial-Sagittal Correlations
163
AntNu
For,B
LDNu
CorCl,Spl
CorCl,G
DMNu
PrTecNu
SMT
Hab
SC
AC
PoCom
IC
RNu
For,Col
HyTh
TroNr
MtTr
MLF
MB
OcNr
FNu
OpNr
SCP,Dec
AbdNu
BP
Py
ML
NuGr
PO
HyNu
LCSp
6-2
Sagittal section through the column of the fornix, anterior thalamic
represents the approximate plane of the axial section shown in Figure
nucleus, red nucleus, and medial portions of the pons (abducens nucleus),
6-1 (facing page). Many of the structures labeled in this photograph can
cerebellum (fastigial nucleus), and medulla (nucleus gracilis). The heavy line
be clearly identified in the adjacent T1-weighted MRI.
Abbreviations
AbdNu Abducens nucleus
MB Mammillary body
AC Anterior commissure
ML Medial lemniscus
AntNu Anterior nucleus of thalamus
MLF Medial longitudinal fasciculus
BP Basilar pons
MtTr Mammillothalamic tract
CorCI,G Corpus callosum, genu
NuGr Nucleus gracilis
CorCI,Spl Corpus callosum, splenium
OcNr Oculomotor nerve
DMNu Dorsomedial nucleus of thalamus
OpNr Optic nerve
FNu Fastigial nucleus (medial cerebellar nucleus)
PO Principal olivary nucleus
For,B Fornix, body
PoCom Posterior commissure
For,Col Fornix, column
PrTecNu Pretectal nuclei
Hab Habenular nuclei
Py Pyramid
HyNu Hypoglossal nucleus
RNu Red nucleus
HyTh Hypothalamus
SC Superior colliculus
IC Inferior colliculus
SCP,Dec Superior cerebellar peduncle, decussation
LCsp Lateral corticospinal tract
SMT Stria medullaris thalami
LDNu Lateral dorsal nucleus
TroNr Trochlear nerve
164
Internal Morphology of the Brain in Stained Sections
Anterior horn of
lateral ventricle
Sep
CaNu,H
For,Col
GPL
Ins
Put
VA
MtTr
IntCap,AL
Cl
DMNu
ExtCap
VL
InTCap,PL
CM
VPM
VPL
HabCom
MGNu
SC,Br
StTer
SC
PulNu
CaNu, T
OpRad
Hip
Tap
6-3
Axial section through the head of the caudate nucleus, centrome-
ure 6-4 (facing page). Many of the structures labeled in this photograph
dian nucleus, medial geniculate body, and superior colliculus. The heavy line
can be clearly identified in the adjacent T2-weighted MRI.
represents the approximate plane of the sagittal section shown in Fig-
Abbreviations
CaNu,H Caudate nucleus, head
MGNu Medial geniculate nucleus
CaNu,T Caudate nucleus, tail
MtTr Mammillothalamic tract
Cl Claustrum
OpRad Optic radiations
CM Centromedian nucleus of thalamus
PulNu Pulvinar nuclear complex
DMNu Dorsomedial nucleus of thalamus
Put Putamen
ExtCap External capsule
SC Superior colliculus
For,Col Fornix, column
SC,Br Superior colliculus, brachium
Sep Septum pellucidum
StTer
Stria terminalis
GPL Globus pallidus, lateral segment
VA Ventral anterior nucleus of thalamus
Hab,Com Habenular commissure
VL Ventral lateral nucleus of thalamus
Hip Hippocampal formation
VPL Ventral posterolateral nucleus of thalamus
Ins Insula
VPM Ventral posteromedial nucleus of thalamus
IntCap,AL Internal capsule, anterior limb
Tap Tapetum
IntCap,PL Internal capsule, posterior limb
Axial-Sagittal Correlations
165
LatVen,AH
MtTr
CorCl,B
LDNu
For,B
CorCl, Spl
AntNu
DMNu
CM
PulNu
VA
AC
SC
H
ThFas
RNu
IC
Hyth
LenFas
AnLen
SN
OpTr
SCP
CC
OlfTr
ML
ForVen
BP
NuGr
FacNu
PO
SolNu & Tr
NuCu
6-4
Sagittal section through anterior and ventral anterior thalamic nu-
sents the approximate plane of the axial section shown in Figure 6-3
clei, red nucleus and central areas of the pons, cerebellum (and superior pe-
(facing page). Many of the structures labeled in this photograph can be
duncle), and medulla (solitary nuclei and tract). Note the position of the
clearly identified in the adjacent T1-weighted MRI.
facial motor nucleus at the pons-medulla junction. The heavy line repre-
Abbreviations
AC
Anterior commissure
LDNu Lateral dorsal nucleus
AnLen
Ansa lenticularis
ML Medial lemniscus
AntNu
Anterior nucleus of thalamus
MtTr Mammillothalamic tract
BP
Basilar pons
NuCu Nucleus cuneatus
CC
Crus cerebri
NuGr Nucleus gracilis
CM Centromedian nucleus
OlfTr Olfactory tract
CorCl,B
Corpus callosum, body
OpTr Optic tract
CorCl, Spl Corpus callosum, splenium
PO Principal olivary nucleus
DMNu Dorsomedial nucleus of thalamus
PulNu Pulvinar nuclear complex
FacNu
Facial nucleus
RNu Red nucleus
For,B
Fornix, body
SC Superior colliculus
ForVen
Fourth ventricle
SCP Superior cerebellar peduncle (brachium
H Prerubral field
conjunctivum)
HyTh
Hypothalamus
SN Substantia nigra
IC
Inferior colliculus
SolNu&Tr Solitary nuclei and tract
LatVen,AH Lateral ventricle, anterior horn
ThFas Thalamic fasciculus
LenFas
Lenticular fasciculus
VA Ventral anterior nucleus of thalamus
166
Internal Morphology of the Brain in Stained Sections
AC
CaNu,H
LT
GPL
Put
For,Col
GPM
Hyth
MtTr
IntCap,AL
Cl
VL
VPM
SC
IntCap:
CM VPL
PL
RL
MGNu
CeGy
SC
PulNu
Hip,F
OpRad
Hip
CP
ALV
6-5
Axial section through the head of the caudate nucleus, ventral post-
shown in Figure 6-6 (facing page). Many of the structures labeled in
eromedial nucleus, medial geniculate body, and ventral parts of the pulvinar.
this photograph can be clearly identified in the adjacent T1-weighted
The heavy line represents the approximate plane of the sagittal section
MRI.
Abbreviations
AC Anterior commissure
IntCap,AL Internal capsule, anterior limb
ALV Atrium of lateral ventricle
IntCap,Pl Internal capsule, posterior limb
CaNu,H Caudate nucleus, head
IntCap,RL Internal capsule, retrolenticular limb
CeGy Central gray (periaqueductal gray)
LT Lamina terminalis
CI Claustrum
MGNu Medial geniculate nucleus
CM Centromedian nucleus of thalamus
MtTr Mammillothalamic tract
CP Choroid plexus
OpRad Optic radiations
For,Col Fornix, column
PulNu Pulvinar nuclear complex
GPL Globus pallidus, lateral segment
Put Putamen
GPM Globus pallidus, medial segment
SC Superior colliculus
Hip Hippocampal formation
VL Ventral lateral nucleus of thalamus
Hip,F Hippocampus, fimbria
VPL Ventral posterolateral nucleus of thalamus
HyTh Hypothalamus
VPM Ventral posteromedial nucleus of thalamus
Axial-Sagittal Correlations
167
CorCl,G
LDNu
CorCl,Spl
DMNu
VL
PulNu
VA
CaNu,H
CM
SC
H
RNu
IC
LL
AC
SN
SCP
ENu
OpTr
CC
AnLen
SOpNu
LenFas
CSNu
ML
TriMoNu
FacNr
OCblF
NuCu
6-6
Sagittal section through central regions of the diencephalon (cen-
heavy line represents the approximate plane of the axial section shown
tromedian nucleus) and midbrain (red nucleus), and through lateral areas of
in Figure 6-5 (facing page). Many of the structures labeled in this pho-
the pons (trigeminal motor nucleus) and medulla (nucleus cuneatus). The
tograph can be clearly identified in the adjacent T1-weighted MRI.
Abbreviations
AC Anterior commissure
LL Lateral lemniscus
AnLen Ansa lenticularis
ML Medial lemniscus
CaNu,H Caudate nucleus, head
NuCu Nucleus cuneatus
CC Crus cerebri
OCblF Olivocerebellar fibers
CM Centromedian nucleus of thalamus
OpTr Optic tract
CorCl,G Corpus callosum, genu
PulNu Pulvinar nuclear complex
CorCl,Spl Corpus callosum, splenium
RNu Red nucleus
CSNu Chief (prinicipal) sensory nucleus of trigeminal nerve
SC Superior colliculus
DMNU Dorsomedial nucleus of thalamus
SCP Superior cerebellar peduncle (brachium con-
ENu Emboliform nucleus (anterior interposed cerebellar nucleus)
junctivum)
FacNr Facial nerve
SN Substantia nigra
H Field of Forel (prerubral field)
SOpNu Supraoptic nucleus
IC Inferior colliculus
TriMoNu Trigeminal motor nucleus
LenFas Lenticular fasciculus
VA Ventral anterior nucleus of thalamus
LDNu Lateral dorsal nucleus of thalamus
VL Ventral lateral nucleus of thalamus
168
Internal Morphology of the Brain in Stained Sections
CaNu
Ins
LT
HyTh
For
MtTr
AC
RNu
OpTr
LGNu
CC
ML
StTer
IC,Br
CaNu,T
MGNu
Hip
IC
OpRad
6-7
Axial section through the hypothalamus, red nucleus, inferior col-
the midbrain, represents a slightly oblique section through the mesen-
liculus, and lateral geniculate body. The heavy line represents the ap-
cephalon. The position of the lamina terminalis is indicated by the dou-
proximate plane of the sagittal section shown in Figure 6-8 (facing
ble-dashed lines. Many of the structures labeled in this photograph can
page). The axial plane through the hemisphere, when continued into
be clearly identified in the adjacent T1-weighted MRI.
Abbreviations
AC Anterior commissure
LGNu Lateral geniculate nucleus
CaNu Caudate nucleus
LT Lamina terminalis
CaNu,T Caudate nucleus, tail
MGNu Medial geniculate nucleus
CC Crus cerebri
ML Medial lemniscus
For Fornix
MtTr Mammillothalamic tract
Hip Hippocampal formation
OpRad Optic radiation (geniculocalcarine fibers)
HyTh Hypothalamus
OpTr Optic tract
IC Inferior colliculus
RNu Red nucleus
IC,Br Inferior colliculus, brachium
StTer Stria terminalis
Ins Insula
Axial-Sagittal Correlations
169
Zl
AC
ThFas
Lenfas
VPM
CaNu
BrSC
VL
PulNu
SThNu
GPL
GPM
MGNu
Put
SN
Hip
OpTr
AmyNu
CC
DNu
MCP
PCNu
6-8
Sagittal section through the caudate nucleus, central parts of the
proximate plane of the axial section shown in Figure 6-7 (facing page).
diencephalon (ventral posteromedial nucleus), and lateral portions of the
Many of the structures labeled in this photograph can be clearly iden-
pons and cerebellum (dentate nucleus). The heavy line represents the ap-
tified in the adjacent T1-weighted MRI.
Abbreviations
AC Anterior commissure
MGNu Medial geniculate nucleus
AmyNu Amygdaloid nucleus (complex)
OpTr Optic tract
BrSC Brachium of superior colliculus
PCNu Posterior cochlear nucleus
CaNu Caudate nucleus
PulNu Pulvinar nuclear complex
CC Crus cerebri
Put Putamen
DNu Dentate nucleus (lateral cerebellar nucleus)
SN Substantia nigra
GPL Globus pallidus, lateral segment
SThNu Subthalamic nucleus
GPM Globus pallidus, medial segment
ThFas Thalamic fasciculus
Hip Hippocampal formation
VL Ventral lateral nucleus of thalamus
LenFas Lenticular fasciculus
VPM Ventral posteromedial nucleus of thalamus
MCP Middle cerebellar peduncle (brachium pontis)
ZI Zona incerta
170
Internal Morphology of the Brain in Stained Sections
LT
SOR
OpTr
IR
HyTh
AmyNu
MB
CC
CP
SN
Hip
ML
CaNu,T
SCP,Dec
FHip
LatVen,IH
MLF
DenGy
LL
SCP
OpRad
6-9
Axial section through ventral portions of the hypothalamus
hemisphere, when continued into the midbrain, represents a slightly
(supraoptic recess and mammillary body) and forebrain (amygdaloid nu-
oblique section through the mesencephalon. Many of the structures la-
cleus), and through the superior cerebellar peduncle decussation in the mid-
beled in this photograph can be clearly identified in the adjacent T1-
brain. The heavy line represents the approximate plane of the sagittal
weighted MRI.
section shown in Figure 6-10 (facing page). The axial plane through the
Abbreviations
AmyNu Amygdaloid nucleus (complex)
LT Lamina terminalis
CaNu,T Caudate nucleus, tail
MB Mammillary body
CC Crus cerebri
ML Medial lemniscus
CP Choroid plexus
MLF Medial longitudinal fasciculus
DenGy Dentate gyrus
OpRad Optic radiations
FHip Fimbria of hippocampus
OpTr Optic tract
Hip Hippocampal formation
SCP Superior cerebellar peduncle (brachium
HyTh Hypothalamus
conjunctivum)
IR Infundibular recess of third ventricle
SCP,Dec Superior cerebellar peduncle, decussation
LatVen,lH Lateral ventricle, inferior (temporal) horn
SN Sustantia nigra
LL Lateral lemniscus
SOR Supraoptic recess of third ventricle
Axial-Sagittal Correlations
171
VL + VPL
CaNu,B
OpRad
EML + ThRetNu
OpTr
AC
PulNu
CalSul
ALV
Hip
GPL
LGNu
Put
GPM
FHip
CP
DenGy
Hip
DNu
AmyNu
LatVen,IH
6-10
Sagittal section through the putamen, amygdaloid nucleus, and
line represents the approximate plane of the axial section shown in Fig-
hippocampus and through the most lateral portions of the diencephalon
ure 6-9 (facing page). Many of the structures labeled in this photograph
(external medullary lamina and ventral posterolateral nucleus). The heavy
can be clearly identified in the adjacent T1-weighted MRI.
Abbreviations
AC Anterior commissure
GPM Globus pallidus, medial segment
ALV Atrium of lateral ventricle
Hip Hippocampal formation
AmyNu Amygdaloid nucleus (complex)
LatVen,lH Lateral ventricle, inferior (temporal) horn
CalSul Calcarine sulcus
LGNu Lateral geniculate nucleus
CaNu,B Caudate nucleus, body
OpRad Optic radiations
CP Choroid plexus
OpTr Optic tract
DenGy Dentate gyrus
PulNu Pulvinar nuclear complex
DNu Dentate nucleus
Put Putamen
EML External medullary lamina
ThRetNu Thalamic reticular nuclei
FHip Fimbria of hippocampus
VL Ventral lateral nucleus of thalamus
GPL Globus pallidus, lateral segment
VPL Ventral posterolateral nucleus of thalamus
CHAPTER
7
Synopsis of Functional Components,
Tracts, Pathways, and Systems
The study of regional neurobiology (brain structures in gross spec-
the CD that comes with this atlas; these are taken from the cur-
imens, in brain slices, in stained sections, and in MRI and CT) is
rent edition of Stedman’s Medical Dictionary. In this respect, the
the basis for the study of systems neurobiology (tracts, pathways,
full definitions are actually available in this book. Researching the
cranial nerves and their functions), which, in turn, is the basis for
full definition of a clinical term or phrase is a powerful and ef-
understanding and diagnosing the neurologically impaired pa-
fective learning tool. Also, each clinical term or phrase is avail-
tient. Building on the concepts learned in earlier chapters on ex-
able in any standard medical dictionary or comprehensive neu-
ternal and internal brain anatomy in specimens and in MRI and
rology text.
CT, on brain vascular patterns, and on the relationships of cra-
The layout of the drawings in this chapter clearly shows the
nial nerves with long tracts, this chapter explores systems neurobi-
laterality of the tract/pathway. That is, the relationship between
ology with a particular emphasis on clinical correlations.
the location of the cell of origin and the termination of the fibers
The format of each set of facing pages is designed to summa-
making up a tract/pathway or the projections of cranial nerve
rize, accurately and consisely, the relationships of a given tract
nuclei. This information is absolutely essential to understanding the po-
or pathway. This includes, but is not limited to, 1) the location
sition of a lesion and correlating this fact with the deficits seen in the
of the cells of origin for a given tract/pathway, 2) its entire
neurologically compromised patient. For example, is the deficit on
course throughout the neuraxis and cerebrum, 3) the location of
the same side as the lesion (ipsilateral), on the opposite side (con-
the decussation of these fibers, if applicable, 4) the neurotrans-
tralateral), or on both sides (bilateral)? The concept of laterality
mitters associated with the neurons comprising the tract/pathway,
is usually expressed as “right,” “left,” or “bilateral” in reference
5) a brief review of its blood supply, and 6) a summary of a num-
to the side of the deficit(s) when written on the patient’s chart.
ber of deficits seen as a result of lesions at various points in the
This chapter is designed to maximize the correlation between
tract/pathway. The structure of an atlas does not allow a detailed
structure and function, to provide a range of clinical examples
definition of each clinical term on the printed page. However,
for each tract/pathway, and to help the user develop a knowl-
the full definition of each clinical term or phrase is available on
edge base that can be easily integrated into the clinical setting.
174
Synopsis of Functional Components, Tracts, Pathways, and Systems
SSA
SL
SL
SVA
G
GVA
G
V
S
E
G
G
G
S
SG
S
G
E
S
S
V
V
V
V
S
S
A
E
E
E
AA
A
A
S
V
E
posterior
GSA
GVA
medial
lateral
SL
GVE
GSE
anterior
GSA
G
G
G
G
GVA
S
V
V
S
GVE
E
E
A
A
GSE
SL
7-1
A semidiagrammatic summary of the positions of functional
In the brainstem, however, there is a slight transposition of the SVE
components as seen in the developing neural tube (left) and in the
and GSA functional components. Embryologically, SVE cell groups ap-
spinal cord and brainstem of the adult (right). In the neural tube, the
pear between those associated with GSE and GVE components. As de-
alar plate and its associated GSA and GVA components are posterior
velopment progresses, however, SVE cell groups migrate (open ar-
(dorsal) to the sulcus limitans (SL) while the basal plate and its related
row) to anterolateral areas of the tegmentum. Cell groups associated
GVE and GSE components are anterior (ventral) to the SL. In the adult
with the GSA functional component are displaced from their postero-
spinal cord, this general posterior/anterior relationship is maintained,
lateral position in the developing brainstem by the newly acquired cell
although the neural canal (as central canal) is reduced and/or absent.
groups having SSA components (as well as other structures). Conse-
Two major changes occur in the transition from spinal cord to brain-
quently, structures associated with the GSA component are located
stem in the adult. First, as the central canal of the cervical cord enlarges
(open arrow) in more anterolateral and lateral areas of the brainstem.
into the fourth ventricle and the cerebellum develops, the posterior
The approximate border between motor and sensory regions of the
portion of the neural tube is rotated laterally. Consequently, in the
brainstem is represented by an oblique line drawn through the brain-
adult, the sulcus limitans is present in the brainstem with motor com-
stem beginning at the SL. The medial (from midline) to lateral posi-
ponents (adult derivatives of the basal plate) medial to it, and sensory
tions of the various functional components, as shown on the far right
components (adult derivatives of the alar plate) are located laterally.
of this figure, are taken from their representative diagrams of brain-
Second, in the brainstem, special functional components (SVE to mus-
stem and cord and are directly translatable to Figure 7-2 (facing page).
cles of pharyngeal arch origin; SVA taste and olfaction; SSA vestibular,
The color-coding of the components on this figure correlate with that
auditory, and visual systems) are intermingled with the rostral contin-
in Figure 7-2 on the facing page.
uation of the general functional components as found in the spinal cord.
Abbreviations
GSA General somatic afferent
SSA Special somatic afferent
GSE General somatic efferent
SVA Special visceral afferent
GVA General visceral afferent
SVE Special visceral efferent
GVE General visceral efferent
SL Sulcus limitans
Cranial nerves
Components of Cranial and Spinal Nerves
175
G
G
S
S
G
S
G
S
V
V
V
V
S
S
E E
E
A
A
A
A
Midbrain
2
1. Oculomotor nuc. (GSE)
2. Edinger-Westphal nuc. (GVE)
1
3. Trochlear nuc. (GSE)
4. Mesencephalic nuc. & tr.
4
of V (GSA)
3
Pons
5. Abducens nuc. (GSE)
9
6. Sup. salivatory nuc. (GVE)
7
SL
7. Motor trigeminal nuc. (SVE)
8. Motor facial nuc. (SVE)
10
9. Principle sensory nuc of V (GSA)
10. Spinal trigeminal nuc. (GSA)
(pars oralis)
5
8
S
6
L
a
Medulla oblongata
17
13
M
11. Hypoglossal nuc. (GSE)
12. Dorsal motor nuc. of vagus (GVE)
SP
13. Inf. salivatory nuc. (GVE)
14
14. Nuc. ambiguus (SVE)
15
15. Solitary nuc. and tr.
15a: gustatory nuc. (SVA)
15b: cardiorespiratory nuc (GVA)
11
16. Vestibular nuclei (SSA)
18
S = Sup; L = Lat; M = Med; Sp. = Spinal
17. Cochlear nuc. (SSA)
12
b
18. Spinal trigeminal nuc. (GSA)
SL
(pars interpolaris, pars caudalis)
16
20
SL
Spinal cord
Cervical
24
cord
19. Medial motor cell column (GSE)
21
20. Accessory nuc. (GSE)
21. Lateral motor cell columns (GSE)
22. Intermediolateral cell column (GVE)
23. Visceral afferent receptive areas (GVA)
24. Substantia gelatinosa, nucleus proprious
19
and associated GSA receptive areas
22
Thoracic
25. Sacral parasympathetics (GVE)
23
cord
21
Lumbosacral
cord
25
G
G
G
G
S
V
V
S
E
E
A
A
Spinal nerves
7-2
The medial to lateral positions of brainstem cranial nerve
continuous cell groups) from one division of the brainstem to the
and spinal cord nuclei as shown here are the same as in Figure 7-1.
next or from brainstem to spinal cord. The nucleus ambiguus is a
This diagrammatic posterior (dorsal) view shows 1) the relative po-
column of cells composed of distinct cell clusters interspersed with
sitions and names of specific cell groups and their associated func-
more diffusely arranged cells, much like a string of beads. Nuclei as-
tional components, 2) the approximate location of particular nuclei
sociated with cranial nerves I (olfaction, SVA) and II (optic, SSA) are
in their specific division of brainstem and/or spinal cord, and 3) the
not shown. The color-coding used on this figure correlates with that
rostrocaudal continuity of cell columns (either as continuous or dis-
on Figure 7-1 (facing page).
176
Synopsis of Functional Components, Tracts, Pathways, and Systems
7-3
Orientation drawing for pathways. The trajectory of most
cospinal fibers are found in the internal capsule, crus cerebri, basilar
pathways illustrated in Chapter 7 appears on individualized versions of
pons, pyramid, and lateral corticospinal tract). Third, the location of ter-
this representation of the central nervous system (CNS). Although
minals containing specific neurotransmitters is indicated by the site(s) of
slight changes are made in each drawing, so as to more clearly diagram
termination of each tract (glutaminergic terminals of corticospinal fibers
a specific pathway, the basic configuration of the CNS is as represented
are located in the spinal cord gray matter). In addition, the action of most
here. This allows the user to move from pathway to pathway without
neuroactive substances is indicated as excitatory (
) or inhibitory (
).
being required to learn a different representation or drawing for each
This level of neurotransmitter information, as explained here for gluta-
pathway; also, laterality of the pathway, a feature essential to diagno-
minergic corticospinal fibers, is repeated for each pathway drawing.
sis (see introduction), is inherently evident in each illustration.
Clinical Correlations: The clinical correlations are designed to
The forebrain (telencephalon and diencephalon) is shown in the
give the user an overview of specific deficits (i.e., hemiplegia, athetosis)
coronal plane, and the midbrain, pons, medulla, and spinal cord are
seen in lesions of each pathway and to provide examples of some syn-
represented through their longitudinal axes. The internal capsule is
dromes or diseases (i.e., Brown-Sequard syndrome, Wilson disease) in
represented in the axial plane in an effort to show the rostrocaudal dis-
which these deficits are seen. Although purposefully brief, these cor-
tribution of fibers located therein.
relations highlight examples of deficits for each pathway and provide a
The reader should become familiar with the structures and regions
built-in mechanism for expanded study. For example, the words in
as shown here because their locations and relationships are easily trans-
italics in each correlation are clinical terms and phrases that are defined
ferable to subsequent illustrations. It may also be helpful to refer back
on the CD (from Stedman’s) included with this atlas or can be found
to this illustration when using subsequent sections of this chapter.
in standard medical dictionaries and clinical neuroscience textbooks.
Neurotransmitters: Three important facts are self-evident in the
Consulting these sources, especially the CD available in this atlas, will
descriptions of neurotransmitters that accompany each pathway draw-
significantly enhance understanding of the deficits seen in the neuro-
ing. These are illustrated by noting, as an example, that glutamate is
logically compromised patient. Expanded information, based on the
found in corticospinal fibers (see Figure 7-10). First, the location of neu-
deficits mentioned in this chapter, is integrated into some of the ques-
ronal cell bodies containing a specific transmitter is indicated (glutamate-
tions for chapter 7. Referring to such sources will allow the user to
containing cell bodies are found in cortical areas that project to the spinal
glean important clinical points that correlate with the pathway under
cord). Second, the trajectory of fibers containing a particular neurotrans-
consideration, and enlarge his or her knowledge and understanding by
mitter is obvious from the route taken by the tract (glutaminergic corti-
researching the italicized words and phrases.
Abbreviations
CE Cervical enlargement of spinal cord
IntCap,PL Internal capsule, posterior limb
Cer Cervical levels of spinal cord
LatSul Lateral sulcus (Sylvian sulcus)
CinSul Cingulate sulcus
LatVen Lateral ventricle
CaNu Caudate nucleus ( Put neostriatum)
LSE Lumbosacral enlargement of spinal cord
CM Centromedian (and intralaminar) nuclei
LumSac Lumbosacral level of spinal cord
CorCI Corpus callosum
L-VTh Lateral and ventral thalamic nuclei
Dien Diencephalon
excluding VPM and VPL
DMNu Dorsomedial nucleus of thalamus
Mes Mesencephalon
For Fornix
Met Metencephalon
GP Globus pallidus (paleostriatum)
Myelen Myelencephalon
GPl Globus pallidus, lateral segment
Put Putamen ( CaNu neostriatum)
GPm Globus pallidus, medial segment
SThNu Subthalamic nucleus
HyTh Hypothalamic area
Telen Telencephalon
IC Internal capsule
Thor Thoracic levels of spinal cord
IntCap,AL Internal capsule, anterior limb
VPL Ventral posterolateral nucleus of thalamus
IntCap,G Internal capsule, genu
VPM Ventral posteromedial nucleus of thalamus
Orientation
177
Midline
Cerebral cortex
CinSul
LatVen
Basal ganglia
CorCl
CaNu
For
Telen
&
DMNu
Dien
L-VTh
LatSul
CM
IC
IC
VPL
GPl
GPm
Put
VPM
HyTh
SThNu
Internal Capsule
Midbrain
IntCap, AL
Mes
Rostral
CaNu
Put
IntCap, G
GP
Met
Pons & Cerebellum
IntCap, PL
Dien
Caudal
Medulla
Myelen
Cer
CE
Spinal Cord
Thor
LSE
LumSac
Midline
178
Synopsis of Functional Components, Tracts, Pathways, and Systems
Posterior (Dorsal) Column-Medial Lemniscus System
7-4
The origin, course, and distribution of fibers composing the
used to describe deficits seen in lesions of the parietal cortex. Bilateral
posterior (dorsal) column (PC)-medial lemniscus (ML) system. This il-
damage (as in tabes dorsalis or subacute combined degeneration of the spinal
lustration shows the longitudinal extent, the positions in representa-
cord) produces bilateral losses. Although ataxia is the most common fea-
tive cross-sections of brainstem and spinal cord, and the somatotopy of
ture in patients with tabes dorsalis, they also have a loss of deep tendon
fibers in the PC and ML. The ML undergoes positional changes as it
reflexes, severe lancinating pain over the body below the head (more com-
courses from the myelencephalon (medulla) rostrally toward the mes-
mon in the lower extremity), and bladder dysfunction. The ataxia that
encephalic-diencephalic junction. In the medulla, ML and ALS fibers
may be seen in patients with posterior column lesions (sensory ataxia) is
are widely separated and receive different blood supplies, whereas in
due to a lack of proprioceptive input and position sense. These individ-
the midbrain, they are served by a common arterial source. As the ML
uals tend to forcibly place their feet to the floor in an attempt to stimu-
makes positional changes, the somatotopy therein follows accordingly.
late such sensory input. A patient with mild ataxia due to posterior col-
Fibers of the postsynaptic posterior column system (shown in green)
umn disease may compensate for the motor deficit by using visual cues.
are considered in detail in Figure 7-6 on page 182.
Patients with subacute combined degeneration (SCD) of the spinal cord
Neurotransmitters: Acetylcholine and the excitatory amino
first have signs and symptoms of posterior column involvement, fol-
acids glutamate and aspartate are associated with some of the large-
lowed later by signs of corticospinal tract damage (spastic weakness of legs,
diameter, heavily myelinated fibers of the posterior horn and posterior
increased deep tendon reflexes, Babinski sign).
columns.
Rostral to the sensory decussation, medial lemniscus lesions result
Clinical Correlations: Damage to posterior column fibers on one
in contralateral losses that include the entire body excluding the head.
side of the spinal cord (as in the Brown-Sequard syndrome) results in an ip-
Brainstem lesions involving medial lemniscus fibers usually include ad-
silateral loss of vibratory sensation, position sense, and discriminative
jacent structures, result in motor and additional sensory losses, and
touch (astereognosis, stereoagnosis) below the level of the lesion. The term
may reflect the distribution patterns of vessels (as in medial medullary or
stereoanesthesia is frequently used to specify a lesion of peripheral nerves
medial pontine syndromes). Large lesions in the forebrain may result in a
that results in an inability to perceive proprioceptive and tactile sensa-
complete contralateral loss of modalities carried in the posterior
tions. The term tactile agnosia is sometimes considered to be synony-
columns and anterolateral systems, or may produce pain (as in the thal-
mous with these preceding three terms. However, tactile agnosia is also
amic syndrome).
Abbreviations
ALS
Anterolateral system
NuGr Gracile nucleus
BP
Basilar pons
PC Posterior column
CC
Crus cerebri
PO Principal olivary nucleus
CTT
Central tegmental tract
PoCGy Postcentral gyrus
FCu
Cuneate fasciculus
PPGy Posterior paracentral gyrus
FGr
Gracile fasciculus
PRG Posterior (dorsal) root ganglia
IAF
Internal arcuate fibers
Py Pyramid
IC
Internal capsule
RB Restiform body
ML
Medial lemniscus
RNu Red nucleus
MLF
Medial longitudinal fasciculus
SN Substantia nigra
NuCu
Cuneate nucleus
VPL Ventral posterolateral nucleus of thalamus
Somatotopy of Body Areas
A
Fibers conveying input from upper extremity
S5
Fibers from approximately the fifth sacral
L
Fibers conveying input from lower extremity
level
N
Fibers conveying input from neck
T5
Fibers from approximately the fifth thoracic
T
Fibers conveying input from trunk
level
C2
Fibers from approximately the second
cervical level
Review of Blood Supply to DC-ML System
STRUCTURES
ARTERIES
PC in Spinal Cord
penetrating branches of arterial vasocorona (see Figure 5-6)
ML in Medulla
anterior spinal (see Figure 5-14)
ML in Pons
overlap of paramedian and long circumferential branches of basilar
(see Figure 5-21)
ML in Midbrain
short circumferential branches of posterior cerebral and superior
cerebellar (see Figure 5-27)
VPL
thalamogeniculate branches of posterior cerebral (see Figure 5-38)
Posterior Limb of IC
lateral striate branches of middle cerebral (see Figure 5-38)
Sensory Pathways
179
Posterior Column-Medial Lemniscus System
Trunk
Thigh
Leg
Somatosensory cortex
PPGy
Somatotopy in PC and ML
Foot
Post. limb, IC
A
T
L
L
Position of ML
A
ALS
ML
L
VPL
T
A
RNu
CC
ML
SN
ALS
MLF
CTT
L T A
ML
ML
BP
RB
MLF
A
ALS
ML
T
ML
NuCu
PO
L
Py
IAF
NuGr
NuGr
NuCu
FCu
ML
S5
IAF
T5
Py
C2
PRG, T
6
L
T A N
FGr
FCu
FGr
PRG, T6
Laminae III-V
180
Synopsis of Functional Components, Tracts, Pathways, and Systems
Anterolateral System
7-5
The longitudinal extent and somatotopy of fibers composing the
fibers or projection neurons, conveying nociceptive (pain) informa-
anterolateral system (ALS). The ALS is a composite bundle containing
tion.
ascending fibers that terminate in the reticular formation (spinoreticu-
Clinical Correlations: Spinal lesions involving the anterolateral
lar fibers), the mesencephalon (spinotectal fibers to deep layers of the
system (as in the Brown-Sequard syndrome) result in a loss of pain and
superior colliculus, spinoperiaqueductal fibers to the periaqueductal
temperature sensations on the contralateral side of the body beginning
grey), the hypothalamus (spinohypothalamic fibers), and the sensory re-
one to two levels caudal to the lesion. Syringomyelia produces bilateral
lay nuclei of the dorsal thalamus (spinothalamic fibers). Other fibers in
sensory losses restricted to adjacent dermatomes because of damage to
the ALS include spinoolivary projections to the accessory olivary nuclei.
the anterior (ventral) white commissure. Vascular lesions in the spinal
Spinothalamic fibers terminate primarily in the VPL and reticulothala-
cord (such as acute central cervical cord syndrome) may result in a bilateral
mic fibers terminate in some intralaminar nuclei, and in medial areas of
and splotchy loss of pain and thermal sense below the lesion because
the posterior thalamic complex.
the ALS has a dual vascular supply.
Fibers from the PAG and nucleus raphe dorsalis enter the nucleus
Vascular lesions in the lateral medulla (posterior inferior cerebellar artery
raphe magnus and adjacent reticular area. These latter sites, in turn,
syndrome) or lateral pons (anterior inferior cerebellar artery occlusion)
project to laminae I, II, and V of the spinal cord via raphespinal and
result in a loss of pain and thermal sensations over the entire contralat-
reticulospinal fibers that participate in the modulation of pain trans-
eral side of the body (ALS) as well as on the ipsilateral face (spinal
mission in the spinal cord.
trigeminal tract and nucleus), coupled with other motor and/or sensory
Neurotransmitters: Glutamate
(
), calcitonin gene-related
deficits based on damage to structures these vessels serve. Note that the
peptide, and substance P(
)-containing posterior (dorsal) root gan-
ALS and PC-ML systems are separated in the medulla (in different vas-
glion cells project into laminae I, II (heavy), V (moderate), and III, IV
cular territories) but are adjacent to each other in the midbrain (basi-
(sparse). Some spinoreticular and spinothalamic fibers contain
cally in the same vascular territory). Consequently, medullary lesions
enkephalin (
), somatostatin (
), and cholecystokinin (
). In addi-
will not result in deficits related to both pathways, while a lesion in the
tion to enkephalin and somatostatin, some spinomesencephalic fibers
midbrain may result in a contralateral loss of pain, thermal, vibratory,
contain vasoactive intestinal polypeptide (
). Neurons in the PAG and
and discriminative touch sensations on the body, excluding the head.
nucleus raphe dorsalis containing serotonin and neurotensin project
Profound loss of posterior column and anterolateral system modali-
into the nuclei raphe magnus and adjacent reticular formation. Cells in
ties, or intractable pain and/or paresthesias (as in the thalamic syndrome),
these latter centers that contain serotonin and enkephalin send
may result from vascular lesions in the posterolateral thalamus. So-
processes to spinal cord laminae I, II, and V. Serotonergic raphespinal
called thalamic pain may also be experienced by patients who have
or enkephalinergic reticulospinal fibers may inhibit primary sensory
brainstem lesions.
Abbreviations
A
Input from upper extremity regions
PRG Posterior (dorsal) root ganglion
ALS
Anterolateral system
Py Pyramid
AWCom
Anterior (ventral) white commissure
RaSp Raphespinal fibers
CC
Crus cerebri
RB Restiform body
IC
Internal capsule
RetF Reticular formation (of midbrain)
L
Input from lower extremity regions
RetTh Reticulothalamic fibers
MCP
Middle cerebellar peduncle
RNu Red nucleus
ML
Medial lemniscus
S Input from sacral regions
MLF
Medial longitudinal fasciculus
SC Superior colliculus
Nu
Nuclei
SpRet Spinoreticular fibers
NuDark
Nucleus of Darkschewitsch
SpTec Spinotectal fibers
NuRa,d
Nucleus raphe, dorsalis
SpTh Spinothalamic fibers
NuRa,m
Nucleus raphe, magnus
T Input from thoracic regions
PAG
Periaqueductal gray
VPL Ventral posterolateral nucleus of thalamus
PoCGy
Postcentral gyrus
I-VIII Laminae I-VIII of Rexed
PPGy
Posterior paracentral gyrus
Review of Blood Supply to ALS
STRUCTURES
ARTERIES
ALS in Spinal Cord penetrating branches of arterial vasocorona and branches of central
(see Figures 5-6 and 5-14)
ALS in Medulla
caudal third, vertebral; rostral two-thirds, posterior inferior cerebellar
(see Figure 5-14)
ALS in Pons
long circumferential branches of basilar (see Figure 5-21)
ALS in Midbrain
short circumferential branches of posterior cerebral, superior
cerebellar (see Figure 5-27)
VPL
thalamogeniculate branches of posterior cerebral (see Figure 5-38)
Posterior Limb of IC
lateral striate branches of middle cerebral (see Figure 5-38)
Sensory Pathways
181
Anterolateral System
Trunk
Thigh
Leg
Somatosensory cortex
PPGy
Foot
Post. limb, IC
Somatotopy of ALS fibers
Intralaminar Nu
A
Position of ALS fibers
T
L
SpTec
SC
SC, RetF, PAG
VPL
NuDark
PAG
SpTh
LTA
ML
RNu
PAG, NuRa,d
CC
RetTh
ALS
MLF
LTA
ALS
ML
NuRa,m
BP
SpRet
RaSp
RB
MLF
S
A
ALS
ALS
L T
ML
Py
RaSp
ALS
PRG
Laminae I-VIII
PRG
AWCom
S
L
RaSp to
T
Laminae
A
AWCom
I, II, V
ALS
182
Synopsis of Functional Components, Tracts, Pathways, and Systems
Postsynaptic-Posterior (Dorsal) Column System and the
Spinocervicothalamic Pathway
7-6
The origin, course, and distribution of fibers composing the
cal nucleus and the dorsal column nuclei, glutamate (and substance P)
postsynaptic-posterior column system (upper) and the spinocervi-
may also be present in some postsynaptic dorsal column fibers.
cothalamic pathway (lower). Postsynaptic-posterior column fibers
Clinical Correlations: The postsynaptic-posterior column and
originate primarily from cells in lamina IV (some cells in laminae III and
spinocervicothalamic pathways are not known to be major circuits in
V-II also contribute), ascend in the ipsilateral dorsal fasciculi, and end
the human nervous system. However, the occurrence of these fibers
in their respective nuclei in the caudal medulla. Moderate-to-sparse
may explain a well known clinical observation. Patients that have re-
collaterals project to a few other medullary targets.
ceived an anterolateral cordotomy (this lesion is placed just ventral to the
Fibers of the spinocervical part of the spinocervicothalamic pathway
denticulate ligament) for intractable pain may experience complete or
also originate from cells in lamina IV (less so from III and V). The axons
partial relief, or there may be a recurrence of pain perception within
of these cells ascend in the posterior part of the lateral funiculus (this is
days or weeks. Although the cordotomy transects fibers of the antero-
sometimes called the dorsolateral funiculus) and end in a topographic
lateral system (the main pain pathway), this lesion spares the posterior
fashion in the lateral cervical nucleus: lumbosacral projections termi-
horn, posterior columns, and spinocervical fibers. Consequently, the
nate posterolaterally and cervical projections anteromedially. Cells of
recurrence of pain perception (or even the partial relief of pain) in
the posterior column nuclei and the lateral cervical nucleus convey in-
these patients may be explained by these postsynaptic-dorsal column
formation to the contralateral thalamus via the medial lemniscus.
and spinocervicothalamic projections. Through these connections,
Neurotransmitters: Glutamate (
) and possibly substance P
some nociceptive (pain) information may be transmitted to the ventral
(
) are present in some spinocervical projections. Because some cells
posterolateral nucleus and on to the sensory cortex, via circuits that by-
in laminae III-V have axons that collateralize to both the lateral cervi-
pass the anterolateral system and are spared in a cordotomy.
Abbreviations
ALS Anterolateral system
AWCom Anterior (ventral) white commissure
FCu Cuneate fasciculus
FGr Gracile fasciculus
IAF Internal arcuate fibers
LCerNu Lateral cervical nucleus
ML Medial lemniscus
NuCu Cuneate nucleus
NuGr Gracile nucleus
PRG Posterior (dorsal) root ganglion
Review of Blood Supply to Dorsal Horn, FGr, FCu, LCerNu
STRUCTURES
ARTERIES
FGr, FCu in Spinal Cord penetrating branches of arterial vasocorona and some branches
from central (sulcal) (see Figure 5-6)
LCerNu
penetrating branches of arterial vasocorona and branches from
central (see Figure 5-6)
NuGr NuCu
posterior spinal (see Figure 5-14)
Sensory Pathways
183
Postsynaptic-Posterior (Dorsal) Column System and the
Spinocervicothalamic Pathway
FGr
ML
FCu
Other
brainstem
targets
IAF
NuCu
PRG
NuGr
FGr
FCu
Laminae IV
(III-VII)
PRG
ALS
FGr
FGr
Laminae IV
(III-VII)
ALS
ML
LCerNu
AWCom
FGr
FCu
Laminae IV (III-VII)
PRG
PRG
Dorsolateral region
of lateral funiculus
ALS
184
Synopsis of Functional Components, Tracts, Pathways, and Systems
Trigeminal Pathways
7-7
The distribution of general sensory (GSA) information origi-
tile sensation from the ipsilateral face, oral cavity, and teeth; 2) ipsilat-
nating on cranial nerves V (trigeminal), VII (facial), IX (glossopharyn-
eral paralysis of masticatory muscles; and 3) ipsilateral loss of the
geal), and X (vagus). Some of these primary sensory fibers end in the
corneal reflex. Damage to peripheral portions of the trigeminal nerve
chief sensory nucleus, but most form the spinal trigeminal tract and
may be traumatic (skull fracture, especially of supraorbital and infraor-
end in the spinal trigeminal nucleus.
bital branch), inflammatory (as in herpes zoster), or result from tumor
Neurons in the spinal trigeminal nucleus and in ventral parts of the
growth. The deficit would reflect the peripheral portion of the trigem-
chief sensory nucleus give rise to crossed anterior (ventral) trigeminothal-
inal nerve damaged.
amic fibers. Collaterals of these ascending fibers influence the hy-
Trigeminal neuralgia (tic douloureux) is a severe burning pain restricted
poglossal, facial (corneal reflex, supraorbital, or trigeminofacial reflex), and
to the peripheral distribution of the trigeminal nerve, usually its V2
trigeminal motor nuclei; mesencephalic collaterals are involved in the
(maxillary) division. This pain may be initiated by any contact to areas
jaw reflex, also called the jaw-jerk reflex. Collaterals also enter the dorsal
of the face such as the corner of the mouth, nose, lips, or cheek (e.g.,
motor vagal nucleus (vomiting reflex), the superior salivatory nucleus
shaving, putting make-up on, chewing, or even smiling). The attacks
(tearing/lacrimal reflex), and the nucleus ambiguus and adjacent reticular
frequently occur without warning, may happen only a few times a
formation (sneezing reflex). Uncrossed posterior (dorsal) trigeminothal-
month to many times in a single day, and are usually seen in patients
amic fibers arise from posterior regions of the chief sensory nucleus.
40 years of age or older. One probable cause of trigeminal neuralgia is
Neurotransmitters: Substance P (
)-containing and cholecys-
compression of the trigeminal root by aberrant vessels, most com-
tokinin (
)-containing trigeminal ganglion cells project to the spinal
monly a loop of the superior cerebellar artery (see page 41). Other
trigeminal nucleus, especially its caudal part (pars caudalis). Glutamate
causes may include tumor, multiple sclerosis, and ephaptic transmission
(
) is found in many trigeminothalamic fibers arising from the chief sen-
(ephapse) in the trigeminal ganglion. This is the most common type of
sory nucleus and the pars interpolaris of the spinal nucleus. It is present
neuralgia.
in fewer trigeminothalamic fibers from the pars caudalis and in almost
In the medulla, fibers of the spinal trigeminal tract and ALS are
none from the pars oralis. The locus ceruleus (noradrenergic fibers) and
served by the posterior inferior cerebellar artery (PICA). Conse-
the raphe nuclei (serotonergic fibers) also project to the spinal nucleus.
quently, an alternating hemianesthesia is one characteristic feature of the
Enkephalin (
)-containing cells are present in caudal regions of the
PICA syndrome. This is a loss of pain and thermal sensations on one side
spinal nucleus, and enkephalinergic fibers are found in the nucleus am-
of the body and the opposite side of the face. Pontine gliomas may pro-
biguus and in the hypoglossal, facial, and trigeminal motor nuclei.
duce a paralysis of masticatory muscles (motor trigeminal damage) and
Clinical Correlations: Lesions of the trigeminal ganglion or nerve
some loss of tactile input (chief sensory nucleus damage), as well as
proximal to the ganglion result in 1) a loss of pain, temperature, and tac-
other deficits based on what adjacent structures may be involved.
Abbreviations
ALS
Anterolateral system
MesNu Mesencephalic nucleus
VPM Ventral posteromedial nucleus
CC
Crus cerebri
ML Medial lemniscus
of thalamus
CSNu
Chief (principal) sensory
OpthV Ophthalmic division of
VTTr Ventral trigeminothalamic
nucleus
trigeminal nerve
tract
DTTr
Dorsal trigeminothalamic tract
RB Restiform body
FacNu
Facial nucleus
RetF Reticular formation
Ganglia
GSA
General somatic afferent
RNu Red nucleus
1
Trigeminal ganglion
HyNu
Hypoglossal nucleus
SpTNu Spinal trigeminal nucleus
2
Geniculate ganglion
IC
Internal capsule
SpTTr Spinal trigeminal tract
3
Superior of glossopharyngeal
ManV
Mandibular division of
TriMoNu Trigeminal motor nucleus
4
Superior of vagus
trigeminal nerve
TMJ Temporomandibular joint
MaxV
Maxillary division of trigeminal
VPL Ventral posterolateral nucleus
nerve
of thalamus
Review of Blood Supply to SpTT, SpTNu,
and Trigeminothalamic Tracts
STRUCTURES
ARTERIES
SpTTr and SpTNu
caudal third, vertebral; rostral two-thirds, posterior
in Medulla
inferior cerebellar (see Figure 5-14)
SpTTr and SpTNu in Pons long circumferential branches of basilar (see Figure 5-21)
Trigeminothalamic Fibers
short circumferential branches of posterior cerebral and
in Midbrain
superior cerebellar (see Figure 5-27)
VPM
thalamogeniculate branches of posterior cerebral
(see Figure 5-38)
Posterior Limb of IC
lateral striate branches of middle cerebral (see Figure 5-38)
Sensory Pathways
185
Trigeminal Pathways
Trunk
Thigh
Leg
Somatosensory
cortex
Foot
Posterior
limb, IC
VPM
DTTr
VTTr
MesNu
Origin of SA Data
CSNu
Position of Trigeminal tracts
GSA, skin of face, forehead
1
and part of scalp; mem-
branes of nose and of
MesNu
nasal, maxillary and frontal
sinuses; oral cavity, teeth;
ALS
ant. 2/3 of tongue; muscles
DTTr
TriMoNu
of mastication, TMJ; cornea
and conjunctiva; dura of
VTTr
TriMoNu
mid. and ant. cranial fossae
ML
RNu
SpTTr
FacNu
CC
SpTNu
RetF
GSA, external auditory meatus,
med. and lat. surfaces
VTTr
2
of ear (conchae)
3
GSA, small area on ear
Somatotopy in SpTTr and SpTNu
GSA, med. and lat. surfaces
4
of ear (conchae); post.
Input from 7,9,10
wall and floor of external
SpTTr
auditory meatus; tympanic
RB
membrane; dura of post.
cranial fossa
Man. V
SpTTr
HyNu
Max. V
SpTNu
SpTNu
Opth. V
ALS
SpTTr
SpTNu
186
Synopsis of Functional Components, Tracts, Pathways, and Systems
Solitary Pathways
7-8
Visceral afferent input (SVA-taste; GVA general visceral sen-
adjacent dorsal motor vagal nucleus. Cholecystokinin (
), somato-
sation) on cranial nerves VII (facial), IX (glossopharyngeal), and X (va-
statin (
), and enkephalin (
) are present in solitary neurons, in cells
gus) enters the solitary nuclei via the solitary tract. What we com-
of the parabrachial nuclei, and in some thalamic neurons that project
monly call the solitary “nucleus” is actually a series of small nuclei that
to taste, and other visceral areas, of the cortex.
collectively form this rostrocaudal-oriented cell column.
Clinical Correlations: Lesions of the geniculate ganglion, or fa-
Solitary cells project to the salivatory, hypoglossal, and dorsal mo-
cial nerve proximal to the ganglion, result in 1) ipsilateral loss of taste
tor vagal nuclei and the nucleus ambiguus. Solitary projections to the
(ageusia) from the anterior two-thirds of the tongue and 2) an ipsilateral
nucleus ambiguus are largely bilateral and are the intermediate neurons
facial (Bell) palsy. Although a glossopharyngeal nerve lesion will result in
in the pathway for the gag reflex. The afferent limb of the gag-reflex is
ageusia from the posterior third of the tongue on the ipsilateral side, this
carried on the glossopharyngeal nerve, and the efferent limb originates
loss is difficult to test. On the other hand, glossopharyngeal neuralgia is an
from the nucleus ambiguus. In this respect, the efferent limb travels on
idiopathic pain localized to the peripheral sensory branches of the IXth
both the glossopharyngeal and vagus nerves. Although not routinely
nerve in the posterior pharynx, posterior tongue, and tonsillar area. Al-
tested, the gag-reflex should be evaluated in patients with dysarthria, dys-
though comparatively rare, glossopharyngeal neuralgia may be aggra-
phagia, or hoarnessness. Solitariospinal fibers are bilateral with a con-
vated by talking or even swallowing. Occlusion of the posterior inferior
tralateral preponderance and project to the phrenic nucleus, the inter-
cerebellar artery (as in the posterior inferior cerebellar artery or lateral
mediolateral cell column, and the ventral horn. The VPM is the
medullary syndrome), in addition to producing an alternate hemianesthesia,
thalamic center through which visceral afferent information is relayed
will also result in ageusia from the ipsilateral side of the tongue because
onto the cerebral cortex.
the posterior inferior cerebellar artery serves the solitary tract and nu-
Neurotransmitters: Substance P (
)-containing and cholecys-
clei in the medulla.
tokinin (
)-containing cells in the geniculate ganglion (facial nerve)
Interestingly, lesions of the olfactory nerves or tract (anosmia, loss
and in the inferior ganglia of the glossopharyngeal and vagus nerves
of olfactory sensation; dysosmia, distorted olfactory sense) may affect
project to the solitary nucleus. Enkephalin (
), neurotensin, and
how the patient perceives taste. Witness the fact that the nasal conges-
GABA (
) are present in some solitary neurons that project into the tion accompanying a severe cold will markedly affect the sense of taste.
Abbreviations
AmyNu Amygdaloid nucleus (complex)
SalNu Salivatory nuclei
CardResp Cardiorespiratory portion (caudal) of
SolTr & Nu Solitary tract and nuclei
solitary nucleus
SVA Special visceral afferent
GustNu Gustatory nucleus (rostral portion of
Tr Tract
solitary nucleus)
VA Visceral afferent
GVA General visceral afferent
VPM Ventral posteromedial nucleus of
HyNu Hypoglossal nucleus
thalamus
HyTh Hypothalamus
Inf VNu Inferior (or spinal) vestibular nucleus
Number Key
MVNu Medial vestibular nucleus
1
Geniculate ganglion of facial
NuAm Nucleus ambiguus
2
Inferior ganglion of glossopharyngeal
PBNu Parabrachial nuclei
3
Inferior ganglion of vagus
RB Restiform body
4
Dorsal motor vagal nucleus
Review of Blood Supply to SolNu and SolTr
STRUCTURES
ARTERIES
SolNu and Tr in
caudal medulla, anterior spinal; rostral medulla, posterior inferior
inferior cerebellar
cerebellar (See Figure 5-14)
Ascending Fibers
long circumferential branches of basilar and branches of superior
in Pons
cerebellar (see Figure 5-21)
VPM
thalamogeniculate branches of posterior cerebral (see Figure 5-38)
Posterior Limb of IC
lateral striate branches of middle cerebral (see Figure 5-38)
Sensory Pathways
187
Solitary Pathways
Trunk
Thigh
Leg
Foot
HyTh
AmyNu
VPM
PBNu
Origin of VA data
to HyNu,
SalNu
SVA, taste, ant. 2/3 of tongue
1
GVA, submand., subling., lac.
glds.
SVA
2
(GustNu)
SVA, taste, post. 1/3 of tongue
SolTr and Nu
4
GVA, parotid gld.; mucosa of
pharynx; tonsillar sinus;
3
GVA
post. 1/3 of tongue;
(CardResp)
carotid body
SVA, taste buds at root of
NuAm
tongue and on epiglottis
GVA, pharynx; larynx; aortic
bodies; thoracic and
abdominal viscera
Solitariospinal Tr
Position of SolTr & Nu
MVNu
InfVNu
RB
SolTr and Nu
188
Synopsis of Functional Components, Tracts, Pathways, and Systems
7-9
Blank master drawing for sensory pathways. This illustration is
provided for self-evaluation of sensory pathway understanding, for the
instructor to expand on sensory pathways not covered in the atlas, or
both.
190
Synopsis of Functional Components, Tracts, Pathways, and Systems
Corticospinal Tracts
7-10
The longitudinal extent of corticospinal fibers and their posi-
is a hallmark of this disease. Ocular muscles are usually affected first
tion and somatotopy at representative levels within the neuraxis. The
(diplopia, ptosis), and in approximately 50% of patients, facial and
somatotopy of corticospinal fibers in the basilar pons is less obvious than
oropharyngeal muscles are commonly affected ( facial weakness, dys-
in the internal capsule, crus cerebri, pyramid, or spinal cord. In the de-
phagia, dysarthria). Weakness may also be seen in limb muscles but al-
cussation of the pyramids, fibers originating from upper extremity ar-
most always in combination with facial/oral weaknesses.
eas of the cerebral cortex cross rostral to those that arise from lower ex-
Injury to corticospinal fibers on one side of the cervical spinal cord
tremity areas. In addition to fibers arising from the somatomotor area
(as in the Brown-Sequard syndrome) results in weakness (hemiparesis) or
of the cerebral cortex (area 4), a significant contingent also originate
paralysis (hemiplegia) of the ipsilateral upper and lower extremities. In
from the postcentral gyrus (areas 3, 1, 2); the former terminate pri-
addition, and with time, these patients may exhibit features of an upper
marily in laminae VI-IX, while the latter end mainly in laminae IV and
motor neuron lesion (hyperreflexia, spasticity, loss of superficial abdominal
V. Prefrontal regions, especially area 6, and parietal areas 5 and 7 also
reflexes, and the Babinski sign). Bilateral cervical spinal cord damage
contribute to the corticospinal tract.
above C4-C5 may result in paralysis of all four extremities (quadriple-
Neurotransmitters: Acetylcholine, gamma-aminobutyric acid
gia). Unilateral spinal cord lesions in thoracic levels may result in paral-
(
), and substance P (
, plus other peptides) are found in small cor-
ysis of the ipsilateral lower extremity (monoplegia). If the thoracic spinal
tical neurons presumed to function as local circuit cells or in cortico-
cord damage is bilateral both lower extremities may be paralyzed ( para-
cortical connections. Glutamate (
) is present in cortical efferent
plegia). Small lesions within the decussation of the pyramids may result
fibers that project to the spinal cord. Glutaminergic corticospinal fibers
in a bilateral paresis of the upper extremities (lesion in rostral portions)
and terminals are found in all spinal levels but are especially concen-
or a bilateral paresis of the lower extremities (lesion in caudal portions)
trated in cervical and lumbosacral enlargements. This correlates with
based on the crossing patterns of fibers within the decussation.
the fact that approximately 55% of all corticospinal fibers terminate in
Rostral to the pyramidal decussation, vascular lesions in the medulla
cervical levels of the spinal cord, approximately 20% in thoracic lev-
(the medial medullary syndrome), pons (the Millard-Gubler or Foville syn-
els, and approximately 25% in lumbosacral levels. Some corticospinal
dromes), or midbrain
(the Weber syndrome) all produce alternating
fibers may branch and terminate at multiple spinal levels. Lower mo-
(crossed) hemiplegias. These present as a contralateral hemiplegia of the
tor neurons are influenced by corticospinal fibers either directly or in-
upper and lower extremities, coupled with an ipsilateral paralysis of
directly via interneurons. Acetylcholine and calcitonin gene-related
the tongue (medulla), facial muscles or lateral rectus muscle (pons),
peptides are present in these large motor cells and in their endings in
and most eye movements (midbrain). Sensory deficits are frequently
skeletal muscle.
seen as part of these syndromes. Lesions in the internal capsule (lacu-
Clinical Correlations: Myasthenia gravis, a disease characterized
nar strokes) produce contralateral hemiparesis sometimes coupled with
by moderate to profound weakness of skeletal muscles, is caused by
various cranial nerve signs due to corticonuclear (corticobulbar) fiber
circulating antibodies that react with postsynaptic nicotinic acetyl-
involvement. Bilateral weakness, indicative of corticospinal involve-
choline receptors. Progressive muscle fatigability throughout the day
ment, is also present in amyotrophic lateral sclerosis.
Abbreviations
ACSp Anterior corticospinal tract
LCSp Lateral corticospinal tract
Somatotopy of CSp Fibers
ALS Anterolateral system
ML Medial lemniscus
A Position of fibers coursing to
APGy Anterior paracentral gyrus
MLF Medial longitudinal fasciculus
upper extremity regions of
BP Basilar pons
PO Principal olivary nucleus
spinal cord
CC Crus cerebri
PrCGy Precentral gyrus
L Position of fibers coursing to
CNu Corticonuclear (corticobulbar)
Py Pyramid
lower extremity regions of
fibers
RB Restiform body
spinal cord
CSp Corticospinal fibers
RNu Red nucleus
T Position of fibers coursing to
IC Internal capsule
SN Substantia nigra
thoracic regions of spinal cord
Review of Blood Supply to Corticospinal Fibers
STRUCTURES
ARTERIES
Posterior Limb of IC
lateral striate branches of middle cerebral (see
Figure 5-38)
Crus Cerebri in
paramedian and short circumferential
Midbrain
branches of basilar and posterior
communicating (see Figure 5-27)
CSp in BP
paramedian branches of basilar (see Figure 5-21)
Py in Medulla
anterior spinal (see Figure 5-14)
LCSp in Spinal Cord
penetrating branches of arterial vasocorona (leg
fibers), branches of central artery (arm fibers)
(See Figure 5-6)
Motor Pathways
191
Corticospinal Tracts
Trunk
Thigh
Somatomotor cortex
Leg
APGy
Foot
Somatotopy of CSp
Post. limb, IC
A
T
Position of CSp
L
ALS
ML
RNu
SN
L
CC
T
A
CSp fibers in CC
VesNu
Face
ALS
(CNu Fibers)
MLF
CSp fibers in BP
ML
L
T
A
BP
CSp
RB
CSp fibers in Py
MLF
ALS
ML
PO
L T A
Py
Pyramidal (motor) decussation
LCSp
L
LCSp
T
ACSp
A
Laminae
IV-IX
ALS
ACSp
192
Synopsis of Functional Components, Tracts, Pathways, and Systems
Corticonuclear (Corticobulbar) Fibers
7-11
The origin, course, and distribution of corticonuclear (corti-
may produce a transient gaze palsy in which the eyes deviate toward the
cobulbar) fibers to brainstem motor nuclei. These fibers influence—ei-
lesioned side and away from the side of the hemiplegia. In addition to
ther directly or through neurons in the immediately adjacent reticular
a contralateral hemiplegia, common cranial nerve findings in capsular le-
formation—the motor nuclei of oculomotor, trochlear, trigeminal, ab-
sions may include 1) deviation of the tongue toward the side of the
ducens, facial, glossopharyngeal and vagus (both via nucleus ambiguus),
weakness and away from the side of the lesion when protruded and 2)
spinal accessory, and hypoglossal nerves.
paralysis of facial muscles on the contralateral lower half of the face
Corticonuclear (corticobulbar) fibers arise in the frontal eye fields (ar-
(central facial palsy). This reflects the fact that corticonuclear (cortico-
eas 6 and 8 in caudal portions of the middle frontal gyrus), the precen-
bulbar) fibers to genioglossus motor neurons and to facial motor neu-
tral gyrus (somatomotor cortex, area 4), and some originate from the
rons serving the lower face are primarily crossed. Interruption of cor-
postcentral gyrus (areas 3,1, 2). Fibers from area 4 occupy the genu of
ticonuclear fibers to the nucleus ambiguus may result in weakness of
the internal capsule, but those from the frontal eye fields (areas 8,6) may
palatal muscles contralateral to the lesion; the uvula will deviate to-
traverse caudal portions of the anterior limb, and some (from areas
wards the ipsilateral (lesioned) side on attempted phonation. In addi-
3,1,2), may occupy the most rostral portions of the posterior limb.
tion, a lesion involving corticonuclear fibers to the accessory nucleus
Fibers that arise in areas 8 and 6 terminate in the rostral interstitial nucleus
may result in drooping of the ipsilateral shoulder (or an inability to el-
of the medial longitudinal fasciculus (vertical gaze center) and in the parame-
evate the shoulder against resistance) due to trapezius weakness, and
dian pontine reticular formation (horizontal gaze center); these areas, in turn,
difficulty in turning the head (against resistance) to the contralateral
project respectively to the IIIrd and IVth, and to the VIth nuclei. Fibers
side due to weakness of the sternocleidomastoid muscle. In contrast to
from area 4 terminate in, or adjacent to, cranial nerve motor nuclei ex-
the alternating hemiplegia seen in some brainstem lesions, hemisphere
cluding those of III, IV, and VI.
lesions result in spinal and cranial nerve deficits that are generally, but
Although not illustrated here, the superior colliculus receives cor-
not exclusively, contralateral to the cerebral injury.
tical input from area 8 and from the parietal eye field (area 7) and also
Brainstem lesions, especially in the midbrain or pons, may result in
projects to the riMLF and PPRF. In addition, it is important to note
the following: 1) vertical gaze palsies (midbrain), 2) the Parinaud syn-
that descending cortical fibers (many arising in areas 3, 1, 2) project to
drome—paralysis of upward gaze (tumors in area of pineal), 3) internu-
sensory relay nuclei of some cranial nerves and to other sensory relay
clear ophthalmoplegia (lesion in MLF between motor nuclei of III and
nuclei in the brainstem, such as those of the posterior column system.
VI), 4) horizontal gaze palsies (lesion in PPRF), or 5) the one-and-a-half
Neurotransmitters: Glutamate (
) is found in many corticofu-
syndrome. In the latter case, the lesion is adjacent to the midline and in-
gal axons that directly innervate cranial nerve motor nuclei and in
volves the abducens nucleus and adjacent PPRF, internuclear fibers
those fibers that terminate near (indirect), but not in, the various mo-
from the ipsilateral abducens that are crossing to enter the contralat-
tor nuclei.
eral MLF, and internuclear fibers from the contralateral abducens nu-
Clinical Correlations: Lesions involving the motor cortex (as in
cleus that cross to enter the MLF on the ipsilateral (lesioned) side. The
cerebral artery occlusion) or the internal capsule (as in lacunar strokes
result is a loss of ipsilateral abduction (lateral rectus) and adduction
or occlusion of lenticulostriate branches of M1) give rise to a con-
(medial rectus, the “one”) and a contralateral loss of adduction (medial
tralateral hemiplegia of the arm and leg (corticospinal fiber involve-
rectus, the “half ”); the only remaining horizontal movement is con-
ment) coupled with certain cranial nerve signs. Strictly cortical lesions
tralateral abduction via the intact abducens motor neurons.
Abbreviations
AbdNu Abducens nucleus
OcNu Oculomotor nucleus
AccNu Accessory nucleus (spinal accessory nu.)
PPRF Paramedian pontine reticular formation
FacNu Facial nucleus
riMLF Rostral interstitial nucleus of the medial
HyNu Hypoglossal nucleus
longitudinal fasciculus
IC Internal capsule
TriMoNu Trigeminal motor nucleus
NuAm Nucleus ambiguus
TroNu Trochlear nucleus
Review of Blood Supply to Cranial Nerve Motor Nuclei
STRUCTURES
ARTERIES
OcNu and EWNu
paramedian branches of basilar bifurcation and medial branches
of posterior cerebral and posterior communicating (see Figure
5-27)
TriMoNu
long circumferential branches of basilar (see Figure 5-21)
AbdNu and FacNu
long circumferential branches of basilar (see Figure 5-21)
NuAm
posterior inferior cerebellar (see Figure 5-14)
HyNu
anterior spinal (see Figure 5-14)
Motor Pathways
193
Corticonuclear (Corticobulbar) Fibers
Motor cortex,
precentral gyrus
Frontal eye fields
riMLF
OcNu
Genu of IC
TroNu
TriMoNu
Bilateral for upper face
PPRF
AbdNu
FacNu
Crossed for lower face
NuAm
= Direct to motor
neurons of nucleus
Crossed for uvula
(soft palate)
= Indirect to motor
neurons via adjacent
Crossed for
reticular formation
genioglossus muscle
HyNu
= Bilateral projections
AccNu
= Primarily crossed
projections
194
Synopsis of Functional Components, Tracts, Pathways, and Systems
Tectospinal and Reticulospinal Tracts
7-12
The origin, course, position in representative cross-sections
ing system that modulates pain transmission at the spinal level. Many
of brainstem and spinal cord, and the general distribution of tectospinal
reticulospinal fibers influence the activity of lower motor neurons.
and reticulospinal tracts. Tectospinal fibers originate from deeper lay-
Clinical Correlations: Isolated lesions of only tectospinal and
ers of the superior colliculus, cross in the posterior (dorsal) tegmental
reticulospinal fibers are essentially never seen. Tectospinal fibers pro-
decussation, and distribute to cervical cord levels. Several regions of
ject to upper cervical levels where they influence reflex movement of
cerebral cortex (e.g., frontal, parietal, temporal) project to the tec-
the head and neck. Such movements may be diminished or slowed in
tum, but the most highly organized corticotectal projections arise from
patients with damage to these fibers. Pontoreticulospinal (medial retic-
the visual cortex. Pontoreticulospinal fibers (medial reticulospinal)
ulospinal) fibers are excitatory to extensor motor neurons and to neu-
tend to be uncrossed, while those from the medulla (bulboreticu-
rons innervating axial musculature; some of these fibers may also in-
lospinal or lateral reticulospinal) are bilateral but with a pronounced
hibit flexor motor neurons. In contrast, some bulboreticulospinal
ipsilateral preponderance. Corticoreticular fibers are bilateral with a
(lateral reticulospinal) fibers are primarily inhibitory to extensor mo-
slight contralateral preponderance and originate from several cortical
tor neurons and to neurons innervating muscles of the neck and back;
areas.
these fibers may also excite flexor motor neurons via interneurons.
Neurotransmitters: Corticotectal projections, especially those
Reticulospinal (and vestibulospinal) fibers contribute to the spasticity
from the visual cortex, utilize glutamate (
). This substance is also
that develops in patients having lesions of corticospinal fibers. These
present in most corticoreticular fibers. Some neurons of the giganto-
reticulospinal and vestibulospinal fibers (see Figure 7-13 on page 196)
cellular reticular nucleus that send their axons to the spinal cord, as
also contribute to the tonic extension of the arms and legs seen in de-
reticulospinal projections, contain enkephalin (
) and substance P
cerebrate rigidity when spinal motor neurons are released from de-
(
). Enkephalinergic reticulospinal fibers may be part of the descend- scending cortical control.
Abbreviations
ALS Anterolateral system
PO Principal olivary nucleus
ATegDec Anterior tegmental decussation
PTegDec Posterior tegmental decussation
(rubrospinal fibers)
(tectospinal fibers)
BP Basilar pons
Py Pyramid
CC Crus cerebri
RB Restiform body
CRet Corticoreticular fibers
RetNu Reticular nuclei
CTec Corticotectal fibers
RetSp Reticulospinal tract(s)
GigRetNu Gigantocellular reticular nucleus
RNu Red nucleus
LCSp Lateral corticospinal tract
RuSp Rubrospinal tract
ML Medial lemniscus
SC Superior colliculus
MLF Medial longitudinal fasciculus
SN Substantia nigra
MVNu Medial vestibular nucleus
SpVNu Spinal (or inferior) vestibular nucleus
OcNu Oculomotor nucleus
TecSp Tectospinal tract
Review of Blood Supply to SC, Reticular Formation of Pons
and Medulla, and TecSp and RetSp Tracts in Cord
STRUCTURES
ARTERIES
SC
long circumferential branches (quadrigeminal branch) of posterior
cerebral plus some from superior cerebellar and posterior
choroidal (see Figure 5-27)
Pontine Reticular
long circumferential branches of basilar plus branches of superior
Formation
cerebellar in rostral pons (see Figure 5-21)
Medullary Recticular
branches of vertebral plus paramedian branches of basilar at
Formation
medulla-pons junction (see Figure 5-14)
TecSp and RetSp
branches of central artery (TecSp and Medullary RetSp); Tracts
penetrating branches of arterial vasocorona (Pontine RetSp) (see
Figures 5-14 and 5-6)
Motor Pathways
195
Tectospinal and Reticulospinal Tracts
CRet
CTec
Postition of TecSp and RetSp
SC
CTec
SC
PTegDec
TecSp
ML
RNu
SN
CRet
PTegDec (TecSp)
CRet
CC
ATegDec (RuSp)
MLF
Pontine RetNu:
oralis
TecSp
RetNu of Pons
caudalis
ML
ALS
BP
InfVNu
Pontine RetSp
MVNu
RB
MLF
TecSp
ALS
GigRetNu
GigRetNu
PO
ML
Py
Pontine RetSp
TecSp
Medullary RetSp
LCSp
Medullary RetSp
ALS
to Laminae VII
(VI,VII, IX)
TecSp
to Laminae VI, VII (VIII)
Pontine RetSp
of cervical levels
to Laminae VIII
(VII,IX)
196
Synopsis of Functional Components, Tracts, Pathways, and Systems
Rubrospinal and Vestibulospinal Tracts
7-13
The origin, course, and position in representative cross-sec-
Clinical Correlations: Isolated injury to rubrospinal and vestibu-
tions of brainstem and spinal cord, and the general distribution of
lospinal fibers is really not seen in humans. Deficits in fine distal limb
rubrospinal and vestibulospinal tracts. Rubrospinal fibers cross in the an-
movements seen in monkeys following experimental rubrospinal le-
terior (ventral) tegmental decussation and distribute to all spinal levels
sions may be present in humans. However, these deficits are over-
although projections to cervical levels clearly predominate. Cells in dor-
shadowed by the hemiplegia associated with injury to the adjacent cor-
somedial regions of the red nucleus receive input from upper extremity
ticospinal fibers. The contralateral tremor seen in patients with the
areas of the motor cortex and project to cervical cord, but those in ven-
Claude syndrome (a lesion of the medial midbrain) is partially related to
trolateral areas of the nucleus receive some fibers from lower extremity
damage to the red nucleus as well as to the adjacent cerebellothalamic
areas of the motor cortex and may project in sparse numbers to lum-
fibers. These patients may also have a paucity of most eye movement
bosacral levels. The red nucleus also projects, via the central tegmental
on the ipsilateral side and a dilated pupil (mydriasis) due to concurrent
tract, to the ipsilateral inferior olivary complex (rubroolivary fibers).
damage to exiting rootlets of the oculomotor nerve.
Medial and lateral vestibular nuclei give rise to the medial and lateral
Medial vestibulospinal fibers primarily inhibit motor neurons inner-
vestibulospinal tracts, respectively. The former tract is primarily ipsi-
vating extensors and neurons serving muscles of the back and neck. Lat-
lateral, projects to upper spinal levels, and is considered a component
eral vestibulospinal fibers may inhibit some flexor motor neurons, but
of the medial longitudinal fasciculus in the spinal cord. The latter tract
they mainly facilitate spinal reflexes via their excitatory influence on spinal
is ipsilateral and somatotopically organized; fibers to lumbosacral levels
motor neurons innervating extensors. Vestibulospinal and reticulospinal
originate from dorsal and caudal regions of the lateral nucleus, while
(see Figure 7-12 on page 194) fibers contribute to the spasticity seen in pa-
those to cervical levels arise from its rostral and more ventral areas.
tients with damage to corticospinal fibers or to the tonic extension of the
Neurotransmitters: Glutamate (
) is present in corticorubral
extremities in patients with decerebrate rigidity. In the case of decerebrate
fibers. Some lateral vestibulospinal fibers contain aspartate
(
),
rigidity, the descending influences on spinal flexor motor neurons (corti-
whereas glycine (
) is present in a portion of the medial vestibu-
cospinal, rubrospinal) is removed; the descending brainstem influence on
lospinal projection. There are numerous gamma-aminobutyric acid
spinal extensor motor neurons predominates; this is augmented by exci-
(
)-containing fibers in the vestibular complex; these represent the
tatory spinoreticular input (via ALS) to some of the centers giving rise to
endings of cerebellar corticovestibular fibers.
reticulospinal fibers (see also Figure 7-12 on page 194).
Abbreviations
ATegDec Anterior tegmental decussation
MVessp Medial vestibulospinal tract
(rubrospinal fibers)
MVNu Medial vestibular nucleus
CC Crus cerebri
OcNu Oculomotor nucleus
CorRu Corticorubral fibers
PTegDec Posterior tegmental decussation (tectospinal
FacNu Facial nucleus
fibers)
InfVNu Inferior (or spinal) vestibular nucleus
Py Pyramid
LCSp Lateral corticospinal tract
RNu Red nucleus
LRNu Lateral reticular nucleus
RuSp Rubrospinal tract
LVNu Lateral vestibular nucleus
SC Superior colliculus
LVesSp Lateral vestibulospinal tract
SVNu Superior vestibular nucleus
ML Medial lemniscus
TecSp Tectospinal tract
MLF Medial longitudinal fasciculus
VesSp Vestibulospinal tracts
Review of Blood Supply to RNu, Vestibular Nuclei, MFL
and RuSp, and Vestibulospinal Tracts in Cords
STRUCTURES
ARTERIES
RNu
medial branches of posterior cerebral and posterior communicating
plus some from short circumferential branches of posterior cerebral
(see Figure 5-27)
Vestibular Nuclei
posterior inferior cerebellar in medulla (see Figure 5-14) and long
circumferential branches in pons (see Figure 5-21)
MLF
long circumferential branches of basilar in pons (see Figure 5-21)
and anterior spinal in medulla (see Figure 5-14)
MVesSp
branches of central artery (see Figures 5-6 and 5-14)
LVesSp and RuSp
penetrating branches of arterial vasocorona plus terminal branches
of central artery (see Figure 5-6)
Motor Pathways
197
Rubrospinal and Vestibulospinal Tracts
Thigh
Leg
Foot
CorRu
Position of RuSp and VesSp
PTegDec
SC
OCNu
ML
RNu
RNu
PTegDec (TecSp)
RuSp
CC
SVNu
ATegDec (RuSp)
LVNu
MVNu
MVNu
FacNu
InfVNu
MLF
RuSp
SpVNu
LRNu
Py
ML
LVesSp
MVesSp
in MLF
LCSp
RuSp
to Laminae
RuSp
LVesSp
V-VIII
MVesSp
to Laminae
LVesSp
VII and VIII
MVesSp
LCSp
RuSp
LVesSp
198
Synopsis of Functional Components, Tracts, Pathways, and Systems
7-14
Blank master drawing for motor pathways. This illustration
is provided for self-evaluation of motor pathways understanding, for
the instructor to expand on motor pathways not covered in this atlas,
or both.
200
Synopsis of Functional Components, Tracts, Pathways, and Systems
Cranial Nerve Efferents (III, IV, VI, XI-AccNu, XII)
7-15
The origin and peripheral distribution of GSE fibers from the
addition, the pupil may be unaffected (pupillary sparing) or dilated and
oculomotor, trochlear, abducens, spinal accessory, and hypoglossal
fixed. Lesions in the midbrain that involve the root of the IIIrd nerve
nuclei. Also shown are GVE fibers arising from the Edinger-Westphal
and the crus cerebri give rise to a superior alternating (crossed) hemiplegia.
nucleus and the distribution of postganglionic fibers from the ciliary
This is a paralysis of most eye movement and possibly a dilated pupil
ganglion. Internuclear abducens neurons project, via the MLF, to con-
on the ipsilateral side and a contralateral hemiplegia of the extremities.
tralateral oculomotor neurons that innervate the medial rectus muscle
Damage to the MLF (as in multiple sclerosis or small vessel occlusion) be-
(internuclear ophthalmoplegia pathway).
tween the VIth and IIIrd nuclei results in internuclear ophthalmoplegia; on
Some authors specify the functional component of neurons in the
attempted lateral gaze, the opposite medial rectus muscle will not adduct.
accessory nucleus as special visceral efferent, some specify it as somatic
A lesion of the IVth nerve (frequently caused by trauma) produces diplopia
efferent, and some are noncommittal. Because, in humans, the trapez-
on downward and inward gaze (tilting the head may give some relief ), and
ius and sternocleidomastoid muscles originate from cervical somites
the eye is slightly elevated when the patient looks straight ahead.
located caudal to the last pharyngeal arch, the functional component is
Diabetes mellitus, trauma, or pontine gliomas are some causes of VIth
designated here as GSE. In addition, experiments in animals reveal that
nerve dysfunction. In these patients, the affected eye is slightly ad-
motor neurons innervating the trapezius and sternocleidomastoid
ducted, and diplopia is pronounced on attempted gaze to the lesioned
muscles are found in cervical cord levels C1 to approximately C6.
side. Damage in the caudal and medial pons may involve the fibers of
Neurotransmitters: Acetylcholine
(and probably calcitonin
the VIth nerve and the adjacent corticospinal fibers in the basilar pons,
gene-related peptide, CGRP) is found in the motor neurons of cranial
giving rise to a middle alternating (crossed ) hemiplegia. The deficits are an
nerve nuclei and in their peripheral endings. This substance is also
ipsilateral paralysis of the lateral rectus muscle and a contralateral
found in cells of the Edinger-Westphal nucleus and the ciliary ganglion.
hemiplegia of the extremities. The XIth nerve may be damaged cen-
Clinical Correlations: Myasthenia gravis (MG) is a disease caused
trally (as in syringobulbia or amyotrophic lateral sclerosis) or at the jugular
by autoantibodies that may directly block nicotinic acetylcholine
foramen with resultant paralysis of the ipsilateral sternocleidomastoid
receptors or damage the postsynaptic membrane (via complement me-
and upper parts of the trapezius muscle.
diated lysis) thereby reducing the number of viable receptor sites. Oc-
Central injury to the XIIth nucleus or fibers (as in the medial
ular movement disorders (diplopia, ptosis) are seen first in approxi-
medullary syndrome or in syringobulbia) or to its peripheral parts (as in
mately 50% of patients and are present in approximately 85% of all
polyneuropathy or tumors) results in deviation of the tongue toward the
MG patients. Movements of the neck and tongue may also be impaired,
lesioned side on attempted protrusion. A lesion in the medial aspects
with the latter contributing to dysphagia and dysarthria.
of the medulla will give rise to an inferior alternating (crossed ) hemiplegia.
Lesions of the IIIrd nerve (as in the Weber syndrome or in carotid cav-
This is characterized by a paralysis of the ipsilateral side of the tongue
ernous aneurysms) may result in 1) ptosis, 2) lateral and downward devi-
(XIIth root damage) and contralateral hemiplegia of the extremities
ation of the eye, and 3) diplopia (except on ipsilateral lateral gaze). In
(damage to corticospinal fibers in the pyramid).
Abbreviations
AbdNr Abducens nerve
OcNu Oculomotor nucleus
AbdNu Abducens nucleus
PO Principal olivary nucleus
AccNr Accessory nerve
Py Pyramid
AccNu Accessory nucleus (spinal accessory nu.)
RNu Red nucleus
BP Basilar pons
SC Superior colliculus
CC Crus cerebri
SCP,Dec Superior cerebellar peduncle, decussation
EWNu Edinger-Westphal nucleus
TroDec Trochlear decussation
FacCol Facial colliculus
TroNr Trochlear nerve
HyNr Hypoglossal nerve
TroNu Trochlear nucleus
HyNu Hypoglossal nucleus
ML Medial lemniscus
Ganglion
MLF Medial longitudinal fasciculus
1
Ciliary
OcNr Oculomotor nerve
Review of Blood Supply to OcNu, TroNu, AbdNu and HyNu,
and the Internal Course of their Fibers
STRUCTURES
ARTERIES
OcNu and Fibers
medial branches of posterior cerebral and posterior
communicating (see Figure 5-27)
TroNu
paramedian branches of basilar bifurcation (see Figure 5-27)
AbdNu
long circumferential branches of basilar (see Figure 5-21)
Abducens Fibers in BP
paramedian branches of basilar (see Figure 5-21)
HyNu and Fibers
anterior spinal (see Figure 5-14)
Motor Pathways
201
Cranial Nerve Efferents (III, IV, VI, XI-AccNu, and XII)
Position of Nucleus
and Internal
Route of Fibers
SC
ML
SN
OcNu and
EWNu
OcNu
EWNu
OcNr
RNu
CC
1
Muscles Innervated
OcNr
TroNu
Ciliary; Sphincter of iris
TroNr
Med. Rectus
Inf. Oblique; Inf. and
TroDec
Med. recti
Exit of TroNr
TroNu
Sup. rectus
MLF
Levator palpebrae
TroDec
TroNr
MLF
CC
Sup. Oblique
AbdNu
SCP, Dec
Lat. rectus
AbdNr
FacCol
AbdNu
MLF
HyNu
ML
Intrinsic tongue muscles,
BP
and styloglossus,
HyNr
hyoglossus,
genioglossus
AbdNr
AccNr
AccNu
HyNu
Sternocleidomastoid
MLF
Trapezius
PO
ML
Py
HyNr
202
Synopsis of Functional Components, Tracts, Pathways, and Systems
Cranial Nerve Efferents (V, VII, IX, and X)
7-16
The origin and peripheral distribution of fibers arising from
viates to the lesioned side when closed);, and 3) loss of the afferent limb
the SVE motor nuclei of the trigeminal, facial, and glossopharyngeal and
of the corneal reflex. If especially large, a vestibular schwannoma may
vagus (via the nucleus ambiguus) nerves. Also shown are the origin of
compress the trigeminal nerve root and result in a hemifacial sensory
GVE preganglionic parasympathetic fibers from the superior (to facial
loss that may include the oral cavity. Trigeminal neuralgia (tic douloureux)
nerve) and inferior (to glossopharyngeal nerve) salivatory nuclei and
is an intense, sudden, intermittent pain emanating from the area of the
from the dorsal motor vagal nucleus. Their respective ganglia are indi-
cheek, oral cavity, or adjacent parts of the nose (distribution of V2 or
cated as well as the structures innervated by postganglionic fibers aris-
V3, see also Figure 7-7 on page 184).
ing from each. The SVE functional component specifies cranial nerve
Tumors (such as chordoma or vestibular schwannoma), trauma, or
motor nuclei that innervate head muscles that arose, embryologically,
meningitis may damage the VIIth nerve, resulting in 1) an ipsilateral fa-
from pharyngeal arches. Muscles innervated by the trigeminal nerve (V)
cial palsy (or Bell palsy); 2) loss of taste from the ipsilateral two-thirds
come from the 1st arch, those served by the facial nerve (VII) from the
of the tongue; and (3) decreased secretion from the ipsilateral lacrimal,
2nd arch; the stylopharyngeal muscle originates from the 3rd arch and
nasal, and sublingual and submandibular glands. Injury distal to the
is innervated by the glossopharyngeal nerve (IX), and the muscles de-
chorda tympani produces only an ipsilateral facial palsy. A paralysis of
rived from the 4th arch are served by the vagus nerve (X).
the muscles on one side of the face with no paralysis of the extremities
Neurotransmitters: The transmitter found in the cells of cra-
is a facial hemiplegia. On the other hand, intermittent and involuntary
nial nerve motor nuclei, and in their peripheral endings, is acetyl-
contraction of the facial muscles is called hemifacial spasm. One cause is
choline; CGRP is also colocalized in these motor neurons. This
compression of the facial root by an artery, most commonly a loop of
substance is also present in preganglionic and postganglionic para-
the anterior inferior cerebellar artery or its branches. These patients
sympathetic neurons.
may also have deficits (vertigo, tinnitus, hearing loss) suggesting involve-
Clinical Correlations: Patients with myasthenia gravis frequently
ment of the adjacent vestibulocochlear nerve.
have oropharyngeal symptoms and complications that result in
Because of their common origin from NuAm, adjacent exit from the
dysarthria, and dysphagia. These individuals have difficulty chewing and
medulla, and passage through the jugular foramen, the IXth and Xth
swallowing, their jaw may hang open, and the mobility of facial mus-
nerves may be damaged together (as in amyotrophic lateral sclerosis or in
cles is decreased. Imparied hearing (weakness of tensor tympani) and
syringobulbia). The results are dysarthria, dysphagia, dyspnea, loss of taste
hyperacusis (weakness of stapedius) may also be present.
from the ipsilateral caudal tongue, and loss of the gag reflex, but no sig-
Lesions of the Vth nerve (as in meningiomas or trauma) result in 1) loss
nificant autonomic deficits. Bilateral lesions of the Xth nerve are life-
of pain, temperature, and touch on the ipsilateral face and in the oral
threatening because of the resultant total paralysis (and closure) of the
and nasal cavities; 2) paralysis of ipsilateral masticatory muscles ( jaw de-
muscles in the vocal folds (vocalis muscle).
Abbreviations
AbdNu Abducens nucleus
SpTNu Spinal trigeminal nucleus
ALS Anterolateral system
SpTTr Spinal trigeminal tract
BP Basilar pons
SSNu Superior salivatory nucleus
DVagNu Dorsal motor nucleus of vagus
TecSp Tectospinal tract
FacNr Facial nerve
TriMoNu Trigeminal motor nucleus
FacNu Facial nucleus
TriNr Trigeminal nerve
GINr Glossopharyngeal nerve
VagNr Vagus nerve
HyNu Hypoglossal nucleus
ISNu Inferior salivatory nucleus
Ganglia
MesNu Mesencephalic nucleus
1
Pterygopalatine
ML Medial lemniscus
2
Submandibular
MLF Medial longitudinal fasciculus
3
Otic
NuAm Nucleus ambiguus
4
Terminal and/or intramural
PSNu Principal (chief ) sensory nucleus
Review of Blood Supply to TriMoNu, FacNu, DMNu and NuAm,
and the Internal Course of Their Fibers
STRUCTURES
ARTERIES
TriMoNu and Trigeminal Root long circumferential branches of basilar (see Figure 5-21)
FacNu and Internal Genu
long circumferential branches of basilar (see Figure 5-21)
DMNu and NuAm
branches of vertebral and posterior inferior cerebellar
(see Figure 5-14)
Motor Pathways
203
Cranial Nerve Efferents (V, VII, IX, and X)
Position of Nucleus
and Internal
Route of Fibers
TriMotNu
MesNu
MLF
PSNu
TecSp
Motor root
Structures Innervated
ALS
of TriNr
Masticatory muscles and
ML
tensor tympani,
TriMotNu
tensor veli palatini,
Motor root
mylohyoid,
BP
of TriNr
digastric (ant. belly)
AbdNu
SpTTr &
FacNu
Muscles of facial expression,
SpTNu
and stapedius, buccinator,
SSNu
FacNr
stylohyoid, platysma
MLF
digastric (post. belly)
AbdNu
ISNu
GINr
SSNu
1
NuAm
ML
Lacrimal gld.; mucous
FacNr
membranes of nose
and mouth
FacNu
2
Submandibular and
VagNr
sublingual glds.
3
DVagNu
DVagNu
HyNu
Parotid gld.
MLF
Stylopharyngeus
TecSp
NuAm
VagNr
Striated mus. of pharynx,
larynx, esophagus
SpTTr and
ML
4
SpTNu
Thoracic and abdomnal
viscera; smooth and
cardiac muscle; glandular
epithelium
204
Synopsis of Functional Components, Tracts, Pathways, and Systems
Spinocerebellar Tracts
7-17
The origin, course, and distribution pattern of fibers to the
bellar fibers, in their mossy fiber terminals in the cerebellar cortex, and
cerebellar cortex and nuclei from the spinal cord (posterior [dorsal]
in their collateral branches that innervate the cerebellar nuclei.
and anterior [ventral] spinocerebellar tracts, rostral spinocerebellar
Clinical Correlations: Lesions, or tumors, that selectively dam-
fibers) and from the external cuneate nucleus
(cuneocerebellar
age only spinocerebellar fibers are rarely, if ever, seen in humans. The
fibers). Also illustrated is the somatotopy of those fibers originating
ataxia one might expect to see in patients with a spinal cord hemisection
from the spinal cord. These fibers enter the cerebellum via the resti-
(as in the Brown-Sequard syndrome) is masked by the hemiplegia resulting
form body, the larger portion of the inferior cerebellar peduncle, or
from the concomitant damage to lateral corticospinal (and other) fibers.
in relationship to the superior cerebellar peduncle. After these fibers
Friedreich ataxia (hereditary spinal ataxia) is an autosomal recessive dis-
enter the cerebellum, collaterals are given off to the cerebellar nuclei
order the symptoms of which usually appear between 8 and 15 years
while the parent axons of spinocerebellar and cuneocerebellar fibers
of age. There is degeneration of anterior and posterior spinocerebellar
pass on to the cortex, where they end as mossy fibers in the graunular
tracts plus the posterior columns and corticospinal tracts. Degenera-
layer. Although not shown here, there are important ascending spinal
tive changes are also seen in Purkinje cells in the cerebellum, in poste-
projections to the medial and dorsal accessory nuclei of the inferior
rior root ganglion cells, in neurons of the Clarke column, and in some
olivary complex (spino-olivary fibers). The accessory olivary nuclei
nuclei of the pons and medulla. The axial and appendicular ataxia seen
(as well as the principal olivary nucleus) project to the cerebellar cor-
in these patients correlates partially with the spinocerebellar degener-
tex and send collaterals into the nuclei (see Figure 7-18 on page 206).
ation and also partially with proprioceptive losses via the degeneration
Neurotransmitters: Glutamate (
) is found in some spinocere- of posterior column fibers.
Abbreviations
ACNu Accessory (external or lateral) cuneate
PSCT Posterior (dorsal) spinocerebellar tract
nucleus
PSNu Principal (chief ) sensory nucleus of
ALS Anterolateral system
trigeminal nerve
AMV Anterior medullary velum
Py Pyramid
ASCT Anterior (ventral) spinocerebellar tract
RB Restiform body
Cbl Cerebellum
RSCF Rostral spinocerebellar fibers
CblNu Cerebellar nuclei
RuSp Rubrospinal tract
CCblF Cuneocerebellar fibers
S Sacral representation
DNuC Dorsal nucleus of Clarke
SBC Spinal border cells
FNL Flocculonodular lobe
SCP Superior cerebellar peduncle
IZ Intermediate zone
SpTNu Spinal trigeminal nucleus
L Lumbar representation
SpTTr Spinal trigeminal tract
MesNu Mesencephalic nucleus
T Thoracic representation
ML Medial lemniscus
TriMoNu Trigeminal motor nucleus
PRG Posterior (dorsal) root ganglion
VesNu Vestibular nuclei
Review of Blood Supply to Spinal Cord Grey Matter,
Spinocerebellar Tracts, RB, and SCP
STRUCTURES
ARTERIES
Spinal Cord Grey
branches of central artery (see Figure 5-6)
PSCT and ASCT in Cord
penetrating branches of arterial vasocorona (see Figure 5-6)
RB
posterior inferior cerebellar (See Figure 5-14)
SCP
long circumferential branches of basilar and superior
cerebellar (see Figure 5-21)
Cerebellum
posterior and anterior inferior cerebellar and superior
cerebellar
Cerebellum and Basal Nuclei (Ganglia)
205
Spinocerebellar Tracts
Position of SCP
AMV
ASCT
SCP
MesNu
TriMoNu
ML
PSNu
ASCT
on SCP
Lobules II-IV
Lobules II-IV
Ant.
Lobe
Lobule V
Lobule
V
Recrossing ASCT
fibers in Cbl
CblNu
CblNu
RB
RB
FNL
Post. Lobe
CCblF
Lobule VIII
Lobule VIII
ACNu
RSCF
Somatotopy Position
PRG
Lamina VII
VesNu
at C4-C8
PSCT
RB
ASCT
SpTTr & Nu
DNuC
ASCT
ALS + RuSp
Intermediate zone (IZ)
and "spinal border"
Py
cells (SBC)
PRG
DNuC
PSCT
PSCT
T
L
S
IZ
ASCT
L
T
ASCT
SBC
206
Synopsis of Functional Components, Tracts, Pathways, and Systems
Pontocerebellar, Reticulocerebellar, Olivocerebellar, Ceruleocerebellar,
Hypothalamocerebellar, and Raphecerebellar Fibers
7-18
Afferent fibers to the cerebellum from selected brainstem ar-
ticotropin (
)-releasing factor are present in many olivocerebellar
eas and the organization of corticopontine fibers in the internal capsule
fibers. Ceruleocerebellar fibers contain noradrenalin, histamine is
and crus cerebri as shown here. The cerebellar peduncles are also indi-
found in hypothalamocerebellar fibers, and some reticulocerebellar
cated. Pontocerebellar axons are mainly crossed, reticulocerebellar
fibers contain enkephalin. Serotonergic fibers to the cerebellum arise
fibers may be bilateral (from RetTegNu) or mainly uncrossed (from
from neurons found in medial areas of the reticular formation (open
LRNu and PRNu), and olivocerebellar fibers (OCblF) are exclusively
cell in Figure 7-18) and, most likely, from some cells in the adjacent
crossed. Raphecerebellar, hypothalamocerebellar, and ceruleocerebel-
raphe nuclei.
lar fibers are, to varying degrees, bilateral projections. Although all af-
Clinical Correlations: Common symptoms seen in patients with
ferent fibers to the cerebellum give rise to collaterals to the cerebellar
lesions involving nuclei and tracts that project to the cerebellum are
nuclei, those from pontocerebellar axons are relatively small, having
ataxia (of trunk or limbs), an ataxic gait, dysarthria, dysphagia, and dis-
comparatively small diameters. Olivocerebellar axons end as climbing
orders of eye movement such as nystagmus. These deficits are seen in
fibers, reticulocerebellar and pontocerebellar fibers as mossy fibers, and
some hereditary diseases (such as olivopontocerebellar degeneration, ataxia
hypothalamocerebellar and ceruleocerebellar axons end in all cortical
telangiectasia, or hereditary cerebellar ataxia), in tumors
(brainstem
layers. These latter fibers have been called multilayered fibers in the lit-
gliomas), in vascular diseases (lateral pontine syndrome), or in other con-
erature because they branch in all layers of the cerebellar cortex.
ditions such as alcoholic cerebellar degeneration or pontine hemorrhages
Neurotransmitters: Glutamate (
) is found in corticopontine
(see Figure 7-19 on page 208 for more information on cerebellar le-
projections and in most pontocerebellar fibers. Aspartate (
) and cor- sions).
Abbreviations
AntLb Anterior limb of internal capsule
PonNu Pontine nuclei
CblNu Cerebellar nuclei
PO Principal olivary nucleus
CerCblF Ceruleocerebellar fibers
PPon Parietopontine fibers
CPonF Cerebropontine fibers
PRNu Paramedian reticular nuclei
CSp Corticospinal fibers
Py Pyramid
DAO Dorsal accessory olivary nucleus
RB Restiform body
FPon Frontopontine fibers
RCblF Reticulocerebellar fibers
Hyth Hypothalamus
RetLenLb Retrolenticular limb of internal capsule
HythCblF Hypothalamocerebellar fibers
RNu Red nucleus
IC Internal capsule
RetTegNu Reticulotegmental nucleus
LoCer Nucleus (locus) ceruleus
SCP Superior cerebellar peduncle
LRNu Lateral reticular nucleus
SubLenLb Sublenticular limb of internal capsule
MAO Medial accessory olivary nucleus
SN Substantia nigra
MCP Middle cerebellar peduncle
TPon Temporopontine fibers
ML Medial lemniscus
NuRa Raphe nuclei
Number Key
OCblF Olivocerebellar fibers
1
Nucleus raphe, pontis
OPon Occipitopontine fibers
2
Nucleus raphe, magnus
PCbIF Pontocerebellar fibers
3
Raphecerebellar fibers
PostLb Posterior limb of internal capsule
Review of Blood Supply to Precerebellar Relay Nuclei in Pons
and Medulla, MCP, and RB
STRUCTURES
ARTERIES
Pontine Tegmemtum
long circumferential branches of basilar plus some from superior
cerebellar (see Figure 5-21)
Basilar Pons
paramedian and short circumferential branches of basilar (See
Figure 5-21)
Medulla RetF and IO branches of vertebral and posterior inferior cerebellar (see
Figure 5-14)
MCP
long circumferential branches of basilar and branches of anterior
inferior and superior cerebellar (see Figure 5-21)
RB
posterior inferior cerebellar (see Figure 5-14)
Cerebellum and Basal Nuclei (Ganglia)
207
Pontocerebellar, Reticulocerebellar, Olivocerebellar, Ceruleocerebellar,
Hypothalamocerebellar, and Raphecerebellar Fibers
Position of Associated
Tracts and Nuclei
AntLb
(FPon)
PostLb
(PPon)
IC
SubLenLb
Hyth
(TPon)
RetLenLb
(OPon)
CPonF
HythCblF
LoCer
ML
RetTegNu
CerCblF
RNu
SCP
SN
PPon
MCP
OPon
1
TPon
PonNu
FPon
NuRa
PCblF
3
2
CblNu
RetTegNu
OCblF
MCP
ML
DAO
RB
CPonF
RCblF
PCblF
CSp
PO
LRNu
PonNu
PRNu
MAO
PRNu
RB
OCblF
LRNu
PO
Py
OCblF
PCblF
CerCblF
208
Synopsis of Functional Components, Tracts, Pathways, and Systems
Cerebellar Cortioconuclear, Nucleocortical, and Corticovestibular Fibers
7-19
Cerebellar corticonuclear fibers arise from all regions of the
Lesions involving midline structures (vermal cortex, fastigial nu-
cortex and terminate in an orderly (mediolateral and rostrocaudal) se-
clei) and/or the flocculonodular lobe result in truncal ataxia (titubation
quence in the ipsilateral cerebellar nuclei. For example, corticonuclear
or tremor), nystagmus, and head tilting. These patients may also have a
fibers from the vermal cortex terminate in the fastigial nucleus, those
wide-based (cerebellar) gait, are unable to walk in tandem (heel to toe),
from the intermediate cortex terminate in the emboliform and globo-
and may be unable to walk on their heels or on their toes. Generally,
sus nuclei, and those from the lateral cortex terminate in the dentate nu-
midline lesions result in bilateral motor deficits affecting axial and
cleus. Also, cerebellar corticonuclear fibers from the anterior lobe typ-
proximal limb musculature.
ically terminate in more rostral regions of these nuclei while those from
Damage to the intermediate and lateral cortices and the globose,
the posterior lobe terminate more caudally. Cerebellar corticovestibu-
emboliform, and dentate nuclei results in various combinations of the
lar fibers originate primarily from the vermis and flocculonodular lobe,
following deficits: dysarthria, dysmetria (hypometria, hypermetria), dysdi-
exit the cerebellum via the juxtarestiform body, and end in the ipsilat-
adochokinesia, tremor (static, kinetic, intention), rebound phenomenon, un-
eral vestibular nuclei. These projections arise from Purkinje cells.
steady and wide-based (cerebellar) gait, and nystagmus. One of the more
Nucleocortical processes originate from cerebellar nuclear neurons
commonly observed deficits in patients with cerebellar lesions is an in-
and pass to the overlying cortex in a pattern that basically reciprocates
tention tremor, which is best seen in the finger-nose test. The finger-to-fin-
that of the corticonuclear projection; they end as mossy fibers. Some
ger test is also used to demonstrate an intention tremor and to assess
nucleocortical fibers are collaterals of cerebellar efferent axons. The
cerebellar function. The heel-to-shin test will show dysmetria in the lower
cerebellar cortex may influence the activity of lower motor neurons
extremity. If the heel-to-shin test is normal in a patient with his/her
through, for example, the cerebellovestibular-vestibulospinal route.
eyes open, the cerebellum is intact. If this test is repeated in the same
Neurotransmitters: Gamma-aminobutyric acid (GABA) (
) is
patient with eyes closed and is abnormal, this would suggest a lesion in
found in Purkinje cells and is the principal transmitter substance pres-
the posterior column-medial lemniscus system.
ent in cerebellar corticonuclear and corticovestibular projections.
Cerebellar damage in intermittent and lateral areas (nuclei or cor-
However, taurine (
) and motilin (
) are also found in some Purk-
tex plus nuclei) causes movement disorders on the side of the lesion
inje cells. GABA-ergic terminals are numerous in the cerebellar nuclei
with ataxia and gait problems on that side; the patient may tend to fall
and vestibular complex. Some of the glutamate-containing mossy
toward the side of the lesion. This is because the cerebellar nuclei pro-
fibers in the cerebellar cortex represent the endings of nucleocortical
ject to the contralateral thalamus, which projects to the motor cortex
fibers that originate from cells in the cerebellar nuclei.
on the same side, which projects to the contralateral side of the spinal
Clinical Correlations: Numerous disease entities can result in
cord via the corticospinal tract. Other circuits
(cerebellorubal-
cerebellar dysfunction including viral infections (echovirus), hereditary
rubospinal) and feedback loops (cerebelloolivary-olivocerebellar) fol-
diseases (see Figure 7-18), trauma, tumors (glioma, medulloblastoma),
low similar routes. Consequently, the motor expression of unilateral
occlusion of cerebellar arteries (cerebellar stroke), arteriovenous malfor-
cerebellar damage is toward the lesioned side because of these doubly
mation of cerebellar vessels, developmental errors (such as the Dandy-
crossed pathways.
Walker syndrome or the Arnold-Chiari deformity), or the intake of toxins.
Lesions of cerebellar efferent fibers, after they cross the midline in
Usually, damage to only the cortex results in little or no dysfunction
the decussation of the superior cerebellar peduncle, will give rise to
unless the lesion is quite large or causes an increase in intracranial pres-
motor deficits on the side of the body (excluding the head) contralat-
sure. However, lesions involving both the cortex and nuclei, or only
eral to the lesion. This is seen in midbrain lesions such as the Claude syn-
the nuclei, will produce obvious cerebellar signs.
drome.
Abbreviations
CorNu Corticonuclear fibers
MVesSp Medial vestibulospinal tract
CorVes Corticovestibular fibers
MVNU Medial vestibular nucleus
Flo Flocculus
NL, par Lateral cerebellar nucleus, parvocellular
IC Intermediate cortex
region
InfVesNu Inferior (spinal) vestibular nucleus
NM, par Medial cerebellar nucleus,
JRB Juxtarestiform body
parvocellular region
LC Lateral cortex
NuCor Nucleocortical fibers
LVesSp Lateral vestibulospinal tract
SVNu Superior vestibular nucleus
LVNu Lateral vestibular nucleus
VC Vermal cortex
MLF Medial longitudinal fasciculus
Review of Blood Supply to Cerebellum and Vestibular Nuclei
STRUCTURES
ARTERIES
Cerebellar Cortex
branches of posterior and anterior inferior cerebellar and superior
cerebellar
Cerebellar Nuclei
anterior inferior cerebellar and superior cerebellar
Vestibular Nuclei
posterior inferior cerebellar in medulla, long circumferential
branches of basilar in pons
Cerebellum and Basal Nuclei (Ganglia)
209
Cerebellar Corticonuclear, Nucleocortical, and Corticovestibular Fibers
NuCor
VC
IC
CorNu
CorVes
NuCor
4
2
3
CorNu
LC
1
NM, par
Nodulus
NL, par
JRB
Flo
SVNu
LVNu
MLF
InfVNu
MVNu
LVesSp
MVesSp
Cerebellar Nuclei:
1= Medial (Fastigial)
2= Posterior Interposed (Globose)
3= Anterior Interposed (Emboliform)
4= Lateral (Dentate)
210
Synopsis of Functional Components, Tracts, Pathways, and Systems
Cerebellar Efferent Fibers
7-20
The origin, course, topography, and general distribution of
belloreticular-reticulospinal,
3) cerebellothalamic-thalamocortical-
fibers arising in the cerebellar nuclei. Cerebellofugal fibers project to
corticospinal, and others. In addition, some direct cerebellospinal
several thalamic areas (VL and VA), to intralaminar relay nuclei in ad-
fibers arise in the fastigial nucleus as well as in the interposed nuclei.
dition to the centromedian, and to a number of midbrain, pontine, and
Neurotransmitters: Many cells in the cerebellar nuclei contain
medullary targets. Most of the latter nuclei project back to the cere-
glutamate (
), aspartate (
), or gamma-aminobutyric acid (
). Glu-
bellum (e.g., reticulocerebellar, pontocerebellar), some in a highly or-
tamate and aspartate are found in cerebellorubral and cerebellothala-
ganized manner. For example, cerebello-olivary fibers from the den-
mic fibers, whereas some GABA-containing cells give rise to cerebel-
tate nucleus (DNu) project to the principal olivary nucleus (PO), and
lopontine and cerebello-olivary fibers. Some cerebelloreticular
neurons of the PO send their axons back to the lateral cerebellar cor-
projections may also contain GABA.
tex, with collaterals going to the DNu.
Clinical Correlations: Lesions of the cerebellar nuclei result in a
The cerebellar nuclei can influence motor activity through, as ex-
range of motor deficits depending on the location of the injury. Many
amples, the following routes: 1) cerebellorubral-rubrospinal, 2) cere- of these are described in Figure 7-19 on page 208.
Abbreviations
ALS
Anterolateral system
OcNu
Oculomotor nucleus
AMV
Anterior medullary velum
PO
Principal olivary nucleus
BP
Basilar pons
PonNu
Pontine nuclei
CblOl
Cerebello-olivary fibers
RetForm
Reticular formation
CblTh
Cerebellothalamic fibers
RNu
Red nucleus
CblRu
Cerebellorubral fibers
RuSp
Rubrospinal tract
CC
Crus cerebri
SC
Superior colliculus
CeGy
Central grey (periaqueductal grey)
SCP
Superior cerebellar peduncle
CM
Centromedian nucleus of thalamus
SCP, Dec
Superior cerebellar peduncle, decussation
CSp
Corticospinal fibers
SN
Substantia nigra
DAO
Dorsal accessory olivary nucleus
SVNu
Superior vestibular nucleus
DNu
Dentate nucleus (lateral cerebellar nucleus)
ThCor
Thalamocortical fibers
ENu
Emboliform nucleus (anterior interposed
ThFas
Thalamic fasciculus
cerebellar nucleus)
TriMoNu
Trigeminal motor nucleus
EWNu
Edinger-Westphal nucleus
VL
Ventral lateral nucleus of thalamus
FNu
Fastigial nucleus (medial cerebellar nucleus)
VPL
Ventral posterolateral nucleus of thalamus
GNu
Globose nucleus (posterior interposed
VSCT
Ventral spinocerebellar tract
cerebellar nucleus)
ZI
Zona incerta
IC
Inferior colliculus
InfVNu
Inferior (spinal) vestibular nucleus
Number Key
INu
Interstitial nucleus
1
Ascending projections to superior
LRNu
Lateral reticular nucleus
colliculus, and possibly ventral lateral and
LVNu
Lateral vestibular nucleus
ventromedial thalamic nuclei
MAO
Medial accessory olivary nucleus
2
Descending crossed fibers from superior
ML
Medial lemniscus
cerebellar peduncle
MLF
Medial longitudinal fasciculus
3
Uncinate fasciculus (of Russell)
MVNu
Medial vestibular nucleus
4
Juxtarestiform body to vestibular nuclei
NuDark
Nucleus of Darkschewitsch
5
Reticular formation
Review of Blood Supply to Cerebellar Nuclei and
Their Principal Efferent Pathways
STRUCTURES
ARTERIES
Cerebellar Nuclei
anterior inferior cerebellar and superior cerebellar
SCP
long circumferential branches of basilar and superior cerebellar
(see Figure 5-21)
Midbrain Tegmemtum
paramedian branches of basilar bifurcation, short circumferential
(RNu, CblTh,
branches of posterior cerebral, branches of superior cerebellar
CblRu, OcNu)
(see Figure 5-27)
VPL, CM, VL, VA
thalamogeniculate branches of posterior cerebral, thalamo-
perforating branches of the posteromedial group of posterior
cerebral (see Figure 5-38)
IC
lateral striate branches of middle cerebral (see Figure 5-38)
Cerebellum and Basal Nuclei (Ganglia)
211
Cerebellar Efferent Fibers
CSp
VL
ThCor
CM
VPL
ThFas
Zl
Position of SCP,
NuDark, INu,
CblTh, and CblRu
OcNu, EWNu
RNu
SC
CeGy
CeGy
1
SCP
ML
CblTh &
2
RetForm
RNu
CblRu
3
4
CC
SN
PonNu
DNu
IC
FNu
SVNu
MLF
5
ML
ENu
GNu
CblOl
SN
LVNu
5
SCP, Dec
InfVNu
LRNu
MVNu
5
VSCT
AMV
DAO
SCP
5
PO
TriMoNu
MAO
ALS & RuSp
ML
BP
Cerebellospinal
fibers
212
Synopsis of Functional Components, Tracts, Pathways, and Systems
7-21
Blank master drawing for pathways projecting to the cere-
bellar cortex, and for efferent projections of cerebellar nuclei. This il-
lustration is provided for self-evaluation of understanding of pathways
to the cerebellar cortex and from the cerebellar nuclei, for the in-
structor to expand on cerebellar afferent/efferent pathways not cov-
ered in the atlas, or both.
Cerebellum and Basal Nuclei (Ganglia)
213
214
Synopsis of Functional Components, Tracts, Pathways, and Systems
Striatal Connections
7-22
The origin, course, and distribution of afferent fibers to, and
enkephalinergic cells in the neostriatum (primarily the caudate) and
efferent projections from, the neostriatum. These projections are ex-
cell loss in the cerebral cortex. Loss of neostriatal cell terminals in the
tensive, complex, and in large part, topographically organized; only
lateral and medial segments of the globus pallidus correlate, respec-
their general patterns are summarized here. Afferents to the caudate
tively, with the development of choreiform movements and later with
and putamen originate from the cerebral cortex (corticostriate fibers),
rigidity and dystonia. Loss of cortical neurons correlate, respectively,
from several of the intralaminar thalamic nuclei (thalamostriate), from
with personality changes and eventual dementia. Huntington chorea is
the substantia nigra-pars compacta (nigrostriate), and from some of the
rapid, unpredictable, and may affect muscles of the extremities, face,
raphe nuclei. Neostriatal cells send axons into the globus pallidus (pa-
and trunk; abnormal movements seem to flow through the body. Pa-
leostriatum) as striopallidal fibers and into the substantia nigra pars
tients commonly attempt to mask the abnormal movement by trying
reticulata as a strionigral projection.
to make it appear to be part of an intended movement (parakinesia).
Neurotransmitters: Glutamate (
) is found in corticostriate
Symptoms in Wilson disease (hepatolenticular degeneration) appear in
fibers, and serotonin is found in raphestriatal fibers from the nucleus
persons between 10 to 20 years of age. Copper accumulates in the basal
raphe dorsalis. Four neuroactive substances are associated with striatal
nuclei (ganglia) and the frontal cortex, with resultant spongy degener-
efferent fibers, these being gamma-aminobutyric acid (GABA)(
),
ation in the putamen and cortex. These patients may show athetoid
dynorphin, enkephalin(
), and substance P(
). Enkephalinergic and
movements, rigidity and spasticity, dysarthria, dysphagia, contractures, and
GABA-ergic striopallidal projections are numerous to the lateral pal-
tremor. A unique movement of the hand and/or upper extremity in
lidum (origin of pallidosubthalamic fibers), while dynorphin-contain-
these patients is called a flapping tremor (asterixis) sometimes described
ing terminals are more concentrated in its medial segment (source of
as a wing-beating tremor. Copper can also be seen in the cornea
pallidothalamic fibers). Enkephalin and GABA are also present in stri-
(Kayser-Fleischer ring) in these patients.
onigral projections to the pars reticulata. Because substance P and
In Parkinson disease (onset at 50 to 60 years of age), there is a pro-
GABA are found in striopallidal and strionigral fibers, some of the for-
gressive loss of dopaminergic cells in the substantia nigra-pars com-
mer may be collaterals of the latter. Dopamine is present in nigrostri-
pacta, of their terminals in the caudate and putamen, and of their den-
atal projection neurons and in their terminals in the neostriatum.
drites that extend into the substantia nigra-pars reticulata. Patients
Clinical Correlations: Degenerative changes and neuron loss in
with Parkinson disease characteristically show a resting tremor (pill-
the caudate nucleus and putamen result in movement disorders. Ex-
rolling), rigidity (cog wheel or lead pipe), and bradykinesia or hypokinesia.
amples are seen in Sydenham chorea (rheumatic chorea), Huntington disease
The slowness of movement may also be expressed in speech (dysarthria,
(a dominantly inherited disease), and Wilson disease (a genetic error in
hypophonia, tachyphonia) and in writing (micrographia). These patients
the patient’s ability to metabolize copper). In persons with Parkinson
have a distinct stooped flexed posture and a festinating gait. Behavioral
disease, a loss of the dopamine-containing cells in the pars compacta of
changes are also seen. Parkinson disease and Huntington disease are
the substantia nigra and of their nigrostriatal terminals in the caudate
progressive neurodegenerative disorders.
nucleus and putamen occurs.
Dystonia, a movement disorder seen in some patients with basal nu-
Sydenham chorea is a disease usually seen in children between 5 and
clei disease, is characterized by increased/sustained muscle contrac-
15 years of age, resulting from infection with hemolytic streptococcus.
tions that cause twisting of the trunk or extremities resulting in abnor-
The choreiform movements are brisk and flowing, irregular, and may in-
mal posture. These patients may also have unusual and repetitive
volve muscles of the limbs, face, oral cavity, and trunk. Dystonia may
movements of the extremities or of the neck (cervical dystonia or spas-
be seen; muscle weakness is common. In most patients, the disease re-
modic torticollis). Dystonia may be an inherited progressive disease or
solves following successful treatment of the infection.
have other causes and may be seen in children or young adults. The
Huntington disease is a progressive genetic disorder the symptoms of
symptoms may initially appear during movements or when talking but
which appear at 35 to 45 years of age. There is loss of GABA-ergic and
in later stages may be present at rest.
Abbreviations
CaNu Caudate nucleus
SNpc Substantia nigra, pars compacta
CorSt Corticostriate fibers
SNpr Substantia nigra, pars reticulata
GPL Globus pallidus, lateral segment
StNig Striatonigral fibers
GPM Globus pallidus, medial segment
StPal Striatopallidal fibers
NigSt Nigrostriatal fibers
SThNu Subthalamic nucleus
Put Putamen
ThSt Thalamostriatal fibers
RaNu Raphe nuclei
ZI Zona incerta
RaSt Raphestriatal fibers
Review of Blood Supply to Caudate, Putamen, SN, CC, and IC
STRUCTURES
ARTERIES
Caudate, Putamen and IC
medial striate a. for head of caudate and lateral striate
branches of middle cerebral for Put and IC (see Figure
5-38)
SN and CC
paramedian branches of basilar bifurcation, short
circumferential branches of posterior cerebral and some
from superior cerebellar (see Figure 5-27)
Cerebellum and Basal Nuclei (Ganglia)
215
Striatal Connections
Cerebral cortex
CorSt
CorSt
Ca,Nu
StPal
ThSt
Intralaminar
nuclei
Put
StPal
NigSt
CorSt
Zl
SThNu
GPL
NigSt
StNig
GPM
SNpc
SNpr
RaSt
RaNu
216
Synopsis of Functional Components, Tracts, Pathways, and Systems
Pallidal Efferents and Nigral Connections
7-23
The origin, course, and distribution of efferent projections
rons, which give rise to nigrostriatal, nigroamygdaloid, and several
of the globus pallidus (upper illustration), and connections of the sub-
other projections; GABA in pars reticulata cells, which give rise to ni-
stantia nigra (lower drawing) that were not shown in relation to the
grocollicular and nigrothalamic fibers; and glycine in some local circuit
pallidum or in Figure 7-22 on page 215. The ansa lenticularis (dashed
nigral neurons. Glutamate (
) is found in corticonigral fibers, and
line) arches around the internal capsule and passes caudally to join in
serotonin (
) is associated with raphenigral fibers; these latter fibers
the formation of the thalamic fasciculus. Pallidosubthalamic fibers orig-
originate primarily from the nucleus raphe dorsalis.
inate primarily from the lateral pallidal segment, but pallidothalamic
The dopaminergic projections to the frontal cortex, shown here as
projections, via the ansa lenticularis and lenticular fasciculus, arise
arising only from SNpc, originates from this cell group as well as from
mainly from its medial segment. The substantia nigra has extensive
the immediately adjacent ventral tegmental area. Excessive activity in
connections, the clinically most important being the dopaminergic ni-
neurons comprising this projection may play a partial role in schizo-
grostriatal fibers. The globus pallidus influences motor activity by way
phrenia.
of pallidothalamic-thalamocortical-corticospinal (and corticonuclear
Clinical Correlation: Movement disorders associated with le-
[corticobulbar]) pathways.
sions in the neostriatum and substantia nigra are reviewed in Figure
Neurotransmitters: Gamma-aminobutyric acid (
)-containing
7-22 on page 214. Hemorrhage into, the occlusion of vessels serving
cells in the globus pallidus give rise to pallidonigral projections, which
or a tumor within, the subthalamic nucleus will result in violent flail-
end primarily in the substantia nigra-pars reticulata. Although GABA
ing movements of the extremities, a condition called hemiballismus.
is also found in some subthalamopallidal axons, this latter projection
Hemiballistic movements are seen contralateral to the lesion because
contains many glutaminergic (
) fibers. Dopamine-containing, GABA
the motor expression of this lesion is through the corticospinal tract.
(
)-containing, and glycine (
)-containing cells are present in the
Lesions confined to the globus pallidus, as in hemorrhage of lenticu-
substantia nigra. Of these, dopamine is found in pars compacta neu-
lostriate arteries, may result in hypokinesia and rigidity without tremor.
Abbreviations
AmyNig Amygdalonigral fibers
PedPonNu Pedunculopontine nucleus
AmyNu Amygdaloid nucleus (complex)
Put Putamen
AnLent Ansa lenticularis
RaNu Raphe nuclei
CaNu Caudate nucleus
SC Superior colliculus
CM Centromedian nucleus of thalamus
SNpc Substantia nigra, pars compacta
CorNig Corticonigral fibers
SNpr Substantia nigra, pars reticulata
CSp Corticospinal fibers
SThFas Subthalamic fasciculus
GPL Globus pallidus, lateral segment
SThNig Subthalamonigral fibers
GPM Globus pallidus, medial segment
SthNu Subthalamic nucleus
LenFas Lenticular fasciculus (H2)
ThCor Thalamocortical fibers
NigAmy Nigroamygdaloid fibers
ThFas Thalamic fasciculus (H1)
NigCol Nigrocollicular fibers
VA Ventral anterior nucleus of thalamus
NigTec Nigrotectal fibers
VL Ventral lateral nucleus of thalamus
NigSTh Nigrosubthalamic fibers
VM Ventromedial nucleus of thalamus
NigTh Nigrothalamic fibers
ZI Zona incerta
PalNig Pallidonigral fibers
Review of Blood Supply to Pallidum, Subthalamic Area, and SN
STRUCTURES
ARTERIES
GPM/GPL lateral striate branches of middle cerebral and branches of anterior choroidal
(see Figure 5-38)
SThNu
posteromedial branches of posterior cerebral and posterior communicating
(see Figure 5-38)
SN
branches of basilar bifurcation, medial branches of posterior cerebral and
posterior communicating, short circumferential branches of posterior
cerebral (see Figure 5-27)
Cerebellum and Basal Nuclei (Ganglia)
217
Pallidal Efferents and Nigral Connections
Motor cortex
ThCor
CaNu
VA
Intralaminar
nuclei
VL
Put
ThFas
CSp
LenFas
GPL
Zl
Forel's Field H
SThNu
GPM
SThNig and
Nig Sth
SThFas
AnLent
SNpc
PalNig
SNpr
PedPonNu
CorNig
Nigral efferents
to frontal cortex
Ca,Nu
VM
VL
Put
NigTh
GPL
Zl
GPM
SthNu
Nigral efferents to
SC
olfactory tubercle
and bed nucleus of
the stria terminalis
SNpc
AmyNu
NigCol
NigAmy and AmyNig
SNpr
RaNu
218
Synopsis of Functional Components, Tracts, Pathways, and Systems
7-24
Blank master drawing for connections of the basal ganglia.
This illustration is provided for self-evaluation of understanding of
basal ganglia connections, for the instructor to expand on basal nuclei
pathways not covered in this atlas, or both.
Cerebellum and Basal Nuclei (Ganglia)
219
220
Synopsis of Functional Components, Tracts, Pathways, and Systems
Pupillary Pathways
7-25
The origin, course, and distribution of fibers involved in the
ducing a bitemporal hemianopsia) or the uncrossed fibers in the right (or
pathway for the pupillary light reflex. In addition, the pathway for sym-
left) side of the optic chiasm. These lateral lesions produce a right (or
pathetic innervation of the dilator muscle of the iris is also depicted. The
left) nasal hemianopsia.
pathway from the midbrain reticular formation to the intermediolateral
Optic (geniculocalcarine) radiations (see Figures 7-26 and 7-27 on
cell column may also have a multisynaptic component, with relay sta-
pages 222 and 223) may pass directly caudal to the upper lip (cuneus)
tions in the pontine and medullary reticular formations. Postganglionic
of the calcarine sulcus or follow an arching route (the Meyer, or Meyer-
sympathetic fibers to the head originate from the superior cervical gan-
Archambault loop) through the temporal lobe to the lower bank (lingual
glion. Although not shown, descending projections to the intermedio-
gyrus) of the calcarine sulcus. Temporal lobe lesions involving the
lateral cell column also originate from various hypothalamic areas and
Meyer-Archambault loop, or involving fibers entering the lingual
nuclei (hypothalamospinal fibers), some of which receive retinal input.
gyrus, can produce a homonymous superior quadrantanopia. A homonymous
Neurotransmitters: Acetylcholine is the transmitter found in
inferior quandrantanopia is seen in patients with damage to upper (pari-
the preganglionic and postganglionic autonomic fibers shown in this il-
etal) parts of the geniculocalcarine radiations or to these fibers as they
lustration. In addition, N-acetylaspartylglutamate is present in some
enter the cuneus.
retinal ganglion cells (retinogeniculate projections).
Damage to the visual cortex adjacent to the calcarine sulcus (distal
Clinical Correlations: Total or partial blindness in one or both
posterior cerebral artery occlusion) results in a right (or left) homony-
eyes may result from a variety of causes (such as gliomas, meningiomas,
mous hemianopsia. With the exception of macular sparing, this deficit is
strokes, aneurysms, infections, and demyelinating diseases); lesions may
the same as that seen in optic tract lesions.
occur at any locus along the visual pathway. A complete lesion (for ex-
Vascular lesions (as in the lateral medullary syndrome), tumors (such
ample, a transection) of the optic nerve will result in blindness and loss
as brainstem gliomas), or syringobulbia may interrupt the descending pro-
of the pupillary light reflex (direct response) in the eye on the injured
jections from hypothalamus (hypothalamospinal fibers) and midbrain
side and a loss of the pupillary light reflex (consensual response) in the
to the intermediolateral cell column at upper thoracic levels. This may
opposite eye when shining a light in the blind eye. On the other hand, shin-
result in a Horner syndrome (ptosis, miosis, and anhidrosis) on the ipsilat-
ing a light in the normal eye will result in a pupillary light reflex (direct
eral side. The enophthalmos (a slight sinking of the eyeball into the or-
response) in that eye and a consensual response in the blind eye. A pi-
bit) frequently mentioned in relation to Horner syndrome is not really
tuitary adenoma may damage the crossing fibers in the optic chiasm (pro-
very apparent in afflicted patients.
Abbreviations
CC Crus cerebri
PoCom Posterior commissure
CilGang Ciliary ganglion
PrTecNu Pretectal nucleus
EWNu Edinger-Westphal nucleus
PulNu Pulvinar nuclear complex
ILCC Intermediolateral cell column
RetF Reticular formation
LGNu Lateral geniculate nucleus
RNu Red nucleus
MGNu Medial geniculate nucleus
SC Superior colliculus
ML Medial lemniscus
SC,Br Superior colliculus, brachium
OcNr Oculomotor nerve
SCerGang Superior cervical ganglion
OpCh Optic chiasm
SN Substantia nigra
OpNr Optic nerve
WRCom White ramus communicans
OpTr Optic tract
Review of Blood Supply to OpTr, MGB, LGB, SC,
and Midbrain Tegementum, Including PrTecNu
STRUCTURES
ARTERIES
OpTr
anterior choroidal (see Figure 5-38)
MGNu, LGNu
thalamogeniculate branches of posterior cerebral (see Figure 5-38)
SC and PrTecNu
long circumferential branches (quadrigeminal) of posterior cerebral,
posterior choroidal, and some from superior cerebellar (to SC) (see
Figures 5-27 and 5-38)
Midbrain
paramedian branches of basilar bifurcation, medial branches
Tegmentum
of posterior cerebral and posterior communicating, short
circumferential branches of posterior cerebral (see Figure 5-27)
Optic, Auditory, and Vestibular Systems
221
Pupillary Pathways
Dilator muscles of iris
Sphincter mus. of iris
Sphincter mus. of ciliary body
Ganglion cells
of retina
CilGang
OpNr
OcNr
OpCh
OpTr
OcNr
Via blood
Midbrain
vessels
RetF
CC
RNu
ML
LGNu
SN
LGNu
MGNu
MGNu
SC,Br
SC
SC,Br
PulNu
PulNu
PoCom
SCerGang
PrTecNu
EWNu
HySpF
ILCC
Spinal nerve
Thoracic cord
WRCom
T1-T3
Anterior root
222
Synopsis of Functional Components, Tracts, Pathways, and Systems
Visual Pathways
Visual fields
Ganglion cells
of retina
Retinae
LGNu Laminae
Optic nerve
1
mc
2
3
Optic chiasm
4
pc
5
Optic tract
6
Red nucleus
Substantia nigra
Crus cerebri
Medial lemniscus
Meyer's Loop
LGNu
MGNu
MGNu
SC,Br
SC,Br
PulNu
PulNu
Oculomotor Nu.
Sup. colliculus
Edinger-Westphal Nu.
Pretectal Nu.
Optic radiations
(in retrolenticular
limb of internal
Cuneus
capsule)
Lingual gyrus
CalSul
7-26
The origin, course, and distribution of the visual pathway are
Neurotransmitters: Cholecystokinin (
) is present in some
shown. Uncrossed retinogeniculate fibers terminate in laminae 2, 3,
geniculocalcarine fibers. N-acetylaspartylglutamate is found in some
and 5, while crossed fibers end in laminae 1, 4, and 6. Geniculocal-
retinogeniculate fibers, and in some lateral geniculate and visual cor-
carine fibers arise from laminae 3 through 6. Retinogeniculate and
tex neurons.
geniculocalcarine pathways are retinotopically organized (see facing
Clinical Correlations: Deficits seen following lesions of various
page).
parts of visual pathways are described in Figure 7-25 on p. 220.
Abbreviations
CalSul Calcarine sulcus
MGNu Medial geniculate nucleus
LGNu Lateral geniculate nucleus
PulNu Pulvinar nuclear complex
mc Magnocellular
SC,Br Superior colliculus, brachium
pc Parvocellular
Optic, Auditory, and Vestibular Systems
223
Visual Pathways
LEFT
RIGHT
B'
A'
B
A
Visual fields
Orientation fo all Levels
overlapped for
M
Except Visual Cortex
both eyes
D'
C'
C
D
Dor
Dor
Lat
Med
Lat
Ven
Ven
B'
A'
B
A
B
A
Visual fields for
individual eyes
M
M
C
D
C
D
C'
D'
C'
D'
D
C
D
C
Retinae
M
M
A
B
A
B
B'
A'
C'
D'
D
C
D
C
Optic nerves
M
M
A
B
A
B
B'
A'
Optic chiasm
A
B
M
Optic
M
tracts
A'
C
D
B'
D'
C'
M
Lateral
D
geniculate
C
M
nuclei
A
D'
C'
B
A'
B'
D'
C'
D
C
M
M
Optic radiations
A
B
A'
B'
Primary visual cortex
C'
D'
Cuneus
C
M
M D
Calcarine sulcus
B
M
M A
B'
A'
Lingual gyrus
7-27
Semidiagrammatic representation of the retinographic
Clinical Correlations: Deficits seen following lesions of various
arrangement of visual and retinal fields, and the subsequent topogra-
parts of the visual pathway are described in Figure 7-25 on p. 220.
phy of these projections throughout the visual system. Upper-case let-
ters identify the binocular visual fields (A, B, C, D), the macula (M),
and the monocular visual fields (A , B , C , D ).
224
Synopsis of Functional Components, Tracts, Pathways, and Systems
7-28
Blank master drawing of visual pathways. This illustration is
provided for self-evaluation of visual pathway understanding, for the
instructor to expand on aspects of the visual pathways not covered in
the atlas, or both.
Optic, Auditory, and Vestibular Systems
225
226
Synopsis of Functional Components, Tracts, Pathways, and Systems
Auditory Pathways
7-29
The origin, course, and distribution of the fibers collectively
hearing loss and conduction hearing loss, and to lateralize the deficit. In
composing the auditory pathway. Central to the cochlear nerve and
the Weber test, a tuning fork (512 Hz) is applied to the midline of the fore-
dorsal and ventral cochlear nuclei this system is, in a general sense, bi-
head or apex of the skull. In the normal patient, the sound (conducted
lateral and multisynaptic, as input is relayed through brainstem nuclei
through the bones of the skull) is heard the same in each year. In the case
en route to the auditory cortex. Synapse and crossing (or re-crossing)
of nerve deafness (lesions of the cochlea or cochlear nerve), the sound is best
of information can occur at several levels in the neuraxis. Conse-
heard in the normal ear, while in conductive deafness, the sound is best heard
quently, central lesions rarely result in a total unilateral hearing loss.
in the abnormal ear. In the Rinne test, a tuning fork (512 Hz) is placed
The medial geniculate body is the thalamic station for the relay of au-
against the mastoid process. When the sound is no longer perceived, the
ditory information to the temporal cortex.
prongs are moved close to the external acoustic meatus, where the sound
Neurotransmitters: Glutamate (
) and aspartate (
) are found in
is again heard; this is the situation in a normal individual (positive Rinne
some spiral ganglion cells and in their central terminations in the cochlear
test). In middle ear disease, the sound is not heard at the external meatus
nuclei. Dynorphin-containing and histamine-containing fibers are also
after it has disappeared from touching the mastoid bone (abnormal or neg-
present in the cochlear nuclei; the latter arises from the hypothalamus. A
ative Rinne test). Therefore, a negative Rinne test signifies conductive
noradrenergic projection to the cochlear nuclei and to the inferior col-
hearing loss in the ear tested. In mild nerve deafness (cochlea or cochlear
liculus originates from the nucleus locus ceruleus. Cells in the superior
nerve lesions), the sound is heard by application of the tuning fork to the
olive that contain cholecystokinin and cells in the nuclei of the lateral lem-
mastoid and movement to the ear (the Rinne test is positive). In severe
niscus that contain dynorphin project to the inferior colliculus. Although
nerve deafness, the sound may not be heard at either position.
the olivocochlear bundle is not shown, it is noteworthy that enkephalin is
In addition to hearing loss and tinnitus, large vestibular schwannomas
found in some of the cells that contribute to this projection.
may result in other signs and symptoms. These include nausea, vomiting
Clinical Correlations: There are three categories of deafness.
and ataxia/unsteady gait (vestibular root involvement), weakness of fa-
Conductive deafness is due to problems of the external ear (obstruction
cial muscles (facial root involvement), and altered sensation from the
of the canal, wax build-up) or disorders of the middle ear (otitis media,
face and a diminished corneal reflex (trigeminal root involvement).
otosclerosis). Nerve deafness (sensorineural hearing loss) results from diseases
There may also be general signs associated with increased intracranial
involving the cochlea or the cochlear portion of the vestibulocochlear
pressure (lethargy, headache, and vomiting).
nerve. Central deafness results from damage to the cochlear nuclei or
Central lesions (as in gliomas or vascular occlusions) rarely produce
possibly their central connections.
unilateral or bilateral hearing losses that can be detected, the possible
Hearing loss may result from trauma (such as fracture of the petrous
exception being pontine lesions that damage the trapezoid body and
bone), demyelinating diseases, tumors, certain medications (strepto-
nuclei. Injury to central auditory pathways and/or primary auditory
mycin), or occlusion of the labyrinthine artery. Damage to the cochlear
cortex may diminish auditory acuity, decrease the ability to hear cer-
part of the VIIIth nerve (as in vestibular schwannoma) results in tinnitus
tain tones, or make it difficult to precisely localize sounds in space. Pa-
and/or deafness (partial or total) in the ipsilateral ear. High-frequency
tients with damage to secondary auditory cortex in the temporal lobe
hearing losses are most common.
experience difficulty in understanding and/or interpreting sounds (au-
The Weber test and Rinne test are used to differentiate between neural
ditory agnosia).
Abbreviations
AbdNu Abducens nucleus
MLF Medial longitudinal fasciculus
ACNu Anterior (ventral) cochlear nucleus
PCNu Posterior (dorsal) cochlear nucleus
ALS Anterolateral system
PulNu Pulvinar nuclear complex
CC Crus cerebri
RB Restiform body
FacNu Facial nucleus
RetF Reticular formation
IC Inferior colliculus
SC Superior colliculus
IC,Br Inferior colliculus, brachium
SCP,Dec Superior cerebellar peduncle, decussation
IC,Com Inferior colliculus, commissure
SO Superior olive
IC,SL Internal capsule, sublenticular limb
SpGang Spiral ganglion
LGNu Lateral geniculate nucleus
SpTTr Spinal trigeminal tract
LL Lateral lemniscus
TrapB Trapezoid body
LL,Nu Lateral lemniscus, nucleus
TrapNu Trapezoid nucleus
MGNu Medial geniculate nucleus
TTGy Transverse temporal gyrus
ML Medial lemniscus
Review of Blood Supply to Cochlear Nuclei, LL (and associated
structures), Pontine Tegmentum, IC, and MGB
STRUCTURES
ARTERIES
Cochlear Nuclei
anterior inferior cerebellar (see Figure 5-14)
LL, SO in Pons
long circumferential branches of basilar (see Figure 5-21)
IC
long circumferential branches (quadrigeminal branches) of basilar,
superior cerebellar (see Figure 5-27)
MGB
thalamogeniculate branches of posterior cerebral (see Figure 5-38)
Optic, Auditory, and Vestibular Systems
227
Auditory Pathways
PulNu
LGNu
MGNu
TTGY
Positions of LL and
Related Structures
IC,SL
IC,Br
IC,Com
SC
IC
IC,Com
LL
IC
ALS
LL,Nu
CC
ML
SCP,Dec
FacNu
LL
LL
SpTTr
FacNu
LL
SO
RetF
ALS
PCNu
SO
ML
TrapNu
TrapB
ACNu
RetF
SpGang
PCNu
ACNu
Hair cells in
LL
organ of corti
RB
LL
SO
ML
TrapB
228
Synopsis of Functional Components, Tracts, Pathways, and Systems
Vestibular Pathways
7-30
The origin, course, and distribution of the main afferent and
Clinical Correlations: The vestibular part of the VIIIth nerve can
efferent connections of the vestibular nuclei (see also Figures 7-13,
be damaged by many of the same insults that affect the cochlear nerve
7-19, and 7-20). Primary vestibular afferent fibers may end in the
(see Figure 7-29). Damage to vestibular receptors of the vestibular nerve
vestibular nuclei or pass to cerebellar structures via the juxtarestiform
commonly results in vertigo. The patient may feel that his or her body is
body. Secondary vestibulocerebellar axons originate from the vestibu-
moving (subjective vertigo) or that objects in the environment are moving
lar nuclei and follow a similar path to the cerebellum. Efferent projec-
(objective vertigo). They have equilibrium problems, an unsteady (ataxic)
tions from the vestibular nuclei also course to the spinal cord through
gait, and a tendency to fall to the lesioned side. Deficits seen in nerve le-
vestibulospinal tracts (see Figure 7-13), as well as to the motor nuclei
sions—or in brainstem lesions involving the vestibular nuclei, include
of the oculomotor, trochlear, and abducens nerves via the MLF. Cere-
nystagmus, nausea, and vomiting, along with vertigo and gait problems. A
bellar structures most extensively interconnected with the vestibular
facial palsy may also appear in concert with VIIIth nerve damage in pa-
nuclei include the lateral regions of the vermal cortex of anterior and
tients who have a vestibular schwannoma. These vestibular deficits, along
posterior lobes, the flocculonodular lobe, and the fastigial (medial)
with partial or complete deafness, are seen in Ménière disease.
cerebellar nucleus.
Lesions of those parts of the cerebellum with which the vestibular
Neurotransmitters: Gamma-aminobutyric (
) is the transmit-
nerve and nuclei are most intimately connected (flocculonodular lobe and
ter associated with many cerebellar corticovestibular fibers and their
fastigial nucleus) result in nystagmus, truncal ataxia, ataxic gait, and a
terminals in the vestibular complex; this substance is also seen in cere-
propensity to fall to the injured side. The nystagmus seen in patients with
bellar corticonuclear axons. The medial vestibular nucleus also has
vestibular lesions and the internuclear ophthalmoplegia seen in some patients
fibers that are dynorphin-positive and histamine-positive; the latter
with multiple sclerosis are signs that correlate with the interruption of
arise from cells in the hypothalamus.
vestibular projections to the motor nuclei of III, IV, and VI via the MLF.
Abbreviations
AbdNu Abducens nucleus
PAG Periaqueductal gray
ALS Anterolateral system
Py Pyramid
Cbl Cerebellar
RB Restiform body
Cbl-CoVes Cerebellar corticovestibular fibers
RNu Red nucleus
CblNu Cerebellar nuclei
SC Superior colliculus
HyNu Hypoglossal nucleus
SCP,Dec Superior cerebellar peduncle,
IC Inferior colliculus
decussation
InfVNu Inferior (spinal) vestibular nucleus
SN Substantia nigra
JRB Juxtarestiform body
SolNu Solitary nucleus
LVesSp Lateral vestibulospinal tract
SolTr Solitary tract
LVNu Lateral vestibular nucleus
SpTTr Spinal trigeminal tract
MesNu Mesencephalic nucleus
SVNu Superior vestibular nucleus
ML Medial lemniscus
TroNu Trochlear nucleus
MLF Medial longitudinal fasciculus
VesGang Vestibular ganglion
MVesSp Medial vestibulospinal tract
VesCbl,Prim Vestibulocerebellar fibers, primary
MVNu Medial vestibular nucleus
VesCbl,Sec Vestibulocerebellar fibers, secondary
OcNu Oculomotor nucleus
Review of Blood Supply to Vestibular Nuclei, TroNu, and OcNu
STRUCTURES
ARTERIES
Vestibular Nuclei
posterior inferior cerebellar in medulla (see Figure 5-14), long
circumferential branches of basilar in pons (see Figure 5-21)
TroNu and OcNu
paramedian branches of basilar bifurcation, medial branches of
posterior cerebral and posterior communicating, short
circumferential branches of posterior cerebral (see Figure 5-27)
Optic, Auditory, and Vestibular Systems
229
Vestibular Pathways
Position of Vestibular Nuclei,
MLF, and Related Structures
SC
OcNu
PAG
MesNu
ML
RNu
SN
IC
OcNu
ALS
TroNu
TroNu
MLF
ML
SCP,Dec
CblNu
MLF
AbdNu
Cbl-CoVes
SVNu
LVNu
JRB
JRB
AbdNu
SVNu
MLF
SpTTr
VesCbl, Sec
LVNu
Cbl cortex
VesCbl, Prim
ALS
ML
VesGang
MVNu
HyNu
Crista ampullaris
InfVNu
MVNu and
Macula utriculi
InfVNu
Macula sacculi
RB
MLF
SpTTr
ML
MVesSp in MLF
SolTr and Nu
Py
LVesSp
230
Synopsis of Functional Components, Tracts, Pathways, and Systems
7-31
Blank master drawing for auditory or vestibular pathway.
This illustration is provided for self-evaluation of auditory or vestibu-
lar pathway understanding, for the instructor to expand on aspects of
these pathways not covered in the atlas, or both.
Optic, Auditory, and Vestibular Systems
231
232
Synopsis of Functional Components, Tracts, Pathways, and Systems
Hippocampal Connections
7-32
Selected afferent and efferent connections of the hippocam-
to the dentate gyrus, Ammon’s horn, and subiculum; and serotonin-
pus (upper) and the mammillary body (lower) with emphasis on the
ergic fibers arise from the rostral raphe nuclei.
circuit of Papez. The hippocampus receives input from, and projects
Clinical Correlations: Dysfunction associated with damage to
to, diencephalic nuclei (especially the mammillary body via the post-
the hippocampus is seen in patients with trauma to the temporal lobe,
commissural fornix), the septal region, and amygdala. The hippocam-
as a sequel to alcoholism, and as a result of neurodegenerative changes
pus receives cortical input from the superior and middle frontal gyri,
seen in the dementing diseases (such as Alzheimer disease and Pick disease).
superior temporal and cingulate gyri, precuneus, lateral occipital cor-
Bilateral injury to the hippocampus results in loss of recent memory
tex, occipitotemporal gyri, and subcallosal cortical areas. The mam-
(remote memory is unaffected), impaired ability to remember recent
millary body is connected with the dorsal and ventral tegmental nuclei,
(new) events, and difficulty in turning a new experience (something
anterior thalamic nucleus (via the mammillothalamic tract), septal nu-
just done or experienced) into a longer-term memory that can be re-
clei, and through the mammillotegmental tract, to the tegmental pon-
trieved at a later time. Also, memory that depends on visual, tactile,
tine and reticulotegmental nuclei.
or auditory discrimination is noticeably affected. These represent visual
Neurotransmitters: Glutamate
(
)-containing cells in the
agnosia, tactile agnosia, and auditory agnosia, respectively.
subiculum and Ammon’s horn project to the mammillary body, other
In the Korsakoff syndrome (amnestic confabulatory syndrome) there is mem-
hypothalamic centers, and the lateral septal nucleus through the fornix.
ory loss, dementia, amnesia, and a tendency to give confabulated re-
Cholecystokinin (
) and somatostatin (
) are also found in hip-
sponses. This type of response is fluent but consists of a string of unrelated,
pocampal cells that project to septal nuclei and hypothalamic struc-
or even made up, “memories” that never actually occurred or make no
tures. The septal nuclei and the nucleus of the diagonal band give rise
sense. This may lead to an incorrect conclusion that the patient is suffer-
to cholinergic afferents to the hippocampus that travel in the fornix. In
ing from dementia. In addition to lesions in the hippocampus in these pa-
addition, a gamma-aminobutyric acid (
) septohippocampal projec-
tients, the mammillary bodies and dorsomedial nucleus of the thalamus
tion originates from the medial septal nucleus. Enkephalin and gluta-
are noticeably affected. The Korsakoff syndrome (see also the Wernicke-
mate containing hippocampal afferent fibers arise from the adjacent en-
Korsakoff syndrome) as seen in chronic alcoholics is largely owing to thi-
torhinal cortex; the locus ceruleus gives origin to noradrenergic fibers
amine deficiency and can be treated with therapeutic doses of this vitamin.
Abbreviations
AC Anterior commissure
LT Lamina terminalis
AmHrn Ammon’s horn
MB Mammillary body
Amy Amygdaloid nucleus (complex)
MedFCtx Medial frontal cortex
AntNu Anterior nucleus of thalamus
MedTh Medial thalamus
CC, G Corpus callosum, genu
MTegTr Mammillotegmental tract
CC,Spl Corpus callosum, splenium
MtTr Mammillothalamic tract
Cing Cingulum
NuAcc Nucleus accumbens
CingGy Cingulate gyrus
OpCh Optic chiasm
CorHip Corticohippocampal fibers
Pi Pineal
DenGy Dentate gyrus
RSplCtx Retrosplenial cortex
EnCtx Entorhinal cortex
SepNu Septal nuclei
For Fornix
SMNu Supramammillary nucleus
GyRec Gyrus rectus
Sub Subiculum
Hip Hippocampus
TegNu Tegmental nuclei
Hyth Hypothalamus
VmNu Ventromedial hypothalamic nucleus
IC,G Internal capsule, genu
Review of Blood Supply to Hip, MB, Hyth, and CingGy
STRUCTURES
ARTERIES
Hip
anterior choroidal (see Figure 5-38)
MB, Hyth
branches of circle of Willis (see Figure 2-21)
AntNu
thalamoperforating (see Figure 5-38)
CingGy
branches of anterior cerebral
Limbic System
233
Hippocampal Connections
CingGy
Cing
IC,G
For
CC,Spl
RSplCtx
CC,G
AntNu
MedTh
For
Pi
AC
SepNu
LT
GyRec
NuAcc
VmNu
Amy
CorHip
OpCh
MB
EnCtx
DenGy
Hip
AmHrn
Sub
CingGy
Cing
IC,G
For
MTTr
AntNu
For
AC
MTegTr
SepNu
TegNu
Hyth
LT
DenGy
OpCh
AmHrn
Hip
Sub
EnCtx
Amy
MB
234
Synopsis of Functional Components, Tracts, Pathways, and Systems
Amygdaloid Connections
7-33
The origin, course, and distribution of selected afferent and
complex. Acetylcholine is present in afferents to the amygdala from the
efferent connections of the amygdaloid nuclear complex in sagittal (up-
substantia innominata, as well as from the septal area. In patients with
per) and coronal (lower) planes. The amygdala receives input from,
Alzheimer disease and the associated dementia, there is a marked loss
and projects to, brainstem and forebrain centers via the stria terminalis
of acetylcholine-containing neurons in the basal nucleus of the sub-
and the ventral amygdalofugal pathway. Corticoamygdaloid and amyg-
stantia innominata, in the cortex, and in the hippocampus.
dalocortical fibers interconnect the basal and lateral amygdaloid nuclei
Clinical Correlations: Dysfunctions related to damage to the
with select cortical areas.
amygdaloid complex are seen in patients with trauma to the temporal
Neurotransmitters: Cells in the amygdaloid complex contain
lobes, herpes simplex encephalitis, bilateral temporal lobe surgery to treat
vasoactive intestinal polypeptide (VIP,
), neurotensin (NT), so-
intractable epileptic activity, and in some CNS degenerative disorders
matostatin (SOM,
), enkephalin (ENK,
), and substance P (SP,
).
(such as Alzheimer disease and Pick disease). The behavioral changes seen
These neurons project, via the stria terminalis or the ventral amyg-
in individuals with amygdala lesions collectively form the Klüver-Bucy
dalofugal path, to the septal nuclei (VIP, NT), the bed nucleus of the
syndrome. In humans these changes/deficits are 1) hyperorality; 2) visual,
stria terminalis (NT, ENK, SP), the hypothalamus (VIP, SOM, SP), the
tactile, and auditory agnosia; 3) placidity; 4) hyperphagia or other dietary
nucleus accumbens septi, and the caudate and putamen (NT). Sero-
manifestations; 5) an intense desire to explore the immediate environ-
tonergic amygdaloid fibers originate from the nucleus raphe dorsalis
ment (hypermetamorphosis), and 6) what is commonly called hypersexual-
and the superior central nucleus, dopaminergic axons from the ventral
ity. These changes in sexual attitudes are usually in the form of com-
tegmental area and the substantia nigra-pars compacta, and noradren-
ments, suggestions, and attempts to make a sexual contact (such as
alin-containing fibers from the locus ceruleus. Glutamate (
) is found
touching) rather than in actual intercourse or masturbation. These pa-
in olfactory projections to the prepiriform cortex and the amygdaloid
tients may also show aphasia, dementia, and amnesia.
Abbreviations
AC Anterior commissure
NuRa,d Nucleus raphe, dorsalis
Amy Amygdaloid nuclear complex
NuRa,m Nucleus raphe, magnus
AmyCor Amygdalocortical fibers
NuRa,o Nucleus raphe, obscurus
AmyFugPath Amygdalofugal pathway
NuRa,p Nucleus raphe, pallidus
AntHyth Anterior hypothalamus
NuStTer Nucleus of the stria terminalis
Ba-LatNu Basal and lateral nuclei
OlfB Olfactory bulb
CaNu Caudate nucleus
OpCh Optic chiasm
Cen-MedNu Central, cortical and medial nuclei
PAG Periaqueductal (central) gray
CorAmy Corticoamygdaloid fibers
PBrNu Parabrachial nuclei
DVagNu Dorsal motor vagal nucleus
PfNu Parafascicular nucleus
EnCtx Entorhinal cortex
Pi Pineal
For Fornix
POpNu Preoptic nucleus
GP Globus pallidus
PPriCtx Prepiriform cortex
Hyth Hypothalamus
Put Putamen
LT Lamina terminalis
SepNu Septal nuclei
LHAr Lateral hypothalamic area
SNpc Substantia nigra, pars compacta
MedThNu Medial thalamic nuclei
SolNu Solitary nucleus
MGNu Medial geniculate nucleus
StTer Stria terminalis
MidTh Midline thalamic nuclei
Sub Subiculum
NuAcc Nucleus accumbens
Subln Substantia innominata
NuCen,s Nucleus centralis, superior
VenTegAr Ventral tegmental area
NuCer Nucleus ceruleus
VmNu Ventromedial hypothalamic nucleus
Review of Blood Supply to Amy and Related Centers
STRUCTURES
ARTERIES
Amy
anterior choroidal (see Figure 5-38)
Hyth
branches of circle of Willis (see Figure 5-38)
Brainstem
(see Figures 5-14, 5-21, and 5-27)
Thalamus
thalamoperforating, thalamogeniculate (see Figure 5-38)
Limbic System
235
Amygdaloid Connections
MedThNu
MidTh, PfNu, MGNu
StTer
Put, CaNu
StTer
NuAcc
VmNu, LHAr
NuStTer
PAG
AC
LT
SNpc, VenTegAr
AntHyth
NuRa,d
POpNU
OpCh
OlfB
NuCer
AmyFugPath
Cen-MedNu
PBrNu
Amy
Ba-LatNu
NuCen,s
PPriCtx
Sub
NuRa,m
EnCtx
SolNu
NuRa,p
NuRa,o
DVagNu
Prefrontal cortex
Cingulate gyrus
CaNu
NuStTer
Insula
StTer
Temporal
NuAcc
lobe
Put
GP
SepNu
For
AmyCor
Hyth
CorAmy
POpNu
Parahippocampal
SubIn
gyrus
AmyFugPath
Cen-MedNu
to StTer
Amy
Ba-LatNu
236
Synopsis of Functional Components, Tracts, Pathways, and Systems
7-34
Blank master drawing for limbic pathways. This illustration
is provided for self-evaluation of limbic pathways or connections, for
the instructor to expand on aspects of these pathways not covered in
the atlas, or both.
CHAPTER
8
Anatomical-Clinical Correlations:
Cerebral Angiogram, MRA, and MRV
240
Anatomical-Clinical Correlations
A
Callosomarginal branch
(of ACA)
Parietal branches
(of MCA)
Pericallosal branch
(of ACA)
Angular branch
Anterior cerebral artery
(of MCA)
(ACA)
Middle cerebral artery
(MCA)
Internal carotid artery
B
Internal carotid artery
Ophthalmic artery
(petrous part)
Internal carotid artery
(cavernous part)
Internal carotid artery
(cerebral part)
8-1
Internal carotid angiogram
(left lateral projection, arterial
an important source of blood supply to the retina. Occlusion of the
phase) showing the general patterns of the internal carotid, middle,
ophthalamic artery may result in blindness in the eye on that side. The
and anterior cerebral arteries (A, B) and an image with especially good
terminal branches of the ophthalamic artery will anastomose with su-
filling of the ophthalmic artery (B). The ophthalmic artery leaves the
perficial vessels around the orbit. Compare with Figures 2-12 (page 19)
cerebral part of the internal carotid and enters the orbit via the optic
and 2-25 (page 27).
canal. This vessel gives rise to the central artery of the retina, which is
Cerebral Angiogram, MRA, and MRV
241
A
Superior sagittal sinus
Superior cerebral veins
Inferior sagittal sinus
Straight sinus
Thalamostriate vein
Transverse sinus
Internal cerebral vein
Great cerebral vein (of Galen)
Venous angle
Sigmoid sinus
Inferior cerebral veins
Inferior anastomotic vein
(of Labbé)
Superficial middle
cerebral vein
Basal vein
(of Rosenthal)
B
Superior cerebral
veins
Superior anastomotic vein
(of Trolard)
Straight sinus
Inferior anastomotic vein
(of Labbé)
Superficial middle
cerebral vein
8-2
Two internal carotid angiograms (left lateral projection, ve-
bral vein is called the venous angle (A). The interventricular foramen
nous phase). Superficial and deep venous structures are clear in A, but
is located immediately rostral to this point.Compare these images with
B shows a particularly obvious vein of Trolard. The thalamostriate vein
the drawings of veins and sinuses in Figures 2-13 (page 19) and 2-28
(A) at this location can also be called the superior thalamostriate vein.
(page 29).
The junction of the superior thalamostriate vein with the internal cere-
242
Anatomical-Clinical Correlations
Middle cerebral artery
(M4-cortical branches)
Anterior cerebral artery
(A4, A5)
Middle cerebral artery
(M2-insular branches)
Anterior cerebral artery
(A3)
Lenticulostriate branches
(A2)
(of M1)
(A1)
Middle cerebral artery
(M1)
Internal carotid artery
(cavernous part)
Internal carotid artery
(cerebral part)
Internal carotid artery
(petrous part)
8-3
Internal carotid angiogram (anterior-posterior projection,
(supracallosal) and A5 (postcallosal) segments are located superior
arterial phase). Note general distribution patterns of anterior and
(above) the corpus callosum.
middle cerebral arteries and the location of lenticulostriate
The M1 segment of the middle cerebral artery is located between
branches. The A1 segment of the anterior cerebral artery is located
the internal carotid bifurcation and the point at which this vessel
between the internal carotid bifurcation and the anterior communi-
branches into superior and inferior trunks on the insular cortex. As
cating artery. The distal portion of the anterior cerebral artery
branches of the middle cerebral artery pass over the insular cortex they
(ACA) immediately rostral to the anterior communicating artery
are designated as M2, as M3 when these branches are located on the in-
and inferior to the rostrum of the corpus callosum is the A2 segment
ner surface of the frontal, parietal, and temporal opercula, and as M4
(infracallosal). The portion of the ACA arching around the genu of
where they exit the lateral sulcus and fan out over the lateral aspect of
the corpus callosum is the A3 segment (precallosal) and the A4
the cerebral hemisphere. Compare with Figure 2-21 on page 25.
Cerebral Angiogram, MRA, and MRV
243
Arachnoid villi
Superior cerebral veins
Superior sagittal sinus
Superior sagittal
Inferior sagittal sinus
sinus
Confluence of sinuses
Transverse
Transverse sinus
sinus
Sigmoid sinus
8-4
Internal carotid angiogram (anterior-posterior projection, ve-
nous phase). The patient’s head is tilted slightly; this shows the arch-
ing shapes of the superior and inferior sagittal sinuses to full advantage.
Note the other venous structures in this image and compare with the
arterial phase shown in Figure 8-3 on page 242 and the images in Fig-
ures 8-5 and 8-6 on pages 244 and 245. Also compare with Figure 2-
28 on page 29.
244
Anatomical-Clinical Correlations
A
Superior sagittal sinus
Superficial cerebral veins
B
Superior sagittal sinus
Confluence of sinuses
Transverse sinus
Sigmoid sinus
Jugular bold
Internal jugular vein
8-5
Digital subtraction image of an internal carotid angiogram (an-
in the jugular fossa at the point where the sigmoid sinus is continu-
terior-posterior projection, venous phase). Image A is early in the ve-
ous with the IJV; this continuity is through the jugular foramen. The
nous phase (greater filling of cortical veins), whereas image B is later
jugular foramen also contains the roots of cranial nerves IX, X, and
in the venous phase (greater filling of the sinuses and jugular vein). Both
XI, the continuation of inferior petrosal sinus with the IJV and sev-
images are of the same patient.
eral small arteries. Compare with Figures 2-16 (page 21) and 2-19
The jugular bulb is a dilated portion of internal jugular vein (IJV)
(page 23).
Cerebral Angiogram, MRA, and MRV
245
B
ACA
PCA
A
SSS
MCA
SCA
TS
ICA
CS
AICA
BA
VA
Anterior cerebral artery (ACA)
A2 segment
C
A1 segment
Anterior communicating artery
Posterior cerebral artery (PCA)
Superior sagittal sinus (SSS)
Superior cerebellar artery (SCA)
Middle cerebral artery (MCA)
M2 segment
M1 segment
Internal carotid artery (ICA):
Cerebral part
Sigmoid sinus
Cavernous part
Petrous part
Transverse sinus (TS)
Confluence of sinuses (CS)
Anterior inferior cerebellar artery (AICA)
Basilar artery (BA)
Vertebral artery (VA)
8-6
Magnetic resonance angiography
(MRA) is a noninvasive
terior to posterior. C shows the relative position of the major vessels
method for imaging cerebral arteries, veins, and sinuses simultane-
and dural sinuses as imaged in A and B. The superior sagittal sinus, as
ously. A 3-D phase contrast MRA (A) and an inverted video image
seen in A and B, is usually continuous with the right transverse sinus at
window (B) of the same view show major vessels and sinuses from an-
the confluence of sinuses.
246
Anatomical-Clinical Correlations
A
Parieto-occipitial
branches (of PCA)
Thalamogeniculate arteries
Posterior choroidal arteries
Calcarine branch
(of PCA)
Posterior cerebral arteries
(PCA)
Thalamoperforating
arteries
Basilar bifurcation
Posterior communicating
artery
Posterior inferior
Superior cerebellar
cerebellar artery
artery (SCA)
(PICA)
Vertebral artery
Basilar artery (BA)
(VA)
B
Parieto-occipital
branches
Calcarine branch
PCA
Basilar bifurcation
SCA
BA
Anterior inferior
cerebellar artery
PICA
8-7
A vertebral artery angiogram (left lateral
projection, arterial phase) is shown in A, and the
same view, but in a different patient, is shown in B,
using digital subtraction methods. Note the charac-
VA
teristic orientation of the major vessels. Compare
with Figure 2-21 on page 25.
Cerebral Angiogram, MRA, and MRV
247
PCA, Cortical branches
PCA
Thalamoperforating
arteries
SCA
AICA
Basilar artery (BA)
Vertebral artery (VA)
B
Posterior cerebral artery,
Cortical branches
Posterior cerebral arteries
(PCA)
Thalamoperforating arteries
(of the basilar bifurcation)
Superior cerebellar artery
(SCA)
SCA
BA
AICA
Anterior inferior cerebellar
artery (AICA)
PICA
Posterior inferior cerebellar
artery (PICA)
VA
8-8
A vertebral artery angiogram (anterior-posterior projection,
The root of the oculomotor (IIIrd) nerve, after exiting the inferior
arterial phase) is shown in A; the same view, but in a different patient,
aspect of the midbrain, characteristically passes through the interpe-
is shown in B, using digital subtraction methods. Even though the in-
duncular cistern and between the superior cerebellar and posterior
jection is into the left vertebral, there is bilateral filling of the vertebral
cerebral arteries en route to its exit from the skull through the supe-
arteries and of branches of the basilar artery. The thalamoperforating
rior orbital fissure. In this position the IIIrd nerve may be damaged by
arteries are important branches of P1 that generally serve rostral por-
large aneurysms that impinge on the nerve root. Compare with Figures
tions of the diencephalon.
2-40 (page 39) and 2-41 (page 40).
248
Anatomical-Clinical Correlations
A
Middle cerebral
Anterior cerebral artery:
artery (MCA):
A3 segment
M1 segment
A2 segment
M2 segment
A1 segment
MCA, Insular branches
Basilar artery (BA)
Posterior cerebral
artery (PCA)
MCA, Cortical branches
(M4 segment)
PCA, Temporal branch
Internal cerebral vein
Superior petrosal sinus
Lateral ventricular
vein
Great cerebral vein
(of Galen)
Straight sinus (SS)
Transverse sinus (TS)
TS
B
Superior sagittal sinus
Anterior cerebral artery:
Great cerebral vein
(A3)
(A2)
MCA, M2 segment
SS
Internal carotid
artery
Posterior communicating artery
BA
PCA
TS
Superior cerebellar
artery
8-9
MRA images arteries, veins, and sinuses simultaneously, based
sum is the A2 segment (infracallosal). The portion of the ACA arching
on the movement of fluid in these structures. These are inverted video
around the genu of the corpus callosum is the A3 segment (precallosal)
images of 3-D phase contrast MRA images as viewed from the dorsal
and the A4 (supracallosal) and A5 (postcallosal) segments are located
to ventral (A) and from the lateral aspect (B). The distal portion of the
superior to (above) the corpus callosum. Compare these images with
anterior cerebral artery (ACA) immediately rostral to the anterior
arteries and veins as depicted in Figures 2-18 and 2-19 (page 23), 2-21
communicating artery and inferior to the rostrum of the corpus callo-
(page 25), and 2-23 (page 27).
Cerebral Angiogram, MRA, and MRV
249
Anterior cerebral artery:
Cortical branches
A1 segment
A
Internal carotid artery
Branches of middle
cerebral artery
Middle cerebral artery:
Branches on insula (M2)
Internal carotid artery
M1 segment
Posterior communicating
artery
Cortical branches (M4)
Posterior communicating
Posterior cerebral artery:
artery
P2 segment
P1 segment
Posterior cerebral artery
Parieto-occiptal artery
Calcarine artery
Anterior communicating artery
B
Orbit
Anterior cerebral artery
(A1 segment)
Ophthalmic artery
Cavernosus sinus
Middle cerebral artery
(containing internal
(branches on insula)
carotid artery)
Petrosal segment of
internal carotid artery
Middle cerebral artery
Superior cerebellar artery
(M1 segment)
Tumor (vestibular schwannoma)
Posterior communicating
artery
Posterior cerebral artery
Basilar artery
Calcarine artery
Posterior cerebral
artery
Vertebral arteries
8-10
MRA images of the vessels at the base of the brain forming
much of the cerebral arterial circle (of Willis) (A, B). Note the ante-
rior, middle, and posterior cerebral arteries as they extend outward
from the circle. The upper image is from a normal individual, and the
lower image is from a patient with a vestibular schwannoma. Descrip-
tions of the segments of the anterior, middle, and posterior cerebral
arteries are found on pages 25 and 242.
250
Anatomical-Clinical Correlations
Superior sagittal sinus
A
Callosomarginal branch
of ACA
Superficial cerebral veins
Pericallosal branch
of ACA
Anterior cerebral artery
Internal cerebral vein
(ACA)
Middle cerebral
artery
Great cerebral vein (of Galen)
Ophthalmic artery
or vein
Straight sinus
Carotid artery
(cavernous portions)
Vein of Labbé
Transverse sinus
Basal vein (of Rosenthal)
Confluence of sinuses
Sigmoid sinus
Basilar artery
Internal jugular vein
B
Superficial cerebral veins
Superficial cerebral
veins
Superior sagittal sinus
Middle cerebral artery
on insular cortex
Superficial cerebral vein
Confluence of sinuses
Transverse sinus
Basilar artery
Sigmoid sinus
Internal carotid artery
Inferior petrosal sinus
Internal jugular vein
Vertebral artery
8-11
Magnetic resonance venography (MRV) primarily demon-
Note that the continuation of the superior sagittal sinus is most promi-
strates veins and venous sinuses although arteries (seen in A and B) will
nent into the right transverse sinus (B, compare with Figure 8-6 on
also sometimes be visualized. Many veins and venous sinuses can be
page 245). Compare with Figures 2-13 (page 19), 2-16 (page 21), 2-
seen in this lateral view (A) and in the anterior-posterior view (B).
19 (page 27), and 2-28 (page 29).
Blood Supply to the Choroid Plexi
251
A
Choroid plexus (CP) in
body of lateral ventricle
CP in atrium of lateral ventricle
CP in roof of third ventricle
CP in temporal horn
of lateral ventricle
Anterior choroidal artery
CP in fourth ventricle
AICA
PICA
Posterior communicating artery
Lateral posterior choroidal artery
BA
VA
Medial posterior choroidal artery
B
Medial striate artery
Internal carotid artery
Middle cerebral artery (M1)
A
1
P1
Anterior choroidal artery
Anterior choroidal artery
Posterior communicating artery
Posterior cerebral artery (P2)
Lateral posterior choroidal artery
Superior cerebellar artery
Medial posterior choroidal artery
Basilar artery (BA)
Anterior inferior cerebellar artery (AICA)
AICA branch to choroid plexus
at the foramen of Luschka
Vertebral artery (VA)
Posterior inferior cerebellar artery (PICA)
PICA branch to choroid plexus
in the fourth ventricle
8-12
Blood supply to the choroid plexus of the lateral, third, and
rior lateral choroidal arteries serve the plexuses of the lateral and third
fourth ventricles. Those branches of the vertebrobasilar system and of the
ventricles. The choroid plexus in the fourth ventricle and the clump of
internal carotid artery and P2 segment of the posterior cerebral artery that
choroid plexus protruding out of the foramen of Luschka are served by
supply the choroid plexus are accentuated by appearing in a darker red
posterior inferior and anterior inferior cerebellar arteries, respectively. In
shade. In A, a representation of these vessels (origin, course, termination)
B, the origins of these branches from their main arterial trunks are shown.
is shown from the lateral aspect. Anterior, posterior medial, and poste-
See also Figures 2-21 (page 25), 2-24 (page 27), and 2-35 (page 35).
252
Anatomical-Clinical Correlations
A
Anterior cerebral artery
Middle cerebral artery
A2
M2
A1
M1
Internal carotid artery
Cerebral part
Cavernous part
Basilar artery
Petrosal part
Anterior inferior
cerebellar artery
Cervical part
Vertebral artery
Posterior inferior
cerebellar artery
Maxillary artery
(br. of external
carotid artery)
Internal carotid artery
Vertebral artery
External carotid artery
Common carotid artery
B
Posterior cerebral artery
Superior cerebellar artery
Position of
occulomotor nerve
Basilar artery
Anterior inferior
cerebellar artery
Vertebral artery
(intercranial portion)
Vertebral artery
Anterior inferior
(passing caudally and
cerebellar artery
medially around the lateral
mass of the atlas)
Vertebral artery
(passing through transverse
foramen of the atlas)
8-13
Overview (A) of the arteries in the neck that serve the brain (in-
The vertebral artery (VA) is generally described as being composed
ternal carotid and vertebral) and of their main terminal branches (anterior
of 4 segments sometimes designated as V1 to V4. The first segment (V1)
cerebral artery and middle cerebral artery, vertebrobasilar system) as seen
is between the VA origin from the subclavian artery and the entrance
in an MRA (anterior-posterior view). In approximately 40-45% of indi-
of VA into the first transverse foramen (usually C6); the second seg-
viduals the left vertebral artery is larger, as seen here, and in about 5-10%
ment V2 is that part of VA ascending through the transverse foramen
of individuals one or the other of the vertebral arteries may be hypoplas-
of C6 to C2; the third segment (V3) is between the exit of VA from the
tic as seen here on the patient’s right. The MRI in B is a detailed view of
transverse foramen of the axis and the dura at the foramen magnum
the vertebrobasilar system from the point where the vertebral arteries exit
(this includes the loop of the VA that passes through the transverse
the transverse foramen to where the basilar artery bifurcates into the pos-
foramen of C1/the atlas); the fourth segment (V4) pierces the dura and
terior cerebral arteries. Compare this image with Figure 2-21 on page 25.
joins its counterpart to form the basilar artery.
CHAPTER
9
Q & A’s: A Sampling of Study and Review
Questions, Many in the USMLE Style,
All With Explained Answers
D. E. Haines and J. A. Lancon
There are two essential goals of a student studying human neu-
cise, some answers may contain additional relevant informa-
robiology, or, for that matter, the student of any of the medical
tion to extend the educational process.
sciences. The first is to gain the knowledge base and diagnostic
In general, the questions are organized by individual chapters,
skills to become a competent health care professional. Address-
although chapters 1 and 2 and chapters 3 and 4 are combined. Ref-
ing the medical needs of the patient with insight, skill, and com-
erence to the page (or pages) containing the correct answer are
passion is paramount. The second is to successfully negotiate
usually to the chapter(s) from which the question originated.
whatever examination procedures are used in a given setting.
However, recognizing that neuroscience is dynamic and three-di-
These may be standard class examinations, Subject National
mensional, some answers contain references to chapters other
Board Examination (now used/required in many courses), the
than that from which the question originated. This provides a
USMLE Step 1 Examination (required of all U.S. medical stu-
greater level of integration by bringing a wider range of informa-
dents), or simply the desire, on the part of the student, for self-
tion to bear on a single question.
assessment.
Correct diagnosis of the neurologically compromised patient
The questions in this chapter are prepared in two general
not only requires integration of information contained in differ-
styles. First, there are study or review questions that test gen-
ent chapters but may also require inclusion of concepts gained in
eral knowledge concerning the structure of the central ner-
other basic science courses. In this regard a few questions, and
vous system. Many of these have a functional flavor. Second,
their answers, may include such additional basic concepts.
there are single one best answer questions in the USMLE style
This is not an all-inclusive list of questions, but rather a
that use a patient vignette approach in the stem. These ques-
sampling that covers a wide variety of neuroanatomical and
tions have been carefully reviewed for clinical accuracy and
clinically relevant points. There is certainly a much larger va-
relevance as used in these examples. At the end of each ex-
riety of questions that could be developed from the topics cov-
plained answer, page numbers appear in parentheses that
ered in this atlas. It is hoped that this sample will give the user
specify where the correct answer, be it in a figure or in the text,
a good idea of how basic neuroscience information correlates
may be found. In order to make this a fruitful learning exer-
with a range of clinically relevant topics.
254
Q & A’s: A Sampling of Study and Review Questions with Explained Answers
6.
In addition to the vestibulocochlear nerve, which of the following
Review and Study Questions for
structures would most likely also be affected by the tumor in this
Chapters 1 and 2
man?
(A) Anterior inferior cerebellar artery
1.
A 71-year-old man complains to his family physician that his face
(B) Facial nerve
“feels funny.” The examination reveals numbness on his face and
(C) Glossopharyngeal nerve
on the same side of his tongue. MRI shows a lesion in the cerebral
(D) Posterior inferior cerebellar artery
cortex. This man’s lesion is most likely located in which of the fol-
(E) Vagus nerve
lowing cortical regions?
(A) Anterior paracentral
7.
A 67-year-old man complains to his family physician of severe
(B) Lateral one-third of the postcentral
headaches. The examination reveals visual deficits in both eyes,
(C) Lateral one-third of the precentral
and MRI shows a lesion in the cerebral cortex. Which of the fol-
(D) Middle one-third of the postcentral
lowing cortical structures represents the most likely location of
(E) Posterior paracentral
this lesion?
(A) Angular gyrus
2.
A 41-year-old woman complains to her family physician about re-
(B) Cingulate gyrus
curring episodes of sharp pain that seem to originate from around
(C) Lingual gyrus
her mouth and cheek. The pain is so intense that she is unable to
(D) Parahippocampal gyrus
eat, brush her teeth, or apply make-up. Which of the following
(E) Precuneus
cranial nerves is the most likely source of this pain?
(A) Facial (VII)
8.
A sagittal MRI of a 23-year-old woman is located at, or immedi-
(B) Glossopharyngeal (IX)
ately adjacent to, the midline. Which of the following spaces or
(C) Hypoglossal (XII)
structures would be in the image and would indicate a midline
(D) Trigeminal (V)
plane?
(E) Vagus (X)
(A) Cerebral aqueduct
(B) Corpus callosum
3.
The labyrinthine artery is an important source of blood supply to
(C) Interpeduncular fossa
the inner ear. Which of the following arteries represents the ma-
(D) Interventricular foramen
jor vessel from which this branch usually arises?
(E) Superior colliculus
(A) Anterior inferior cerebellar
(B) Basilar
9.
A 20-year-old man is brought to the emergency department from
(C) Posterior inferior cerebellar
the site of a motorcycle accident. He is unconscious and has a bro-
(D) Superior cerebellar
ken femur, humerus, and extensive facial injuries. Axial CT shows
(E) Vertebral
a white layer on the lateral aspect of the left hemisphere that is ap-
proximately 5 mm thick and extends for 12 cm. This observation
4.
The quadrigeminal artery in a 20-year-old man is occluded by a fat
most likely represents:
embolus originating from a compound fracture of the humerus.
(A) Epidural hemorrhage/hematoma
Which of the following structures does this occluded vessel most
(B) Parenchymatous hemorrhage in the cortex
directly affect?
(C) Subarachnoid hemorrhage
(A) Superior cerebellar peduncle
(D) Subdural hemorrhage/hematoma
(B) Mammillary bodies
(E) Ventricular hemorrhage
(C) Medial and lateral geniculate bodies
(D) Pineal and habenula
10.
Which of the following portions of the ventricular system does not
(E) Superior and inferior colliculi
contain choroid plexus?
(A) Cerebral aqueduct
Questions 5 and 6 are based on the following patient.
(B) Fourth ventricle
A 63-year-old man has hearing loss, tinnitus (ringing or buzzing sounds
(C) Lateral ventricle
in the ear), vertigo, and unsteady gait; all of these have developed over
(D) Interventricular foramen
several years. MRI reveals a large tumor (3 cm in diameter) at the cere-
(E) Third ventricle
bellopontine angle, most likely a vestibular schwannoma (sometimes
incorrectly called an acoustic neuroma).
11.
A 47-year-old man presents with an intense pain on his face aris-
ing from stimulation at the corner of his mouth. This is character-
5. What additional deficit could this patient also have?
istic of trigeminal neuralgia (tic douloureux). MRI shows a vessel
compressing the root of the trigeminal nerve. Which of the fol-
(A) Anosmia
lowing vessels would most likely be involved?
(B) Hemianopsia
(C) Numbness on the face
(A) Anterior inferior cerebellar artery
(D) Visual field deficits
(B) Basal vein (of Rosenthal)
(E) Weakness of the tongue
(C) Basilar artery
(D) Posterior cerebral artery
(E) Superior cerebellar artery
Q & A’s: A Sampling of Study and Review Questions with Explained Answers
255
12.
Which of the following cranial nerves contain the afferent and ef-
18.
A lumbar puncture, commonly called a “lumbar tap,” consists of a
ferent limbs of the corneal reflex?
needle being inserted through an intervertebral space into the
lumbar cistern to retrieve a sample of cerebrospinal fluid. Which
(A) II and III (optic and oculomotor)
of the following is the most likely level for the insertion of the nee-
(B) III, IV, VI (oculomotor, trochlear, abducens)
dle?
(C) V and VII (trigeminal, facial)
(D) VIII and IX (vestibulocochlear, glossopharyngeal)
(A) L1-L2
(E) IX and X (glossopharyngeal, vagus)
(B) L2-L3
(C) L4-L5
13.
A 73-year-old man is brought to the emergency department after
(D) S1-S2
being found in his garage in a state of confusion. CT shows an in-
(E) T12-L1
farct involving much of the superior frontal gyrus. Which of the
following vessels is most likely occluded in this patient?
19.
A 59-year-old man complains of persistent headache. An MRA
(Magnetic Resonance Angiography) shows an aneurysm in the in-
(A) Angular artery
terpeduncular fossa (and cistern) arising from the basilar tip.
(B) Callosomarginal artery
Which of the following cranial nerves would be most directly af-
(C) Lenticulostriate arteries
fected by this aneurysm?
(D) Middle cerebral artery, M4 segments
(E) Posterior cerebral artery, P4 segments
(A) Abducens (VI)
(B) Oculomotor (III)
14.
The MRI of a 49-year-old woman shows a tumor located immedi-
(C) Optic (II)
ately superior to the corpus callosum. This lesion is most likely lo-
(D) Trigeminal, V1 (V)
cated in which of the following lobes?
(E) Trochlear (IV)
(A) Frontal
20.
A 71-year-old man presents with a Broca (nonfluent) aphasia. MRI
(B) Limbic
reveals a lesion in Brodmann area 44. As this lesion expands, due
(C) Occipital
to edema, and impinges on the immediately adjacent cortical ar-
(D) Parietal
eas, which of the following deficits would most likely be seen?
(E) Temporal
(A) Loss of hearing in one ear
15.
A 69-year-old woman is brought to the emergency department.
(B) Numbness and prickly sensation on the hand
The daughter reports that her mother suddenly seemed to be un-
(C) Visual field deficits in both eyes
able to speak. The examination reveals that the woman has a non-
(D) Weakness of facial muscles
fluent (Broca) aphasia. A sagittal MRI shows a lesion in which of
(E) Weakness of the upper extremity
the following gyri?
21.
A 47-year-old woman presents with seizures and ill-defined neu-
(A) Angular
rologic complaints. The examination reveals a bruit on the lateral
(B) Inferior frontal
aspect of the head immediately rostral and superior to the ear. A
(C) Lateral one-third of the precentral
CT shows a large arteriovenous malformation in the area of the lat-
(D) Middle frontal
eral sulcus. The feeding artery(ies) is M4 branches. Which of the
(E) Supramarginal
following most likely represents the major draining vein?
16.
Which of the following Brodmann areas represents the primary
(A) Inferior sagittal sinus
somatosensory cortex?
(B) Internal cerebral vein
(C) Ophthalmic vein
(A) Areas 3, 1, 2
(D) Superficial middle cerebral vein
(B) Area 4
(E) Superior petrosal sinus
(C) Area 17
(D) Area 22
22.
The collection of posterior and anterior roots that occupy the lum-
(E) Area 40
bar cistern are collectively known as which of the following?
17.
A 64-year-old man awakens with a profound weakness of his right
(A) Cauda equina
hand. The man is transported by ambulance to a major medical
(B) Conus medullaris
center, a distance of 240 miles and taking several hours. About 2.5
(C) Denticulate ligament
hours after his arrival, an MRI shows a small lesion in the cerebral
(D) Filum terminale externum
cortex. Which of the following gyri represents the most likely lo-
(E) Filum terminale internum
cation of this lesion?
23.
Which of the following Brodmann areas represents the primary
(A) Anterior paracentral
somatomotor cortex?
(B) Medial one-third of precentral
(C) Middle frontal
(A) Areas 3,1,2
(D) Middle one-third of precentral
(B) Area 4
(E) Lateral one-third of precentral
(C) Area 5
(D) Area 6
(E) Area 7
256
Q & A’s: A Sampling of Study and Review Questions with Explained Answers
24.
A 39-year-old woman complains of weakness in her right lower
30.
The abducens nerve exits the brainstem at the pons-medulla junc-
extremity. The history suggests that this deficit has developed
tion generally in line with the preolivary sulcus and passes rostrally
slowly, perhaps over several years. MRI shows a meningioma im-
just lateral to, and in the same cistern as, the basilar artery. Which
posing on the cerebral cortex. Which of the following gyri is most
of the following cisterns contains the abducens nerve and basilar
likely involved in this patient?
artery?
(A) Anterior paracentral
(A) Ambient
(B) Lateral part of precentral
(B) Inferior cerebellopontine
(C) Medial part of precentral
(C) Premedullary
(D) Medial part of postcentral
(D) Prepontine
(E) Posterior paracentral
(E) Superior cerebellopontine
25.
A 71-year-old woman presents with motor and sensory deficits af-
31.
An 81-year-old woman is brought to the emergency department
fecting her face and upper extremity. CT shows a hemorrhage that
by her son with a complaint of weakness on the same side of her
is confined largely to the cortex and adjacent subcortical areas.
body and face. CT shows a hemorrhage in the territory of the
Which of the following vessels/segments are most likely involved?
lenticulostriate arteries. Which of the following represents the
(A) A1
most likely origin of these vessels?
(B) M2
(A) A1
(C) M3
(B) M1
(D) M4
(C) M2
(E) P4
(D) P1
(E) P2
26.
A 22-year-old man is brought to the emergency department with
a gunshot wound to the head. He is decorticate but soon becomes
32.
The MRI of a 27-year-old woman shows a meningioma impinging
decerebrate. This change in status is due to uncal herniation.
on the gyrus rectus in axial and coronal MRI. This lesion is located
Which of the following most specifically describes the position of
on which of the following lobes of the cerebral hemisphere?
the uncus prior to herniation?
(A) Frontal
(A) At the temporal lobe
(B) Insular
(B) Caudal aspect of the cingulate gyrus
(C) Occipital
(C) Caudal aspect of the gyrus rectus
(D) Parietal
(D) Medial edge of occipitotemporal gyri
(E) Temporal
(E) Rostromedial aspect of the parahippocampal gyrus
33.
A 51-year-old man presents with visual field deficits in both eyes
27.
A 73-year-old woman presents with visual deficits in both eyes.
and a right-sided weakness of the upper and lower extremities.
No other cranial nerve deficits or motor or sensory deficits are
MRI shows a lesion in the optic tract that has spread into a struc-
seen. CT shows a hemorrhage in the cerebral cortex. Which of the
ture located immediately adjacent to this tract. Based on its
following vessels/segments is most likely involved in this hemor-
anatomical relationship, which of the following structures is most
rhage?
likely involved in a lesion spreading from the optic tract?
(A) A1
(A) Left basilar pons
(B) M3
(B) Left crus cerebri
(C) M4
(C) Left pyramid
(D) P2
(D) Right crus cerebri
(E) P4
(E) Right optic nerve
28.
The CT of a 77-year-old man shows a calcified tuft of choroid
34. A 19-year-old man presents with significant paralysis of move-
plexus, the glomus choroideum. Which of the following repre-
ment in his left eye and a dilated pupil. No other deficits are
sents the location of this part of the choroid plexus?
seen. Suspecting some type of lesion on the root or along the
(A) Anterior horn of the lateral ventricle
intracranial course of the oculomotor (III) nerve, the neurolo-
(B) Atrium of the lateral ventricle
gist orders an MRI. Which of the following describes the ap-
(C) Body of the lateral ventricle
pearance of the subarachnoid and ventricular spaces in a T2-
(D) Caudal roof of the third ventricle
weighted image?
(E) Temporal horn of the lateral ventricle
(A) Black (hypointense)
(B) Dark grey
29.
Which of the following represents the most common cause of
(C) Light grey
blood in the subarachnoid space (subarachnoid hemorrhage)?
(D) Medium grey
(A) Bleeding from an arteriovenous malformation
(E) White (hyperintense)
(B) Bleeding from a meningioma
(C) Bleeding from a tumor
(D) Rupture of an aneurysm
(E) Trauma to the brain
Q & A’s: A Sampling of Study and Review Questions with Explained Answers
257
35.
A 49-year-old woman presents with ill-defined neurologic deficits
41.
The MRI of an 11-year-old boy shows a tumor in the pontine por-
that have persisted over several months. As part of the evaluation,
tion of the fourth ventricle. The rostral edge of which of the fol-
the neurologist orders an MRI. Which of the following describes
lowing structures represents the border between the medullary
the appearance of CSF in the ventricular spaces, and consequently
and pontine parts of the fourth ventricle?
the outline and shape of the ventricles, in a T1-weighted image?
(A) Facial colliculus
(A) Black (hypointense)
(B) Hypoglossal trigone
(B) Dark grey
(C) Medial eminence
(C) Light grey
(D) Stria medullares
(D) Medium grey
(E) Vagal trigone
(E) White (hyperintense)
42.
A 61-year-old man presents with a tremor and unsteady gait; these
36.
A 71-year-old morbidly obese man is brought to the emergency
problems are on the same side of his body. Sagittal MRI shows a
department by his son. The son reports that the man complained
lesion in the anterior lobe of the cerebellum. Which of the fol-
of a sudden excruciating headache and then became stuporous.
lowing represents the fissure separating the anterior and posterior
Suspecting a ruptured aneurysm the physician orders a CT. Which
lobes of the cerebellum?
of the following describes the appearance of acute blood in the
(A) Horizontal fissure
subarachnoid space in CT?
(B) Posterior superior fissure
(A) Black (hypodense)
(C) Posterolateral fissure
(B) Black to grey
(D) Primary fissure
(C) Light grey
(E) Secondary fissure
(D) Medium grey
(E) White (hyperdense)
43.
The MRI of a 49-year-old woman with a brain tumor shows ton-
sillar herniation. Based on its anatomical position, which of the fol-
37.
Which of the following cranial nerves exits the brainstem via the
lowing portions of the brainstem would be most adversely affected
preolivary sulcus?
by tonsillar herniation?
(A) Abducens (VI)
(A) Caudal midbrain
(B) Facial (VII)
(B) Caudal pons
(C) Hypoglossal (XII)
(C) Medulla
(D) Vagus (X)
(D) Rostral midbrain
(E) Trigeminal (V)
(E) Rostral pons
38.
A 29-year-old woman becomes acutely ill with high fever, a stiff
44.
A 4-year-old boy is brought to the emergency department by his
neck, and stupor. A lumbar puncture reveals cloudy cerebrospinal
mother who explains that the boy fell off a porch onto a concrete
fluid from which organisms are cultured. Which of the following
sidewalk. The examination reveals that the boy has a parietal scalp
represents the most frequently seen organisms in cases of adult
laceration, is stuporous, and has reactive pupils. Suspecting that
bacterial meningitis?
this boy may have a possible skull fracture with some type of in-
tracranial bleeding, which of the following imaging tests would be
(A) Escherichia coli
most immediately (and appropriately) useful?
(B) Haemophilus influenzae
(C) Herpes simplex
(A) CT
(D) Listeria monocytogenes
(B) MRI, gadolinium enhanced
(E) Streptococcus pneumoniae
(C) MRI, T1-weighted
(D) MRI, T2-weighted
39.
Which of the following cranial nerves exits the posterior (dorsal)
(E) PET (Positron Emission Tomography)
aspect of the brainstem?
45.
A sagittal MRI of a 52-year-old man clearly shows a small tumor
(A) Abducens (VI)
in the area of the long and short gyri. These gyri are characteristi-
(B) Hypoglossal (XII)
cally found in which of the following lobes?
(C) Trigeminal (V)
(D) Trochlear (IV)
(A) Frontal
(E) Vestibulocochlear (VIII)
(B) Insular
(C) Limbic
40.
Which of the following cranial nerves passes between the poste-
(D) Occipital
rior cerebral artery and the superior cerebellar artery as it exits the
(E) Parietal
brainstem?
46.
A lesion involving the root of which of the following nerves would
(A) Abducens
most likely have an effect on the gag reflex?
(B) Oculomotor
(C) Optic
(A) Accessory
(D) Trigeminal
(B) Facial
(E) Vestibulocochlear
(C) Glossopharyngeal
(D) Hypoglossal
(E) Trigeminal
258
Q & A’s: A Sampling of Study and Review Questions with Explained Answers
8.
Answer A: The cerebral aqueduct is about 1.5-2.0 mm in di-
Answers for Chapters 1 and 2
ameter, and connects the third ventricle with the fourth ventricle.
When this part of the ventricular system appears in a sagittal MRI,
1.
Answer B: Numbness on the face, resulting from a lesion in the
the plane of the scan is at the midline. Neither the interventricu-
cerebral cortex, indicates a lesion in the lateral one-third of the
lar foramen nor the superior colliculus are on the midline. Both
postcentral gyrus (face area of the somatosensory cortex). The an-
the interpeduncular fossa and the corpus callosum are on the mid-
terior paracentral gyrus and the precentral gyrus are somatomotor
line, but extend off the midline well beyond the width of the cere-
areas of the cerebral cortex. The upper extremity is represented in
bral aqueduct. (p. 28-31, 49, 50, 52)
the middle one-third of the postcentral gyrus and the lower ex-
tremity is represented in the posterior paracentral gyrus. (p. 15)
9.
Answer D: Trauma may cause epidural hemorrhage, subdural
hemorrhage, or subarachnoid hemorrhage. Acute subdural hem-
2.
Answer D: Tic douloureux (trigeminal neuralgia) is a lancinat-
orrhage/hematoma will appear white in CT and will usually pre-
ing pain that originates from the territories of the trigeminal
sent as a comparatively thin but long defect. Epidural hemorrhage
nerve, primarily its V2 or V3 territories. The trigger zone is fre-
will usually be seen as a shorter but thicker lesion and may appear
quently around the corner of the mouth. There is a geniculate neu-
loculated (have some sort of internal structure). The structure
ralgia (related to the ear) and a glossopharyngeal neuralgia (related
(shape) of this lesion does not conform to hemorrhage into the
to the throat or palate), but neither of these originates from the
substance of the brain (brain parenchyma), into the subarachnoid
surface of the face near the oral cavity. The hypoglossal nerve is
space (or cisterns), and certainly not to hemorrhage into the ven-
the motor for the tongue and the vagus is the motor for most of
tricles. (p. 46, 48, 51)
the pharynx and larynx, visceromotor for much of the gut, and
contains viscerosensory fibers from the gut. (p. 41)
10.
Answer A: The only portion of the ventricular system that does
not contain choroid plexus is the cerebral aqueduct. The choroid
3.
Answer A: In most cases (85-100%), the labyrinthine artery,
plexus in the lateral ventricle is continuous from the inferior horn
also called the internal auditory artery, originates from the ante-
into the atrium and into the body of the ventricle, and through the
rior inferior cerebellar artery. It enters the internal acoustic mea-
interventricular foramen with the choroid plexus located along
tus, serves bone and dura of the canal, the nerves of the canal, and
the roof of the third ventricle. There is a tuft of choroid plexus in
vestibular and cochlear structures. In a few cases (15% or less),
the fourth ventricle, a small part of which extends into the lateral
this artery originates from the basilar artery. None of the other
recess and through the lateral foramen (of Luschka) into the sub-
choices gives rise to vessels that serve the inner ear. (p. 25, 27)
arachnoid space at the cerebellopontine angle. (p. 52-53)
4.
Answer E: The quadrigeminal artery is the primary blood sup-
11.
Answer E: Branches of the superior cerebellar artery are most
ply to the superior and inferior colliculi: this vessel originates from
frequently involved in cases of trigeminal neuralgia that are pre-
P1. The geniculate bodies receive their blood supply from the thal-
sumably of vascular origin. The posterior cerebral artery and its
amogeniculate arteries, and the pineal and habenula from the pos-
larger branches serve the midbrain-diencephalic junction or join
terior medial choroidal artery. The superior cerebellar peduncle
the medial surface of the hemisphere. The basilar artery serves the
receives its blood supply via the medial branch of the superior
basilar pons and the anterior inferior cerebellar artery serves the
cerebellar artery, and branches of the cerebral circle (of Willis)
caudal midbrain, inner ear, and the inferior surface of the cere-
serve the mammillary bodies. (p. 25, 35)
bellar surface. The basal vein drains the medial portions of the
hemisphere and passes through the ambient cistern to enter the
5.
Answer C: Vestibular schwannomas larger than 2.0 cm in diame-
great cerebral vein (of Galen). (p. 41)
ter may impinge on the root of the trigeminal nerve and cause numb-
ness on the same side of the face. Although the other deficits listed
12.
Answer C: The afferent limb of the corneal reflex is via the oph-
are not seen in these patients, diplopia (involvement of oculomotor,
thalmic division of the trigeminal nerve (V); the cell body of ori-
abducens or trochlear nerves, singularly or in combination) may be
gin is in the trigeminal ganglion and the central terminations in the
present, but in fewer than 10% of these individuals. (p. 42)
pars caudalis of the spinal trigeminal nucleus. The efferent limb
originates in the motor nucleus of the facial nerve (VII) and dis-
6.
Answer B: The internal acoustic meatus contains the vestibulo-
tributes to the facial muscles around the eye. None of the other
cochlear nerve, the facial nerve, and the labyrinthine artery, a
choices contains fibers related to the corneal reflex. (p. 42)
branch of the anterior inferior cerebellar artery. A vestibular
schwannoma located in the meatus would likely affect the facial
13.
Answer B: The callosomarginal artery, a branch of the anterior
nerve and result in facial weakness. The vagus and glossopharyn-
cerebral artery, serves the medial aspect of the superior frontal
geal nerves exit the skull via the jugular foramen (along with the
gyrus and that portion of this gyrus on the superior and lateral as-
accessory nerve). The cerebellar arteries originate within the skull
pects of the hemisphere. M4 segments of the middle cerebral
and distribute to structures within the skull. (p. 42)
artery serve the lateral aspects of the hemisphere; P4 segments of
the posterior cerebral artery serve the medial aspects of the hemi-
7.
Answer C: The lingual gyrus is the lower bank of the calcarine sul-
sphere caudal to the parietoccipital sulcus, and the angular artery
cus; the upper (cuneus) and lower banks of this sulcus are the loca-
(an M4 branch) serves the angular gyrus of the inferior parietal lob-
tion of the primary visual cortex. The precuneus is the medial aspect
ule. The lenticulostriate arteries are branches of M1 that serve in-
of the parietal lobe, and the angular gyrus is a portion of the inferior
ternal structures of the hemisphere. (p. 17, 29)
parietal lobule on the lateral aspect of the hemisphere. The cingu-
late and parahippocampal gyri are located on the medial aspect of the
14.
Answer B: The limbic lobe, consisting primarily of the cingu-
hemisphere and are parts of the limbic lobe. (p. 13-15, 28)
late gyrus and the parahippocampal gyrus, is located on the most
Q & A’s: A Sampling of Study and Review Questions with Explained Answers
259
medial aspect of the hemisphere; the cingulate gyrus is located im-
of the hemisphere immediately superior to the corpus callosum.
mediately adjacent to the corpus callosum. None of the other
The internal cerebral vein (to the great cerebral vein) drains the
lobes of the cerebral cortex borders directly on the corpus callo-
internal parts of the hemisphere; the ophthalmic vein connects the
sum. (p. 13, 28)
orbit with the cavernous sinus; and the superior petrosal sinus
connects the cavernous sinus with the sigmoid sinus at its junction
15.
Answer B: The inferior frontal gyrus consists of the pars or-
with the transverse sinus. (p. 19, 23, 29)
bitalis (Brodmann area 47), pars triangularis (area 45), and pars
opercularis (area 44). A lesion located primarily in areas 44 and
22.
Answer A: As they descend in the dural sac from their origin
45 in the dominant hemisphere will result in a nonfluent (Broca)
from the spinal cord to their exit at their respective intervertebral
aphasia. The supramarginal (area 40) and angular (area 39) gyri
foramen, the anterior and posterior roots form the cauda equina.
represent what is called the Wernicke area, and the middle frontal
The conus medullaris is the most caudal end of the spinal cord, and
gyrus contains areas 6 and 8. The lateral one-third of the precen-
the filum terminale internum is the strand of pia that extends from
tral gyrus is the face area of the somatomotor cortex. (p. 14)
the conus caudally to attach to the inner aspect of the dural sac at
about S2. The denticulate ligament anchors the spinal cord later-
16.
Answer A: Areas 3, 1, 2 collectively represent the primary so-
ally to the inner surface of the dural sac, and the filum terminale
matosensory cortex. Area 4 is the primary somatomotor cortex,
externum anchors the dural sac caudally to the inner aspect of the
area 17 the primary visual cortex, and area 22 the primary audi-
coccyx. (p. 12, 85, 87)
tory cortex. Area 40 is in the supramarginal gyrus, a large part of
which is called the Wernicke area. (p. 14)
23.
Answer B: The primary somatomotor cortex consists of the
precentral gyrus and the anterior paracentral gyrus; area 4 is found
17.
Answer D: The body is represented in the somatomotor cortex
in these structures. Areas 3, 1, and 2 are the primary somatosen-
(precentral gyrus, anterior paracentral gyrus) in the following pat-
sory cortex; areas 5 and 7 make up the superior parietal lobule and
tern: the face in about the lateral one-third of the precentral gyrus
the precuneus; and area 6 is located rostral to area 4. Portions of
above the lateral sulcus; the hand and upper extremity in about its
area 6 in the caudal region of the middle frontal gyrus are the
middle third; and the trunk and hip in about its medial third. The
frontal eye field. (p. 15)
lower extremity and foot are represented in the anterior paracen-
tral gyrus. Caudal portions of the middle frontal gyrus are the lo-
24.
Answer A: In this patient, the meningioma is located in the falx
cation of the frontal eye field. (p. 15)
cerebri and is impinging on the anterior paracentral gyrus corre-
lating with her motor deficit. The lower extremity is represented
18.
Answer C: The L4-L5 interspace is commonly used for a lum-
in the anterior paracentral gyrus (somatomotor) and in the poste-
bar puncture. The L3-L4 space may also be used. Levels T12 to
rior paracentral gyrus (somatosensory). The precentral gyrus con-
L2-L3 are too high. Because the caudal end of the spinal cord (the
tains the motor representation for the face (lateral part) and the
conus medullaris) may be as low as L2 in some individuals, levels
trunk and hip (medial part). The postcentral gyrus is part of the
T12-L1 to L2-L3 are not used, as this would most likely result in
somatosensory cortex. (p. 15)
damage to the spinal cord. The S1-S2 vertebrae are fused so there
is no intervertebral space through which a needle can pass. Fur-
25.
Answer D: The M4 segments of the middle cerebral artery serve
thermore, the dural sac ends at about S2. (p. 12)
the lateral aspect of the cerebral hemisphere. The named M4 ves-
sels that serve the pre- and postcentral gyri (hemorrhage into ap-
19.
Answer B: The oculomotor nerve (III) exits from the medial as-
proximately the lower two-thirds of these gyri explain the motor
pect of the midbrain into the interpeduncular fossa/cistern. It tra-
and sensory deficits) are the precentral branches (prerolandic),
verses this space, courses through the lateral wall of the cavernous
central branches
(Rolandic branches), and anterior parietal
sinus to eventually enter (along with the trochlear [IV] and ab-
branches. The M2 segment serves the insular cortex, and the M3
ducens [VI] nerves) the superior orbital fissure. Cranial nerves IV,
segment serves the inner surface of the frontal, parietal, and tem-
VI, and V1 (the ophthalmic portion of the trigeminal nerve), along
poral opercula. The A1 segment serves hypothalamic structures,
with III, pass through the cavernous sinus. Cranial nerve II (optic)
the subcallosal and septal areas, and adjacent structures. P4 serves
is quite rostral to the interpeduncular fossa. (p. 24, 30, 40)
the medial aspect of the occipital lobe (visual cortex). (p. 19, 29)
20.
Answer D: A lesion in area 44 (the pars opercularis) that spreads
will affect the lower portions of the precentral gyrus in which the
26.
Answer E: The uncus is a small elevation at the rostral and me-
face is represented. This will result in weakness of facial muscles,
dial aspect of the parahippocampal gyrus adjacent to the crus cere-
accompanied by other cranial nerve deficits. The cortical areas for
bri of the midbrain. In addition to the catastrophic effect of de-
hearing and vision are far separated from area 44. Also, a lesion in
cerebration, herniation of the uncus may also affect corticospinal
the primary auditory cortex will not result in a hearing loss in one
and corticonuclear (corticobulbar) fibers in the crus cerebri and
ear. The hand area of the sensory cortex and the upper extremity
the root of the oculomotor nerve. None of the other areas of the
area of the motor cortex are not adjacent to Brodmann area 44.
forebrain listed as choices is related to uncal herniation. (p. 20, 22)
(p. 14)
27.
Answer E: The P4 segments of the posterior cerebral artery con-
21.
Answer D: The superficial middle cerebral vein is located on the
sist of the parieto-occipital and calcarine branches; the latter being
surface of the cerebral hemisphere in the immediate vicinity of the
located in the calcarine sulcus and a primary blood supply to the
lateral sulcus and, of the choices, is the most likely candidate. The
primary visual cortex. M3 and M4 segments of the middle cerebral
deep middle cerebral vein is located on the surface of the insular
are located, respectively, on the inner aspect of the frontal, pari-
lobe. The inferior sagittal sinus primarily drains the medial aspect
etal, and temporal opercula and on the lateral aspect of the cere-
260
Q & A’s: A Sampling of Study and Review Questions with Explained Answers
bral hemisphere. The P
34.
Answer E: Cerebrospinal fluid in the ventricles, and throughout
2segment of the posterior cerebral artery is
located just distal to the posterior communicating-posterior cere-
the subarachnoid space, appears very white in T2-weighted MRI
bral intersection and gives rise to medial and lateral posterior
images. Structures located in, or traversing the subarachnoid
choroidal and to thalamogeniculate arteries. The A1 segment is lo-
space (such as vessels or cranial nerve roots, including the oculo-
cated between the internal carotid and anterior communicating
motor nerve) appear grey to black against a white background. (p.
artery and gives rise to branches that serve anterior hypothalamic
46-47, 49, 51, 54)
structures, septal areas, and the optic chiasm. (p. 21, 29)
35.
Answer A: Cerebrospinal fluid, and other fluids, appear black in
28.
Answer B: The glomus choroideum is found in the atrium of the
T1-weighted MRI images. Consequently, the ventricles, and more
lateral ventricle. This part of the choroid plexus is rostrally contin-
obvious parts of the subarachnoid space, appear black. Changes in
uous with that in the body of the lateral ventricle and continuous
ventricular shape (i.e., enlargement, midline shift), or obliterated
anteroinferiorly with that in the temporal horn. The roof of the
sulci, or even subarachnoid space, most likely represent a poten-
third ventricle has a small portion of choroid plexus that is contin-
tially serious clinical issue. (p. 2-4, 33 as one example)
uous with that in the body of the ventricle via the interventricular
foramen. The anterior horn contains no choroid plexus. (p. 52)
36.
Answer E: Patients who experience rupture of an intracranial
aneurysm frequently complain of an intense, sudden headache
29.
Answer E: Trauma is the most common cause of subarachnoid
(“the most horrible headache I have ever had”). Acute blood in the
hemorrhage (SAH). The most common cause of spontaneous (also
subarachnoid space will appear white to very white on CT. This
called nontraumatic) SAH is bleeding from a ruptured aneurysm
will contrast with the medium grey of the brain and the black of
(about 75% of all spontaneous cases). Bleeding from an arteriove-
cerebrospinal fluid (CSF) in the ventricles. The degree of white
nous malformation (AVM) is an infrequent cause of SAH (about
may vary somewhat, based on the relative concentration of blood,
5% of cases), and bleeding from brain tumors into the subarach-
from very white (concentrated blood) to white (mostly blood,
noid space is rare. Meningiomas are usually slow-growing tumors
some CSF), to very light grey (mixture of blood and CSF). (p.
that may have a rich vascular supply but rarely hemorrhage spon-
46-47, 51)
taneously. (p. 46)
37.
Answer C: The hypoglossal nerve exits the medulla via the pre-
30.
Answer D: The prepontine cistern is located external to the
olivary sulcus of the medulla immediately (and laterally) adjacent
basilar pons and contains the abducens nerve, basilar artery, ori-
to the pyramid. The abducens nerve exits in line with the preoli-
gin of the anterior inferior cerebellar artery, and small perforating
vary sulcus, but, at the caudal edge of the pons, and the trigemi-
arteries and veins. The ambient cistern is located on the lateral as-
nal nerve exits the lateral aspect of the pons. The vagus nerve ex-
pect of the midbrain and contains the trochlear nerve and several
its the lateral aspect of the medulla via the postolivary sulcus, and
major arteries. The premedullary cistern is located at the anterior
the facial nerve in line with this sulcus, but at the pons-medulla
surface of the medulla and contains the anterior spinal artery. The
junction. (p. 24, 44)
inferior cerebellopontine cistern contains the glossopharyngeal,
vagus, and accessory nerves. The superior cerebellopontine cis-
38.
Answer E: Approximately one-half of cases of bacterial menin-
tern contains the trigeminal, facial, and vestibulocochlear nerves
gitis in adults are caused by S. pneumoniae. E. coli and L. monocyto-
plus a short segment of the trochlear nerve. (p. 50, 51)
genes are causative agents in neonates and children, although the
latter (L. monocytogenes) is present in less than 10% of cases. While
31.
Answer B: Lenticulostriate arteries, also called the lateral stri-
H. influenzae was a major cause of bacterial meningitis in children,
ate arteries, originate from the M1 segment of the middle cerebral
the use of a vaccine has reduced this bacterium as a causative agent
artery and serve much of the lenticular nucleus and adjacent parts
to well under 10% of cases. H. simplex is a virus. (p. 46)
of the internal capsule. A1 branches serve the anterior hypothala-
mus and optic chiasm, and M2 branches serve the insular cortex.
39.
Answer D: The trochlear nerve exits the posterior (dorsal) as-
The P1 and P2 segments give rise to many small perforating
pect of the brainstem just caudal to the inferior colliculus and
branches and to the thalamoperforating and quadrigeminal arter-
passes around the lateral aspect of the midbrain in the ambient cis-
ies (P1), medial and lateral posterior choroidal arteries, and the
tern, en route to its exit from the skull via the superior orbital fis-
thalamogeniculate artery (P2). (p. 25, 49, 242)
sure. The abducens nerve exits at the caudal edge of the pons in
line with the preolivary fissure, and the hypoglossal exits from the
32.
Answer A: The gyrus rectus is located on the inferior and me-
medulla via this fissure. The trigeminal nerve exits the lateral as-
dial aspect of the frontal lobe. It is separated from the orbital gyri
pect of the pons, and the vestibulocochlear nerve exits at the most
by the olfactory sulcus in which the olfactory bulb and tract is lo-
lateral aspect of the pons-medulla junction. (p. 26, 33, 34)
cated. None of the other lobes has a direct relationship to the gyrus
rectus.
(p. 20, 22)
40.
Answer B: As it exits the anterior (ventral) surface of the mid-
brain, the oculomotor nerve passes between the superior cerebel-
33.
Answer B: The optic tract lies immediately on the surface of the
lar artery (which is caudal to the nerve root) and the P1 segment
crus cerebri, a relationship frequently seen in MRI. The fact that this
of the posterior cerebral artery (which is rostral to the nerve root).
patient has a right-sided weakness of the extremities specifies that the
The trigeminal root is adjacent to more distal portions of the su-
lesion is in the left crus cerebri. The bilateral visual deficits correlate
perior cerebellar artery; the labyrinthine artery accompanies the
with damage to the left optic tract. Lesions of the left basilar pons and
vestibulocochlear nerve as it enters the internal acoustic meatus;
pyramid would result in a right-sided weakness but no visual deficits.
and the ophthalmic artery accompanies the optic nerve along part
A lesion in the right optic nerve would result in blindness in that eye
of its extent. The abducens nerve passes rostrally adjacent to the
but no weakness of the extremities. (p. 20, 26, 40, 220-221)
basilar artery in the prepontine cistern. (p. 25, 39, 40)
Q & A’s: A Sampling of Study and Review Questions with Explained Answers
261
41.
Answer D: The rostral edge of the striae medullares (of the
Review and Study Questions for
fourth ventricle) is regarded as the border between the pontine
Chapters 3 and 4
and medullary portions of the fourth ventricle. These fibers pass
from the median fissure in the floor of the ventricle laterally into
1.
A 47-year-old woman presents with signs of increased intracranial
the lateral recess where they arch up into the cerebellum. The fa-
pressure (vomiting, headache, lethargy). MRI shows a large tumor
cial colliculus and median eminence are located in the floor of the
invading the head of the caudate nucleus, the rostral portion of the
pontine portion of the ventricle, and the vagal and hypoglossal
putamen and involving a fiber bundle located between these two
trigones are found in the medial floor of the medullary portion of
structures. This fiber bundle is most likely the
the fourth ventricle. (p. 34, 35, 36)
(A) anterior commissure
42.
Answer D: The primary fissure is the deepest fissure in the cere-
(B) anterior limb of the internal capsule
bellum and it separates the anterior lobe from the posterior lobe
(C) column of the fornix
and extends from the vermis to the lateral cerebellar margin. The
(D) external capsule
posterolateral fissure is located between the flocculonodular lobe
(E) posterior limb of the internal capsule
and the posterior lobe. The horizontal, secondary, and posterior
superior fissures are all located within the posterior lobe. (p. 32,
2.
A 76-year-old woman is diagnosed as having
“probable”
33)
Alzheimer’s disease based on a steady decline in cognitive func-
tion. It is likely that this woman has cell dropout in the nucleus ac-
43.
Answer C: The tonsil of the cerebellum is found on the caudal
cumbens. Which of the following most specifically describes the
and inferior aspect of the cerebellar hemisphere, adjacent to the
location of this cell group?
midline and immediately posterior (dorsal) to the medulla. The
(A) At the junction of the caudate head and putamen
cisterna magna is located in this area. Sudden tonsillar herniation
(B) At the junction of the pallidum and putamen
may compress the medulla and damage respiratory and cardiac
(C) At the junction of the pallidum and substantia nigra
centers resulting in sudden death. The tonsil herniates downward
(D) In the anterior wall of the temporal horn
through the foramen magnum. Consequently, no other part of the
(E) Internal to the uncus
brainstem is directly affected. (p. 32, 44)
3.
Which of the following structures is located in the medial wall of
44.
Answer A: A CT is a fast method, does not require sedation of
the temporal horn of the lateral ventricle and, if severely dam-
young patients, and shows bone fractures and acute intracranial
aged, may result in memory deficits?
blood in detail. MRI (T1- and T2-weighted) does not show acute
(A) Amygdaloid complex
blood or bone fracture to advantage, takes much longer to do, and
(B) Calcar avis
may require sedation in a child. Enhanced MRI is uniquely useful
(C) Hippocampus
for tumors, and PET is useful in identifying metabolic activity of
(D) Pulvinar
brain tissue, not anatomic detail. (p. 4-6)
(E) Tail of the caudate
45.
Answer B: The long and short gyri (gyri longi et breves) are
4.
Which of the following represents the fibers that fan out from the
components of the insular lobe. This lobe is located deep to the
internal capsule into the white matter of the hemisphere?
lateral sulcus, has a central sulcus that separates the short gyri (ros-
(A) Cingulum
tral to this sulcus) from the long gyri (caudal to this sulcus). The
(B) Corona radiata
cortex of the insular lobe is separated from the adjacent frontal,
(C) Genu of the corpus callosum
parietal, and temporal opercula by the circular sulcus of the insula.
(D) Superior longitudinal fasciculus
None of the other lobes has gyri that are specifically named long
(E) Uncinate fasciculus
and short gyri. (p. 13, 45, 56)
5.
The lamina of white matter located immediately internal to the
46.
Answer C: The glossopharyngeal nerve contains the afferent
cortex of the insula is the:
limb of the gag reflex and, through its innervation of the stylopha-
ryngeus muscle, is an important part of the efferent limb of this
(A) Arcuate fasciculus
reflex. The nucleus ambiguus, the location of the motor neurons
(B) External capsule
serving the stylopharyngeus, also contributes to the innervation of
(C) Extreme capsule
muscle served by the vagus nerve and, therefore, to the efferent
(D) Internal capsule
limb of the gag reflex. The trigeminal and facial nerves participate
(E) Tapetum
in the afferent and efferent limbs (respectively) of the corneal re-
flex. The accessory nerve innervates the ipsilateral trapezius and
6.
A 48-year-old man presents with a movement disorder (chorea)
sternocleidomastoid muscles, and the hypoglossal nerve inner-
and mental deterioration. MRI shows the loss of a structure in the
vates the ipsilateral genioglossus muscle. (p. 24, 43)
wall of the anterior horn of the lateral ventricle. Which of the fol-
lowing is most likely lost in this patient?
(A) Anterior thalamic nucleus
(B) Body of the caudate nucleus
(C) Column of the fornix
(D) Dorsomedial nucleus
(E) Head of the caudate nucleus
262
Q & A’s: A Sampling of Study and Review Questions with Explained Answers
7.
A 76-year-old man presents with a resting tremor, bradykinesia,
13.
A 29-year-old woman presents with neurologic deficits that wax and
and stooped posture. These observations suggest loss of a promi-
wane over time suggestive of multiple sclerosis. MRI (especially T2-
nent population of cells in the brain. Which of the following struc-
weighted) shows small, demyelinated areas at several locations in
tures is most likely affected in this patient?
her brain, one of these being the mammillothalamic tract. Which of
the following structures is most intimately associated with this tract?
(A) Lateral cerebellar nucleus
(B) Locus ceruleus
(A) Anterior thalamic nucleus
(C) Red nucleus
(B) Centromedian nucleus
(D) Substantia nigra
(C) Dorsomedial nucleus
(E) Subthalamic nucleus
(D) Ventral anterior thalamic nucleus
(E) Ventral lateral thalamic nucleus
8.
Which of the following represents the larger, more laterally lo-
cated portion of the basal nuclei (also called the basal ganglia)?
14.
Which of the following structures is a primary target of the optic
tract as it passes caudally from the optic chiasm?
(A) Caudate nucleus
(B) Globus pallidus
(A) Lateral geniculate nucleus
(C) Putamen
(B) Mammillary body
(D) Subthalamic nucleus
(C) Medial geniculate nucleus
(E) Substantia nigra
(D) Pulvinar
(E) Ventral posterolateral nucleus
9.
The MRI of a 59-year-old woman shows a large arteriovenous
malformation (AVM) located between the lenticular nucleus and
15.
An 82-year-old man presents with a severe motor deficit (resting
the dorsal thalamus. Based on its location, this AVM most likely
tremor) and dementia. The former correlates with degenerative
involves which of the following structures?
changes in the putamen and globus pallidus and the latter with de-
generative changes in the ventral striatum and ventral pallidum.
(A) Anterior limb of the internal capsule
Which of the following structures separates these two areas in the
(B) Crus cerebri
basal forebrain?
(C) External capsule
(D) Posterior limb of the internal capsule
(A) Anterior commissure
(E) Retrolenticular limb of the internal capsule
(B) Lamina terminalis
(C) Massa intermedia
10.
A 29-year-old man is brought to the emergency department with
(D) Posterior commissure
a severe and persistent headache. MRI shows a large tumor of the
(E) Septum pellucidum
pineal gland. Based on its location, this pineal lesion would most
likely impinge on which of the following structures?
16.
A 23-year-old man is brought to the emergency department by
emergency medical personnel after an automobile collision. CT
(A) Anterior thalamic nucleus
shows bilateral damage to the temporal pole and the uncus. Which
(B) Body of the caudate nucleus
of the following structures is also most likely damaged in this patient?
(C) Globus pallidus
(D) Pulvinar nucleus(i)
(A) Amygdaloid complex
(E) Ventral posteromedial nucleus
(B) Anterior thalamic nucleus
(C) Cingulum
11.
The hippocampal commissure contains fibers from one hippocam-
(D) Gracile nucleus
pal formation that cross the midline to distribute to targets on the
(E) Hippocampal formation
opposite side of the hemisphere. Which of the following struc-
tures is directly adjacent to this commissure?
17.
The optic radiations are closely associated with which of the fol-
lowing spaces?
(A) Body of the corpus callosum
(B) Genu of the corpus callosum
(A) Anterior horn of the lateral ventricle
(C) Splenium of the corpus callosum
(B) Body of the lateral ventricle
(D) Spiral fibers of the hippocampus
(C) Cisterns adjacent to the midbrain
(E) Precommissural fornix
(D) Posterior horn of the lateral ventricle
(E) Third ventricle
12.
An 85-year-old woman is brought to the emergency department
by her family because she suddenly became confused and lethar-
18.
A 31-year-old man presents with ill-defined neurologic com-
gic. CT shows a hemorrhage into the medial and lateral geniculate
plaints (persistently tired, headache, confusion). CT shows an ar-
bodies. Which of the following structures would also likely be in-
teriovenous malformation occupying most of the dorsomedial nu-
volved in this lesion due to its apposition to the geniculate bodies?
cleus (DM) of the thalamus. Which of the following structures
separates the DM from the lateral thalamic nuclei and encom-
(A) Anterior thalamic nucleus
passes the centromedial nucleus?
(B) Rostral dorsomedial nucleus
(C) Globus pallidus
(A) Ansa lenticularis
(D) Pulvinar nucleus(i)
(B) External medullary lamina
(E) Subthalamic nucleus
(C) Internal medullary lamina
(D) Lamina terminalis
(E) Stria medullaris thalami
Q & A’s: A Sampling of Study and Review Questions with Explained Answers
263
19.
A 48-year-old woman presents with violent, flailing movements
5.
Answer C: The layer of white matter located internal to the in-
of her left upper extremity. CT shows a small hemorrhage in the
sular cortex, and external to the claustrum, is the extreme cap-
subthalamic nucleus. Which of the following structures is located
sule. The external capsule is found between the claustrum and the
directly adjacent to the subthalamic nucleus?
putamen, and the internal capsule is a large bundle of fibers located
primarily between the lenticular nucleus on one side and the head
(A) Centromedian nucleus
of the caudate and the diencephalon on the other side. The tape-
(B) Globus pallidus
tum is located in the lateral wall of the posterior horn of the lat-
(C) Medial geniculate nucleus
eral ventricle. Arcuate fasciculi are small bundles of fibers passing
(D) Putamen
between gyri. (p. 65, 67-68, 77)
(E) Substantia nigra
6.
Answer E: The large bulge in the lateral wall of the anterior horn
20.
Which of the following structures is located immediately caudal to
of the lateral ventricle is the head of the caudate nucleus. The po-
the anterior commissure and appears as a distinct black spot in a
sition of the interventricular foramen represents the point at which
T2-weighted axial MRI?
the head of the caudate becomes the body of the caudate. The dor-
(A) Anterior limb of internal capsule
somedial nucleus borders on the third ventricle; the anterior thal-
(B) Column of the fornix
amic nucleus is located at the rostral end of the diencephalon and is
(C) Crus of the fornix
caudomedial to the interventricular foramen; and the column of
(D) Lenticular fasciculus
the fornix is rostromedial to this foramen. (p. 64-65, 76)
(E) Mammillothalamic tract
7.
Answer D: These deficits are characteristic of Parkinson’s dis-
ease and are directly correlated with loss of the dopamine (and
melanin)-containing cells of the substantia nigra of the midbrain.
Answers for Chapters 3 and 4
The locus (nucleus) ceruleus, also called the nucleus pigmentosus
pontis, also contains cells with melanin, but loss of these cells does
1.
Answer B: The anterior limb of the internal capsule is insinu-
not cause motor deficits. The other choices do not contain pig-
ated between the head of the caudate nucleus and the rostral as-
mented cells, but damage to these structures does cause a differ-
pect of the lenticular nucleus, mostly the putamen. The posterior
ent series of motor deficits. (p. 68-69, 78)
limb is between the lenticular nucleus and the thalamus; the col-
umn of the fornix is rostromedial to the interventricular foramen;
8.
Answer C: The putamen is the most lateral part of the basal nu-
and the anterior commissure traverses the midline at the level of
clei; taken together, the putamen and the globus pallidus comprise
the genu of the internal capsule. The external capsule is a thin
the lenticular nucleus. The caudate nucleus, specifically its head
sheet of white matter lateral to the lenticular nucleus and medial
and body portions, is located medial to the internal capsule. While
to the claustrum. (p. 64-65, 76-77)
the subthalamic nucleus and the substantia nigra function in con-
cert with the basal nuclei, these structures are medially located
2.
Answer A: The nucleus accumbens is located in the rostral and
and are not part of the basal nuclei. (p. 65-68, 76)
basal forebrain at the point where the head of the caudate is con-
tinuous with the putamen. The amygdaloid nucleus is located in-
9.
Answer D: The posterior limb of the internal capsule, contain-
ternal to the uncus and in the anterior wall of the temporal horn.
ing important cortical afferent and efferent fibers, is located be-
The pallidum (globus pallidus) and the substantia nigra do not have
tween the lenticular nucleus and the dorsal thalamus. Damage to
a continuum with the nucleus accumbens. (p. 64, 78)
this structure may result in sensory and/or motor deficits on the
opposite side of the body. The anterior limb is located between
3.
Answer C: The hippocampal formation, commonly called the
the head of the caudate and the putamen, while the retrolenticu-
hippocampus, is located in the medial wall of the temporal (infe-
lar limb is found caudal to the lenticular nucleus. The crus cerebri
rior) horn of the lateral ventricle. Damage to the hippocampus
is on the inferolateral aspect of the midbrain. The external capsule
may result in memory problems. The amygdaloid complex is lo-
is lateral to the putamen. (p. 67-69, 76-77)
cated in the rostral wall of the temporal horn, the tail of the cau-
date in its lateral wall, and the calcar avis (also called the calcarine
10.
Answer D: The pineal gland is located in the quadrigeminal cis-
spur, a ridge in the wall of the posterior horn indicating the depth
tern, superior to the colliculi, and between the pulvinar nuclei of
of the calcarine sulcus) is in the medial wall of the posterior horn
the thalamus. At this location, the lesion would potentially involve
of the lateral ventricle. The pulvinar is part of the diencephalon.
the colliculi and pulvinar. The other thalamic nuclei are not adja-
(p. 58, 68-71)
cent to the pineal, the globus pallidus is lateral to the posterior
limb of the internal capsule, and the body of the caudate is located
4.
Answer B: The corona radiata (radiating crown) are those fibers
in the lateral wall of the body of the lateral ventricle. (p. 71)
of the internal capsule that fan out in all directions from its supe-
rior edge. These fibers contain a variety of fibers traveling to and
11.
Answer C: The hippocampal commissure is located immedi-
from the cerebral cortex. The superior longitudinal and uncinate
ately inferior to the splenium of the corpus callosum; the crossing
fasciculi are organized bundles of corticocortical fibers on the ip-
of these fibers takes place at this point. Other parts of the corpus
silateral side, and the cingulum is a fiber bundle located internal to
callosum are not related to the hippocampal commissure, and the
the cingulate cortex. The fibers of the genu of the corpus callo-
spiral fibers of the hippocampus are bundles within the hippocam-
sum contain corticocortical fibers that pass between the cerebral
pal formation in the temporal lobe. Some of the fibers in the hip-
hemispheres. (p. 57, 65-69)
pocampal commissure enter the precommissural fornix, but by no
means all. (p. 72)
264
Q & A’s: A Sampling of Study and Review Questions with Explained Answers
12.
Answer D: The geniculate bodies are tucked-up under the cau-
18.
Answer C: The internal medullary lamina is a vertically ori-
dal and inferior aspect of the pulvinar. The groove between the
ented sheet of fibers that extends from the rostral portion of the
medial geniculate body and the pulvinar contains the brachium of
thalamus caudally to surround the centromedian nucleus; this nu-
the superior colliculus. The geniculate bodies and the pulvinar
cleus is frequently referred to as “in” the internal medullary lam-
have a common blood supply from the thalamogeniculate artery,
ina due to its position. This lamina separates the dorsomedial nu-
a branch of P2. None of the other choices have a close apposition
cleus from the laterally adjacent ventral anterior, ventral lateral,
with the geniculate bodies. The anterior thalamic, rostral dorso-
and ventral posterolateral nuclei. The external medullary lamina
medial, and subthalamic nuclei do not share a common blood sup-
is located between the thalamus and the posterior limb of the in-
ply with the pulvinar. (p. 58-59, 70)
ternal capsule, and the lamina terminalis is the rostral wall of the
third ventricle. The stria medullaris is a small bundle of fibers pass-
13.
Answer A: The mammillothalamic tract extends from the
ing rostrocaudally along the upper medial edge of the thalamus
mammillary bodies to the anterior nucleus of the thalamus; the
from the general location of the interventricular foramen to the
cells of origin are in the mammillary nuclei and the axons termi-
habenula, and the ansa lenticularis contains pallidothalamic fibers.
nate in the anterior nucleus. This tract is frequently visible in ax-
(p. 68-69, 76, 144-149, 162)
ial T2-weighted MRI. The ventral anterior nucleus is laterally ad-
jacent to the mammillothalamic tract, but does not receive input
19.
Answer E: The subthalamic nucleus is separated from the sub-
therefrom. The other choices are nuclei located more caudally in
stantia nigra by only a thin layer of myelinated fibers; these two
the diencephalon. (p. 67, 77)
structures are directly adjacent to each other. Damage to the sub-
thalamic nucleus gives rise to hemiballistic movements (described
in this question) while loss of cells in the substantia nigra results in
14.
Answer A: Many of the fibers contained in the optic tract ter-
the motor deficits seen in Parkinson’s disease. The putamen,
minate in the lateral geniculate nucleus. Some of these fibers by-
globus pallidus, and the medial geniculate nucleus are all lateral to
pass this nucleus to traverse the brachium of the superior collicu-
the internal capsule; the subthalamic nucleus is medial. The cen-
lus and a few enter the suprachiasmatic nucleus. The medial
tromedial nucleus is separated from the subthalamic nucleus by
geniculate nucleus receives input via the brachium of the inferior
other thalamic nuclei. (p. 68-69, 148-149)
colliculus (auditory); the pulvinar has interconnections with the
visual cortex and superior colliculus; and the ventral posterolat-
20.
Answer B: The column of the fornix is that portion of this
eral nucleus receives input from the anterolateral system and the
fiber bundle that arches around the rostromedial end of the thal-
medical lemniscus. The mammillary body is located rostral to the
amus. As it does so, the column joins its counterpart on the op-
interpeduncular fossa and medial to the optic tract. (p. 58, 59)
posite side and “leans against” the anterior commissure. The col-
umn of the fornix also signifies, in cross section or axial planes,
15.
Answer A: The anterior commissure, as it passes laterally from
the laterally adjacent interventricular foramen and genu of the
the midline, separates the dorsal basal nuclei (putamen and globus
internal capsule. The mammillothalamic tract is located caudal
pallidus) from the ventral striatum and ventral pallidum. The pos-
to the fornix, and the crus of the fornix is found along the mid-
terior commissure is located at the caudal aspect of the third ven-
line superior to the thalamus. The anterior limb of the internal
tricle just above the opening of the cerebral aqueduct, and the
capsule is found between the head of the caudate nucleus and the
Massa intermedia bridges the third ventricle in about 80% of indi-
lenticular nucleus (mainly the putamen). The lenticular fascicu-
viduals. The rostral wall of the third ventricle is formed by the
lus contains pallidothalamic fibers and traverses the posterior
lamina terminalis and the septum pellucidum forms the medial
limb of the internal capsule en route to the dorsal thalamus. (p.
wall of the anterior horn of the lateral ventricle. (p. 65, 152-153)
31, 77, 163-164)
16.
Answer A: The amygdaloid complex is located immediately in-
ternal to the uncus. Bilateral damage to rostral portions of the
temporal lobe may include the amygdala and result in a constella-
Review and Study Questions for
tion of deficits known as the Klüver-Bucy syndrome. The hip-
Chapter 5
pocampal formation is internal to the cortex of the parahip-
pocampal gyrus, and the anterior thalamic nucleus is internal to
1.
A 16-year-old boy is brought to the emergency department fol-
the anterior thalamic tubercle. The cingulate gyrus overlies the
lowing a diving accident at a local quarry. The examination reveals
longitudinally oriented fibers of the cingulum and the gracile tu-
a bilateral loss of motor and sensory function in the trunk and
bercle is the external elevation formed by the gracile nucleus. (p.
lower extremities. At 36 hours after the accident the boy is able
58-59, 65-66, 78, 170)
to dorsiflex his toes, barely move his right lower extremity at the
knee, and is able to perceive pinprick stimulation of the perianal
17.
Answer D: The optic radiations are located in the lateral wall of
skin (sacral sparing). Which of the following most specifically de-
the posterior horn of the lateral ventricle as they pass through the
scribes the spinal cord lesion in this patient?
retrolenticular limb of the internal capsule from the lateral genic-
ulate nucleus to the primary visual cortex. A thin layer of white
(A) Central cord
matter, the tapetum, separates the optic radiations from the wall
(B) Complete
of the ventricle. The cisterns at the midbrain on the basal aspect of
(C) Hemisection
the hemisphere contain the optic tract. The other ventricular
(D) Incomplete
spaces listed have no direct relationship to the optic radiations. (p.
(E) Large syringomyelia
71-72, 77, 138-141, 162)
Q & A’s: A Sampling of Study and Review Questions with Explained Answers
265
2.
A 54-year-old morbidly obese and hypertensive man is brought to
6.
Which of the following represents the most likely level of damage
the emergency department after experiencing sudden onset of
to the spinal cord resulting from the fracture to the vertebral col-
weakness of his left upper and lower extremities. CT shows an in-
umn in this man?
farcted area in the medulla. Damage to which of the following
(A) T6 on the left
tracts or fiber bundles of the medulla would most likely explain
(B) T8 on the left
this deficit?
(C) T8 on the right
(A) Anterolateral system
(D) T10 on the left
(B) Corticospinal fibers
(E) T10 on the right
(C) Medial lemniscus
(D) Rubrospinal tract
7.
The artery of Adamkiewicz is an especially large spinal medullary
(E) Vestibulospinal fibers
artery supplementing the arterial blood supply to the spinal cord.
Which of the following represents the most consistent location of
this vessel?
3.
A 78-year-old healthy, active woman experiences a sudden weak-
(A) At C7-C8 on the left
ness of her right upper extremity during an angiogram to deter-
(B) At L5-S1 on the left
mine the patency of her carotid bifurcation. The immediate ex-
(C) At L5-S1 on the right
amination reveals weakness of both extremities on the right and a
(D) At T6-T7 on the right
partial loss of vision in both eyes (homonymous hemianopsia).
(E) At T12-L1 on the left
These observations suggest an embolic stroke resulting in a lesion
involving motor and visual structures. The infarcted area in CT
8.
The CT of a 73-year-old woman shows an infarcted area in the ros-
points to the occlusion of one vessel. Which of the following ves-
tral portions of the dorsomedial nucleus, the anterior nucleus, and
sels is most likely occluded?
the ventral anterior nucleus. Which of the following arteries sup-
(A) Anterior cerebral artery
ply blood to this area of the brain?
(B) Anterior choroidal artery
(A) Anterior choroidal
(C) Ophthalmic artery
(B) Lateral striate (lenticulostriate)
(D) Lateral posterior choroidal artery
(C) Medial striate
(E) Posterior cerebral artery (P3 and P4 segments)
(D) Thalamogeniculate
(E) Thalamoperforating
4.
A 69-year-old man is brought to the emergency department by his
wife after complaining of a bad headache and becoming stuporous.
9.
Which of the following structures is insinuated between the ex-
CT shows a hemorrhage into the head of the caudate nucleus that
ternal and extreme capsules and is functionally related to the in-
has ruptured into the anterior horn of the lateral ventricle. This
sular cortex?
hemorrhage has most likely originated from which of the follow-
(A) Claustrum
ing vessels?
(B) External medullary lamina
(C) Lamina terminalis
(A) Anterior choroidal artery (branch of internal carotid)
(D) Putamen
(B) Lenticulostriate branches (of M1)
(E) Stria terminalis
(C) Medial posterior choroidal artery (branch of P2)
(D) Medial striate artery (branch of A2)
10.
An 83-year-old man is brought to the emergency department by his
(E) Thalamoperforating artery(ies)
daughter, who explains that her father started having “fits”. The ex-
amination reveals an alert, otherwise healthy, man who frequently
Questions 5 and 6 are based on the following patient.
has uncontrollable flailing movements of his left arm. Which of the
A 23-year-old man is brought to the emergency department from the
following structures is most likely involved in this lesion?
site of an automobile collision. The neurologic examination reveals
(A) Cerebellar cortex plus nuclei
weakness of the right lower extremity and a loss of pain and thermal
(B) Lenticular nucleus
sensations on the left side beginning at the level of the umbilicus. CT
(C) Subthalamic nucleus
shows a fracture of the vertebral column with displacement of bone
(D) Ventral lateral nucleus
fragments into the vertebral canal.
(E) Ventral posterolateral nucleus
11.
A 17-year-old girl presents with a bilateral loss of pain and ther-
5. Damage to which of the following tracts would correlate with
mal sensations at the base of the neck (C3 dermatome) and ex-
weakness of the lower extremity in this man?
tending over the upper extremity and down to the level of the nip-
(A) Left lateral corticospinal tract
ple (C4 to T4 dermatomes). MRI shows a cavitation in the spinal
(B) Reticulospinal fibers on the right
cord at these levels. Damage to which of the following structures
(C) Right lateral corticospinal tract
would most likely explain this deficit?
(D) Right rubrospinal tract
(A) Anterior white commissure
(E) Vestibulospinal fibers on the right
(B) Left anterolateral system
(C) Medial longitudinal fasciculus
(D) Posterior columns
(E) Right anterolateral system
266
Q & A’s: A Sampling of Study and Review Questions with Explained Answers
12.
Which of the following structures is located within the territory
17.
A 92-year-old woman is brought to the emergency department by
of the medulla that is served by the anterior spinal artery?
her caregiver. The woman had suddenly become drowsy and con-
fused. The examination revealed no cranial nerve deficits and age-
(A) Anterolateral system
normal motor function, but a loss of pain, thermal, vibratory, and
(B) Gracile fasciculus
discriminative touch sensations on one side of the body excluding
(C) Medial lemniscus
the head. CT shows a small infarcted area. Which of the following
(D) Rubrospinal tract
structures is the most likely location of this lesion?
(E) Spinal trigeminal tract
(A) Anterolateral system
(B) Medial geniculate nucleus
13.
A 59-year-old man complains to his family physician that he has
(C) Subthalamic nucleus
trouble chewing. The examination reveals a weakness of mastica-
(D) Ventral posterolateral nucleus
tory muscles on the left side. Which of the following nuclei is
(E) Ventral posteromedial nucleus
specifically related to the deficit seen in this man?
18.
In its location immediately internal to the anterior spinocerebel-
(A) Left facial motor
lar tract, which of the following fiber bundles would most likely
(B) Left hypoglossal
be damaged in a lesion to this area of the spinal cord?
(C) Left trigeminal motor
(D) Right facial motor
(A) Anterolateral system
(E) Right trigeminal motor
(B) Anterior corticospinal tract
(C) Anterior white commissure
(D) Cuneate fasciculus
14.
A 15-year-old boy with signs of increased intracranial pressure
(E) Lateral corticospinal tract
(stupor, vomiting, headache) is referred to a neurologist. The ex-
amination reveals a paralysis of upward gaze, and MRI shows a
19.
A 37-year-old man is brought to the emergency department with
large tumor of the pineal gland. Damage to which of the follow-
a severe head injury. Within a few hours he is decerebrate (upper
ing structures would be most specifically related to the gaze
and lower extremities extended) and comatose. The extension of
deficit?
his extremities indicates a dominant input to extensor motor neu-
rons through vestibulospinal and reticulospinal fibers/tracts.
(A) Exit of the trochlear nerve
Which of the following most specifically describes the position of
(B) Inferior colliculus
these activated fibers within the spinal cord?
(C) Occlusion of the great cerebral vein
(D) Posterior commissure
(A) Anterolateral area (area of anterolateral system)
(E) Superior colliculus
(B) Posterolateral area (area of lateral corticospinal tract)
(C) Posterior columns
(D) Posterolateral (dorsolateral) tract
15.
A 61-year-old man is brought to the emergency department after
(E) Intermediate zone
a fall from his garage roof. The examination reveals a hemiplegia
on the left, a loss of vibratory sense on the left, and a loss of pain
Question 20 and 21 are based on the following patient.
and thermal sensation on the right side involving the upper and
A 71-year-old woman presents to her family physician with the com-
lower extremities. These deficits are characteristically seen in
plaint that “food dribbles out of my mouth when I eat”. The examina-
which of the following syndromes?
tion reveals a unilateral weakness of muscles around the eye (palpebral
(A) Benedikt
fissure) and the opening of the mouth (oral fissure). She also has a loss
(B) Brown-Séquard
of pain and thermal sensations on the opposite side of the body ex-
(C) Claude
cluding the head. CT shows an infarcted area in the lateral portion of
(D) Wallenberg
the pontine tegmentum.
(E) Weber
20. Damage to which of the following nuclei would most likely ex-
plain the muscle weakness experienced by this woman?
16.
Based on their relative locations in the spinal cord, which of the
(A) Abducens
following tracts or fiber bundles would most likely be involved in
(B) Arcuate
a lesion located in the immediate vicinity of the lateral corti-
(C) Facial motor
cospinal tract?
(D) Hypoglossal
(A) Anterolateral system
(E) Trigeminal motor
(B) Anterior spinocerebellar tract
(C) Gracile fasciculus
21. The loss of pain and thermal sensations experienced by this woman
(D) Medial longitudinal fasciculus
would most likely correlate with a lesion involving which of the
(E) Rubrospinal tract
following structures?
(A) Anterior (ventral) trigeminothalamic tract
(B) Anterolateral system
(C) Lateral lemniscus
(D) Medial lemniscus
(E) Spinal trigeminal tract
Q & A’s: A Sampling of Study and Review Questions with Explained Answers
267
22.
A 77-year-old woman is discovered slumped on the floor in the gro-
28.
A 67-year-old woman is brought to the emergency department.
cery store; emergency medical personnel transport her to a local
She is stuporous and has signs that suggest a lesion in the brain-
hospital. The examination reveals a drowsy somewhat stuporous
stem; CT confirms this. Her right pupil is constricted (small)
woman who is difficult to arouse. CT shows a large hemorrhage
when compared with the left. Damage to which of the following
within the brain medial to the internal medullary lamina. Which of
tracts or fiber bundles in the pons or medulla would most likely
the following structures is most likely involved in this lesion?
explain this observation?
(A) Anterior thalamic nucleus
(A) Anterolateral system
(B) Dorsomedial nucleus
(B) Hypothalamospinal fibers
(C) Globus pallidus
(C) Medial longitudinal fasciculus
(D) Ventral lateral and anterior nuclei
(D) Reticulospinal fibers
(E) Ventral posterolateral nucleus
(E) Vestibulospinal fibers
23.
A 78-year-old man presents with deficits suggesting an occlusion
29.
In addition to the medial and lateral geniculate nuclei, which of the
of the posterior spinal artery at spinal cord levels C4-T2. Which
following structures is also served by the thalamogeniculate
of the following structures are in the territory served by this ves-
artery, a branch of P2?
sel at these levels?
(A) Anterior thalamic nucleus
(A) Anterolateral system
(B) Globus pallidus
(B) Cuneate fasciculus
(C) Pulvinar nucleus(i)
(C) Gracile nucleus
(D) Substantia nigra
(D) Lateral corticospinal tract
(E) Ventral anterior thalamic nucleus
(E) Medial longitudinal fasciculus
30. A 71-year- old man is brought to the emergency department by
24.
Based partially on their embryological origin from a common
his wife. She explains that he suddenly became weak in his left
group of cells, which of the following combinations of structures
lower extremity. She immediately rushed him to the hospital,
appear to be the same shade of grey in a T1- weighted MRI?
a trip of about 20 minutes. The examination reveals an alert
(A) Dorsomedial nucleus and Globus pallidus
man who is obese and hypertensive. He has no cranial nerve
(B) Globus pallidus and Caudate
deficits, is slightly weak on his left side, and has no sensory
(C) Globus pallidus and Putamen
deficits. Within 2 hours the weakness has disappeared. An MRI
(D) Putamen and Caudate nucleus
obtained the following day shows no lesions. Which of the fol-
(E) Putamen and Pulvinar
lowing most specifically describes this man’s medical experi-
ence?
25.
Which of the following portions of the trigeminal nuclear complex
(A) Central cord syndrome
is found in lateral areas of the brainstem between the level of the
(B) Small embolic stroke
obex and the spinal cord-medulla junction and is the source of
(C) Small hemorrhagic stroke
trigeminothalamic fibers conveying pain and thermal information
(D) Syringobulbia
originating from the face and oral cavity?
(E) Transient ischemic attack
(A) Mesencephalic nucleus
(B) Principal sensory nucleus
Questions 31 and 32 are based on the following patient.
(C) Spinal trigeminal nucleus, pars caudalis
A 41-year-old man is brought to the emergency department after an
(D) Spinal trigeminal nucleus, pars interpolaris
accident at a construction site. The examination reveals a weakness
(E) Spinal trigeminal nucleus, pars oralis
(hemiplegia) and a loss of vibratory sensation and discriminative touch
26.
Which of the following structures is located within the territory
all on the left lower extremity, and a loss of pain and thermal sensa-
served by branches of the posterior inferior cerebellar artery
tions on the right lower extremity. CT shows a fracture of the verte-
(commonly called PICA by clinicians)?
bral column adjacent to the T8 level of the spinal cord.
(A) Corticospinal fibers
31. Damage to which of the following fiber bundles or tracts would
(B) Hypoglossal root
most likely explain the loss of vibratory sensation in this man?
(C) Medial lemniscus
(D) Nucleus raphe magnus
(A) Anterolateral system on the right
(E) Solitary nucleus
(B) Cuneate fasciculus on the left
(C) Cuneate fasciculus on the right
27.
Space-occupying lesions within the posterior cranial fossa, or
(D) Gracile fasciculus on the left
events that increase pressure within this infratentorial region, may
(E) Gracile fasciculus on the right
result in herniation of a portion of the cerebellum through the
foramen magnum. Which of the following parts of the cerebellum
32. The loss of pain and thermal sensation in this man reflects damage
is most likely involved in this event?
to which of the following fiber bundles or tracts?
(A) Anterior lobe
(A) Anterolateral system on the left
(B) Flocculus
(B) Anterolateral system on the right
(C) Nodulus
(C) Cuneate fasciculus on the left
(D) Simple lobule
(D) Gracile fasciculus on the left
(E) Tonsil
(E) Posterior spinocerebellar tract on the left
268
Q & A’s: A Sampling of Study and Review Questions with Explained Answers
33.
Which of the following is the prominent population of melanin-
39.
Recognizing that this patient’s lesion involves the territory served
containing cells located immediately internal to the crus cerebri?
by paramedian branches of the basilar artery, which of the follow-
The loss of these cells may result in motor deficits.
ing structures is also most likely included in the area of infarction?
(A) Locus ceruleus
(A) Anterolateral system
(B) Pontine nuclei
(B) Facial motor nucleus
(C) Red nucleus
(C) Hypoglossal nucleus
(D) Reticular formation
(D) Medial lemniscus
(E) Substantia nigra
(E) Spinal trigeminal tract
34.
Which of the following structures receives visceral sensory input
40.
A 77-year-old man presents with a weakness of his right upper and
and is located immediately inferior to the medial and spinal
lower extremities and he is unable to abduct his left eye on at-
vestibular nuclei at medullary levels?
tempted gaze to the left. Which of the following most specifically
(A) Cochlear nuclei
describes this deficit?
(B) Inferior salivatory nucleus
(A) Alternating hemianesthesia
(C) Nucleus ambiguus
(B) Hemihypesthesia
(D) Spinal trigeminal nucleus
(C) Inferior alternating hemiplegia
(E) Solitary nucleus
(D) Middle alternating hemiplegia
35.
Which of the following groups of visceromotor (autonomic) cell
(E) Superior alternating hemiplegia
bodies is located lateral to the abducens nucleus, directly adjacent
to the exiting fibers of the facial nerve, and sends its axons out of
41.
In axial MRI which of the following structures is an important
the brainstem via this cranial nerve?
landmark that separates the third ventricle (rostral to this point)
(A) Dorsal motor nucleus
from the quadrigeminal cistern (caudal to this point)?
(B) Edinger-Westphal nucleus
(A) Lamina terminalis
(C) Inferior salivatory nucleus
(B) Habenular nucleus
(D) Intermediolateral cell column
(C) Massa intermedia
(E) Superior salivatory nucleus
(D) Pulvinar
(E) Superior colliculus
36.
A 56-year-old woman presents to her family physician with per-
sistent headache and nausea. MRI shows a tumor in the fourth ven-
tricle impinging on the facial colliculus. Which of the following
42.
A 77-year-old woman presents with deficits that suggest a lesion
nuclei is found immediately internal to this elevation?
involving long tracts and a cranial nerve. CT shows an infarct in
the region served by the penetrating branches of the basilar bifur-
(A) Abducens
cation. Which of the following structures is most likely located in
(B) Facial
this vascular territory?
(C) Hypoglossal
(D) Trigeminal
(A) Abducens nerve
(E) Vestibular
(B) Anterolateral system
(C) Corticospinal fibers in pyramid
Questions 37 through 39 are based on the following patient.
(D) Medial lemniscus
(E) Red nucleus
An 88-year-old man is brought to the emergency department by his
daughter. She indicates that he complained of weakness of his “arm”
Questions 43 through 46 are based on the following patient.
and “leg” (upper and lower extremities) on the right side and of “see-
ing two of everything” (double vision—diplopia). CT shows an in-
A 69-year-old man is brought to the emergency department with the
farcted area in the medial area of the pons at the pons-medulla junc-
complaint of a sudden loss of sensation. The history reveals that the
tion. The infarcted area is consistent with the vascular territory served
man is overweight, hypertensive, and does not regularly take medica-
by paramedian branches of the basilar artery.
tion. When the man speaks his voice is gravely and hoarse. The exam-
ination further reveals a loss of pain and thermal sensations on the right
37. Weakness of the extremities on the right can be explained by dam-
side of his body and on the left side of his face. CT shows an infarcted
age to which of the following structures?
area in the medulla.
(A) Corticospinal fibers on the left
(B) Corticospinal fibers on the right
43. Damage to which of the following structures would most likely
(C) Middle cerebellar peduncle on the left
explain the man’s hoarse, gravely voice?
(D) Rubrospinal fibers on the left
(A) Facial nucleus
(E) Rubrospinal fibers on the right
(B) Gracile nucleus
38. The diplopia (double vision) this man is having is most likely the
(C) Hypoglossal nucleus
result of damage to which of the following structures?
(D) Nucleus ambiguus
(E) Spinal trigeminal nucleus
(A) Abducens nerve root
(B) Facial nerve root
(C) Oculomotor nerve root
(D) Optic nerve
(E) Trochlear nerve or root
Q & A’s: A Sampling of Study and Review Questions with Explained Answers
269
44.
Injury to which of the following structures in this man is most
50. An 82-year-old woman presents to the emergency department
specifically related to the loss of pain and thermal sensations on the
with difficulty swallowing (dysphagia). Which of the following nu-
body below the neck?
clei of the medulla contain motor neurons that innervate muscles
involved in swallowing?
(A) Anterolateral system
(B) Cuneate fasciculus
(A) Dorsal motor vagal
(C) Gracile fasciculus
(B) Hypoglossal
(D) Medial lemniscus
(C) Inferior salivatory
(E) Spinal trigeminal tract
(D) Medial vestibular
(E) Nucleus ambiguus
45.
Damage to which of the following structures would most specifi-
cally explain the loss of pain and thermal sensations on the man’s
Questions 51 through 53 are based on the following patient.
face?
(A) Anterolateral system
A 73-year-old man is brought to the emergency department after los-
(B) Medial lemniscus
ing consciousness at his home. CT shows a hemorrhage into the right
(C) Medial longitudinal fasciculus
hemisphere. The man regains consciousness, but is not fully alert. Af-
(D) Solitary tract
ter 3-4 days the man begins to rapidly deteriorate: his pupils are large
(E) Spinal trigeminal tract
(dilated) and respond slowly to light, eye movement becomes re-
stricted, there is weakness in the extremities on the left side, and the
man becomes comatose. Repeat CT shows an uncal herniation.
46.
The CT shows an infarcted area in the medulla in this man. Based
on the deficits described, and the corresponding structures in-
volved, which of the following vessels is most likely occluded?
51.
Based on its location, which of the following parts of the brainstem
is most likely to be directly affected by uncal herniation, especially
(A) Anterior spinal artery
in the early stages?
(B) Posterior spinal artery
(C) Posterior inferior cerebellar artery
(A) Diencephalon/thalamus
(D) Anterior inferior cerebellar artery
(B) Mesencephalon/midbrain
(E) Penetrating branches of the vertebral artery
(C) Myelencephalon/medulla
(D) Pons and cerebellum
47.
A 77-year-old man presents with an ataxic gait. There are no other
(E) Pons only
deficits. CT shows an infarcted area in the medulla in the territory
served by the posterior inferior cerebellar artery. Damage to
52.
Damage to corticospinal fibers in which of the following locations
which of the following structures would most likely explain the
would most likely explain the weakness in his extremities?
symptoms experienced by this man?
(A) Left basilar pons
(A) Anterolateral system
(B) Left crus cerebri
(B) Corticospinal tract
(C) Right basilar pons
(C) Nucleus ambiguus
(D) Right crus cerebri
(D) Restiform body
(E) Right posterior limb of the internal capsule
(E) Vestibular nuclei
53.
The dilated, and slowly responsive, pupils in this man are most
48.
Which of the following cranial nerve nuclei is located in the ante-
likely explained by damage to fibers in which of the following?
rior (ventral or inferior) and medial portion of the periaqueductal
(A) Abducens nerve
grey at the cross-sectional level of the superior colliculus?
(B) Corticonuclear fibers in the crus
(A) Abducens
(C) Oculomotor nerve
(B) Mesencephalic
(D) Optic nerve
(C) Oculomotor
(E) Sympathetic fibers on cerebral vessels
(D) Trigeminal motor
(E) Trochlear
54.
The sagittal MRI of a 26-year-old man shows a dark shadow in the
midbrain tegmentum on the midline at the cross-sectional level of
49.
A 53-year-old woman presents with motor deficits that the exam-
the inferior colliculus. Which of the following structures does this
ining neurologist describes as a superior alternating hemiplegia.
dark area represent?
Which of the following cranial nerve roots is most likely involved
(A) Central portions of the red nucleus
in this lesion?
(B) Compact and reticular parts of the substantia nigra
(A) Abducens
(C) Decussation of the superior cerebellar peduncle
(B) Hypoglossal
(D) Decussation of trigeminothalamic fibers
(C) Oculomotor
(E) Motor (pyramidal) decussation
(D) Trigeminal
(E) Trochlear
270
Q & A’s: A Sampling of Study and Review Questions with Explained Answers
55.
The CT of a 39-year-old man with untreated hypertension shows
60.
Damage to which of the following tracts or fiber bundles would
a small hemorrhage in the brainstem. This lesion encompasses the
most likely give rise to the sensory deficits experienced by this pa-
brachium of the inferior colliculus and the brain substance imme-
tient?
diately internal to this structure. Which of the following struc-
(A) Anterolateral system
tures is also most likely involved in this lesion?
(B) Medial lemniscus
(A) Anterolateral system
(C) Medial longitudinal fasciculus
(B) Central tegmental tract
(D) Solitary tract
(C) Corticospinal fibers
(E) Spinal trigeminal tract
(D) Mesencephalic tract
(E) Oculomotor nerve
61.
The MRI of a 12-year-old boy reveals a cavity within the medulla.
Which of the following terms most specifically describes this con-
56.
A 69-year-old man complains of difficulty walking. The examina-
dition?
tion reveals no weakness, but does reveal a loss of discriminative
(A) Brown-Séquard syndrome
touch and vibratory sense on the left lower extremity. MRI shows
(B) Central cord syndrome
a small infarcted area in the midbrain. Which of the following
(C) Hydromyelia
structures is most likely involved in the infarcted area?
(D) Syringobulbia
(A) Anterolateral system
(E) Syringomyelia
(B) Corticospinal fibers
(C) Lateral part of the medial lemniscus
62.
Which of the following cell groups within the white matter of the
(D) Medial part of the medial lemniscus
cerebellum characteristically appears as a long undulating line,
(E) Rubrospinal fibers
looking somewhat like the principle olivary nucleus in the medulla?
(A) Dentate nucleus
57.
Which of the following nuclei containing visceromotor (autonomic)
(B) Emboliform nucleus
cell bodies is located immediately inferior to the medial vestibular
(C) Fastigial nucleus
nucleus, medial to the solitary tract and nucleus, and has axons that
(D) Globose nucleus
exit the brainstem on the glossopharyngeal nerve?
(E) Red nucleus
(A) Dorsal motor nucleus
(B) Edinger-Westphal nucleus
(C) Inferior salivatory nucleus
Answers for Chapter 5
(D) Intermediolateral cell column
(E) Superior salivatory nucleus
1.
Answer D: Although this patient initially presented with com-
plete motor and sensory losses, some function had returned by 36
58.
An 81-year-old woman is brought to the emergency department
hours; in this case the lesion is classified as an incomplete lesion of
by her adult grandson. He explains that during dinner she slumped
the spinal cord. Patients with no return of function at 24
hours
off of her chair, did not lose consciousness, but had trouble speak-
and no sacral sparing have suffered a lesion classified as complete
ing. The examination reveals weakness of the upper and lower ex-
and it is unlikely that they will recover useful neurologic function.
tremities on the left and deviation of the tongue to the right on
In a central cord and a large syringomyelia there is sparing of pos-
protrusion. Which of the following most specifically describes this
terior column sensations and in a hemisection the loss of motor
deficit in this elderly patient?
function is unilateral. (p. 94-95)
(A) Alternating hemianesthesia
(B) Hemihypesthesia
2.
Answer B: A medullary lesion that results in weakness of the
(C) Inferior alternating hemiplegia
extremities on one side indicates involvement of the corticospinal
(D) Middle alternating hemiplegia
fibers located in the pyramid on the contralateral side; these fibers
(E) Superior alternating hemiplegia
largely cross in the pyramidal (motor) decussation. Rubrospinal
and vestibulospinal fibers influence the activity of spinal motor
Questions 59 and 60 are based on the following patient.
neurons, but isolated lesions of these fibers would not result in a
A 79-year-old woman is brought to the emergency department after a
unilateral weakness of upper and lower extremities. The antero-
fall in her home from which she was unable to get up. The examination
lateral system and the medial lemniscus are sensory tracts. (p.
reveals a deviation of the tongue to the left on protrusion, a pronounced
98-108, 110-111)
weakness of the right upper and lower extremities, and a loss of posi-
tion and vibratory sense and discriminative touch on the right side of the
3.
Answer B: The anterior choroidal artery serves the optic tract
body below the neck. CT shows an infarcted area in the medulla.
(homonymous hemianopsia) and the inferior portions of the pos-
terior limb of the internal capsule (weakness of the extremities).
59. Which of the following represents the best localizing sign in this
The ophthalmic artery, via its central retinal branch, serves the
patient?
retina; the anterior cerebral artery serves the lower extremity ar-
eas of the motor and sensory cortices; and distal segments of the
(A) Deviation of the tongue
posterior cerebral artery serve the medial temporal cortex and the
(B) Motor loss on lower extremity
visual cortex. The lateral posterior choroidal artery serves the
(C) Motor loss on upper extremity
choroid plexus in the lateral ventricle and some adjacent struc-
(D) Sensory loss on lower extremity
tures. (p. 21, 25, 29, 35, 158-159)
(E) Sensory loss on upper extremity
Q & A’s: A Sampling of Study and Review Questions with Explained Answers
271
4.
Answer D: The head of the caudate nucleus is located in the lat-
10.
Answer C: Wild flailing movements of the extremities, espe-
eral wall of the anterior horn of the lateral ventricle and receives
cially the upper, are hemiballistic movements (hemiballismus);
its blood supply from the medial striate artery (also called the
these are characteristic of a lesion in the subthalamic nucleus.
artery of Heubner). This vessel also serves much of the anterior
Damage to the cerebellar cortex and nuclei and the lenticular nu-
limb of the internal capsule. The lenticulostriate arteries serve a
cleus will result in motor deficits, but these are usually described
large part of the lenticular nucleus and portions of the surround-
as involving tremor, ataxia, and related motor problems. The ven-
ing internal capsule, and thalamoperforating arteries serve ante-
tral lateral nucleus is a thalamic relay center for motor informa-
rior portions of the dorsal thalamus. The anterior choroidal artery
tion and the ventral posterolateral nucleus is a sensory relay nu-
provides blood supply to inferior portions of the internal capsule,
cleus. Lesions of these nuclei will result in motor
(but not
optic tract, and structures in the medial portions of the temporal
hemiballismus) and sensory deficits. (p. 146-149, 158)
lobe. The medial posterior choroidal artery serves choroid plexus
in the lateral and third ventricles and adjacent areas of the lateral
11.
Answer A: Fibers conveying pain and thermal sensations cross
midbrain and caudomedial thalamus. (p. 154-158)
the midline in the anterior white commissure. Consequently, a le-
sion of this structure, as in syringomyelia, would result in a bilat-
5.
Answer C: In this patient the weakness of the right lower ex-
eral loss of these sensations, reflecting the levels of the syrinx.
tremity is related to a lesion of lateral corticospinal tract fibers on
Damage to fibers of the anterolateral system results in a loss of
the right side of the spinal cord. The left corticospinal tract serves
these sensations on the contralateral side and the posterior
the left side of the spinal cord and the left lower extremity.
columns convey proprioception, discriminative touch, and vibra-
Rubrospinal, reticulospinal, and vestibulospinal fibers influence
tory sense. The medial longitudinal fasciculus does not contain
the activity of spinal motor neurons; however, the deficits related
fibers conveying sensory input. (p. 90-91, 94)
to corticospinal tract damage (significant weakness) will dominate
over the lack of excitation to flexor or extensor motor neurons in
12.
Answer C: The anterior spinal artery serves the medial portion
the spinal cord via these tracts. (p. 86-88, 94)
of the medulla, an area that encompasses the medial lemniscus, ex-
iting roots of the hypoglossal nerve, and the corticospinal fibers in
the pyramid. The anterolateral system, spinal trigeminal tract, and
6.
Answer C: The loss of pain and thermal sensations beginning at
rubrospinal tract are in the territory of the posterior inferior cere-
the level of the umbilicus (T10 dermatome) on the left side results
bellar artery (commonly called PICA by clinicians). The posterior
from damage to fibers of the anterolateral system at about the T8
spinal artery in the caudal medulla and spinal cord serves the
level on the right. These fibers ascend 1 to 2 levels as they cross
gracile fasciculus. (p. 110-111)
the midline. Damage at the T6 level would result in a loss begin-
ning at the T8 level on the contralateral side and damage at the T10
13.
Answer C: The masticatory muscles receive their motor inner-
level would result in a loss beginning at about the T12 level. (p.
vation via the motor neurons located in the trigeminal motor nu-
88-89, 94)
cleus on the ipsilateral side; this excludes the right trigeminal nu-
cleus. Facial motor neurons innervate the muscles of facial
7.
Answer E: The artery of Adamkiewicz is usually located at the
expression on the ipsilateral side and the hypoglossal nucleus in-
T12-L1 spinal cord levels and is more frequently (about 65% of
nervates the ipsilateral side of the tongue. (p. 120-121, 124)
the time) seen on the left side. The other cord levels listed may
have small spinal medullary arteries but not the large diameter
14.
Answer E: A pineal tumor impinging on the superior colliculus
vessel characteristic of Adamkiewicz. (p. 94)
may result in a paralysis of upward gaze (Parinaud syndrome). The
inferior colliculus is related to the auditory system, trochlear fibers
8.
Answer E: The thalamoperforating arteries serve the more ros-
innervate the ipsilateral superior oblique muscle, and the posterior
tral portions of the dorsal thalamus. These vessels may originate as
commissure contains fibers related to the pupillary light pathway.
a single trunk or as several vessels from the P1 segment of the pos-
Occlusion of the great cerebral vein may cause serious neurologic
terior cerebral artery. The anterior choroidal artery serves the op-
deficits but not specifically a paralysis of upward gaze. (p. 136)
tic tract, inferior portions of the internal capsule, choroid plexus
in the temporal horn, and structures in the medial region of the
15.
Answer B: Alternating sensory losses accompanied by a motor
temporal lobe. The thalamogeniculate artery supplies blood to the
deficit on the same side as the loss of vibratory sensation are char-
caudal thalamus, the medial striate arteries to the head of the cau-
acteristics of the Brown-Séquard syndrome (also commonly called
date nucleus, and the lateral striate arteries to much of the lentic-
a spinal cord hemisection). The Wallenberg syndrome is seen in
ular nucleus. (p. 25, 158-159)
lesions of the medulla, and the Benedikt, Claude, and Weber syn-
dromes are seen in lesions of the midbrain. In these brainstem syn-
9.
Answer A: The claustrum is the thin layer of grey matter that is
dromes there are usually characteristic cranial nerve and long tract
located between the extreme and external capsules. It is generally
signs and symptoms. (p. 90-91, 94, 110, 136)
regarded as being functionally related to the insular cortex. The
external medullary lamina is found at the interface of the lateral
16.
Answer E: The rubrospinal tract lies immediately anterior
portions of the thalamus with the internal capsule and the lamina
(ventral) to, and partially overlaps with, the lateral corticospinal
terminalis is the thin structure forming the rostral wall of the third
tract. The anterolateral system is in the anterolateral area of the
ventricle. The putamen is located medial to the external capsule
spinal cord and is spatially separated from the lateral corticospinal
and lateral to the globus pallidus and the stria terminalis is a fiber
tract. The gracile fasciculus is in the posterior columns, the me-
bundle in the groove between the body of the caudate nucleus and
dial longitudinal fasciculus is in the ventral funiculus, and the an-
the dorsal thalamus. (p. 144-153, 162)
terior spinocerebellar tract is located on the anterolateral surface
of the spinal cord. (p. 90-91, 94-95, 100)
272
Q & A’s: A Sampling of Study and Review Questions with Explained Answers
17.
Answer D: The ventral posterolateral nucleus of the thalamus
23.
Answer B: Penetrating branches of the posterior spinal artery
receives the pathways (medial lemniscus and anterolateral system)
serve the posterior columns (gracile and cuneate fasciculi) of the
that relay the information lost as a result of the lesion in this
spinal cord at all levels. Branches of the posterior spinal artery also
woman. The ventral posteromedial nucleus relays comparable in-
serve the gracile nucleus, but this structure is in the medulla, not
formation from the face and the medial geniculate nucleus is re-
in the spinal cord. The lateral corticospinal tract and the antero-
lated to the auditory system. Lesions in the subthalamic nucleus
lateral system are served by the arterial vasocorona on the surface
result in hemiballismus. The anterolateral system relays pain and
of the cord and the internal branches of the anterior spinal artery.
thermal sense; this is only part of the sensory deficits experienced
The medial longitudinal fasciculus is in the territory of the ante-
by this woman. (p. 142, 158-159)
rior spinal artery. (p. 95, 111)
18.
Answer A: The anterolateral system is located internal to the
24.
Answer D: The putamen and the caudate nucleus originate from
position of the anterior spinocerebellar tract; damage to this area
the same group of developing neurons, are collectively referred to
of the spinal cord would most likely result in a loss of pain and
as the neostriatum, and appear in the same shade of grey in a T1-
thermal sensations on the contralateral side of the body below the
weighted MRI. In general, the globus pallidus and pulvinar are dis-
lesion. The lateral corticospinal tract is located internal to the pos-
tinctly lighter than the putamen and the dorsomedial nucleus fre-
terior spinocerebellar tract, the anterior white commissure and
quently appears dark in a shade of grey distinctly different from
the anterior corticospinal tract are located in the anterior funicu-
that of the globus pallidus. (p. 151, 153, 155, 162)
lus of the cord, and the cuneate fasciculus is in the posterior col-
umn medial to the posterior horn at upper thoracic and cervical
25.
Answer C: The pars caudalis portion of the spinal trigeminal nu-
levels. (p. 88-91, 95)
cleus is located in the lateral medulla adjacent to the spinal trigem-
inal tract in cross-sectional levels between the obex and the C1
19.
Answer A: Reticulospinal fibers (medial and lateral) and lateral
level of the spinal cord. This portion of the spinal trigeminal nu-
vestibulospinal fibers are found predominately in the anterolateral
cleus is responsible for relaying pain and thermal information orig-
area of the spinal cord; medial vestibulospinal fibers are located in
inating from the face and oral cavity on one side to the ventral pos-
the medial longitudinal fasciculus. In the decerebrate patient, the
teromedial nucleus on the contralateral side. The pars interpolaris
descending influence of rubrospinal fibers on spinal flexor motor
is found at levels between the obex and the rostral end of the hy-
neurons is removed, and descending influence on extensor motor
poglossal nucleus and the pars oralis between the interpolaris and
neurons is predominant. The posterior columns, posterolateral
the principal sensory nucleus. The principal sensory nucleus is in
area of the cord, and the posterolateral tract do not contain
the pons and the mesencephalic nucleus is in the midbrain. (p.
vestibulospinal or reticulospinal fibers. The intermediate zone, a
98-106, 120, 130)
part of the spinal cord grey matter, contains some of the terminals
of these fibers but not the descending tracts in toto. (p. 86, 88, 90,
26.
Answer E: The solitary nucleus receives general visceral affer-
95)
ent (GVA) and special visceral afferent information (SVA, this in-
put is taste) and is located in the region of the medulla served by
20.
Answer C: Weakness of the muscles of the face, particularly
posterior inferior cerebellar artery. All of the other choices are in
when upper and lower portions of the face are involved, indicate
the territory served by the anterior spinal artery. (p. 111)
a lesion of either the facial motor nucleus or the exiting fibers of
the facial nerve; both are located in the lateral pontine tegmentum
27.
Answer E: The tonsil of the cerebellum is located close to the
at caudal levels. The hypoglossal nucleus innervates muscles of the
midline and immediately above the medulla: its position relative to
tongue, the trigeminal nucleus innervates masticatory muscles,
the cerebellum is caudal, medial, and inferior. Tonsillar herniation
and the abducens nucleus innervates the lateral rectus muscle, all
may compress the medulla, and if sudden, may result in death. The
on the ipsilateral side. The arcuate nucleus is a group of cells lo-
other portions of the cerebellum do not herniate. (p. 110, 123)
cated on the surface of the pyramid. (p.106, 116-120, 124)
28.
Answer B: In addition to other signs or symptoms, lesions in lat-
21.
Answer B: The fibers of the anterolateral system are located in
eral areas of the brainstem may also interrupt hypothalamospinal
the lateral portion of the pontine tegmentum anterior (ventral) to
fibers descending from the hypothalamus to the intermediolateral
the facial motor nucleus; these fibers convey pain and thermal in-
cell column in upper thoracic levels of the spinal cord. In this case
puts. The spinal trigeminal tract and the anterior trigeminothala-
the patient may present with a Horner syndrome, part of which is
mic tract also convey pain and thermal input but from the ipsilat-
a small (constricted) pupil. In addition, the affected pupil may re-
eral and contralateral sides of the face, respectively. The lateral
act slowly to reduced light. The anterolateral system conveys so-
lemniscus is auditory in function and the medial lemniscus conveys
matosensory input and fibers of the medial longitudinal fasciculus
proprioception, vibratory sense, and discriminative touch. (p.
(originating from the medulla) are primarily descending to spinal
116-120, 124)
cord levels. Reticulospinal and vestibulospinal tracts influence
spinal motor neurons. (p. 124)
22.
Answer B: The dorsomedial nucleus is located medial to the in-
ternal medullary lamina and, through its connections, one if its
29.
Answer C: The pulvinar, geniculate nuclei, ventral posterome-
functions is to participate in arousal of the cerebral cortex. The
dial and posterolateral nuclei, centromedian, and some other ad-
other choices are in (anterior nucleus) or lateral to the internal
jacent nuclei are served by the thalamogeniculate artery. The an-
medullary lamina, or, in the case of the globus pallidus, lateral to
terior and ventral anterior thalamic nuclei receive their blood
the internal capsule. (p. 144-149)
supply from thalamoperforating arteries, the substantia nigra via
branches of P1 and P2, and globus pallidus from the lenticulostri-
ate branches of M1. (p. 140-141, 158-159)
Q & A’s: A Sampling of Study and Review Questions with Explained Answers
273
30.
Answer E: The short-term loss of function, frequently involv-
36.
Answer A: The facial colliculus is an elevation in the floor of the
ing a specific part of the body, is characteristic of a transient is-
fourth ventricle located medial to the sulcus limitans and formed
chemic attack (commonly called a TIA). The follow-up MRI
by the underlying abducens nucleus and fibers (internal genu)
shows no lesion because there has been no permanent damage.
originating from the facial nucleus. The vestibular area, indicating
TIAs are caused by a brief period of inadequate perfusion of a lo-
the position of the vestibular nuclei, is lateral to the sulcus limi-
calized region of the nervous system; recovery is usually rapid and
tans and the hypoglossal nucleus is internal to the hypoglossal
complete. However, TIAs, especially if repeated, may be indica-
trigone in the medial floor of the ventricle in the medulla. The
tive of an impending stroke. Hemorrhagic strokes frequently re-
trigeminal and facial nuclei are located in the pontine tegmentum
sult in some type of permanent deficit, and the central cord syn-
and do not border on the ventricular space. (p. 34-36, 114-117)
drome has bilateral deficits. A small embolic stroke would be
visible on the follow-up MRI, and in this patient would have re-
37.
Answer A: In this case the weakness of the upper and lower ex-
sulted in a persistent deficit. Syringobulbia may include long tract
tremities on the right reflects damage to corticospinal fibers on the
signs as well as cranial nerve signs. (p.158)
left side of the basilar pons. A lesion of these fibers on the right side
of the pons would produce a left-sided weakness. Rubrospinal
31.
Answer D: Damage to the gracile fasciculus on the left (at the
fibers are not located in the territory of paramedian branches of
T8 level this is the only part of the posterior columns present) ac-
the basilar artery. Also, lesions of rubrospinal fibers and of the
counts for the loss of vibratory sensation (and discriminative
middle cerebellar peduncle do not cause weakness but may cause
touch). Injury to the gracile fasciculus on the right would result is
other types of motor deficits. (p. 24, 116, 124, 190-191)
this type of deficit on the right side. The level of the cord damage
is caudal to the cuneate fasciculi and the anterolateral system con-
38.
Answer A: The exiting fibers of the abducens nerve (on the left)
veys pain and thermal sensations. (p. 86, 88, 90, 94)
are in the territory of the paramedian branches of the basilar artery
and are laterally adjacent to corticospinal fibers in the basilar pons.
32.
Answer A: The loss of pain and thermal sensations on the right
Diplopia may result from lesions of the oculomotor and trochlear
side of the body correlates with a lesion involving the anterolateral
nerves, but these structures are not in the domain of the parame-
system on the left side of the spinal cord. A lesion of the right an-
dian basilar branches. A lesion of the optic nerve results in blind-
terolateral system would result in a left-sided deficit. The gracile
ness in that eye and damage to the facial root does not affect eye
and cuneate fasciculi convey discriminative touch, vibratory sen-
movement but may cause a loss of view of the external world if the
sation, and proprioception. The posterior spinocerebellar tract
palpebral fissure is closed due to facial muscle weakness. (p. 24,
conveys similar information, but it is not perceived/recognized as
116, 124)
such (consciously) by the brain. (p. 88, 90, 94)
39.
Answer D: At caudal pontine levels most, if not all, of the me-
33.
Answer E: The substantia nigra contains a large population of
dial lemniscus is located within the territory served by paramedian
melanin-containing cells, is located in the midbrain just internal to
branches of the basilar artery. Penetrating branches of the anterior
the crus cerebri, and the loss of these cells gives rise to the motor
spinal artery serve the hypoglossal nucleus. The other choices are
deficits characteristic of Parkinson disease. The neurotransmitter
generally in the territories of short or long circumferential
associated with these cells is dopamine. The reticular formation is
branches of the basilar artery. (p. 124-125)
in the core of the brainstem and the pontine nuclei are in the basi-
lar pons; neither of these contain cells with melanin. The red nu-
40.
Answer D: Weakness of the extremities accompanied by paralysis
cleus is in the midbrain, but its reddish tone is related to a rich vas-
of the lateral rectus muscle (innervated by the abducens nerve) on the
cular supply, not to cells containing a pigment. (p. 128-133)
contralateral side indicates a lesion in the caudal and medial pons in-
volving the abducens nerve root and corticospinal fibers. This is a
34.
Answer E: The solitary nucleus is located immediately inferior
middle alternating hemiplegia. Inferior alternating hemiplegia speci-
(ventral) to the medial and spinal vestibular nuclei and is the only
fies involvement of the hypoglossal root and the pyramid, and supe-
nucleus in the choices to receive a general visceral afferent (GVA)
rior alternating hemiplegia indicates damage to the oculomotor root
and special visceral afferent (SVA-taste) input. The inferior saliva-
and the crus cerebri. Alternating (or alternate) hemianesthesia and
tory nucleus and the nucleus ambiguus are visceromotor (general
hemihypesthesia are sensory losses. (p. 116, 124)
visceral efferent [GVE] and special visceral efferent [SVE], respec-
tively) and the spinal trigeminal and cochlear nuclei are sensory
41.
Answer B: The prominent elevation formed on the caudal and
(general somatic afferent [GSA] and special somatic afferent [SSA],
medial wall of the third ventricle, at the general level of the pos-
respectively). (p. 104, 106, 174-175)
terior commissure, represents the location of the habenular nu-
cleus. This is an excellent landmark to use in axial MRI when des-
35.
Answer E: The superior salivatory nucleus lies adjacent to the
ignating the separation between the third ventricle (rostral to this
exiting fibers of the facial nerve in a position just lateral to the ab-
point on the midline) and the quadrigeminal cistern (caudal to this
ducens nucleus in caudal levels of the pons. The preganglionic ax-
point). The pulvinar is lateral to the quadrigeminal cistern, the
ons originating from these cells distribute on peripheral branches
lamina terminalis forms the rostral wall of the third ventricle, and
of the facial nerve. The dorsal motor and inferior salivatory nuclei
the massa intermedia bridges the space of the third ventricle.
are in the medulla and associated, respectively, with the vagus and
When present (in about 80% of patients) the Massa intermedia ap-
glossopharyngeal nerves. The Edinger-Westphal nucleus is related
pears as a shadow in T2-weighted MRI bridging the third ventri-
to the oculomotor nucleus and the intermediolateral cell column
cle. The superior colliculus is a mesencephalic structure found in
is located primarily in thoracic levels of the spinal cord. (p.116,
the quadrigeminal cistern. (p. 76, 138-143, 162)
203)
274
Q & A’s: A Sampling of Study and Review Questions with Explained Answers
42.
Answer E: The red nucleus, exiting fibers of the oculomotor
48.
Answer C: The oculomotor nucleus (containing general somatic
nerve, portions of the corticospinal fibers in the crus cerebri, and
efferent [GSE] cell bodies), along with the Edinger-Westphal (con-
a number of other medially located structures are found in the ter-
taining general visceral efferent [GVE] cell bodies) nucleus, is
ritory of the penetrating branches of the basilar bifurcation. The
found in the most anterior and medial portion of the periaqueduc-
paramedian branches of the basilar artery and the corticospinal
tal grey at the superior colliculus level. The trochlear nucleus is
fibers in the pyramid serve the abducens nerve by branches of the
found at a comparable position, but at the cross-sectional level of
anterior spinal artery. The anterolateral system and the medial
the inferior colliculus. The mesencephalic nucleus is found in the
lemniscus are mainly, if not entirely, in the region of the midbrain
lateral area of the periaqueductal grey, and the trigeminal and ab-
served by branches of the quadrigeminal and posterior medial
ducens nuclei are located in the pons. (p. 130-133, 201)
choroidal arteries. (p. 137)
49.
Answer C: A superior alternating (or alternate) hemiplegia is char-
43.
Answer D: The vocalis muscle (this muscle is actually the me-
acterized by a loss of most eye movement (damage to oculomotor
dial portion of the thyroarytenoid muscle) is innervated, via the
nerve fibers) on the ipsilateral side and weakness of the upper and
vagus nerve, by motor neurons located in the nucleus ambiguus.
lower extremities (damage to corticospinal fibers in the crus cerebri)
The gracile nucleus conveys sensory input from the body and the
on the contralateral side. The abducens nerve is the cranial nerve in-
spinal trigeminal nucleus relays sensory input from the face. The
volved in a middle alternating hemiplegia and the hypoglossal is that
hypoglossal nucleus is motor to the tongue and the facial nucleus
nerve involved in an inferior alternating hemiplegia. The trigeminal
is motor to the muscles of facial expression. (p. 100-106, 110)
nerve innervates the muscles of mastication and the trochlear nerve
innervates the superior oblique muscle. (p. 132, 136, 200)
44.
Answer A: Fibers comprising the anterolateral system convey
pain and thermal sensations from the body, excluding the face.
50.
Answer E: Motor neurons in the nucleus ambiguus innervate,
These fibers are located in lateral portions of the medulla adjacent
primarily through the vagus nerve, the muscles of the throat that
to the spinal trigeminal tract; this latter tract relays pain and ther-
move a bolus of food from the oral cavity to the esophagus. The
mal sensations from the face. The gracile and cuneate fasciculi con-
tongue, via the hypoglossal nucleus and nerve, may move food
vey proprioception, discriminative touch, and vibratory sense in
around in the mouth and toward the back of the oral cavity, but
the spinal cord and the medial lemniscus conveys this same infor-
the actual act of swallowing is through the action of pharyngeal and
mation from the medulla to the dorsal thalamus. (p. 100, 102,
laryngeal musculature. The dorsal motor vagal and inferior saliva-
104, 106, 110)
tory nuclei are both visceromotor (autonomic) nuclei, and the me-
dial vestibular nucleus is involved in the regulation of eye move-
45.
Answer E: The loss of pain and thermal sensations on one side
ment and in balance and equilibrium. (p. 100-106, 110)
of the face correlates with damage to the spinal trigeminal tract;
in this case the loss is ipsilateral to the lesion. The anterolateral sys-
51.
Answer B: The uncus is at the rostral and medial aspect of the
tem relays pain and thermal sensations from the contralateral side
parahippocampal gyrus, and, in this position, is directly adjacent to
of the body, the solitary tract conveys visceral sensory input (es-
the anterolateral aspect of the midbrain. The diencephalon is ros-
pecially taste), and the medial lemniscus contains fibers relaying
tral to this point and the medulla, the most caudal part of the brain-
information related to position sense and discriminative touch.
stem, is located in the posterior fossa. Late stages of uncal hernia-
The medial longitudinal fasciculus does not contain sensory fibers.
tion may, but not always, result in damage to the rostral pons; this
(p. 100-108, 110)
is especially so if the patient becomes decerebrate. The cerebellum
is not involved in uncal herniation. (p. 20, 22, 24, 38, 78, 136)
46.
Answer C: The posterior inferior cerebellar artery (commonly
called PICA by clinicians) serves the posterolateral portion of the
52.
Answer D: Uncal herniation compresses the lateral portion of
medulla, which encompasses the anterolateral system, spinal trigem-
the brainstem, eventually resulting in compression of the corti-
inal tract, and nucleus ambiguus. The anterior and medial areas of the
cospinal fibers in the crus cerebri. Weakness on the patient’s left
medulla (containing the pyramid, medial lemniscus, and hypoglossal
side indicates damage to corticospinal fibers in the right crus. In sit-
nucleus/nerve) are served by the anterior spinal artery and the an-
uations of significant shift of the midbrain due to the herniation, the
terolateral area of the medulla (the region of the olivary nuclei) is
contralateral crus may also be damaged resulting in bilateral weak-
served by penetrating branches of the vertebral artery. The posterior
ness. While all other choices contain corticospinal fibers, none of
spinal artery serves the posterior column nuclei in the medulla and
these areas are directly involved in uncal herniation. (p, 136)
the anterior inferior cerebellar artery (commonly called AICA)
serves caudal portions of the pons and cerebellum. (p. 111)
53.
Answer C: The root of the oculomotor nerve conveys GSE
fibers to four of the six major extraocular muscles and GVE
47.
Answer D: The restiform body is a large fiber bundle located in
parasympathetic preganglionic fibers to the ciliary ganglion from
the posterolateral area of the medulla in the region served by pos-
which postganglionic fibers travel to the sphincter muscle of the
terior inferior cerebellar artery (PICA). This structure contains a
iris. Pressure on the oculomotor root, as in uncal herniation, will
variety of cerebellar afferent fibers including those of the posterior
usually compress the smaller diameter, and more superficially lo-
spinocerebellar tract. Damage to the vestibular nuclei will result
cated GVE fibers first. Optic nerve damage results in blindness in
in a tendency to fall to the ipsilateral side but will also produce
that eye, injury to sympathetic fibers to the eye results in con-
diplopia (double vision) and nausea; symptoms not experienced
striction of the pupil, and an abducens root injury results in an in-
by this patient. The anterolateral system is sensory, the nucleus
ability to abduct that eye. A lesion of corticonuclear fibers in the
ambiguus is motor to muscles of the throat (including the vocalis),
crus results primarily in motor deficits related to the facial, hy-
and the corticospinal tract is not in the PICA territory. (p. 104,
poglossal, and accessory nerves. (p. 136, 201, 221)
106, 110-111)
Q & A’s: A Sampling of Study and Review Questions with Explained Answers
275
54.
Answer C: The decussation of the superior cerebellar peduncle
59.
Answer A: The deviation of the tongue to the left on attempted
is a prominent fiber bundle located in the tegmentum of the mid-
protrusion is the best localizing sign in this woman. This is espe-
brain directly on the midline at the level of the inferior colliculus.
cially the case when the deviation of the tongue is seen in concert
This bundle is made up of cerebellar efferent fibers. The red nu-
with the motor and sensory losses described for this patient. This
cleus is located in the midbrain tegmentum, but not on the mid-
clearly indicates a lesion in the medial medulla encompassing the
line. Decussating trigeminothalamic fibers are found in the
corticospinal fibers, medial lemniscus, and exiting fibers on the
medulla and do not form a visible structure on the midline. The
hypoglossal nerve. Motor and sensory losses, without the cranial
motor decussation is a compact bundle on the midline, but it is in
nerve sign, could suggest a lesion at several different levels of the
the medulla, not the midbrain. The main parts of the substantia ni-
neuraxis. (p. 83, 110-111)
gra are in the midbrain, are seen in sagittal MRI, but they are def-
initely not on the midline. (p. 128, 163, 211)
60.
Answer B: All of the sensory deficits seen in this woman reflect
a lesion in the medial lemniscus, which is located in the medial
medulla in the territory of the anterior spinal artery. The antero-
55.
Answer A: The anterolateral system is located just internal to
lateral system and the spinal trigeminal tract convey pain and ther-
the brachium of the inferior colliculus in the lateral portions of the
mal sensations from the body (sans face) and face, respectively.
midbrain tegmentum. This tract conveys pain and thermal sensa-
The solitary tract is made up of the central processes of vis-
tions from the contralateral side of the body excluding the face.
cerosensory fibers and the medial longitudinal fasciculus at this
Corticospinal fibers are located in the crus cerebri, the mesen-
level contains descending fibers that influence spinal motor neu-
cephalic tract at the lateral edge of the periaqueductal (central)
rons. (p. 100-108, 110-111)
grey, and the central tegmental tract is, as its name indicates, in
the central part of the tegmentum. Oculomotor fibers within the
61.
Answer D: Syringobulbia is a cavitation within the medulla. A
midbrain leave the nucleus, arch through the tegmentum, and exit
cavitation in this location may communicate with a cavity in cer-
on the medial surface of the basis pedunculi into the interpedun-
vical levels of the spinal cord (syringomyelia). Hydromyelia refers
cular cistern. (p. 128-131)
to a cavity of the spinal cord that is lined with ependymal cells. The
central cord and Brown-Séquard syndromes are lesions of the
56.
Answer C: Fibers conveying discriminative touch, vibratory
spinal cord that give rise to characteristic motor and sensory
sensations, and proprioception are located in the lateral lemnis-
losses. (p. 110)
cus; those from the contralateral upper extremity are medial while
those from the contralateral lower extremity are lateral. This man
62.
Answer A: The dentate nucleus appears as a long thin undulat-
has difficulty walking due to a lesion of fibers conveying position
ing line within the white matter core of the cerebellar hemisphere.
sense from the lower extremity, not due to a lesion influencing de-
It is frequently described as having the three-dimensional shape of
scending fibers passing to spinal motor neurons. Fibers of the an-
a crumpled bag with its hilus (the opening of the bag) directed ros-
terolateral system convey pain and thermal sensation. Rubrospinal
tromedially. The other cerebellar nuclei (fastigial, globose, em-
and corticospinal are motor in function; however this man has no
boliform) are small clumps of cells, and the red nucleus is found
weakness. (p. 126-132, 178-179)
in the midbrain, not in the cerebellum. (p. 112-115)
57.
Answer C: The inferior salivatory nucleus is located in the ros-
tral medulla, medial to the solitary tract and nuclei and inferior
to the medial vestibular nucleus. Preganglionic axons that orig-
Review and Study Questions for
inate from these cells distribute on branches of the glossopha-
Chapter 6
ryngeal nerve. The dorsal motor nucleus is in the medulla, its ax-
ons travel on the vagus nerve. The superior salivatory nucleus is
1.
The MRI of a 66-year-old man shows a tumor 2.0 cm in diameter
in the caudal pons and is associated with the facial nerve. Cells
located in the lateral wall of the atrium of the lateral ventricle.
of the Edinger-Westphal nucleus are associated with the oculo-
Which of the following structures does this lesion most likely
motor nucleus of the midbrain and the intermediolateral cell
damage?
column is located primarily in thoracic levels of the spinal cord.
(A) Corticonuclear (corticobulbar) fibers
(p. 106, 203)
(B) Corticospinal fibers
(C) Optic radiations
58. Answer C: Weakness of the extremities accompanied by
(D) Pulvinar nucleus
paralysis of muscles on the contralateral side of the tongue (seen
(E) Splenium of the corpus callosum
as a deviation of the tongue to that side on protrusion) indicates
a lesion in the medulla involving the corticospinal fibers in the
2.
Which of the following structures is clearly seen in coronal and ax-
pyramid and the exiting hypoglossal roots. This is an inferior al-
ial brain slices, and in many MRIs, in planes extending from the
ternating hemiplegia. Middle alternating hemiplegia refers to a
midline laterally through the basal nuclei?
lesion of the pontine corticospinal fibers and the root of the ab-
(A) Anterior commissure
ducens nerve, and superior alternating hemiplegia specifies
(B) Column of the fornix
damage to the oculomotor root and crus cerebri. Alternating
(C) Genu of the internal capsule
(alternate) hemianesthesia and hemihypesthesia are sensory
(D) Optic chiasm
losses. (p. 102, 110)
(E) Posterior commissure
276
Q & A’s: A Sampling of Study and Review Questions with Explained Answers
3.
The MRI of a 49-year-old woman with movement and personality
9.
Which of the following nuclei is located within the internal
disorders and with cognitive dysfunction shows a large anterior
medullary lamina and may be visible in an axial MRI in either T1-
horn of the lateral ventricle. The attending physician suspects that
or T2-weighted images?
her disease has resulted in loss of brain tissue in the lateral wall of
(A) Centromedian
the anterior horn. A loss of which of the following structures would
(B) Dorsomedial
result in this portion of the ventricular system being enlarged?
(C) Pulvinar
(A) Body of the caudate nucleus
(D) Ventral anterior
(B) Head of the caudate nucleus
(E) Ventral lateral
(C) Lenticular nucleus
(D) Pulvinar nucleus (i)
10.
The sagittal MRI of a 23-year-old woman shows a mass in the right
(E) Septum pellucidum and fornix
interventricular foramen (possibly a colloid cyst); the right lateral
ventricle is enlarged. Based on its location, this mass is most likely
4.
The axial MRI of a 54-year-old man shows an arteriovenous mal-
impinging on which of the following structures?
formation located between the thalamus and the lenticular nu-
(A) Anterior nucleus of thalamus
cleus. Which of the following structures is probably most affected
(B) Posterior limb of internal capsule
by this malformation?
(C) Habenular nucleus
(A) Anterior commissure
(D) Head of caudate nucleus
(B) Anterior limb of the internal capsule
(E) Lamina terminalis
(C) Extreme capsule
(D) Retrolenticular limb of the internal capsule
11.
The sagittal MRI of a 42-year-old woman taken adjacent to the
(E) Posterior limb of the internal capsule
midline shows a round structure immediately rostral to the in-
terpeduncular fossa on the inferior surface of the hemisphere.
5.
In a sagittal MRI, and in a sagittal brain slice, both taken just off
Which of the following most likely represents this elevation?
the midline (2-4 mm), which of the following structures would
(A) Anterior commissure
be clearly evident immediately caudal to the anterior commissure?
(B) Basilar pons
(A) Column of the fornix
(C) Lamina terminalis
(B) Lamina terminalis
(D) Mammillary body
(C) Mammillothalamic tract
(E) Optic chiasm
(D) Optic chiasm
(E) Precommissural fornix
12.
Which of the following structures is located immediately inferior
to the pulvinar and, in the sagittal plane (MRI or brain section),
6.
The coronal MRI of a 15-year-old boy shows a 2.0 cm-diameter
forms a distinct elevation immediately adjacent to the lateral as-
tumor in the rostral tip of the temporal (inferior) horn of the lat-
pect of the crus cerebri?
eral ventricle. It is possibly arising from the choroid plexus in this
area of the ventricle. In addition to the hippocampus, this tumor
(A) Mammillary nuclei
is most likely impinging on which of the following structures?
(B) Medial geniculate nucleus
(A) Amygdaloid nucleus
(C) Optic tract
(B) Body of the caudate nucleus
(D) Subthalamic nucleus
(C) Hypothalamus
(E) Uncus
(D) Optic radiations
(E) Putamen
Answers for Chapter 6
7.
Which of the following structures is located immediately internal
to the crus cerebri and appears as a dark shade of grey (hy-
pointense) in a sagittal T1-weighted MRI?
1.
Answer C: The optic radiations are located in the lateral wall of
the atrium of the lateral ventricle, represent projections from the
(A) Brachium of the inferior colliculus
lateral geniculate nucleus to the calcarine cortex, pass through the
(B) Periaqueductal grey
retrolenticular limb of the internal capsule, and are separated from
(C) Pretectal area
the ventricular space by a thin layer of fibers called the tapetum.
(D) Red nucleus
The pulvinar and splenium are located rostromedial and medial,
(E) Substantia nigra
respectively, to the atrium. Corticonuclear and corticospinal
8.
An 81-year-old man is brought to the emergency department fol-
fibers are found in the genu, and the posterior limb of the internal
lowing a fall while walking in the park. The examination reveals
capsule within the hemisphere. (p. 76, 77, 138, 162)
mild confusion and memory loss, but no obvious motor or sensory
deficits. MRI shows an old infarct in the territory of the thalamus
2.
Answer A: The anterior commissure is a mediolaterally oriented
served by the thalamoperforating artery. Which of the following
bundle of fibers that crosses the midline and extends laterally, im-
nuclei is most likely involved in this lesion?
mediately inferior to the basal nuclei. In sagittal section, or in a sagit-
tal MRI, this bundle can be followed into planes of the hemisphere
(A) Centromedian
that include the most lateral portions of the thalamus and the lentic-
(B) Medial geniculate
ular nucleus. The column of the fornix and optic chiasm are located
(C) Ventral anterior
immediately adjacent to the midline. The posterior commissure is
(D) Ventral posterolateral
located at the caudal aspect of the third ventricle and immediately
(E) Ventral posteromedial
Q & A’s: A Sampling of Study and Review Questions with Explained Answers
277
superior to the opening of the cerebral aqueduct. The genu of the
9.
Answer A: The centromedian nucleus is found within the inter-
internal capsule is medial to the lenticular nucleus and rostrolateral
nal medullary lamina in a position just rostral to the pulvinar. The
to the anterior nucleus of the thalamus. (p. 163, 165, 167, 169, 171)
ventral anterior and ventral lateral nuclei are lateral to the inter-
nal medullary lamina, the dorsomedial nucleus is medial to this
3.
Answer B: The head of the caudate nucleus forms a prominent
lamina, and the pulvinar is the large nucleus forming the caudal
bulge in the lateral wall of the anterior horn of the lateral ventri-
part of the dorsal thalamus. (p. 76, 142-143, 162, 164)
cle. In Huntington’s disease, this elevation disappears, and the
wall of the ventricle may become concave laterally; the result be-
10.
Answer A: The interventricular foramen is the space formed
ing an enlarged anterior horn (hydrocephalus ex vacuo). The body
between the column of the fornix (located somewhat rostrome-
of the caudate is located in the lateral wall of the body of the lat-
dially) and the anterior nucleus of the thalamus (located some-
eral ventricle. The lenticular nucleus lies within the hemisphere
what caudolaterally). The anterior nucleus is located internal to
and does not border on any ventricular space. The septum and the
the anterior tubercle of the thalamus. The head of the caudate is
fornix are located in the medial wall of the ventricle, and the pul-
found in the lateral wall of the anterior horn of the lateral ven-
vinar borders on the superior cistern. (p. 75, 76, 152-156, 162)
tricle, and the posterior limb is located in the hemisphere be-
tween the thalamus and the lenticular nucleus. The lamina ter-
4.
Answer E: The posterior limb of the internal capsule is located
minalis extends from the anterior commissure inferiority to the
between the lenticular nucleus, which is lateral, and the thalamus,
upper edge of the optic chiasm. The habenula is a small elevation
which is medial. This large fiber bundle contains thalamocortical
in the caudal and medial wall of the third ventricle. (p. 76, 162,
projections related to motor and sensory function and descending
164)
corticospinal fibers. The anterior limb of the internal capsule is lo-
cated between the head of the caudate and the lenticular nucleus,
11.
Answer D: The mammillary body forms an obvious elevation
and the retrolenticular limb is found caudal to the lenticular nu-
on the inferior aspect of the hemisphere rostral to the interpe-
cleus. The anterior commissure is in the rostroventral portion of
duncular fossa/cistern; this small bulge is clearly evident in MRI.
the hemisphere, and the extreme capsule is immediately internal
The optic chiasm and the basilar pons are both on the inferior as-
to the insular cortex. (p. 162, 164, 166)
pect of the brain at the midline. The former is rostral to the in-
fundibulum (and the mammillary body) and the latter is caudal to
5.
Answer A: The column of the fornix, commonly called the post-
the interpeduncular fossa. The lamina terminalis forms the rostral
commissural fornix, lies caudal to, and against, the anterior com-
end of the third ventricle and the anterior commissure is adjacent
missure as it arches around the interventricular foramen and the an-
to the column of the fornix. (p. 31, 163, 170)
terior tubercle of the thalamus. The precommissural fornix is a
diffuse bundle of fibers rostral to the anterior commissure, and the
12.
Answer B: The medial and lateral geniculate nuclei are located
mammillothalamic tract is located between the mammillary body
inferior to the pulvinar, and form elevations on the surface of the
and the anterior nucleus of the thalamus. The lamina terminalis and
dorsal thalamus; the medial geniculate is adjacent to the lateral
the optic chiasm are inferior to the anterior commissure. (p. 163)
edge of the crus cerebri. The subthalamic nucleus is located inter-
nally, the mammillary nuclei (medial and lateral) are on the infe-
6.
Answer A: The amygdaloid nucleus is in the rostral wall of the
rior aspect of the thalamus, and the uncus is on the medial portion
temporal horn of the lateral ventricle. In this position the amyg-
on the temporal pole. The optic tract lies on the surface of the crus
dala is separated from the rostral tip of the hippocampus (the hip-
cerebri, but it does not form a distinct elevation on the brain sur-
pocampus occupies the medial and inferior wall of the temporal
face inferior to the pulvinar; rather, it has a structural relationship
horn) by a narrow space of the ventricle. The optic radiations are
to the lateral geniculate nucleus. (p. 26, 59, 169)
in the lateral wall of the temporal horn, but are quite caudal to its
rostral tip. The other choices do not have direct structural rela-
tionship to the rostral portions of the temporal horn. (p. 170, 171)
Review and Study Questions for
7.
Answer E: The substantia nigra is located internal to the crus
Chapter 7
cerebri and, in T1-weighted MRI, appears a darker shade of grey
(hypointense) than does the crus. The red nucleus and the peri-
1.
A 15-year-old boy is brought to the emergency department after
aqueductal grey are located in the midbrain, but do not border on
an accident on his father’s farm. The examination reveals weak-
the crus cerebri. The brachium of the inferior colliculus is found
ness of the left lower extremity, but no frank paralysis. There is a
on the lateral surface of the midbrain, and the pretectal area is ad-
loss of pinprick sensation on the right side beginning at the T8 der-
jacent to the cerebral aqueduct at the midbrain-diencephalic junc-
matome (about half way between the nipple and umbilicus), and
tion. (p. 165, 167)
dorsiflexion of the great toe in response to plantar stimulation.
Based on this examination, which of the following represents the
8.
Answer C: The ventral anterior nucleus is located in the rostral
most likely approximate location of this lesion?
portions of the thalamus, is in the territory of the thalamoperfo-
(A) T6 on the left side
rating artery, and projects to large regions of the frontal lobe. An
(B) T6 on the right side
occlusion of the vessels serving this portion of the thalamus may
(C) T8 on the left side
result in a decreased level of alertness. The other choices are in
(D) T8 on the right side
caudal regions of the thalamus, are not in the territory served by
(E) T10 on the left side
the thalamoperforating artery, and, with the exception of the cen-
tromedian nucleus, do not relate to the cortex of the frontal lobe.
(p. 159, 162, 164)
278
Q & A’s: A Sampling of Study and Review Questions with Explained Answers
2.
A 47-year-old man is transported to the emergency department
Questions 8 through 9 are based on the following patient.
from the site of an automobile collision. The examination reveals
A 62-year-old woman presents with tremor and ataxia on the right side
a paralysis of both lower extremities. Which of the following most
of the body excluding the head, and with a loss of most eye movement
specifically identifies this clinical picture?
on the left; the woman’s eye is rotated slightly down and out at rest.
(A) Alternating hemiplegia
The left pupil is dilated. There are no sensory losses on her face or
(B) Hemiplegia
body.
(C) Monoplegia
(D) Quadriplegia
8.
Based on the deficits seen in this woman, which of the following
(E) Paraplegia
represents the most likely location of the causative lesion?
(A) Cerebellum on the left
3.
A 68-year-old woman presents with a complaint of difficulty swal-
(B) Cerebellum on the right
lowing. Which of the following most specifically identifies this
(C) Midbrain on the left
condition in this patient?
(D) Midbrain on the right
(A) Dysarthria
(E) Rostral pons on the right
(B) Dysmetria
(C) Dysphagia
9.
The dilated pupil in this woman is most likely a result of which of
(D) Dyspnea
the following?
(E) Dysdiadochokinesia
(A) Intact parasympathetic fibers on the left
4.
A 37-year-old man presents to his family physician with a com-
(B) Intact parasympathetic fibers on the right
plaint of pain on his face. The examination shows that gentle stim-
(C) Intact sympathetic fibers on the left
ulation of the cheek and corner of the mouth precipitates a severe,
(D) Intact sympathetic fibers on the right
sharp, lancinating pain. A consulting neurologist orders an MRI
(E) Interrupted hypothalamospinal fibers on the left
(T2-weighted), which reveals a vascular loop that appears to be
pressing on the trigeminal root proximal to the ganglion. Which
10.
Which of the following nuclei are the primary target of cerebellar
of the following vessels is most likely involved?
efferent fibers that arise in the dentate, emboliform, and globose
nuclei on the left side?
(A) Anterior inferior cerebellar artery
(B) Posterior cerebral artery
(A) Ventral anterior nucleus on the right
(C) Posterior inferior cerebellar artery
(B) Ventral lateral nucleus on the left
(D) Quadrigeminal artery
(C) Ventral lateral nucleus on the right
(E) Superior cerebellar artery
(D) Ventral posterolateral nucleus on the left
(E) Ventral posterolateral nucleus on the right
5.
Which of the following brainstem structures receives input from
the frontal eye field (in the caudal part of the middle frontal gyrus,
11.
A 22-year-old man presents to his family physician with motor
areas 6 and 8) and is regarded as a vertical gaze center?
deficits. The examination reveals that the man has jerky up-down
(A) Abducens nucleus
movements of his upper extremities especially noticeable in his
(B) Edinger-Westphal nucleus
hands when his arms are extended. Which of the following most
(C) Oculomotor nucleus
specifically designate this abnormal movement?
(D) Paramedian pontine reticular formation (PPRF)
(A) Akinesia
(E) Rostral interstitial nucleus of the medial longitudinal
(B) Asterixis
fasciculus (MLF)
(C) Dystonia
(D) Intention tremor
6.
A newborn girl baby is unable to suckle. The examination reveals that
(E) Resting tremor
muscles around the oral cavity and of the cheek are poorly developed
or absent. A failure in proper development of which of the following
12.
A 59-year-old man is brought to his family physician by his wife.
structures would most likely contribute to this problem for this baby?
He complains of frequent and severe headaches. His wife states
(A) Head mesoderm
that he does not seem to understand what she is saying when she
(B) Pharyngeal arch 1
talks to him. The examination reveals that the man can speak flu-
(C) Pharyngeal arch 2
ently and clearly, can read notes written on paper, can hear noise,
(D) Pharyngeal arch 3
but has great difficulty understanding or interpreting sounds. MRI
(E) Pharyngeal arch 4
shows a tumor in the temporal lobe. This man is most likely suf-
fering from which of the following?
7.
Which of the following neurotransmitters is associated with hy-
(A) Agnosia
pothalamic fibers that project to the cerebellar cortex (hypothala-
(B) Agraphia
mocerebellar fibers)?
(C) Alexia
(A) Gamma aminobutyric acid
(D) Aphasia
(B) Glutamate
(E) Aphonia
(C) Histamine
(D) Noradrenalin
(E) Serotonin
Q & A’s: A Sampling of Study and Review Questions with Explained Answers
279
13. A 47-year-old man is brought to the emergency department by lo-
18.
During a busy day in the emergency department, the neurology
cal law enforcement personnel. The man is thin, undernourished,
resident sees three patients with brainstem lesions. The first is an
somnolent, and clearly intoxicated. Other indicators, such as a
83-year-old woman with a lesion in the territory of the midbrain
lack of personal hygiene, suggest that the man’s condition has been
served by the quadrigeminal and lateral posterior choroidal arter-
long-term. When the physician asks the man his name and where
ies. The second is a 68-year-old man with a posterior inferior cere-
he lives the man give a nonsensical response. This man is most
bellar artery (lateral medullary or Wallenberg) syndrome. The
likely suffering from which of the following?
third is a 47-year-old woman with a presumptive glioblastoma
multiforme invading the mid- to lateral portions of the pontine
(A) Broca aphasia
tegmentum and adjacent portions of the middle cerebellar pedun-
(B) Klüver-Bucy syndrome
cle. Which of the following would most likely be seen in all three
(C) Korsakoff syndrome
patients assuming a thorough neurologic examination?
(D) Munchausen syndrome
(E) Pick disease
(A) Claude syndrome
(B) Contralateral hemiplegia
(C) Facial hemiplegia
Questions 14 through 15 are based on the following patient.
(D) Horner syndrome
A 69-year-old man is diagnosed with dysarthria. The history reveals
(E) Medial medullary syndrome
that the man has had this problem for several weeks. MRI shows an in-
farcted area in the brainstem on the right side.
19.
Which of the following structures serves as an important landmark
in the placement of the intentional division of the spinal cord
14.
Damage to which of the following structures would most likely
(myelotomy) in an anterolateral cordotomy?
explain this deficit in this man?
(A) Anterior median sulcus
(A) Cuneate nucleus
(B) Anterolateral sulcus
(B) Nucleus ambiguus
(C) Denticulate ligament
(C) Solitary tract and nuclei
(D) Posterior intermediate sulcus
(D) Spinal trigeminal tract
(E) Posterolateral sulcus
(E) Vestibular nuclei
20.
A 17-year-old boy is brought to the emergency department from
a high school football game. The examination reveals a loss of vi-
15.
Assuming that the infarcted area in the brain of this man is the re-
bratory sensation and discriminative touch on the left lower ex-
sult of a vascular occlusion, which of the following arteries is most
tremity and to the level of the umbilicus. CT shows a vertebral
likely involved?
fracture with bone displacement into the vertebral canal. Which
(A) Anterior inferior cerebellar
of the following indicates the most likely level of damage to the
(B) Labyrinthine
spinal cord in this boy?
(C) Posterior inferior cerebellar
(A) T7-8 on the left
(D) Posterior spinal
(B) T10 on the left
(E) Superior cerebellar
(C) T12 on the left
(D) T8-9 on the right
16.
Which of the following neurotransmitters is associated with the
(E) T10 on the right
cells in the somatomotor cortex that project to the spinal cord as
corticospinal fibers?
21.
During the neurologic examination of a 52-year-old man, the
(A) Acetylcholine
physician decides to test the gag reflex. Which of the following dif-
(B) Dopamine
ficulties does this man have that would cause the physician to de-
(C) Gamma aminobutyric acid
cide to test this particular reflex?
(D) Glutamate
(A) Dysgeusia
(E) Serotonin
(B) Dysmetria
(C) Dysphagia
17.
A 77-year-old woman presents with a loss of pain and thermal sen-
(D) Dyspnea
sations on the right side of her face and on the left side of her body.
(E) Gustatory agnosia
Which of the following most specifically describe this deficit in this
woman?
22.
A 57-year-old woman presents with the main complaint of diffi-
(A) Alternating hemianesthesia
culty speaking. The examination reveals that the woman’s tongue
(B) Epidural anesthesia
deviates to the right on attempted protrusion. When she says “Ah”
(C) Facial hemiplegia
her soft palate elevates slightly on the left and the uvula deviates
(D) Hemifacial spasm
to the same side. This combination of deficits would most likely
(E) Superior alternating hemiplegia
indicate a small lesion in which of the following?
(A) Crus cerebri on the right
(B) Genu of the internal capsule on the left
(C) Genu of the internal capsule on the right
(D) Lateral medulla on the right
(E) Medial medulla on the right
280
Q & A’s: A Sampling of Study and Review Questions with Explained Answers
23. A 36 year-old-woman is diagnosed with myasthenia gravis. Which
28. A 45-year-old-man is brought to his family physician by his wife.
of the following deficits are seen first in about one-half of patients
The man’s main complaint is that he feels “ real dizzy” and a little
with this disease and is present in most at some time during its
nauseated. The examination reveals that the man has a disease of
course?
his semicircular canals. While sitting still the man perceives that
his body is actually moving around the room. Which of the fol-
(A) Diplopia
lowing most specifically describes this condition?
(B) Dysmetria
(C) Lower extremity weakness
(A) Ataxia
(D) Tremor
(B) Hysterical vertigo
(E) Upper extremity weakness
(C) Nystagmus
(D) Objective vertigo
(E) Subjective vertigo
Questions 24 through 26 are based on the following patient.
An 80-year-old woman is brought to the emergency department from
Questions 29 and 30 are based on the following patient.
an assisted care facility. The woman, who is in a wheelchair, complains
of not feeling well, of numbness on her face, and of being hoarse, al-
A 37-year-old-man is brought to the emergency department from the
though she claims not to have a cold. The examination reveals a loss of
site of an automobile collision. He was unrestrained and, as a result,
pain and thermal sensations on the right side of her face and on the left
has extensive injuries to his face and head. CT shows numerous frac-
side of her body. CT shows an infarcted area in the lateral portion of
tures of the facial bones and skull and blood in the rostral areas of the
the medulla.
frontal lobes and in the rostral 3-4 cm of the temporal lobes, bilater-
ally. After several weeks of recovery the man is moved to a long-term
care facility. His behavior is characterized by (1) difficulty recognizing
24.
A lesion of which of the following structures in this woman would
sounds such as music or words; (2) a propensity to place inappropriate
explain the loss of pain and thermal sensations on her body ex-
objects in his mouth; (3) a tendency to eat excessively or to eat non-
cluding the head?
food items such as the leaves on the plant in his room; and (4) a ten-
(A) Anterolateral system on the left
dency to touch his genitalia.
(B) Anterolateral system on the right
(C) Medial lemniscus on the left
(D) Spinal trigeminal nucleus on the left
29.
Which of the following most specifically describes the tendency of
(E) Spinal trigeminal tract on the left
this man to eat excessively?
(A) Aphagia
25.
The hoarseness in this woman is most likely due to which of the
(B) Dysphagia
following?
(C) Dyspnea
(D) Hyperorality
(A) Lesion of the facial nucleus
(E) Hyperphagia
(B) Lesion of the hypoglossal nucleus/nerve
(C) Lesion of the nucleus ambiguus
(D) Lesion of the spinal trigeminal tract
30.
Based on the totality of this man’s deficits he is most likely suffer-
(E) Lesion of the trigeminal nucleus
ing from which of the following?
(A) Klüver-Bucy syndrome
26.
Assuming this woman suffered a vascular occlusion, which of the
(B) Korsakoff syndrome
following vessels is most likely involved?
(C) Senile dementia
(A) Anterior inferior cerebellar artery
(D) Wallenberg syndrome
(B) Anterior spinal artery
(E) Wernicke aphasia
(C) Posterior inferior cerebellar artery
(D) Posterior spinal artery
31.
A 31-year-old woman is examined by an otolaryngologist pur-
(E) Superior cerebellar artery
suant to her complaint of hearing difficulties. The physician places
a tuning fork against the woman’s mastoid bone until she no longer
27.
In the course of a neurologic examination of a 23-year-old man,
perceives sound, then moves the prongs to her external ear where
the physician places her index finger on the midline of the
a faint sound is again heard. This maneuver is best described as:
mandible and taps it with a percussion hammer stimulating the af-
(A) A negative (abnormal) Rinne test
ferent limb of the jaw ( jaw-jerk) reflex. Collateral fibers from
(B) A normal Binet test
which of the following brainstem nuclei enter the trigeminal mo-
(C) A normal Weber test
tor nucleus to initiate the motor response?
(D) A positive (normal) Rinne test
(A) Hypoglossal
(E) Weber test localizing to the deaf side
(B) Mesencephalic
(C) Principal sensory
(D) Spinal trigeminal, pars caudalis
(E) Spinal trigeminal, pars interpolaris
Q & A’s: A Sampling of Study and Review Questions with Explained Answers
281
32.
A 64-year-old man is brought to a rural health clinic by a neigh-
37. A 59-year-old man, who is a family physician, confides in a neu-
bor. The history reveals that the man is a recluse, lives by himself,
rology colleague that he believes he has early stage Parkinson’s dis-
and does not regularly visit a physician. The examination reveals
ease. The neurologic examination reveals a slight resting tremor
that the man has difficulty walking, chorea and dystonia, and is suf-
of the left hand, a slow gait, and a lack of the normal range of fa-
fering dementia. The neighbor believes that the man’s father died
cial expression for this man. Which of the following is the most
from a similar disease. A tentative diagnosis of Huntington’s dis-
likely location of the degenerative changes at this stage of this
ease is made. Absence of which of the following structures in an
physician’s disease?
MRI of this man would be consistent with this diagnosis?
(A) Bilateral substantia nigra
(A) Anterior lobe of cerebellum
(B) Left globus pallidus
(B) Head of the caudate
(C) Left substantia nigra
(C) Lateral thalamic nuclei
(D) Right globus pallidus
(D) Substantia nigra
(E) Right substantia nigra
(E) Subthalamic nucleus
38. A 14-year-old boy is brought to the emergency department after
33.
A 23-year-old man is brought to the emergency department from an
an accident on his BMX bicycle. The examination reveals that the
accident at a construction site. CT shows a fracture of the left mas-
boy has severe facial injuries. Craniofacial CT shows fracture of fa-
toid bone with total disruption of the stylomastoid foramen. Which
cial bones and probable crushing of the structures traversing the
of the following deficits would most likely be seen in this man?
superior orbital fissure. Damage to which of the following struc-
tures passing through this fissure would result in diplopia when at-
(A) Alternating hemianesthesia
tempting to look down and in?
(B) Alternating hemiplegia
(C) Central seven
(A) Abducens nerve
(D) Facial hemiplegia
(B) Oculomotor nerve
(E) Hemifacial spasm
(C) Ophthalmic nerve
(D) Ophthalmic vein
34.
Cell bodies located in which of the following ganglia of the head
(E) Trochlear nerve
supply postganglionic fibers to the parotid gland?
Questions 39 through 41 are based on the following patient.
(A) Ciliary
(B) Intramural
A 67-year-old man is brought to the emergency department by his
(C) Otic
wife. She explains that he fell suddenly, could not get up, and com-
(D) Pterygopalatine
plained of feeling sick. The examination revealed a left-sided weakness
(E) Submandibular
of the upper and lower extremities, a lack of most movement of the
right eye, and a dilated pupil on the right. MRI shows an infarcted area
Questions 35 and 36 are based on the following patient.
in the brainstem.
A 23-year-old man is brought to the emergency department from the
39. The weakness of this man’s extremities is explained by damage to
site of an automobile collision. CT shows fractures of the facial bones
the axons of cell bodies that are located in which of the following
and evidence of bilateral trauma to the temporal lobes (blood in the
regions of the brain?
substance of the brain).
(A) Left somatomotor cortex
35. As this man recovers, which of the following deficits is most likely
(B) Right anterior paracentral gyrus
to be the most obvious in this man?
(C) Right crus cerebri
(D) Right precentral gyrus
(A) A bilateral sensory loss in the lower body
(E) Right somatomotor cortex
(B) A loss of immediate and short-term memory
(C) A loss of long-term (remote) memory
40. This man’s dilated pupil is due to damage to which of the follow-
(D) Dementia
ing fiber populations?
(E) Dysphagia and dysarthria
(A) Preganglionic fibers from the Edinger-Westphal nu-
36. Assuming that this man has also sustained bilateral injury to the
cleus
Meyer-Archambault loop, which of the following deficits would
(B) Preganglionic fibers from the inferior salivatory nucleus
this man also most likely have?
(C) Postganglionic fibers from the ciliary ganglion
(D) Postganglionic fibers from the geniculate ganglion
(A) Bitemporal hemianopsia
(E) Postganglionic fibers from the superior cervical gan-
(B) Bilateral inferior quadrantanopia
glion
(C) Bilateral superior quadrantanopia
(D) Left superior quadrantanopia
41. Which of the following descriptive phrases best describes the con-
(E) Right superior quadrantanopia
stellation of signs and symptoms seen in the man?
(A) Alternating hemianesthesia
(B) Brown-Séquard syndrome
(C) Inferior alternating hemiplegia
(D) Middle alternating hemiplegia
(E) Superior alternating hemiplegia
282
Q & A’s: A Sampling of Study and Review Questions with Explained Answers
42.
Which of the following structures contains the cell bodies of ori-
46.
The facial sensory deficits experienced by this woman are ex-
gin for fibers conveying taste information from the anterior two-
plained by a lesion to the axons of cell bodies located in which of
thirds of the tongue?
the following structures?
(A) Ciliary ganglion
(A) Anterior trigeminothalamic fibers on the left
(B) Geniculate ganglion
(B) Left trigeminal ganglion
(C) Superior ganglion of the vagus nerve
(C) Principal sensory nucleus on the left
(D) Superior ganglion of the glossopharyngeal nerve
(D) Right trigeminal ganglion
(E) Trigeminal ganglion
(E) Spinal trigeminal nucleus on the right
43.
During a screening neurologic examination of a 39-year-old man,
47.
The loss of pain and thermal sensations experienced by this woman
the physician taps the supraorbital ridge, stimulating the supraor-
on the right side of her body (excluding the face) is most likely the
bital nerve, and elicits a motor response. Which of the following
result of damage to which of the following structures?
most likely represents the motor response in this man?
(A) Anterolateral system fibers on the left
(A) Constriction of the masticatory muscles
(B) Anterolateral system fibers on the right
(B) Constriction of the orbicularis oculi muscle
(C) Anterior trigeminothalamic fibers on the left
(C) Constriction of the pupil
(D) Medial lemniscus on the left
(D) Dilation of the pupil
(E) Medial lemniscus on the right
(E) Horizontal nystagmus
48.
Taking into account all the deficits experienced by this woman,
44.
A 67-year-old man has a bilateral anterolateral cordotomy at T10
which of the following characterizes the syndrome, and the side,
for intractable pelvic pain. Four months after this procedure the
in this patient?
man begins to experience pain sensations. Which of the following
(A) Benedikt syndrome on the left
would most likely explain this apparent recurrence of pain in this
(B) Lateral medullary syndrome on the left
man?
(C) Lateral medullary syndrome on the right
(A) Activation of postsynaptic posterior column and spin-
(D) Parinaud syndrome (bilateral)
ocervicothalamic pathways
(E) Weber syndrome on the right
(B) Activation of recurrent corticospinal fibers
(C) Activation of spinoreticular-reticulothalamic-thalamo-
49.
A 17-year-old boy from a poor rural community is diagnosed with
cortical pathways
hepatolenticular degeneration (Wilson’s disease). Which of the
(D) Regeneration of anterolateral system fibers in the spinal
following is accumulating in certain tissues of his body and pro-
cord
ducing health problems?
(E) Regeneration of anterolateral system fibers into the
(A) Arsenic
posterior column system
(B) Copper
(C) Lead
45.
An 84-year-old woman presents to her physician with the com-
(D) Magnesium
plaint of difficulty walking. The examination reveals that the
(E) Mercury
woman has an unsteady gait and tends to forcibly slap her feet to
the floor as she walks. She has no other deficits. The physician con-
50.
Which of the following represents the location of the postgan-
cludes that the woman has sensory ataxia. Degenerative changes
glionic fibers that influence the dilator pupillae muscle of the iris
in which of the following would most likely explain this deficit?
on the ipsilateral side?
(A) Anterolateral system fibers
(A) Ciliary ganglion
(B) Corticospinal fibers
(B) Edinger-Westphal nucleus
(C) Posterior column fibers
(C) Hypothalamus
(D) Posterior root fibers
(D) Intermediolateral cell column
(E) Vestibulospinal and reticulospinal fibers
(E) Superior cervical ganglion
Questions 46 through 48 are based on the following patient.
51.
A 37-year-old man presents with vertigo, nystagmus, ataxia, and
hearing loss in his right ear. MRI shows a tumor in the cerebello-
A 70-year-old woman is brought to the emergency department by her
pontine angle. A biopsy specimen of this tumor indicates that this
daughter after becoming ill during a trip to the mall. The woman is
mass most likely originated from myelin-forming cells on the root
conscious but lethargic, and she has trouble speaking and swallowing.
of the vestibulocochlear nerve. Which of the following terms most
The examination reveals a loss of pain and thermal sensation on the left
correctly identifies this tumor?
side of the face and a hoarse gravely voice (as if the woman has a sore
throat). Movements of the extremities are normal for the woman’s
(A) Acoustic neuroma
age, but she has a loss of pain and thermal sensations on the right side
(B) Ependymoma
of her body. The corneal reflex is absent on the left side. MRI shows
(C) Glioblastoma multiforme
an infarcted area in the brainstem.
(D) Meningioma
(E) Vestibular schwannoma
Q & A’s: A Sampling of Study and Review Questions with Explained Answers
283
52.
An inherited (autosomal recessive) disorder may appear early in
57. An 11-year-old girl is brought to the family physician by her
the teenage years. These patients have degenerative changes in the
mother. The mother explains that the girl has been complaining
spinocerebellar tracts, posterior columns, corticospinal fibers,
that her hands and arms “feel funny”. In fact, the mother states that
cerebellar cortex, and at select places in the brainstem. The symp-
the girl cut her little finger, but did not realize it until she saw
toms of these patients may include ataxia, paralysis, dysarthria,
blood. The examination reveals a bilateral loss of pain and thermal
and other clinical manifestations. This constellation of deficits is
sensation on the upper extremities and shoulder. Which of the fol-
most characteristically seen in which of the following?
lowing is the most likely cause of this deficit in this girl?
(A) Friedreich ataxia
(A) Brown-Séquard syndrome
(B) Huntington disease
(B) Posterior inferior cerebellar artery syndrome
(C) Olivopontocerebellar degeneration (atrophy)
(C) Tabes dorsalis
(D) Parkinson disease
(D) Syringobulbia
(E) Wallenberg syndrome
(E) Syringomyelia
53.
A 45-year-old man complains to his family physician that there seems
58. A 57-year-old obese man is brought to the emergency department
to be something wrong with his mouth. The examination reveals a
by his wife. The examination reveals that cranial nerve function is
weakness of the masticatory muscles, a deviation of the jaw to the left
normal but the man has bilateral weakness of his lower extremities.
on closure, and a sensory loss on the same side of the lower jaw. MRI
He has no sensory deficits. MRI shows a small infarcted area in the
shows a tumor, presumably a trigeminal schwannoma, in the fora-
general region of the cervical spinal cord-medulla junction. Which
men ovale. Compression of which of the following structures would
of the following represents the most likely location of this lesion?
most likely be the cause of the deficits experienced by this man?
(A) Caudal part of the pyramidal decussation
(A) Maxillary and mandibular nerves and motor fibers on
(B) Lateral corticospinal tract on the left
the left
(C) Pyramids bilaterally
(B) Motor fibers and mandibular nerve on the left
(D) Pyramid on the right
(C) Motor fibers and mandibular nerve on the right
(E) Rostral part of the pyramidal decussation
(D) Motor fibers and maxillary nerve on the left
(E) Motor fibers and maxillary nerve on the right
Questions 59 through 61 are based on the following patient.
A 34-year-old woman presents with the complaint of seeing “two of
54.
A 49-year-old man visits his ophthalmologist with what the man
everything” (diplopia). The history reveals that the woman becomes
interprets as “trouble seeing”. The history reveals that the man had
tired during the workday to the point where she frequently must leave
a sudden event a few days before in which he felt sick and was nau-
her workplace early. The woman said that her vision problems ap-
seated. The man said his trouble “seeing” started after this sudden
peared first, and later she noticed that, when she drank, it would “go
sickness. The examination reveals a loss of abduction and adduc-
down the wrong pipe”. The examination reveals weakness of the ocu-
tion of the right eye and a loss of adduction of the left eye. MRI
lar muscle, difficulty in swallowing (dysphagia), and mild weakness of
confirms an infarcted area in the caudal and medial pontine
the upper extremities. Sensation is normal. Further laboratory tests
tegmentum. Which of the following most specifically identifies
indicate that the woman has a neurotransmitter disease.
this man’s clinical problem?
(A) Horizontal gaze palsy
59. Based on the history and symptoms experienced by this woman,
(B) Internuclear ophthalmoplegia
which of the following is the most likely cause of her medical con-
(C) One-and-a-half syndrome
dition?
(D) Parinaud syndrome
(A) Amyotrophic lateral sclerosis
(E) Vertical gaze palsy
(B) Huntington disease
(C) Myasthenia gravis
55.
Collaterals of ascending anterior
(ventral) trigeminothalamic
(D) Multiple sclerosis
fibers that contribute to the vomiting reflex would most likely
(E) Parkinson disease
project into which of the following brainstem structures?
(A) Dorsal motor vagal nucleus
60. Which of the following represents the most likely location of the
(B) Facial nucleus
neurotransmitter dysfunction in this woman?
(C) Nucleus ambiguus
(A) At the termination of corticonuclear fibers
(D) Superior salivatory nucleus
(B) At the termination of corticospinal fibers
(E) Trigeminal motor nucleus
(C) At the neuromuscular junction
(D) Within the basal nuclei
56.
The topographical arrangement of fibers in the medial lemniscus at
(E) Within the cerebellum
mid-olivary levels is such that the sensory information being con-
veyed by those fibers located most anterior (ventral) in this bundle
61. Which of the following represents the neurotransmitter most
will eventually terminate in which of the following structures?
likely affected in this woman?
(A) Anterior paracentral gyrus
(A) Acetylcholine
(B) Lateral one-third of the postcentral gyrus
(B) Dopamine
(C) Medial one-third of the postcentral gyrus
(C) Glutamate
(D) Middle one-third of the postcentral gyrus
(D) GABA
(E) Posterior paracentral gyrus
(E) Serotonin
284
Q & A’s: A Sampling of Study and Review Questions with Explained Answers
62.
A 39-year-old woman complains to her family physician that
67.
A 17-year-old boy presents with the major complaint that he is
“sometimes I see two of everything, but not always”. The exami-
having trouble playing baseball on the high school varsity team.
nation reveals that the woman can abduct both eyes and can adduct
The examination reveals a healthy, well-nurtured, athletic boy
her left eye but cannot adduct her right eye. All other eye move-
with normal motor and sensory function. The visual examina-
ment is normal. MRI shows a small lesion suggesting an area of de-
tion reveals a superior right quadrantanopia. MRI shows a small
myelination in the pons. Which of the following represents the
lesion in a position consistent with the visual field loss. Which of
most likely location of this lesion?
the following represents the most likely location of the lesion in
this boy?
(A) Left abducens nucleus
(B) Left medial longitudinal fasciculus
(A) Crossing fibers in the optic chiasm
(C) Right abducens nucleus
(B) Lower portions of the optic radiations in the left tem-
(D) Right medial longitudinal fasciculus
poral lobe
(E) Right PPRF
(C) Lower portions of the optic radiations in the right tem-
poral lobe
63.
A 20-year-old man is brought to the emergency department from
(D) Upper portions of the optic radiations in the left pari-
the site of a motorcycle accident. The examination reveals multi-
etal lobe
ple head injuries and a broken humerus. Cranial CT shows a basal
(E) Upper portions of the optic radiations in the right pari-
skull fracture extending through the jugular foramen. Assuming
etal lobe
that the nerve or nerves that traverse this opening are damaged,
which of the following deficits would most likely be seen in this
68.
A 68-year-old man is brought to the emergency department by his
man?
daughter. She explains that he unexpectedly began to have sudden
movements of his left “arm”. The examination reveals a slender
(A) Deviation of the tongue to the injured side on protrusion
man with hypertension and with periodic, uncontrollable flailing
(B) Diplopia and ptosis
movements of his left upper extremity suggestive of hemiballis-
(C) Drooping and difficulty elevating the shoulder
mus. Assuming this to result from a vascular occlusion, MRI
(D) Drooping of the face on the ipsilateral side
would most likely show an infarction in which of the following
(E) Loss of the efferent limb of the corneal reflex
structures?
64.
A 17-year-old boy is brought to the pediatrician by his mother. The
(A) Left substantia nigra
examination reveals that the boy has rigidity, athetoid movements
(B) Left subthalamic nucleus
(athetosis), and difficulty speaking. His ophthalmologist reports
(C) Right motor cortex
that the boy has a greenish-brown ring at the corneoscleral margin.
(D) Right substantia nigra
This boy is most likely suffering from which of the following?
(E) Right subthalamic nucleus
(A) Huntington disease
Questions 69 through 72 are based on the following patient.
(B) Parkinson disease
(C) Pick disease
A 67-year-old man visits his family physician with the complaint that
(D) Sydenham chorea
he is not able to “do things like I used to”. The examination reveals that
(E) Wilson disease
the man is not able to perform rapid alternating movements with his
left upper extremity, and is not able to touch his left index finger to his
65.
A 32-year-old woman complains to her gynecologist that her
nose because of a tremor that worsens as the finger approaches the
breasts are tender and a white fluid issues from her nipples. The
nose. He is able to do these movements on the right. When he walks,
examination reveals that the woman is not pregnant (she had her
he is unsteady with a tendency to fall to the left. He has no sensory
ovaries removed at age 28 resultant to a diagnosis of ovarian can-
deficits.
cer), that a milky substance can be expressed from her nipples, and
that she has a visual field deficit. MRI shows a tumor impinging on
69. Which of the following terms specifically designates the inability
the midline portion of the optic chiasm. Based on the position of
of this man to perform rapid alternating movements?
this tumor which of the following visual deficits would most likely
(A) Dysarthria
be seen in this woman?
(B) Dysdiadochokinesia
(A) Bitemporal hemianopsia
(C) Dysmetria
(B) Left homonymous hemianopsia
(D) Intention tremor
(C) Left superior quadrantanopia
(E) Resting tremor
(D) Right homonymous hemianopsia
(E) Right superior quadrantanopia
70. Which of the following terms specifically designates this man’s in-
ability to touch his nose with his index finger?
66.
Which of the following portions of the cerebellum have a close
(A) Dysmetria
structural and functional relationship with the vestibular appara-
(B) Intention tremor
tus and the vestibular nuclei?
(C) Rebound phenomenon
(A) Dentate nucleus and interposed nuclei
(D) Resting tremor
(B) Dentate nucleus only
(E) Static tremor
(C) Fastigial nucleus and flocculonodular lobe
(D) Hemisphere of the posterior lobe
(E) Interposed nuclei and hemisphere of the anterior lobe
Q & A’s: A Sampling of Study and Review Questions with Explained Answers
285
71.
The MRI of this man shows an infarcted area in the brain. Based
76.
A 39-year-old woman presents with sustained and oscillating mus-
on the deficits this man is experiencing, which of the following
cle contractions that have twisted her trunk and extremities into
represents the most likely location of this lesion?
unusual and abnormal postures. This woman is most likely suffer-
ing from which of the following?
(A) Basal nuclei on the left side
(B) Basal nuclei on the right side
(A) Dysarthria
(C) Cerebellar cortex and nuclei on the left side
(B) Dysmetria
(D) Cerebellar cortex and nuclei on the right side
(C) Dysphagia
(E) Midbrain on the right side
(D) Dyspnea
(E) Dystonia
72.
Assuming this lesion to be the result of the occlusion of an artery,
77.
A 21-year-old man is brought to the emergency department from
which of the following is the most likely candidate?
the scene of an automobile collision. He has a compound fracture
(A) Left anterior inferior cerebellar artery
of the humerus, a fractured tibia, various cuts and bruises, and sig-
(B) Left superior cerebellar artery
nificant facial trauma. Cranial CT shows fractures of the bones of
(C) Lenticulostriate arteries on the left
the face and orbit on the left, and a total collapse of the optic canal
(D) Right anterior inferior cerebellar artery
on that side with probable transection of the optic nerve. Follow-
(E) Right superior cerebellar artery
ing an initial recovery period, which of the following would most
likely be seen during an ophthalmologic examination?
73.
A 61-year-old woman complains to her family physician that the
(A) A loss of both the direct and consensual pupillary re-
muscles of her face sometimes twitch. The examination reveals
sponse when the light is shown in the right eye
that the woman has irregular and intermittent contractions of fa-
(B) A loss of only the consensual pupillary response when
cial muscles; sometimes these are painful. MRI shows an aberrant
the light is shown in the right eye
loop of an artery that appears to be compressing the facial nerve
(C) A loss of the direct but not the consensual pupillary re-
root. Which of the following is most likely the offending vessel in
sponse when a light is shown in the left eye
this case?
(D) Direct and consensual pupillary responses are intact
(A) Anterior inferior cerebellar artery
when light is shown in the left eye
(B) Anterior spinal artery
(E) Direct and consensual pupillary responses are intact
(C) Posterior inferior cerebellar artery
when light is shown in the right eye
(D) Posterior spinal artery
(E) Superior cerebellar artery
78.
A 27-year-old man presents with athetosis (athetoid movements),
rigidity, and dysarthria. He also has a flapping tremor. The man
74.
An 81-year-old man presents with a loss of pain, thermal sensa-
has an obvious greenish-brown ring at the corneoscleral margin.
tions, discriminative touch, and vibratory sense on the right side
A tentative diagnosis of advanced Wilson disease is made. MRI
of his body excluding his head. CT shows a comparatively small
showing which of the following would provide further, if not con-
infarct representing the territory of one vessel. Based on the posi-
clusive evidence, of this disease?
tions and relationships of the pathways conveying the sensations
(A) Atrophy of gyri of the frontal and temporal lobes
lost in this man, which of the following represents the most likely
(B) Degeneration and cavitation of the putamen
location of this lesion?
(C) Lacunae in the thalamus and internal capsule
(A) Caudal pons
(D) Loss of cells in the substantia nigra
(B) Midbrain
(E) Loss of the caudate nucleus
(C) Mid-medulla
(D) Rostral medulla
79.
A 77-year-old man complains to his family physician that he is hav-
(E) Upper cervical spinal cord
ing trouble picking up his coffee cup, shaving with a safety razor,
and picking up the checkers when playing with his grandson. The
examination reveals that the man is unable to control the distance,
75.
The MRI of a 70-year-old man shows an infarcted area in the me-
power, or accuracy of a movement as the movement is taking
dial medulla at a mid-olivary level on the left. This correlates
place. He undershoots or overshoots that target. Which of the fol-
with a loss of position sense from the man’s upper right extrem-
lowing most specifically describes this condition?
ity. Which of the following represents the location of the cell
bodies of origin of those fibers damaged in this patient in the
(A) Bradykinesia
medulla?
(B) Dysarthria
(C) Dysdiadochokinesia
(A) Cuneate nucleus on the left
(D) Dysmetria
(B) Cuneate nucleus on the right
(E) Dysphagia
(C) Gracile nucleus on the left
(D) Gracile nucleus on the right
Questions 80 through 82 are based on the following patient.
(E) Posterior root ganglia on the left
A 70-year-old woman is brought to the emergency department by
members of the volunteer fire department of a small town. She pri-
marily complains of weakness. The examination reveals a hemiplegia
involving the left upper and lower extremities, sensory losses (pain,
thermal sensations, and proprioception) on the left side of the body and
286
Q & A’s: A Sampling of Study and Review Questions with Explained Answers
face, and a visual deficit in both eyes. MRI shows an area of infarction
Answers for Chapter 7
consistent with the territory served by the anterior choroidal artery.
1.
Answer A: The combination of weakness on one side (corti-
80.
Which of the following visual deficits is seen in this woman?
cospinal involvement) and a loss of pain sensation on the opposite
(A) Left homonymous hemianopsia
side specifies components of a Brown-Séquard syndrome. The
(B) Left nasal hemianopsia
motor loss is ipsilateral to the damage and the sensory loss is con-
(C) Left superior quadrantanopia
tralateral; second order fibers conveying pain information cross in
(D) Right homonymous hemianopsia
the anterior white commissure ascending one to two spinal seg-
(E) Right superior quadrantanopia
ments in the process. In this patient, the lesion is on the left side
at about the T6 level; this explains the loss of pain sensation on the
81.
Which of the following most specifically identifies the pattern of
right beginning at the T8 dermatome level. Lesions at T8 or T10
sensory deficits experienced by this woman?
would result in a loss of pain sensation beginning, respectively, at
(A) Alternating hemianesthesia
dermatome levels T10 or T12 on the contralateral side. (p.
(B) Hemianesthesia
180-181)
(C) Paresthesia
(D) Sensory level
2.
Answer E: The paralysis of both lower extremities is paraplegia.
(E) Superior alternating hemiplegia
Monoplegia specifies paralysis of one extremity, hemiplegia of
both extremities on the same side, and quadriplegia of all four ex-
82.
The weakness of the extremities in this woman is most likely due
tremities. An alternating hemiplegia is the combination of a mo-
to damage to which of the following?
tor cranial nerve deficit on one side and a hemiplegia on the con-
tralateral side; this is a brainstem lesion not a spinal cord lesion.
(A) Corticospinal fibers on the left
(p. 190-193)
(B) Corticospinal fibers on the right
(C) Somatomotor cortex on the right
3.
Answer C: While the causes of swallowing difficulties may be
(D) Thalamocortical fibers to motor cortex on the right
central or peripheral (and multiple), this particular problem is
(E) Thalamocortical fibers to sensory cortex on the right
called dysphagia. Dysmetria is an inability to control the distance
or power of a movement and is commonly seen in cerebellar dis-
83.
A 16-year-old boy is brought to the family physician by his
ease. Dysarthria is difficulty in speaking, and dyspnea is a difficulty
mother. The mother explains that her son is having trouble in
in breathing; the latter is usually associated with diseases of the
school even though he is a hard worker and is well behaved. The
lungs or heart. Dysdiadochokinesia, an inability to perform rapid
examination reveals that the boy has a sensorineural hearing loss
alternating movements, is seen most commonly in cerebellar dis-
in his right ear. He has no other deficits. Which of the following
ease. (p. 190, 202)
represents the most likely location of the lesion in this boy?
(A) Auditory cortex
4.
Answer E: One possible cause of trigeminal neuralgia
(tic
(B) Cochlea
douloureux) is compression of the trigeminal root by the superior
(C) External ear
cerebellar artery or its main branches; surgical relocation of the
(D) Inferior colliculus
aberrant vessel (neurovascular decompression) relieves the symp-
(E) Middle ear
toms. Hemifacial spasm may be caused by compression of the fa-
cial nerve by the anterior inferior cerebellar artery (commonly
84.
Which of the following laminae of the lateral geniculate nucleus
called AICA). The other choices do not cause trigeminal neuralgia
receive input from the contralateral retina?
and are not a principal cause of cranial nerve dysfunction via root
(A)
1, 2
compression. (p. 41, 184-185)
(B)
1, 3, 5
(C)
1, 4, 6
5.
Answer E: The rostral interstitial nucleus of the medial longitu-
(D)
2, 3, 5
dinal fasciculus receives cortical input from the frontal eye field on
(E)
3, 4, 5, 6
the ipsilateral side and projects to the ipsilateral (heavy) and con-
tralateral (light) oculomotor and trochlear nuclei. This nucleus is
85.
A 12-year-old girl is brought to the pediatrician by her mother
regarded as the vertical gaze center. The paramedian pontine
who explains that the girl has started to “act funny”. The history
reticular formation is the horizontal gaze center. The oculomotor
reveals that the girl was treated for a hemolytic streptococcus in-
and abducens nuclei do not receive direct input from the frontal
fection 4 weeks before the appearance of her symptoms; the
eye field and the Edinger-Westphal is a visceromotor nucleus con-
mother states that the girl has had this problem for 3 weeks. The
taining preganglionic parasympathetic cell bodies. (p. 192-193)
examination reveals a well-nurtured girl with brisk, flowing, and
irregular movements of her face, neck, and upper extremities.
6.
Answer C: The absence of, or the aberrant development of,
This girl is most likely suffering from which of the following?
muscle around the oral cavity and over the cheek (muscles of fa-
(A) Huntington disease
cial expression, innervated by the facial [VII] nerve) indicate a fail-
(B) Parkinson disease
ure of proper differentiation of the second (2nd) pharyngeal arch.
(C) Senile chorea
Arch 2 also gives rise to the stapedius, buccinator, stylohyoid,
(D) Sydenham chorea
platysma, and posterior belly of the digastric. Mesoderm of the
(E) Weber syndrome
head outside of the pharyngeal arches gives rise to the extraocular
muscles and muscles of the tongue. The muscles of mastication
Q & A’s: A Sampling of Study and Review Questions with Explained Answers
287
(plus the tensor tympani, tensor veli palati, mylohyoid, anterior
ing of written or printed words. Aphonia is a loss of the voice fre-
belly of the digastric) arise from arch 1, the stylopharyngeus from
quently due to disease of, or injury to, the larynx. Aphasia is seen
arch 3, and striated muscles of the pharynx, larynx, and upper
in individuals with a lesion in the dominant hemisphere, and is
esophagus from arch 4. (p. 202-203)
manifest as an inability to comprehend the meaning of spoken,
written, or various other types of input. (p. 226-227)
7.
Answer C: Hypothalamocerebellar fibers that project to the
cerebellar nuclei and cortex contain histamine. GABA is found in
13.
Answer C: The Korsakoff syndrome is a constellation of deficits
several neurons that are located in the cerebellar cortex, and in
the include memory loss, confabulation, amnesia, and dementia
Purkinje cells glutamate is found in many pontocerebellar fibers
that is seen in chronic alcoholics; the manifestations are related, in
and in granule cells of the cerebellar cortex; and noradrenalin is
part, to excessive alcohol consumption and malnutrition. Thera-
found in ceruleocerebellar fibers. Serotonin is found in cells of the
peutic doses of thiamine are used to treat this disease. Broca apha-
reticular formation and in some raphe cells that project to the
sia (nonfluent or expressive aphasia) results from lesions in the
cerebellum. (p. 206-207)
dominant hemisphere. The Klüver-Bucy syndrome is related to
bilateral lesions to the amygdaloid complex, and Pick disease is de-
8.
Answer C: The best localizing sign in this patient is the paucity
mentia related to atrophy of the frontal and temporal lobes. Mun-
of eye movement and dilated pupil on the left; this indicates a le-
chausen syndrome is the fabrication or feigning of illness or disease
sion of the midbrain on the left at the level of the exiting oculo-
to gain attention or control. (p. 232-233)
motor fibers. The red nucleus is found at the same level and, more
importantly, immediately lateral to the red nucleus is a compact
14.
Answer B: Cell bodies in the nucleus ambiguus innervate mus-
bundle of cerebellothalamic fibers. The ataxia and tremor are re-
cles of the pharynx and larynx, including what is commonly called
lated primarily to damage to these cerebellar efferent fibers. The
the vocalis muscle. A lesion of this nucleus is one cause of
motor deficit is contralateral to the lesion because the corti-
dysarthria. The solitary tract and nuclei are concerned with vis-
cospinal fibers, through which the deficit is expressed, cross at the
ceral afferent information including taste, and the spinal trigemi-
motor (pyramidal) decussation. Lesions at the other choices
nal tract is made the central processes of primary sensory fibers
would not result in a paucity of eye movement and are, therefore,
conveying general somatic afferent (GSA) information from the
not potential candidates. (p. 132-133, 208-211)
ipsilateral side of the face and oral cavity. Proprioceptive infor-
mation from the ipsilateral upper extremity is transmitted via the
9.
Answer C: The lesion on the exiting oculomotor fibers (on the
cuneate nucleus; the vestibular nuclei are related to balance, equi-
left) damages the preganglionic fibers from the Edinger-Westphal
librium, and control of eye movement. (p. 202-203)
nucleus and removes their influence on the pupil. Consequently,
the intact postganglionic sympathetic fibers from the ipsilateral su-
15.
Answer C: The area of the brainstem that contains the nucleus
perior cervical ganglion predominate, and the pupil dilates.
ambiguus is served by branches of the posterior inferior cerebel-
Choices on the right are on the incorrect side. Damage to hypo-
lar artery (PICA). Occlusion of this vessel usually gives rise to the
thalamospinal fibers would remove sympathetic influence at the
PICA (lateral medullary or Wallenberg) syndrome. The anterior
intermediolateral cell column, and the pupil would constrict
inferior cerebellar artery (AICA) serves the lateral and inferior
(parasympathetic domination). (p. 200-201, 208-211, 220-221)
cerebellar surface and the superior cerebellar artery serves the su-
perior surface and much of the cerebellar nuclei. The labyrinthine
10.
Answer C: Cerebellar efferent fibers exit the cerebellum via the
artery, a branch of AICA, serves the inner ear. The posterior
superior cerebellar peduncle, cross in its decussation, and termi-
spinal artery serves the posterior columns and their nuclei. (p.
nate primarily in the ventral lateral nucleus (VL). Consequently,
110-111, 202-203)
the cerebellar nuclei on the left project to the right VL. The ven-
tral anterior nucleus does not receive significant cerebellar input.
16.
Answer D: Glutamate is found in many efferent fibers of the
While the ventral posterolateral nucleus receives a limited amount
cerebral cortex including those of the corticospinal tract. Conse-
of cerebellar input, its major role is the relay of somatosensory in-
quently, there are many glutaminergic terminals in the spinal
formation to the primary somatosensory cortex
(postcentral
cord. Acetylcholine is found at many central nervous system
gyrus). (p. 210-211)
(CNS) sites and at the neuromuscular junction, and dopamine is
found mainly in cells of the substantia nigra-pars compacta and in
11.
Answer B: The jerking movements of the upper extremity (as-
their nigrostriatal terminals. Gamma aminobutyric acid (GAMA)
terixis) are also called a flapping tremor and are seen in patients
is an inhibitory neurotransmitter and is found in many interneu-
with hepatolenticular degeneration (Wilson disease). Akinesia is
rons in the CNS. Serotonin is found in CNS areas such as the hy-
lack of movement. Resting tremor is seen in patients with disease
pothalamus, basal nuclei, and the raphe nuclei. (p. 190)
of the basal nuclei, such as Parkinson disease, and an intention
tremor is a characteristic of patients with cerebellar lesions. Dys-
17.
Answer A: The loss of sensation on one side of the face and the
tonia is the result of sustained muscle contractions that twist the
opposite side of the body is an alternating hemianesthesia (also
extremities, trunk, and neck into distorted and abnormal pos-
called an alternate hemianesthesia or a crossed hemianesthesia).
tures. (p. 214-215)
Epidural anesthesia refers to anesthesia resultant to injection of an
appropriate agent into the epidural space. The other choices are
12.
Answer A: This man is unable to recognize or comprehend the
motor abnormalities. (p. 180-181, 186-187)
meaning of sounds; although he is able to hear sounds, he is not
able to put meaning to the sounds; this man is suffering from au-
18.
Answer D: Lesions in the lateral portions of the brainstem dam-
ditory agnosia. Agraphia is the inability to write in a person with
age descending projections from the hypothalamus to the ipsilat-
no paralysis, and alexia is the inability to comprehend the mean-
eral intermediolateral cell column at spinal levels T1-T4, these be-
288
Q & A’s: A Sampling of Study and Review Questions with Explained Answers
ing the hypothalamospinal fibers. The result is Horner syndrome
with this disease. Weakness of the extremities may be seen, but
on the side ipsilateral to the lesion. Horner syndrome may also be
this almost always follows ocular movement disorders. Dysmetria
seen following cervical spinal cord lesions. A contralateral hemi-
is most commonly seen in cerebellar disease and may be present
plegia is not seen in lesions in lateral areas of the brainstem. The
in patients with lesions involving corticospinal fibers. Tremor is
other choices are syndromes or deficits specific to medial brain-
commonly seen in diseases or lesions of the basal nuclei and the
stem areas or to only a particular level. (p. 110, 124, 136,
cerebellum. (p. 200-201)
220-221)
24.
Answer B: The lesion in this woman is in the medulla, and the
19.
Answer C: The denticulate ligament is located on the lateral as-
sensory loss on the body (excluding the head) is on her left side; a
pect of the spinal cord at a midpoint in the posterior-anterior ex-
lesion in the medulla on the right side, involving fibers of the an-
tent of the spinal cord. The anterolateral system, the tract divided
terolateral system (ALS), accounts for this sensory deficit. A le-
in the anterolateral cordotomy, is located in the anterolateral por-
sion of the ALS on the left side of the medulla would result in sen-
tion of the spinal cord just inferior to the position of the denticu-
sory deficits on the right side of the body. The spinal trigeminal
late ligament. The posterolateral sulcus is the entrance point for
tract and nucleus convey pain and thermal sensations from the ip-
sensory fibers of the posterior roots; the anterolateral sulcus is the
silateral side (right side in this case) of the face, and the medial
exit point for motor fibers of the anterior root; and the posterior
lemniscus conveys vibratory and discriminative touch sensations
intermediate sulcus separates the gracile and cuneate fasciculi. The
from the contralateral side of the body. (p. 180-181, 184-185)
anterior median sulcus is located on the anterior midline and con-
tains the anterior spinal artery.
(p. 182-183)
25.
Answer C: The woman is hoarse because the lesion involves the
region of the medulla that includes the nucleus ambiguus. These
20.
Answer B: Damage to the gracile fasciculus on the left at T10,
motor neurons serve, via the glossopharyngeal (IX) and vagus (X)
the level of the umbilicus, will result in the deficits experienced by
nerves, the muscles of the larynx and pharynx, including the me-
this boy. The gracile fasciculus contains uncrossed ascending fibers
dial portion of the thyroarytenoid, also called the vocalis muscle.
conveying vibratory sensation, discriminative touch, and proprio-
Paralysis of the vocalis on one side will cause hoarseness of the
ception; consequently, the deficits will be seen beginning at the
voice. Hypoglossal nucleus or nerve, or facial nucleus lesions may
level of the lesion and extending caudally on the same side. These
cause difficulty with speech but not hoarseness. The spinal trigem-
fibers are the central processes of primary sensory neurons whose
inal tract conveys sensory input from the ipsilateral side of the
cell bodies are located in the ipsilateral posterior root ganglion.
face. There are no historical or examination findings to support a
The other choices are either on the wrong side (right) or at the
diagnosis of upper respiratory viral findings (cold or flu). (p.
wrong level. (p. 178-179)
180-181, 202-203)
21.
Answer C: The gag reflex is not regularly tested. However, in
26.
Answer C: The posterior inferior cerebellar artery
(PICA)
patients with dysphagia (difficulty swallowing) or dysarthria (dif-
serves the lateral area of the medulla that contains the anterolat-
ficulty speaking), the gag reflex should be evaluated. Dysmetria is
eral system, spinal trigeminal tract (loss of pain and thermal sen-
a movement disorder associated with cerebellar lesions; dysgeusia
sations from the ipsilateral side of the face), and the nucleus am-
is the perception of an abnormal taste or of a tastant when there is
biguus. Many patients that present with a PICA (Wallenberg or
none; and dyspnea is difficulty breathing, usually associated with
lateral medullary) syndrome also have involvement of the verte-
disease of the lung or heart. Gustatory agnosia is the inability to
bral artery on that side. The posterior spinal artery serves the pos-
recognize food or distinguish between different food items. (p.
terior column nuclei in the medulla, and the anterior spinal artery
186-187)
serves the pyramid, medial lemniscus, and exiting roots of the hy-
poglossal nerve. The anterior inferior cerebellar artery and the su-
22.
Answer B: The combination of a deviation of the tongue to one
perior cerebellar artery distribute to the pons and midbrain, re-
side (right) and the uvula to the opposite side (left) indicates a le-
spectively, plus significant portions of the cerebellum.
(p.
sion in the genu of the internal capsule on the left involving corti-
110-111, 180)
conuclear (corticobulbar) fibers. Corticonuclear fibers to the hy-
poglossal nucleus are crossed and the tongue deviates toward the
27.
Answer B: The mesencephalic nucleus, a part of the trigeminal
weak side on protrusion. These fibers to the nucleus ambiguus are
complex, has peripheral processes attached to neuromuscular
also crossed resulting in weakness of the contralateral side of the
spindles in the masticatory muscles, unipolar cell bodies in the ros-
palate. However, on attempted phonation (say “Ah”), the strong
tral pons and midbrain, and central collaterals that distribute bi-
side of the palate will contract and elevate, and the uvula will de-
laterally to the trigeminal motor nucleus. Through these connec-
viate to the intact side (opposite to the tongue). Lesions in the
tions, stretching of the spindle initiates a motor response. The
right genu would result in deficits on the opposite sides. Lesions
principal sensory and spinal trigeminal nuclei relay touch and
in the medial medulla on the right would include the tongue, ex-
pain/thermal sensations respectively. The hypoglossal nucleus is
clude the uvula but also show a left-sided hemiplegia. Lesions of
motor to the ipsilateral side of the tongue. (p. 184-185)
the right lateral medulla could include the uvula, but exclude the
tongue. A lesion in the crus would include a number of additional
28.
Answer E: The patient’s perception that his body is moving
deficits and would have to be on the left, not the right. (p.
around the room when he is actually sitting or laying still is sub-
192-193)
jective vertigo. Objective vertigo is the perception, on the part of
the patient, that he is still and objects in the room are moving. As
23.
Answer A: Deficits of eye movement (resulting in diplopia and
its name clearly implies, hysterical vertigo is a psychosomatic dis-
ptosis) are seen first in about 50% of all patients with myasthenia
order. Nystagmus is abnormal rhythmic movements of the eyes,
gravis and are eventually seen in approximately 85% of all patients
usually with fast and slow components. Ataxia is an inability to co-
Q & A’s: A Sampling of Study and Review Questions with Explained Answers
289
ordinate muscle activity resulting in an unsteady gait or other un-
the parotid gland. The ciliary receives from the Edinger-Westphal
coordinated movements. (p. 228-229)
nucleus and sends to the pupil; the pterygopalatine and sub-
mandibular receives from the superior salivatory nucleus (associ-
29.
Answer E: Excessive eating (gluttony), which may include a
ated with the facial [VII] nerve) and send, to the lacrimal, sub-
propensity to attempt to eat things not considered food items, is
mandibular, and sublingual glands, respectively. Intramural
hyperphagia. Dysphagia is difficulty in swallowing, and aphagia is
ganglia are located in the gut and receive input from the dorsal mo-
the inability to eat. Hyperorality is the tendency to put items in
tor vagal nucleus. (p. 202-203)
the mouth or to appear to be examining objects by placing them
in the oral cavity. Dyspnea is difficulty breathing. (p. 234-235)
35.
Answer B: Bilateral damage to the temporal lobes, as in an au-
tomobile collision, may result in damage to the hippocampus.
30.
Answer A: The constellation of deficits experienced by this man
While remote memory, the ability to recall events that happened
is characteristic of the Klüver-Bucy syndrome; this may be seen
years or decades ago, is intact, the man will have difficulty “re-
following bilateral damage to the temporal poles that includes the
membering” recent or immediate events. That is, he will find it
amygdaloid complex. The Korsakoff syndrome is seen, for exam-
difficult, if not impossible, to turn a new experience into longer-
ple, in chronic alcoholics, and senile dementia is a loss of cognitive
term memory (something that can be recalled in its proper con-
and intellectual function associated with neurodegenerative dis-
text at a later time). Dysphagia
(difficulty swallowing) and
eases of the elderly (such as Alzheimer). Wernicke (receptive or
dysarthria (difficulty speaking) are deficits usually seen in brain-
fluent) aphasia is seen in patients with a lesion in the area of the in-
stem lesions. Bilateral sensory losses of the lower portion of the
ferior parietal lobule, and the Wallenberg syndrome results from
body could be seen with bilateral damage to the posterior para-
a lesion in the medulla characterized by alternating hemisensory
central gyri (falcine meningioma) or to the anterior white com-
losses and, depending on the extent of the damage, other deficits.
missure of the spinal cord. Dementia is a multiregional symptom
(p. 234-235)
that usually involves several areas of the brain, cortical as well as
subcortical. (p. 232-233)
31.
Answer D: Hearing a sound in the ipsilateral ear with the appli-
cation of a tuning fork to the mastoid bone (actually the mastoid
36.
Answer C: The Meyer-Archambault loop is composed of optic
process of the temporal bone), and then hearing the sound again at
radiation fibers that loop through the temporal lobe; these fibers,
the external ear by moving the prongs to the external ear after the
on each side, convey visual input from the contralateral superior
sound disappears at the mastoid is a normal Rinne test. In a nega-
quadrant of the visual field. Consequently, a bilateral lesion of
tive Rinne test, the sound is not heard at the external meatus after
these fibers results as a bilateral superior quadrantanopia. Bilateral
it has disappeared from touching the mastoid. In a normal Weber
inferior quadrantanopia is seen in bilateral lesions that would in-
test, sound is heard equally in both ears with application of a tun-
volve the superior portion of the optic radiations. Right or left su-
ing fork to the midline of the forehead. A localizing Weber test in-
perior quadrantanopia is seen in cases of unilateral damage to, re-
dicates that sound is heard in the normal ear, but not in the ear with
spectively, the left or right Meyer-Archambault loop. A
disease or lesion. The Binet is an intelligence test. (p. 226-227)
bitemporal hemianopsia results in a lesion of the optic chiasm. (p.
220, 223)
32.
Answer B: In Huntington disease, especially in advanced stages,
there is a loss of the caudate nucleus and ex vacuo enlargement of
37.
Answer E: Degenerative changes in the dopamine-containing
the ventricles. The most obvious portion of the caudate missing in
cells of the substantia nigra pars compacta on the right side corre-
MRI coronal or axial planes is the head. The anterior lobe of the
late with a left-sided tremor. The altered message through the
cerebellum is diminished in size in alcoholic cerebellar degenera-
lenticular nucleus and thalamus and on to the motor cortex on the
tion, but not so in Huntington disease. Lesions of the subthalamic
side of the degenerative changes will result in tremor on the op-
nucleus result in hemiballismus, and degenerative changes in the
posite (right) side via altered messages traveling down the corti-
substantia nigra result in the motor deficits seen in Parkinson dis-
cospinal tract. The initial symptoms of Parkinson disease appear
ease. One of the main responsibilities of the lateral thalamic nuclei
on one side in about 80% of patients and extend to bilateral in-
is to convey input to the somatomotor and somatosensory cor-
volvement as the disease progresses. Bilateral changes in the sub-
tices. (p. 214-215)
stantia nigra correlate with bilateral deficits. The globus pallidus
does not receive direct nigral input but rather input via a nigro-
33.
Answer D: The paralysis of facial muscles on one side of the face
striatal-striatopallidal circuit. (p. 214-215)
(left in this case) with no paralysis of the extremities is a facial
hemiplegia; this is also commonly known as Bell palsy or facial
38.
Answer E: Damage to the trochlear nerve will cause diplopia on
palsy. Hemifacial spasms are irregular contractions of the facial
gaze inward and downward on the side of the injury. Abducens
muscles, and a central seven refers to paralysis of muscles on the
damage will result in an inability to look laterally on the side of the
lower half of the face contralateral to a lesion in the genu of the in-
lesion, and oculomotor injury will result in the loss of most eye
ternal capsule. Alternating hemiplegia describes a motor loss re-
movement on that side; the eye will be deviated slightly down and
lated to a cranial nerve on one side of the head and motor deficits
out. The ophthalmic nerve is sensory. (p. 200-201)
of the extremities on the contralateral side of the body. A similar
pattern of sensory losses is called an alternating hemianesthesia.
39.
Answer E: The combination of eye movement disorders and a
(p. 202-203)
contralateral hemiplegia localizes this lesion to the midbrain on the
side of the ocular deficits (right side). This also specifies that cor-
34.
Answer C: The otic ganglion receives preganglionic parasympa-
ticospinal fibers on the right (in the crus) are damaged, and places
thetic fibers from the inferior salivatory nucleus (associated with
the location of the cells of origin for these fibers in the somato-
the glossopharyngeal [IX] nerve) and sends postganglionic fibers to
motor cortex on the right side. The right crus contains the axons
290
Q & A’s: A Sampling of Study and Review Questions with Explained Answers
of these fibers but not the neuronal cell bodies. The left somato-
not normally take place in the human nervous system; spinoretic-
motor cortex influences the right extremities. The right precen-
ular fibers are in the divided ALS; and corticospinal fibers function
tral gyrus does not contain cells projecting to the left lumbosacral
in the motor sphere. (p. 182-183)
spinal cord (left lower extremity), and the right anterior paracen-
tral gyrus does not contain the cells that project to the left cervi-
45.
Answer C: The ataxia seen in patients with lesions of posterior
cal spinal cord (left upper extremity). (p. 15, 190-193)
column fibers is due to the loss of proprioceptive input and the re-
sultant inability of the patient to accurately judge the relative po-
40.
Answer A: The lesion in this man is central (brainstem) and in-
sition of the extremity. Thus, the extremity is forcibly slapped to
volves the IIIrd nerve. Consequently, the damage is to the pre-
the floor partially in an attempt to “create” the missing input. An-
ganglionic parasympathetic fibers in the root of the oculomotor
terolateral system fibers convey pain and thermal sensations, and
(III) nerve; this removes the parasympathetic influence (pupil con-
posterior root fibers convey these sensations plus those related to
striction) that originates from the Edinger-Westphal nucleus.
the posterior columns. Corticospinal, vestibulospinal, and reticu-
Fibers from the superior cervical ganglion are intact, hence the di-
lospinal fibers function in the motor sphere. (p. 178-179)
lated pupil. Fibers from the geniculate ganglion and inferior sali-
vatory nucleus distribute on the facial (VII) and glossopharyngeal
46.
Answer B: The axons of cell bodies located in the left trigemi-
(IX) nerves respectively. Postganglionic fibers from the ciliary
nal ganglion collect inside the brainstem to form the spinal trigem-
ganglion, while involved in this pathway, are not damaged in this
inal tract on the left (this tract is made up of the central processes
lesion. (p. 200-201)
of primary sensory fibers on the trigeminal [V] nerve). A lesion of
these fibers on the left side of the medulla will result in a loss of
41.
Answer E: The loss of most eye movement on one side (oculo-
pain and thermal sensations on the left side of the face. Lesions of
motor nerve root involvement) coupled with a paralysis of the ex-
the right trigeminal ganglion, trigeminothalamic fibers on the left,
tremities on the contralateral side is a superior alternating hemi-
and the right spinal trigeminal nucleus would all result in pain and
plegia (this is also Weber syndrome): superior because it is the most
thermal losses on the right side of the face. The principal sensory
rostral of three; alternating because it is a cranial nerve on one side
nucleus conveys touch information. (p. 184-185)
and the extremities on the other; and hemiplegia because one-half
of the body below the head is involved. A middle alternating hemi-
47.
Answer A: Recognizing that this woman has a sensory loss on
plegia involves the abducens (VI) nerve root and adjacent corti-
the left side of her face, damage to fibers of the anterolateral sys-
cospinal fibers, and an inferior alternating hemiplegia involves the
tem on the left correlates with the loss of pain and thermal sensa-
hypoglossal (XII) nerve root and corticospinal fibers in the pyra-
tions on the right side of her body. These anterolateral system
mid. Alternating hemianesthesia is a sensory loss, and a Brown-
(ALS) fibers cross in the spinal cord within about two levels of
Séquard syndrome is a spinal cord lesion with no cranial nerve
where they enter. Lesions of ALS on the right would result in a
deficits. (p. 200-201)
left-sided deficit on the body. Damage to anterior trigeminothal-
amic fibers on the left would produce a corresponding right-sided
42.
Answer B: Taste fibers (special visceral afferent, SVA) that
deficit on the face. The medial lemniscus conveys vibratory, dis-
serve the anterior two-thirds of the tongue on the ipsilateral side
criminative touch, and proprioceptive sensations. (p. 180-181,
are conveyed on the facial nerve and have their cell bodies of ori-
184-185)
gin in the geniculate ganglion. The trigeminal ganglion contains
cell bodies that convey general sensation (general somatic affer-
48.
Answer B: This patient has a lateral medullary syndrome (also
ent, GSA), and the ciliary ganglion contains visceromotor cell
commonly called a posterior inferior cerebellar artery, or PICA
bodies (general visceral efferent GVE, postganglionic, parasym-
syndrome) on the left; this correlates with the left-sided sensory
pathetic). The superior ganglion of the glossopharyngeal contains
loss on the face and right-sided sensory loss on the body. A lateral
cell bodies for taste from the posterior one-third of the tongue,
medullary lesion on the right would result in the same deficits, but
and the superior ganglion of the vagus nerve contains cell bodies
on the opposite sides. The Parinaud, Weber, and Benedikt syn-
for taste from the root of the tongue. (p. 187)
dromes are all associated with lesions in the midbrain.
(p.
180-181, 184-185, see also p. 136)
43.
Answer B: Stimulation of the supraorbital nerve (Vth nerve, af-
ferent limb of the supraorbital reflex) results in contraction of the
49.
Answer B: Wilson disease (hepatolenticular degeneration) is an
orbicularis oculi muscle (VIIth nerve, efferent limb of the supra-
inherited error of copper metabolism. Plasma levels of copper are
orbital reflex). Changes in pupil size relate to the third nerve, the
decreased; urinary levels are increased; and copper accumulates in
pupillary light reflex, and the distribution of postganglionic fibers
the liver, lenticular nuclei, and kidneys. Wilson disease can be
from the superior cervical ganglion. Contraction of masticatory
treated by reducing the level of dietary copper and administering
muscles is seen in the jaw-jerk reflex, and nystagmus usually re-
a copper-chelating agent. Maintenance can be achieved by taking
sults from cerebellum or brainstem lesions or disease of the
zinc, and treatment must be life-long. Ingestion of the other
vestibular apparatus. (p. 184-185, 202-203)
choices can cause serious illness and death. However, none of
these is the causative agent in hepatolenticular degeneration. (p.
44.
Answer A: Fibers in the postsynaptic posterior column and in
214-215)
the spinocervicothalamic pathways are spared in an anterolateral
cordotomy. These pathways originate from those laminae of the
50.
Answer E: The dilator pupillae muscle of the iris is innervated
posterior horn that also contribute to the anterolateral system. It
by postganglionic sympathetic fibers whose cell bodies of origin
is possible that these pathways remodel to transmit pain and ther-
are located in the ipsilateral superior cervical ganglion. Pregan-
mal sensations in the absence of the normal anterolateral system
glionic sympathetic cell bodies are found in the intermediolateral
(ALS) pathway. Regeneration to a functional state probably does
cell column. Preganglionic parasympathetic cell bodies are found
Q & A’s: A Sampling of Study and Review Questions with Explained Answers
291
in the Edinger-Westphal nucleus; axons of these cells terminate in
terminate in the lateral parts of the ventral posterolateral nu-
the ciliary ganglion, which, in turn, innervates the sphincter pupil-
cleus and, from there, be relayed to the posterior paracentral
lae muscle of the iris. The hypothalamus is the origin of hypothal-
gyrus (the lower extremity area of the primary somatomotor
amospinal fibers that project to the intermediolateral cell column.
cortex). The postcentral gyrus is the primary sensory cortex for
(p. 220-21)
the face (approximately the lateral one-third), upper extremity
(middle one-third), and the trunk (medial). The anterior para-
51.
Answer E: The deficits described for the man are consistent
central gyrus is the somatomotor cortex for the lower extrem-
with a tumor on the root of the vestibulocochlear (VIII) nerve;
ity. (p. 179-180)
these are correctly called a vestibular schwannoma because they
arise from the Schwann cells on the root of the vestibular portion
57.
Answer E: Syringomyelia is a cavitation in central areas of the
of the VIIIth nerve. Acoustic neuroma is an earlier, and now in-
spinal cord that results in damage to fibers conveying pain and
correct, designation for this lesion. Meningiomas arise primarily
thermal sensation as they cross the midline in the anterior white
from the arachnoid layer, ependymomas from the cells lining the
commissure. The loss is bilateral since fibers from both sides are
ventricular spaces, and a glioblastoma multiforme arises from as-
damaged as they cross. Tabes dorsalis presents as posterior column
trocytes within the substance of the brain. (p. 228-229)
deficits and lancinating pain; syringobulbia (cavitation within the
brainstem) may have long tract signs and cranial nerve deficits; and
52.
Answer A: This inherited disease is Friedreich ataxia; it initially
PICA syndrome characteristically has alternating sensory losses
appears in children in the age range of 8-15 years and has the char-
(one side of face, opposite side of body). The Brown-Séquard syn-
acteristic deficits described. Huntington disease is inherited, but
drome has both sensory (anterolateral system and posterior col-
appears in adults; olivopontocerebellar atrophy is an autosomal
umn) and motor (corticospinal) deficits. (p. 180-181)
dominant disease and gives rise to a different set of deficits. The
cause of Parkinson disease is unclear, but it is probably not inher-
58.
Answer A: There are basically only two areas where a relatively
ited; the Wallenberg syndrome is a brainstem lesion resulting
restricted lesion would result in weakness of both lower extrem-
from a vascular occlusion. (p. 204-205)
ities. One is in caudal parts of the pyramidal decussation (damage
to decussating corticospinal fibers traveling to the lumbosacral
53.
Answer B: A tumor in the foramen would damage the motor
cord levels), and the other would be a lesion in the falx cerebri
root of the trigeminal nerve and the mandibular root (sensory) of
(such as a meningioma) damaging the lower extremity areas on the
the Vth nerve. In this patient, the jaw deviates to the left and the
somatomotor cortex bilaterally. Decussating fibers in the rostral
sensory loss is on the left; this indicates that the tumor is on the
part of the pyramidal decussation terminate in cervical levels of
left. The deviation of the jaw to the left is due to the action of the
the spinal cord. Damage to either the pyramid or the lateral cor-
intact pterygoid muscles on the right (unlesioned side). Motor
ticospinal tract would result in a hemiplegia (pyramid-contralateral,
fibers on the trigeminal (V) nerve travel in association with the
lateral corticospinal tract-ipsilateral). Damage to the pyramids bi-
mandibular root and through the foramen ovale. Maxillary fibers
laterally would result in quadriplegia. (p. 190-191)
are sensory for the upper jaw and cheek area of the face. (p.
202-203)
59.
Answer C: The fatigability (progressive weakness), involve-
ment of ocular muscles initially, followed by other muscle weak-
54.
Answer C: The loss of abduction and adduction in one eye and
ness, is characteristic of myasthenia gravis. Amyotrophic lateral
of adduction in the opposite eye (the one-and-a-half syndrome) in-
sclerosis is an inherited disease that affects spinal and/or brainstem
dicates a lesion in the area of the paramedian pontine reticular for-
motor neurons and may result in upper or lower motor neuron
mation and abducens nucleus (in this case on the right side) and the
symptoms; this disease is usually fatal within a few years. Multiple
adjacent medial longitudinal fasciculus (MLF). The lesion damages
sclerosis is a demyelinating disease; Parkinson and Huntington dis-
the ipsilateral abducens motor neurons, internuclear neurons
eases are neurodegenerative conditions that eventually have a de-
passing to the contralateral MLF, and internuclear axons in the ip-
mentia component. (p.190, 202)
silateral MLF coming from the contralateral abducens nucleus.
Parinaud syndrome is a paralysis of upward gaze, and gaze palsies
tend to be toward one side and may result from cortical lesions.
60.
Answer C: The history and the combination of signs and symp-
Internuclear ophthalmoplegia is a deficit of medial gaze in one eye,
toms seen in this woman indicate a probable diagnosis of myas-
assuming a one-sided lesion. (p. 192-193)
thenia gravis and, consequently, a neurotransmitter disease at the
neuromuscular junction. Damage to corticospinal and corticonu-
55.
Answer A: Anterior trigeminothalamic collaterals that project
clear terminals and to synaptic contacts within the basal nuclei and
into the dorsal motor nucleus of the vagus are an important link in
the cerebellum would result in motor deficits but not in the pat-
the reflex pathway for vomiting. The superior salivatory nucleus
tern seen in this woman. (p. 190, 202)
is involved in the tearing or lacrimal reflex, the nucleus ambiguus
in the sneezing reflex, and the facial nucleus in the corneal reflex.
61.
Answer A: The neurotransmitter at the neuromuscular junction
Collaterals of primary afferent fibers to the mesencephalic nucleus
is acetylcholine; a blockage of postsynaptic nicotinic acetylcholine
that branch to enter the trigeminal motor nucleus mediate the jaw
receptors is the cause of the motor deficits characteristically seen
reflex. (p. 184-184)
in patients with myasthenia gravis. A loss of dopamine results in
Parkinson disease, motor deficits that are not seen in this woman.
56.
Answer E: The most anterior (ventral) portion of the medial
Glutamate and GABA are found in many pathways involved in mo-
lemniscus at mid-olivary levels contains second order fibers con-
tor function but are not located at the neuromuscular junction.
veying discriminative touch, vibratory sense, and propriocep-
Serotonin is found in pathways related to the basal nuclei, raphe
tion from the contralateral lower extremity. These axons will
nuclei, and the hypothalamus. (p. 190, 202)
292
Q & A’s: A Sampling of Study and Review Questions with Explained Answers
62.
Answer D: A lesion in the medial longitudinal fasciculus (MLF)
diations (geniculocalcarine radiations). The visual loss is in the vi-
on the right interrupts axons of the interneurons that arise from
sual field contralateral to the side of the lesion. Lesions in the
the left abducens nucleus and pass to oculomotor motor neurons
lower portions of the radiations result in deficits in the contralat-
on the right innervating the medial rectus muscle (internuclear
eral superior quadrants, while lesions in the upper portions of the
ophthalmoplegia). Damage to the abducens nucleus will indeed
radiations result in deficits in the contralateral lower quadrants.
destroy these interneurons, but will also result in an inability to
Consequently, in this boy (with a superior right quadrantanopia),
abduct the eye on the ipsilateral side. Injury to the MLF on the left
the lesion is in the lower portions of the optic radiations in the left
would result in an inability to adduct the left eye, and a lesion in
temporal lobe (Meyer-Archambault loop). The lesion in the chi-
the PPRF would most likely produce a bilateral horizontal gaze
asm would result in a bitemporal hemianopsia. (p. 220-223)
palsy. (p. 192-193, 200-201)
68.
Answer E: Hemiballismus is the result of a lesion largely con-
fined to the subthalamic nucleus on the side contralateral to the
63.
Answer C: A fracture through the jugular foramen would po-
deficit. These movements are violent, flinging, unpredictable, and
tentially damage the glossopharyngeal (IX), vagus (X), and spinal
uncontrollable. The abnormal movements are contralateral to the
accessory (XI) nerves. The major observable deficit would be a
lesion because the expression of the lesion is through the corti-
loss of the efferent limb of the gag reflex and a paralysis of the ip-
cospinal tract. Lesions in the left subthalamic nucleus would result
silateral trapezius and sternocleidomastoid muscles (drooping of
in a right-sided problem. Damage in the motor cortex would be
the shoulder, difficulty elevating the shoulder especially against re-
seen as a contralateral weakness, and cell loss in the substantia ni-
sistance, difficulty turning the head to the contralateral side). In-
gra would result in motor deficits characteristic of Parkinson dis-
volvement of facial muscles would suggest damage to the internal
ease (resting tremor, bradykinesia, stooped posture, festinating
acoustic or stylomastoid foramina; this would also be the case for
gait). (p. 216-217)
the efferent limb of the corneal reflex. Diplopia and ptosis would
suggest injury to the superior orbital fissure, as all three nerves
69.
Answer B: The inability to perform a rapid alternating move-
controlling ocular movement traverse this space. The hypoglossal
ment, such as pronating and supinating the hand on the knee, is
nerve (which supplies muscles of the tongue) passes through the
dysdiadochokinesia. This is one of several cardinal signs of cere-
hypoglossal canal. (p. 200-201)
bellar disease or stroke. Dysmetria is an inability to judge power,
distance, and accuracy during a movement, and dysarthria is diffi-
64.
Answer E: The constellation of signs and symptoms experi-
culty speaking. A resting tremor is seen in diseases of the basal nu-
enced by this boy are characteristic of Wilson disease, also called
clei, and an intention tremor is seen in cerebellar lesions. (p.
hepatolenticular degeneration. These may include movement dis-
208-211)
orders, tremor, the Kayser-Fleischer ring at the corneoscleral
margin, and eventual cirrhosis of the liver. Huntington and
70.
Answer B: The tremor that worsens as this man attempts to
Parkinson diseases are predominately motor problems in the early
bring his index finger to his nose is called an intention tremor,
stages and Pick disease is a degenerative disease of the cerebral cor-
sometimes referred to as a kinetic tremor. This type of tremor is
tex affecting mainly the frontal and temporal lobes; dementia is
one cardinal sign of cerebellar lesions. A resting tremor is seen in
the primary deficit. Sydenham chorea is seen in children follow-
diseases of the basal nuclei and a static tremor (postural tremor) is
ing an infection with hemolytic streptococcus; after treatment
seen in the trunk and extremities in a static position. Dysmetria is
for the infection, the choreiform movements usually resolve. (p.
an inability to judge distance, power, or accuracy during a move-
214-215)
ment. The rebound phenomenon is an inability of agonist and an-
tagonist muscles to rapidly adapt to changes in load. (p. 208-211)
65.
Answer A: A tumor impinging on the midline of the optic chi-
asm would damage crossing fibers from both eyes that are coming
71.
Answer C: The signs and symptoms in this man clearly indicate
from the nasal retinae and would reflect a loss of all, or part of both
a lesion in the cerebellum on the left side. The cerebellar nuclei
temporal retinal fields. Between 60% and 70% of pituitary ade-
on the left (lesion side) project to the contralateral thalamus
nomas are prolactin-secreting tumors. Right or left homonymous
(right) and from here to motor cortical areas (also right). The mo-
hemianopsia (the nasal visual field of one eye and the temporal vi-
tor cortex projects, via the corticospinal tract and its decussation,
sual field of the other eye) are seen following lesions of, respec-
back to the side of the body, excluding the head, on which the le-
tively, the left and right optic tracts. Quadrantanopsias result from
sion is located (left cerebellum). The motor expression of the
lesions in the optic radiations. (p. 220-221)
cerebellar deficit is through the corticospinal tract. The man’s left-
sided deficits are not consistent with a right cerebellar lesion, and
66.
Answer C: The flocculonodular lobe and the fastigial nucleus re-
the deficits are not consistent with a midbrain lesion. Lesions of
ceive input from the vestibular apparatus (primary vestibulocere-
the basal nuclei would result in a different set of motor disorders.
bellar fibers) and from the vestibular nuclei (secondary vestibulo-
(p. 208-211)
cerebellar fibers). In turn, the Purkinje cells of the flocculonodular
cortex and cells of the fastigial nucleus project to the vestibular nu-
72.
Answer B: The superior cerebellar artery serves the cortex on
clei as cerebellar corticovestibular and cerebellar efferent fibers,
the superior surface of the cerebellum and most of the cerebellar
respectively. While other areas of the cerebellar cortex may have
nuclei on the same side; in this case, it is the left artery. The ante-
a small projection to the vestibular nuclei, this is not significant
rior inferior cerebellar artery serves the cortex on the lateral infe-
compared to that of the flocculonodular lobe. (p. 228-229)
rior surface of the cerebellum and a small caudal tip of the dentate
nucleus. A lesion that involves primarily the cerebellar cortex will
67.
Answer B: Quadrantanopia, a loss of approximately one quar-
not result in long-term deficits. A lesion that involves cortex plus
ter (a quadrant) of the visual field, is seen in lesions in the optic ra-
nuclei or primarily nuclei, especially in an older patient (as in this
Q & A’s: A Sampling of Study and Review Questions with Explained Answers
293
man), is likely to result in long-term deficits. The lenticulostriate
its head) is seen in Huntington disease. Lacunae are usually seen in
arteries serve the basal nuclei. (p. 208-211)
patients who have had small strokes. (p. 214-215)
73.
Answer A: One cause of hemifacial spasm (intermittent and ab-
79.
Answer D: The inability of this man to control the distance,
normal contractions of the facial muscles) is compression of the fa-
power, and accuracy of a movement is dysmetria; this is charac-
cial root by a loop of the anterior inferior cerebellar artery, or per-
teristically seen in cerebellar lesions. Dysphagia is difficulty swal-
haps one of its larger branches. Aberrant loops of the superior
lowing, and dysarthria is difficulty speaking. The inability to per-
cerebellar artery may compress the trigeminal root (trigeminal neu-
form rapid alternating movements is dysdiadochokinesia, and
ralgia), and the posterior inferior cerebellar artery serves the lateral
bradykinesia is a slowness to initiate movement. The latter is char-
medulla and medial regions of the cerebellum. The anterior and pos-
acteristic of individuals with disease of the basal nuclei.
(p.
terior spinal arteries serve areas of the medulla. (p. 202-203)
208-211)
74.
Answer B: The anterolateral system and the medial lemniscus
80.
Answer A: The territory served by the anterior choroidal artery
are adjacent to each other in lateral portions of the midbrain and
includes the optic tract, inferior portions of the posterior limb of
are served largely by the same vessel(s), these being penetrating
the internal capsule, thalamocortical radiations within the poste-
branches of the quadrigeminal artery. This area may also receive
rior limb, and structures in the temporal lobe. The left-sided
some blood supply from the posterior choroidal arteries.
deficits indicate a lesion on the right side. A lesion of the right op-
Throughout the spinal cord, medulla, and into about the mid- to
tic tract results in a loss of vision in the opposite (left) visual fields;
more rostral pons, these fiber bundles are spatially separated from
this being the temporal visual field of the left eye and the nasal vi-
each other and have separate blood supplies. (p. 137, 178-181)
sual field of the right eye (left homonymous hemianopsia). This
constellation of deficits is known as the anterior choroidal artery
75.
Answer B: Fibers in the left medial lemniscus conveying posi-
syndrome. Quadrantanopia specifies a lesion in a portion of the
tion sense from the right upper extremity originate from cell bod-
optic radiations, and a nasal hemianopsia indicates a small lesion in
ies located in the right cuneate nucleus. These cuneate neurons
the lateral aspect of the optic chiasm on one side. (p. 158-159,
give rise to axons that form the internal arcuate fibers that arch
220-223)
towards the midline, cross, and collect to form the contralateral
medial lemniscus. The left cuneate nucleus sends axons to the
81.
Answer B: This woman has sensory losses on the left side of her
right medial lemniscus, and the gracile nucleus (right or left) con-
body and face that include pain/thermal sensations and the gen-
veys information from the lower extremity. Posterior root ganglia
eral category of proprioception (discriminative touch, vibratory
neurons project to the gracile or cuneate nuclei. (p. 178-179)
and position sense); this is a hemianesthesia, a loss of sensation on
one side of the body. This is a result of damage to thalamocortical
76.
Answer E: Dystonia is a movement disorder characterized by
fibers projecting from the ventral posteromedial and ventral pos-
abnormal, sometimes intermittent, but frequently sustained, con-
terolateral thalamic nuclei to the somatosensory cortex. Alternat-
tractions of the muscles of the trunk and extremities that force the
ing hemianesthesia refers to a sensory loss on one side of the face
body into a twisted posture. Dystonia may be seen in patients with
and on the contralateral side of the body. A sensory level is a char-
diseases of the basal nuclei. Dysmetria is the inability to judge the
acteristic of lesions in the spinal cord, and paresthesia refers to an
distance and trajectory of a movement. Dyspnea is difficulty
abnormal spontaneous sensation not a loss. A superior alternating
breathing; this may result from heart and/or lung disorders as well
hemiplegia is a motor deficit. (p. 158-159, 178-181, 220-223)
as from neurologic disorders. Dysphagia is difficulty swallowing,
and dysarthria is difficulty speaking. (p. 124-215)
82.
Answer B: The corticospinal fibers traversing the inferior por-
tions of the posterior limb of the internal capsule are damaged by
77.
Answer E: Transection of the optic nerve (on the left in this
an occlusion of the anterior choroidal artery; a left-sided deficit
man) eliminates the afferent limb of the pupillary light reflex, but
correlates with a lesion on the right side, especially when taking
the efferent limb, via the oculomotor nerve, is intact. Conse-
into consideration the concurrent visual loss. Damage to corti-
quently, there is a loss of both the direct response (in the blind eye)
cospinal fibers on the left would result in a right-sided deficit. The
and the consensual response (in the good eye) when light is shined
somatomotor cortex is not involved in the lesion. While thalamo-
in the blind eye, because the afferent limb is eliminated and no in-
cortical fibers are certainly damaged in this lesion, the deficits
put is getting to the center from which the efferent limb origi-
related to corticospinal fiber involvement predominate.
(p.
nates. On the other hand, light shined into the good eye (right in
190-191)
this man) results in a direct pupillary response (in the good eye)
and a consensual pupillary response in the blind eye because the
83.
Answer B: Sensorineural hearing loss, also called nerve deaf-
efferent limb of this reflex is not damaged for the blind eye. Other
ness, results from lesions or diseases that involve the cochlea or
combinations of responses may occur as a result of lesions in other
the cochlear portion of the vestibulocochlear nerve. Obstructions
portions of the nervous system. (p. 220-221)
of the external ear or diseases of the middle ear result in conduc-
tive deafness (conductive hearing loss). Lesions in the inferior col-
78.
Answer B: In addition to the motor deficits characteristic of this
liculus, auditory cortex, or other areas within the brain may result
disease, MRI would reveal a spongy degeneration (with cavita-
in difficulty localizing, interpreting, or understanding sound but
tions) of the lenticular nucleus most noticeable in the putamen.
do not result in total deafness in one ear. (p. 226-227)
There may also be a spongy degeneration in areas of the cerebral
cortex. Atrophy of frontal and temporal lobe gyri is seen in Pick
84.
Answer C: Laminae 1, 4, and 6 receive input from the ganglion
disease; loss of nigral cells is characteristic of Parkinson disease;
cells in the contralateral retina. Laminae 2, 3, and 5 receive an ip-
and loss of the caudate nucleus (especially noticeable as absence of
silateral input; laminae 1 and 2 are the magnocellular layers of the
294
Q & A’s: A Sampling of Study and Review Questions with Explained Answers
lateral geniculate nucleus; and laminae 3, 4, 5, and 6 are its par-
4.
The superficial middle cerebral vein forms a direct anastomotic
vocellular layers. (p. 222)
junction with which of the following venous structures on the lat-
eral aspect of the cerebral hemisphere?
85.
Answer D: Sydenham chorea is a disease of childhood thought
(A) Cavernous sinus
to be an autoimmune disorder seen in children as a sequel to a he-
(B) Confluence of sinuses
molytic streptococcus infection. In most children the disease is
(C) Superior sagittal sinus
self-limiting and the patient recovers with no permanent deficits.
(D) Transverse sinus
Huntington disease, Parkinson disease, and senile chorea present
(E) Veins of Labbé and Trolard
with motor deficits that partially resemble those seen in this girl
but these are diseases of adults or the elderly. Weber syndrome (a
5.
The coronal MRI of a 69-year-old man reveals an infarcted area in
superior alternating hemiplegia) is a motor deficit involving the
the region of the cerebral hemisphere lateral to the internal cap-
oculomotor nerve on one side and the corticospinal tract on the
sule but internal to the insular cortex. A comparison of coronal
opposite side. (p. 214-215)
and sagittal MRI suggests that the vessels involved are branches of
the middle cerebral artery. Which of the following branches or
segments of the middle cerebral artery are most likely involved in
Review and Study Questions for
this man?
Chapter 8
(A) Anterior and polar temporal branches
(B) Insular branches
(C) Lenticulostriate branches
1.
Which of the following arteries is generally found in the area of the
(D) Opercular segment
cingulate sulcus and has branches that serve the lower extremity
(E) Uncal artery
areas of the somatomotor and somatosensory cortex?
(A) Callosomarginal
6.
The anterior and middle cerebral arteries are the terminal
(B) Frontopolar
branches of which of the following vascular trunks?
(C) Internal parietal
(D) Parietooccipital
(A) Basilar artery
(E) Pericallosal
(B) Cavernous part of the internal carotid
(C) Cerebral part of the internal carotid
2.
A 44-year-old woman presents to her family physician with inter-
(D) External carotid artery
mittent headache and the complaint that she can’t see in her left
(E) Petrous part of the internal carotid
eye. The examination reveals that the woman is blind in her left
eye. When a light is shined into her left eye there is no direct or
7.
The superior sagittal sinus, straight sinus, and transverse sinuses
consensual pupillary light reflex. Magnetic resonance angiography
converge at which of the following landmarks?
(MRA) shows a large aneurysm at the origin of the ophthalmic
(A) Clivus
artery. Which of the following represents the usual point of origin
(B) Confluens sinuum
of this vessel?
(C) Great cerebral vein
(A) Cavernous part of the internal carotid artery
(D) Jugular foramen
(B) Cerebral part of the internal carotid artery
(E) Venous angle
(C) First segment (A1) of the anterior cerebral artery
(D) First segment (M1) of the middle cerebral artery
8.
A 47-year-old woman is brought to the emergency department by
(E) Petrous part of the internal carotid artery
her husband. She has a severe headache, nausea, and is somnolent.
The examination reveals that the woman is hypertensive and has
3.
The venous phase of an angiogram of a 52-year-old man suggests
papilledema. MRI shows evidence of cerebral edema, bilateral in-
a small tumor at what the neuroradiologist refers to as the venous
farcted areas in the thalamus, and a large sinus thrombosis that is
angle. Which of the following points most specifically describes
blocking the egress of blood through the vascular system. This
the position of the venous angle?
thrombus is most likely located in which of the following venous
(A) Where the internal cerebral vein meets the great cere-
structures?
bral vein
(A) Inferior sagittal sinus
(B) Where the superficial middle cerebral vein meets the
(B) Left sigmoid sinus
cavernous sinus
(C) Right transverse sinus
(C) Where the thalamostriate vein turns to form the inter-
(D) Straight sinus
nal cerebral vein
(E) Superior sagittal sinus
(D) Where the transverse sinus turns to form the sigmoid
sinus
(E) Where the vein of Labbé meets the vein of Trolard
Q & A’s: A Sampling of Study and Review Questions with Explained Answers
295
9.
A 39-year-old man presents to his family physician with a com-
14.
A 16-year-old boy with developmental delay has been followed
plaint of difficulty swallowing. The history reveals that the man has
since birth by a pediatric neurologist. A recent MRA is done in
had severe recurring headaches over the last 5 days and suffered
which major arteries and venous sinuses are visualized. It is con-
several bouts of vomiting. The examination confirms the difficulty
cluded that the pattern of the boy’s venous sinuses is essentially
swallowing, and reveals that the man’s voice is hoarse and gravely,
normal. Which of the following describes the usual pattern of the
and that he is unable to elevate his left shoulder against resistance.
superior sagittal sinus at the confluence of sinuses?
MRI shows a dural sinus thrombosis. Based on this man’s deficits,
(A) Always drains equally into the right and left transverse
which of the following represents the most likely location of this
sinuses
thrombus?
(B) Always drains into the left transverse sinus
(A) Left cavernous sinus
(C) Always drains into the right transverse sinus
(B) Left jugular bulb
(D) Usually drains into the left transverse sinus
(C) Left transverse sinus
(E) Usually drains into the right transverse sinus
(D) Right jugular bulb
(E) Straight sinus
Answers for Chapter 8
10.
Which of the following vessels forms a characteristic loop in the
cisterna magna that is prominent on lateral angiograms and, in the
process, supplies blood to the choroid plexus of the fourth ventri-
1.
Answer A: The callosomarginal artery lies generally in the re-
cle?
gion of the cingulate sulcus and gives rise to branches (paracentral
branches) that distribute to the anterior and posterior paracentral
(A) Anterior inferior cerebellar artery
gyri. The pericallosal artery is located immediately superior to the
(B) Posterior inferior cerebellar artery
corpus callosum and the frontopolar artery serves the medial as-
(C) Posterior spinal artery
pect of the frontal lobe. The internal parietal arteries are the ter-
(D) Superior cerebellar artery
minal branches of the pericallosal artery; these vessels distribute
(E) Vertebral artery
to the medial portion of the parietal lobe, the precuneus. The pari-
etooccipital artery is one of the terminal branches (part of P4) of
11.
The MRI of a 42-year-old man shows a small tumor in the choroid
the posterior cerebral artery. (p. 29, 240)
plexus of the third ventricle. Angiogram and MRA suggest that
this tumor contains numerous vascular loops. Which of the fol-
2.
Answer B: In most instances (approximately 80-85%), the
lowing represents the blood supply to this portion of the choroid
ophthalmic artery originates from the cerebral portion of the in-
plexus?
ternal carotid artery just as this parent vessel leaves the cavernous
(A) Anterior choroidal artery
sinus and passes through the dura. In a small percentage of cases
(B) Choroidal branches of AICA
the ophthalmic artery may originate from other locations on the
(C) Choroidal branches of PICA
internal carotid artery, including its cavernous portion. This ves-
(D) Lateral posterior choroidal artery
sel does not originate from the petrous portion of the internal
(E) Medial posterior choroidal artery
carotid or from anterior or middle cerebral arteries. (p. 25, 240)
12.
The angiogram of a 56-year-old woman shows an aneurysm orig-
3.
Answer C: The point at which the thalamostriate vein (also
inating from the lateral aspect of the basilar bifurcation and ex-
called the superior thalamostriate vein at this position) abruptly
tending into the space between the posterior cerebral and superior
turns 180
to form the internal cerebral vein is called the venous
cerebellar arteries. Based on the structure(s) located at this point,
angle. This angle is located immediately caudal to the position of
which of the following deficits would most likely be seen in this
the interventricular foramen and is, therefore, an important land-
woman?
mark. The thalamostriate vein is located in the groove between the
(A) Constriction of the ipsilateral pupil
thalamus and the caudate nucleus. At the superior aspect of the
(B) Inability to look down and out with the ipsilateral eye
thalamus, this vein is the superior thalamostriate vein, and, on the
(C) Inability to look laterally with the ipsilateral eye
inferior surface, it is called the inferior thalamostriate vein. None
(D) Inability to look up, down, or medially with the ipsilat-
of the other choices is involved in the formation of the venous an-
eral eye
gle. (p. 241)
(E) Loss of pain and thermal sensation from the ipsilateral
side of the face
4.
Answer E: The superficial middle cerebral vein is a compara-
tively obvious venous structure on the lateral surface of the hemi-
13.
The position of the posterior communicating artery, as frequently
sphere that communicates directly with the veins of Trolard (to
seen in MRA, is an important landmark that specifies the intersec-
the superior sagittal sinus) and Labbé (to the transverse sinus). The
tion of which of the following?
superficial middle cerebral vein also communicates with the cav-
(A) A1 and A2 segments
ernous sinus, but this sinus in not on the lateral aspect of the hemi-
(B) M1and M2 segments
sphere as specified in the question. The other choices do not re-
(C) M2 and M3 segments
ceive venous blood directly from the superficial middle cerebral
(D) P1 and P2 segments
vein. (p. 19, 241)
(E) P2 and P3 segments
5.
Answer C: The position of this lesion is in that portion of the
hemisphere occupied by the lenticular nucleus; the lenticulostri-
296
Q & A’s: A Sampling of Study and Review Questions with Explained Answers
ate branches of the M1 segment of the middle cerebral artery serve
may cause certain deficits, but not those experienced by this man.
this structure. The uncal, anterior, and polar temporal branches
(p. 244, 250)
originate from the M1 segment but do not serve structures in the
area of the hemisphere described. Insular branches (M2) and op-
10.
Answer B: The posterior inferior cerebellar artery (commonly
ercular branches (M3) serve cortical structures. (p. 25, 242)
called PICA) originates from the vertebral artery, courses around
the lateral aspect of the medulla, loops sharply into the space of
6.
Answer C: As the internal carotid artery exits the cavernous si-
the cisterna magna (giving off small branches to the choroid plexus
nus, it becomes the cerebral part of the internal carotid and, after
in the fourth ventricle), then joins the inferior and medial surface
giving rise to three important small branches (ophthalmic, ante-
of the cerebellum. None of the other choices forms prominent
rior choroidal, posterior communicating), bifurcates into the an-
vascular structures in the cisterna magna or serves the choroid
terior and middle cerebral arteries. These two cerebral vessels are
plexus of the fourth ventricle. (p. 246)
the terminal branches of the cerebral part of the internal carotid
artery. In approximately 70-75% of specimens, the anterior cere-
11.
Answer E: The medial posterior choroidal artery originates from
bral artery is the smaller of these two terminal branches. None of
the P2 segment of the posterior cerebral artery, arches around the
the other choices gives rise to the anterior and middle cerebral ar-
midbrain, and enters the caudal end of the third ventricle. The an-
teries. (p. 242)
terior choroidal artery serves the choroid plexus in the temporal
horn, and the lateral posterior choroidal artery serves the glomus
7.
Answer B: The superior sagittal sinus, straight sinus, the two
choroideum and extends into the plexi of the temporal horn and
transverse, and the occipital sinus (when present) converge at the
the body of the ventricle. These patterns may be somewhat vari-
confluence of sinuses (confluens sinuum), which is located inter-
able. Choroidal branches of anterior inferior cerebellar artery
nal to the external occipital protuberance. The venous angle is the
(AICA) serve the choroid plexus extending through the foramen of
junction of the thalamostriate and the internal cerebral veins, and
Luschka, and these branches from the posterior inferior cerebellar
the great cerebral vein (of Galen) receives the internal cerebral
artery (PICA) serve the plexus within the fourth ventricle. (p. 251)
veins and several smaller veins including the basal vein (of Rosen-
thal) and empties into the straight sinus. The jugular foramen con-
12.
Answer D: The oculomotor nerve (III) is located between the
tains the transition from the sigmoid sinus to the internal jugular
posterior cerebral and superior cerebellar arteries and may be
vein and the terminus of the inferior petrosal sinus. The clivus is
damaged by aneurysms at this location. Most eye movement
composed mainly of the basal part of the occipital bone; this is the
would be lost (the trochlear (IV) and abducens (VI) nerves are in-
location of the basilar plexus. (p. 19. 23, 243-245)
tact) and the ipsilateral pupil would be dilated, not constricted.
Sensation from the face is carried on the trigeminal nerve. Move-
8.
Answer D: A key observation in this woman is the bilateral in-
ment deficits related to injury to the IVth nerve (looking down and
farcted areas in the thalamus. The straight sinus receives venous
out) or the VIth nerve (looking laterally) are not affected. (p. 39,
flow from both internal cerebral veins; a blockage of flow through
40, 247, 252)
the straight sinus would adversely affect both thalami. Such a le-
sion would also cause potential damage to the medial temporal
13.
Answer D: The posterior communicating artery originates
lobe due to the disruption of flow through the basal vein (of Rosen-
from the cerebral part of the internal carotid artery and courses
thal). None of the other choices receives venous drainage directly
caudally to join the posterior cerebral artery (PCA). The part of
from the thalamus. (p. 29, 248, 250)
the PCA medial to this intersection is the P1 segment and the part
of the PCA immediately lateral to this junction is the P2 segment.
9.
Answer B: The deficits experienced by this man (difficulty swal-
Important branches arise from both of these parts of the PCA.
lowing, hoarseness, inability to elevate the left shoulder against re-
None of the other choices have any direct relationship to the points
sistance) point to damage to the glossopharyngeal (IXth), vagus
of origin of the posterior communicating artery. (p. 25, 247, 249)
(Xth), and spinal accessory (XIth) nerves or to their roots. All
three of these cranial nerves exit the jugular foramen along with
14.
Answer E: The drainage pattern of the superior sagittal sinus at
the continuity of the sigmoid sinus with the internal jugular vein
the confluence of sinuses is variable, including about equal to both
( jugular bulb or bulb of the jugular vein). In this case, the venous
transverse sinuses or mainly to the right or to the left. However,
thrombosis is at the left jugular bulb and impinging on these three
the usual pattern is for the superior sagittal sinus to drain pre-
cranial nerve roots. Dural sinus thrombosis of the other choices
dominately into the right transverse sinus. (p. 245)
Sources and Suggested Readings
AbuRahma A, Bergan JJ. Noninvasive Vascular Diagnosis. London:
Burt AM. Textbook of Neuroanatomy. Philadelphia: WB Saunders
Springer-Verlag, 2000.
Co, 1993.
Afifi AK, Bergman RA. Functional Neuroanatomy, Text and Atlas.
Cassini P, Ho RH, Martin GF. The brainstem origin of enkephalin- and
New York: McGraw-Hill, 1998.
substance-P-like immunoreactive axons in the spinal cord of the
Airaksinen MS, Panula P. The histaminergic system in the guinea pig cen-
North American opossum. Brain Behav Evol 1989;34:212-222.
tral nervous system: An immunocytochemical mapping study using
Clemente CD. Anatomy: A Regional Atlas of the Human Body, 3rd
an antiserum against histamine. J Comp Neurol 1988;273:163-186.
ed. Baltimore: Urban & Schwarzenberg, 1987.
Airaksinen MS, Flugge G, Fuchs E, Panula P. Histaminergic system in
Council of Biology Editions Style Manual Committee. Scientific Style
the tree shrew brain. J Comp Neurol 1989;286:289-310.
and Format—The CBE Manual for Authors, Editors, and Publish-
Aminoff MJ, Greenberg DA, Simon RP. Clincial Neurology, 3rd ed.
ers, 6th Ed. Cambridge: Cambridge University Press, 1994.
Stanford, CT: Appleton & Lange, 1996.
Craig AD, Jr, Sailer S, Kniffki K-D. Organization of anterogradely la-
Anderson KD, Reiner A. Extensive co-occurrence of substance P and
beled spinocervical tract terminations in the lateral cervical nucleus
dynorphin in striatal projection neurons: An evolutionarily con-
of the cat. J Comp Neurol 1987;263:214-222.
served feature of basal ganglia organization. J Comp Neurol 1990;
Craig AD, Jr, Tapper DN. Lateral cervical nucleus in the cat: Functional
295:339-369.
organization and characteristics. J Neurophysiol 1978;41:1511-1534.
Angevine JB, Mancall EL, Yakovlev PI. The Human Cerebellum: An
Crosby EC, Humphrey T, Lauer EW. Correlative Anatomy of the
Atlas of Gross Topography in Serial Sections. Boston: Little,
Nervous System. New York: Macmillan, 1962.
Brown, 1961.
DeArmond SJ, Fusco MM, Dewev MM. Structure of the Human
Beckstead RM, Morse JR, Norgren R. The nucleus of the solitary tract
Brain: A Photographic Atlas, 3rd ed. New York: Oxford University
in the monkey: Projections to the thalamus and brain stem nuclei. J
Press, 1989.
Comp Neurol 1980;190:259-282.
deGroot J. Correlative Neuroanatomy of Computed Tomography
Benarroch EE, Westmoreland BF, Daube JR, Reagan TJ, Sandok BA.
and Magnetic Resonance Imaging. Philadelphia: Lea & Febiger,
Medical Neuroscience, An Approach to Anatomy, Pathology, and
1984.
Physiology by Systems and Levels, 4th Ed. Baltimore: Lippincott
DeLacalle S, Hersh LB, Saper CB. Cholinergic innervation of the hu-
Williams & Wilkins, 1999.
man cerebellum. J Comp Neurol 1993;328:364-376.
Bishop GA, Ho RH, King JS. Localization of immunoreactivity in the
Dietrichs E, Haines DE. Interconnections between hypothalamus and
opossum cerebellum. J Comp Neurol 1985;235:301-321.
cerebellum. Anat Embryol 1989;179:207-220.
Bobillier P, Seguin S, Petitjean F, Salvert D, Touret M, Jouvert M. The
Donaghy M. Neurology. New York: Oxford, 1997.
raphe nuclei of the cat brainstem: A topographical atlas of their ef-
Dublin AB, Dublin WB. Atlas of Neuroanatomy with Radiologic Cor-
ferent projections as revealed by autoradiography. Brain Res 1976;
relation and Pathologic Illustration. St. Louis: Warren H. Green
113:449-486.
Inc., 1982.
Brodal A. Neurological Anatomy in Relation to Clinical Medicine, 3rd
Duus P. Topical Diagnosis in Neurology: Anatomy, Physiology, Signs,
ed. New York: Oxford University Press, 1981.
Symptons, 2nd ed. Stuttgart: Georg Thieme Verlag, 1989.
Brodal P. The Central Nervous System: Structure and Function. 2nd
Duvernoy HM. The Human Brain Stem and Cerebellum, Surface,
ed. New York: Oxford University Press, 1998.
Structure, Vascularization, and Three-Dimensional Sectional
Broman J, Blomqvist A. Substance P-like immunoreactivity in the lat-
Anatomy with MRI. Wein: Springer-Verlag, 1995.
eral cervical nucleus of the owl monkey (Aotus trivirgatus): A com-
England MA, Wakely J. Color Atlas of the Brain and Spinal Cord. St.
parison with the cat and rat. J Comp Neurol 1989;289:111-117.
Louis: Mosby Year Book, 1991.
Broman J, Westman J, Ottersen OP. Spinocervical tract terminals are
Federative Committee on Anatomical Terminology. Terminologia
enriched in glutamate-like immunoreactivity: An anterograde
Anatomica. Stuttgart and New York: Thieme, 1998.
transport-quantitative immunogold study in the cat. Neurosci Abstr
Fischer HW, Ketonen L. Radiographic Neuroanatomy: A Working At-
1989;15:941.
las. New York: McGraw-Hill, 1991.
Brown AG. Organization in the Spinal Cord. Berlin: Springer-Verlag,
Fix JD. Atlas of the Human Brain and Spinal Cord. Rockville, MD: As-
1981.
pen Publishers, 1987.
Buisseret-Delmas C, Batini C, Compoint C, Daniel H, Menetrey D.
Gasser RF. Personal communication, 1989.
The GABAergic neurones of the cerebellar nuclei: Projection to the
Gasser RF. Atlas of Human Embryos. New York: Harper & Row, 1975.
caudal inferior olive and to the bulbar reticular formation. In: P
Giesler GJ, Jr, Bjorkeland M, Xu Q, Grant G. Organization of the
Strata (ed). The Olivocerebellar System in Motor Control. (also Exp
spinocervicothalamic pathway in the rat. J Comp Neurol 1989;268:
Brain Res Ser 17:108-110). New York: Springer-Verlag, 1989.
223-233.
297
298
Sources and Suggested Readings
Gilman S, Winans SS. Manter and Gatz’s Essentials of Clinical Neu-
Leah J, Menetrey D, dePommery J. Neuropeptides in long ascending
roanatomy and Neurophysiology, 10th ed. Philadelphia: FA Davis
spinal tract cells in the rat: Evidence for parallel processing of as-
Company, 2003.
cending information. Neuroscience 1988;24:195-207.
Giuffrida R, Rustioni A. Glutamate and aspartate immunoreactivity in
Leblanc A. The Cranial Nerves: Anatomy Imaging Vascularization,
corticospinal neurons of rats. J Comp Neurol 1989;288:154-164.
2nd ed. Berlin: Springer-Verlag, 1995.
Gluhbegovic N, Williams TH. The Human Brain: A Photographic
Lechtenberg R. Synopsis of Neurology. Philadelphia: Lea & Febiger,
Guide. Hagerstown, MD: Harper & Row, 1980.
1991.
Greenberg MS. Handbook of Neurosurgery, 5th Ed. New York:
Lehéricy S, Hirsch EC, Cervera-Pierot P, Hersh LB, Bakchine S, Piette
Thieme, 2001.
F, Duyckaerts C, Hauw J-J , Javoy-Agid F, Agid Y. Heterogeneity
Groenewegen HJ, Berendse HW. Connections of the subthalamic nu-
and selectivity of the degeneration of cholinergic neurons in the
cleus with ventral striatopallidal parts of the basal ganglia in the rat.
basal forebrain of patients with Alzheimer’s disease. J Comp Neu-
J Comp Neurol 1990;294:607-622.
rol 1993;330:15-31.
Haines DE (ed). Fundamental Neuroscience, 2nd Edition. Philadel-
Ljungdahl A, Hökfelt T, Nilsson G. Distribution of substance P-like
phia: Churchill Livingstone/Elsevier Science, 2002.
immunoreactivity in the central nervous system of the rat I cell bod-
Haines DE, Dietrichs E. On the organization of interconnections be-
ies and nerve terminals. Neuroscience 1978;3:861-943.
tween the cerebellum and hypothalamus. In: JS King (ed), New
Lilly R, Cummings JL, Benson DF, Frankel M. The human Klüver-
Concepts in Cerebellar Neurobiology, pp. 113-149. New York:
Bucy syndrome. Neurology 33:1141-1145, 1983.
Alan R. Liss, 1987.
Lu GW. Spinocervical tract-dorsal column postsynaptic neurons: A
Haines DE, May PJ, Dietrichs E. Neuronal connections between the
double-projection neuronal system. Somatosens Mot Res 1989;6:
cerebellar nuclei and hypothalamus in Macaca fascicularis: Cerebello-
445-454.
visceral circuits. J Comp Neurol 1990;299:106-122.
Lufkin R, Flannigan BD, Bentson IR, Wilson GH, Rauschning W,
Hamilton WJ, Mossman HW. Human Embryology, 4th ed. Baltimore:
Hanafee W. Magnetic resonance imaging of the brainstem and cra-
Williams & Wilkins, 1972.
nial nerves. Surgical and Radiologic Anatomy 1986;8:49-66.
Heimer L. The Human Brain and Spinal Cord: Functional Neuroanatomy
Magnusson KR, Clements JR, Larson AA, Madl JE, Beitz AJ. Local-
and Dissection Guide. New York: Springer-Verlag, 1995.
ization of glutamate in trigeminothalamic projection neurons: A
Herbert H, Saper CB. Cholecystokinin-, galamin-, and corticotropin-
combined retrograde transport-immunohistochemical study. So-
releasing factor-like immunoreactive projections from the nucleus
matosens Res 1987;4:177-190.
of the solitary tract to the parabrachial nucleus in the rat. J Comp
Mai J, Assheurer J, Paxinos G. Atlas of the Human Brain. New York:
Neurol 1990;293:581-598.
Academic Press, 1997.
Huang X-F, Törk I, Paxinos G. Dorsal motor nucleus of the vagus
Martin JH. Neuroanatomy: Text and Atlas, 2nd ed. Stanford, CT: Ap-
nerve: A cyto-and chemoarchitectonic study in the human. J Comp
pleton & Lange, 1996.
Neurol 1993;330:158-182.
Martin TJ, Corbett JJ. Neuro-Ophthalmology: The Requisites of Oph-
Iverson MA et al. American Medical Association Manual of Style—A
thalmology. St. Louis: Mosby, 2000.
Guide for Authors and Editors, 9th Ed. Baltimore: Williams &
McKusick VA. On the naming of clinical disorders, with particular ref-
Wilkins, 1998.
erence to eponyms. Medicine 77:1-2 (1998a).
Jackson GD, Duncan JS. MRI Neuroanatomy: A New Angle on the
McKusick VA. Mendelian Inheritance in Man: A Catalog of Human
Brain. New York: Churchill Livingstone, 1996.
Genes and Genetic Disorders. 12th edition. Baltimore: Johns Hop-
Jennes L, Traurig HH, Conn PM. Atlas of the Human Brain. Philadel-
kins University Press, 1998b.
phia: JB Lippincott Co, 1995.
Mihailoff GA. Cerebellar nuclear projections from the basilar pontine
Jones SL, Light AR. Serotoninergic medullary raphespinal projection
nuclei and nucleus reticularis tegmenti pontis as demonstrated with
to the lumbar spinal cord in the rat: A retrograde immunohisto-
PHA-L tracing in the rat. J Comp Neurol 1993;330:130-146.
chemical study. J Comp Neurol 1992;322:599-610.
Miller RA, Burack E. Atlas of the Central Nervous System in Man, 3rd
Kandel ER, Schwartz JH, Jessell TM. Principles of Neural Sciences,
ed. Baltimore: Williams & Wilkins, 1982.
4th ed. McGraw-Hill: New York, 2000.
Monaghan PL, Beitz AJ, Larson AA, Altschuler RA, Madl JE, Mullett
Keirman JA. Barr’s The Human Nervous System: An Anatomical
MA. Immunocytochemical localization of glutamate-, glutaminase-
Viewpoint, 7th Ed. Philadelphia: Lippincott-Raven, 1998.
and aspartate aminotransferase-like immunoreactivity in the rat
Kirkwood JR. Essentials of Neuroimaging. New York: Churchill Liv-
deep cerebellar nuclei. Brain Res 1986;363:364-370.
ingstone, 1990.
Montemurro DG, Bruni JE. The Human Brain in Dissection, 2nd ed.
Krammer EB, Lischka MF, Egger TP, Reidl M, Gruber H. The mo-
New York: Oxford University Press, 1988.
toneuronal organization of the spinal accessory nuclear complex.
Nahin RL. Immunocytochemical identification of long ascending, pep-
Adv Anat Embryol Cell Biol 1987;103:1-62.
tidergic lumbar spinal neurons terminating in either the medial or
Kretschmann H-J, Weinrich W. Cranial Neuroimaging and Clinical
lateral thalamus in the rat. Brain Res 1988;443:345-349.
Neuroanatomy: Magnetic Resonance Imaging and Computed To-
Nelson BJ, Mugnaini E. Origins of GABAergic inputs to the inferior olive.
mography, 2nd ed. New York: Georg Thieme Verlag, 1993.
In: P Strata (ed), The Olivocerebellar System in Motor Control. (also
Larsell O, Jansen, J. The Comparative Anatomy and Histology of the
Exp Brain Res Ser 17:86-107). New York: Springer-Verlag, 1989.
Cerebellum: The Human Cerebellum, Cerebellar Connections, and
Newman DB, Hilleary SK, Ginsberg CY. Nuclear terminations of corti-
Cerebellar Cortex. Minneapolis: University of Minnesota Press, 1972.
coreticular fiber systems in rats. Brain Behav Evol 1989;34:223-264.
Sources and Suggested Readings
299
Nicholls JG, Martin AR, Wallace BG. From Neuron to Brain, 3rd ed.
mary afferents to the dorsal column nuclei of the cat. Exp Brain Res
Sunderland, MA: Sinauer Associates, Inc, 1992.
1977;27:1-14.
Nieuwenhuys R. Chemoarchitecture of the Brain. Berlin: Springer-
Salt TE, Hill RG. Neurotransmitter candidates of somatosensory pri-
Verlag, 1985.
mary afferent fibres. Neuroscience 1983;10:1083-1103.
Nieuwenhuys R, Voogd J, van Huijzen C. The Central Nervous Sys-
Schnitzlein HN, Hartley EW, Murtagh FR, Grundy L, Fargher JT.
tem: A Synopsis and Atlas, 3rd ed. Berlin: Springer-Verlag, 1988.
Computed Tomography of the Head and Spine: A Photographic
Noback CR, Strominger NL, Demarest RJ. The Human Nervous
Color Atlas of CT, Gross, and Microscopic Anatomy. Baltimore:
System, Structure and Function, 5th ed. Baltimore: Williams &
Urban & Schwarzenberg, 1983.
Wilkins, 1996.
Schnitzlein HN, Murtagh FR. Imaging Anatomy of the Head and Spine:
Nolte J. The Human Brain: An Introduction to its Functional
A Photographic Color Atlas of MRI, CT, Gross, and Microscopic
Anatomy, 5th ed. St. Louis: CV Mosby, 2002.
Anatomy in Axial, Coronal, and Sagittal Planes, 2nd ed. Baltimore:
Nolte J, Angevine JB. The Human Brain in Photographs and Diagrams.
Urban & Schwarzenberg, 1990.
St. Louis: CV Mosby, 1995.
Schwedtfeger WK, Buhl EH, Germroth P. Disynaptic olfactory input
Nudo RJ, Sutherland DP, Masterton RB. Inter- and intra-laminar dis-
to the hippocampus mediated by stellate cells in the entorhinal cor-
tribution of tectospinal neurons in 23 mammals. Brain Behav Evol
tex. J Comp Neurol 1990;292:163-177.
1993;42:1-23.
Shepherd GM. Neurobiology, 3rd ed. New York: Oxford University
Olszewski J, Baxter D. Cytoarchitecture of the Human Brain Stem,
Press, 1994.
2nd ed. Basel: S. Karger, 1982.
Siegel G, Agranoff BW, Albers RW, Molinoff PB. Basic Neurochem-
Pansky B, Allen DJ, Budd GC. Review of Neuroscience, 2nd ed. New
istry, Molecular, Cellular, and Medical Aspects, 6th ed. Philadel-
York: Macmillan Publishing Company, 1988.
phia: Lippincott-Raven, 1999.
Parent A. Carpenters Human Neuroanatomy, 9th ed. Baltimore:
Singer M, Yakovlev PI. The Human Brain in Sagittal Section. Spring-
Williams & Wilkins, 1996.
field, IL: Charles C Thomas, 1964.
Parent A, DeBellefeuille L. The pallidointralaminar and pallidonigral
Smith CG. Serial Dissections of the Human Brain. Baltimore: Urban &
projections in primate as studied by retrograde double-labeling
Schwarzenberg, 1981.
method. Brain Res 1983;278:11-27.
Smith Y, Bolam JP. The output neurones and the dopaminergic
Platzer W (ed). Pernkopf Atlas of Topographic and Applied Human
neurones of the substantia nigra receive a GABA-containing input
Anatomy, 3rd ed. Volume I, Head and Neck. Baltimore: Urban &
from the globus pallidus in the rat. J Comp Neurol 1990;296:
Schwarzenberg, 1989.
47-64.
Poritsky R. Neuroanatomical Pathways. Philadelphia: WB Saunders
Smith Y, Hazrati L-N, Parent A. Efferent projections of the sub-
Co, 1984.
thalamic nucleus in the squirrel monkey as studied by the PHA-L
Rasmussen AT. Atlas of Cross Section Anatomy of the Brain: Guide to
anterograde tracing method. J Comp Neurol
1990;294:
the Study of the Morphology and Fiber Tracts of the Human Brain.
306-323.
New York: Blakiston Division, McGraw-Hill Book Company, Inc,
Strata P (ed). The Olivocerebellar System in Motor Control. Berlin:
1951.
Springer-Verlag, 1989.
Reddy VK, Cassini P, Ho RH, Martin GF. Origins and terminations of
Sugiura K, Robinson GA, Stuart DG. Illustrated Guide to the Central
bulbospinal axons that contain serotonin and either enkephalin or
Nervous System. St. Louis: Ishiyaku EuroAmerica, Inc, 1989.
substance P in the North American opossum. J Comp Neurol
Swash M, Oxburgy J (eds). Clinical Neurology, volumes 1 and 2. New
1990;294:96-108.
York: Churchill Livingstone, 1991.
Reddy VK, Fung SJ, Zhuo H, Barnes CD. Localization of enkephalin-
Tatu L, Moulin T, Bogousslavsky J, Duvernoy H. Arterial territories
ergic neurons in the dorsolateral pontine tegmentum projecting to
of human brain: Brainstem and cerebellum. Neurology 1996;47:
the spinal cord of the cat. J Comp Neurol 1990;291:195-202.
1125-1135.
Rhoton AL, Jr. The posterior cranial fossa: Microsurgical anatomy and
Terzian H, Ore GD. Syndrome of Klüver and Bucy reproduced in man
surgical approaches. Neurosurgery
2000;
47
(Supplement);
by bilateral removal of the temporal lobes. Neurology 1955;5:
S7-S297.
373-380.
Rhoton AL, Jr. The supratentorial cranial space: Microsurgical
Tieman SB, Butler K, Neale JH. N-acetylaspartylglutamate: A neu-
anatomy and surgical approaches. Neurosurgery 2002; 51 (Supple-
ropeptide in the human visual system. JAMA 1988;259:2020.
ment); S1-S410.
Victor M, Ropper AH. Adams and Victor’s Principles of Neurology,
Rosenberg RN. Neurology, Volume 5 of The Science and Practice
7th Ed. New York: McGraw-Hill, 2001.
of Clinical Medicine, JM Dietschy, Editor-in-Chief. New York:
Walker JJ, Bishop GA, Ho RH, King JS. Brainstem origin of serotonin-
Grune & Stratton, 1980.
and enkephalin-immunoreactive afferents to the opposum’s cere-
Rowland LP (ed.). Merritt’s Textbook of Neurology, 10th ed. Balti-
bellum. J Comp Neurol 1988;276:481-497.
more: Lippincott Williams & Wilkins, 2000.
Walton L. Essentials of Neurology, 6th ed. New York: Churchill Liv-
Rustioni A. Non-primary afferents to the nucleus gracilis from the
ingstone, 1989.
lumbar cord of the cat. Brain Res 1973;51:81-95.
Watson C. Basic Human Neuroanatomy: An Introductory Atlas, 5th
Rustioni A. Non-primary afferents to the cuneate nucleus in the
ed. Boston: Little, Brown, 1995.
brachial dorsal funiculus of the cat. Brain Res 1974;75:247-259.
Waxman SG, deGroot J. Correlative Neuroanatomy, 22nd ed. Nor-
Rustioni A, Kaufman AB. Identification of cells of origin of non-pri-
walk, CT: Appleton & Lange, 1995.
300
Sources and Suggested Readings
Westlund KN, Carlton SM, Zhang D, Willis WD. Glutamate-im-
Witelson SF, Kigas DL. Sylvian fissure morphology and asymmetry in
munoreactive terminals synapse on primate spinothalamic tract
men and women: Bilateral differences in relation to handedness in
cells. J Comp Neurol 1992;322:519-527.
men. J Comp Neurol 1992;323:326-340.
Wicke L. Atlas of Radiologic Anatomy, 3rd ed. Baltimore: Urban &
Woolsey TA, Hanaway J, Gado MH: The Brain Atlas: A Visual Guide
Schwarzenberg, 1982.
to the Human Central Nervous System, 2nd Ed. Hoboken: Wiley,
Willard FH. Medical Neuroanatomy: A Problem-Oriented Manual
2003.
with Annotated Atlas. Philadelphia: JB Lippincott Co, 1993.
Yasargil MG. Microneurosurgery I Microsurgical Anatomy of the
Willis WD, Jr. The Pain System: The Neural Basis of Nociceptive
Basal Cisterns and Vessels of the Brain, Diagnostic Studies, Gen-
Transmission in the Mammalian Nervous System, Volume 8, Pain
eral Operative Techniques and Pathological Considerations of
and Headache. Basel: S Karger, 1985.
the Intracranial Aneurysms. New York: Georg Thieme Verlag,
Willis WD, Grossman RG. Medical Neurobiology: Neuroanatomical
1984.
and Neurophysiological Principles Basic to Clinical Neuroscience,
Zulegar S, Staubesand J. Atlas of the Central Nervous System in Sec-
3rd ed. St. Louis: CV Mosby, 1981.
tional Planes. Baltimore: Urban & Schwarzenberg, 1977.